You are on page 1of 253

Table of Contents

THE APPLICABLE LAWS .................................................................................................................................................................... 1 GENERAL PRINCIPLES ...................................................................................................................................................................... 1 1. Singer Sewing Machine vs. NLRC, 193 SCRA 271 ..................................................................................................................... 1 2. Manila Golf Club vs. IAC, 237 SCRA 207 .................................................................................................................................... 1 3. Encyclopedia Britanica vs. NLRC, 264 SCRA 4 [1996] ................................................................................................................ 2 4. Carungcong vs. Sunlife, 283 SCRA 319 ...................................................................................................................................... 2 5. Ramos vs CA, 380 SCRA 467 ..................................................................................................................................................... 3 6. Sonza vs. ABS-CBN, G.R. No. 138051, June 10, 2004 ............................................................................................................... 3 7. Lazaro vs. Social Security Commission, 435 SCRA 472 [2004] .................................................................................................. 4 8. Phil. Global Communications v. De Vera, 459 SCRA 260 [2005] ................................................................................................ 5 9. ABS-CBN vs. Nazareno, G.R. No. 164156, Sept. 26, 2006 ......................................................................................................... 6 10. Francisco vs. NLRC, 500 SCRA 690 [2006]............................................................................................................................... 6 11. Nogales et. al., vs. Capitol Medical Center et al., G.R. No. 142625, December 19, 2006 ......................................................... 7 12. Coca-Cola Bottlers Phils., vs. Dr. Climaco, G.R. No. 146881, February 15, 2007 ..................................................................... 8 13. Consolidated Broadcasting System vs. Oberio, G.R. No. 168424, June 8, 2007....................................................................... 9 14. Dumpit-Morillo vs. CA, G.R. No. 164652, June 8, 2007, citing 2004 Sonza............................................................................. 10 15. Lopez vs. Bodega City, G.R. No. 155731, Sept. 3, 2007, citing 2004 Abante & 2005 Consulta .............................................. 10 16. Calamba Medical Center vs. NLRC et al., G.R. No. 176484, Nov. 25, 2008............................................................................ 11 17. Escasinas et al., vs. Shangri-las Mactan Island Resort et al., G.R. No. 178827, March 4, 2009 ............................................. 12 18. Tongko v. Manufacturer Life Insurance Co. (MANULIFE) Inc., et al., G.R. No 167622, January 25, 2011 .............................. 14 19. Caong, Jr. v. Begualos, G.R. No. 179428, January 26, 2011 .................................................................................................. 15 20. Atok Big Wedge Company vs. Gison, G.R. No. 169510, August 8, 2011 ................................................................................ 16 21. Semblante vs. CA, G.R. No. 196426, August 15, 2011............................................................................................................ 18 22. Bernarte vs. Phil. Basketball Assoc., G.R. No. 192084, September 14, 2011 ......................................................................... 19 23. Lirio vs. Genovia, G.R. No. 169757, November 23, 2011 ........................................................................................................ 20 24. Jao vs. BCC Product Sales Inc., G.R. No. 163700, April 18, 2012 .......................................................................................... 22 RIGHT TO SECURITY OF TENURE .................................................................................................................................................. 23 1. ALU-TUCP vs. NLRC, 234 SCRA 678 [1994] ............................................................................................................................ 23 2. Cosmos Bottling Corp., vs NLRC, 255 SCRA 358 [1996] .......................................................................................................... 23 3. Purefoods v. NLRC 283 SCRA 136 [1997] ................................................................................................................................ 24 4. Phil. Fruit and Vegetable Industries v. NLRC, 310 SCRA 680 [1999] ........................................................................................ 25 5. Philips Semiconductor vs. Fardiquela, G.R. No. 141717, April 14, 2004 ................................................................................... 26 6. Alcira vs. NLRC, G.R. No. 149859, June 9, 2004 ...................................................................................................................... 28 7. Mitsubishi Motors Phils. vs. Chrysler Phil Labor Union, G.R. No. 148738, June 29, 2004 ......................................................... 29 8. Pangilinan vs. General Milling Co., G.R. No. 149329, July 2, 2004 ........................................................................................... 29 9. Ravago vs. Esso Eastern Marine Ltd., G.R. No. 158324, March 14, 2005 ................................................................................ 30 10. Hacienda Bino/Hortencia Stark vs. Cuenca, G.R. No. 150478, April 15, 2005, citing 2003 Hacienda Fatima ......................... 31 11. Phil Global Communication v. De Vera, G.R. No. 157214, June 7, 2005................................................................................. 32 12. Integrated Contractor and Plumbing Works, Inc. vs. National Labor Relations Commission and Glen Solon, G.R. No. 152427. August 9, 2005 ............................................................................................................................................................................... 35 13. Lacuesta vs. Ateneo de Manila, G.R. No. 152777, December 9, 2005 .................................................................................... 36 14. Poseidon Fishing/Terry De Jesus v. NLRC, G.R. No. 168052, February 20. 2006 .................................................................. 37 15. Abesco Construction vs. Ramirez, G.R. No. 141168, April 10, 2006 ....................................................................................... 38 16. Cebu Metal Corp., vs. Saliling, G.R. No. 154463, September 5, 2006 ..................................................................................... 38 17. Liganza v. RBL Shipyard Corp., G.R. No. 159682, October 17, 2006...................................................................................... 39 18. Fabeza v. San Miguel Corp., G.R. No. 150658, February 9, 2007 ........................................................................................... 41 19. Soriano vs. NLRC, G.R. No. 165594, April 23, 2007, citing 2005 Filipina Pre-fabricated Bldg. System (Filisystem) ............... 42 20. Caseres vs. Universal Robina Sugar Milling Corp., et al., G.R. No. 159343, September 28, 2007.......................................... 42 21. Pier 8 Arrastre & Stevedoring Services, Inc. vs Boclot, G.R. No. 173849, September 28, 2007 ............................................. 44 22. Pacquing vs. Coca-Cola Bottlers Phils., Inc., G.R. No. 157966, January 31, 2008, citing Magsalin vs. National Organization of Workingmen, G.R. No. 148492, May 9, 2003................................................................................................................................. 46 23. Cocomangas Hotel Beach Resort v. Visca, G.R. No. 167045, August 29, 2008...................................................................... 48

24. Price, et al., v Innodata Phils., G.R. No. 178505, September 30, 2008.................................................................................... 50 25. Agusan del Norte Electric Cooperative v. Cagampang, G.R. No. 167627, October 10, 2008 .................................................. 52 26. William Uy Construction et. al vs. Trinidad, GR No. 183250, March 10, 2010 ......................................................................... 53 27. Dacuital vs. L.M. Camus Engineering Corp.,G.R. No. 176748, September 1, 2010 ................................................................ 54 28. Millenium Erectors Corp. vs. Magallanes, G.R. No. 184362, November 15, 2010 ................................................................... 55 29. EXODUS INTERNATIONAL CONSTRUCTION CORPORATION vs. GUILLERMO BISCOCHO et. al.G.R. No. 166109, February 23, 2011 .......................................................................................................................................................................... 56 30. Leyte Geothermal Power Progressive Employees Union v. Phil National Oil Co., G.R. No. 176351, March 30, 2011 ............ 57 31. St. Paul College Quezon City vs. Ancheta II, G.R. No. 169905, September 7, 2011 ............................................................... 59 32. Lynvil Fishing Enterprises vs. Ariola, G.R. No. 181974, February 1, 2012............................................................................... 61 33. D.M. Consunji Inc. vs. Jamin, G.R. No. 192514, April 18, 2012 citing Maraguinot................................................................... 62 MANAGEMENT PREROGATIVE ....................................................................................................................................................... 64 1. Dosch vs. NLRC, 123 SCRA 296 [1983] .................................................................................................................................... 64 2. PT&T v. Court of Appeals, G.R. No. 152057, September 23, 2003 ........................................................................................... 65 3. Mendoza vs. Rural Bank of Lucban, G.R. No. 155421, July 7, 2004 ......................................................................................... 66 4. Duncan Assn. of Detailman-PTFWO vs Glaxo Wellcome Phils. G.R. 162994 ........................................................................... 67 5. Norkis Trading Co., vs. NLRC, G.R. No. 168159, August 19, 2005 ........................................................................................... 67 6. PLDT vs. Paquio, G.R. No. 152689, October 12, 2005 .............................................................................................................. 69 7. Star Paper Corp., vs. Simbol, G.R. No. 164774, April 12, 2006 ................................................................................................. 70 8. Rivera vs. Solidbank, G.R. No. 163269, April 19, 2006.............................................................................................................. 71 9. Tiu v. Platinum Plans, Inc., G.R. No. 163512, February 28, 2007 .............................................................................................. 72 10. Duldulao vs. Court of Appeals, G.R. No. 164893, March 1, 2007 ............................................................................................ 73 11. Almario v. Philippine Airlines, G.R. No. 170928, September 11, 2007 ..................................................................................... 74 12. San Miguel Corp. v. Pontillas, G.R. No. 155178, May 07, 2008 ............................................................................................... 75 13. Bisig Manggagawa sa Tryco vs. NLRC, G.R. No. 151309, Oct. 15, 2008................................................................................ 76 14. Coca-Cola Bottlers Philippines, Inc. v. Del Villar, G.R. No. 163091, October 6, 2010............................................................. 77 15. Manila Electric Co. vs. Lim, G.R. No. 184769, October 5, 2010 .............................................................................................. 80 16. Bello vs. Bonifacio Security Services, G.R. No. 188086, August 3, 2011 ................................................................................ 80 17. Alert Security and Investigation Agency vs. Pasawilan, G.R. No. 182397, September 14, 2011............................................. 81 18. Manila Pavilion Hotel vs. Delada, G.R. No. 189947, January 25, 2012 ................................................................................... 82 TERMINATION OF EMPLOYMENT ................................................................................................................................................... 84 1. Retuya v. NLRC, G.R. No. 148848, August 5, 2003, citing Bustamante .................................................................................... 84 2. Agabon vs. NLRC, G.R. No. 158693, November 17, 2004 ........................................................................................................ 85 3. Jaka Food Processing vs. Pacot, G.R. No. 151378, March 28, 2005 ........................................................................................ 87 4. Mauricio v. NLRC, G.R. No. 164635, November 17, 2005 ......................................................................................................... 88 5. Industrial Timber Corp. vs. Ababon, G.R. No. 164518, Janury 25, 2006 and March 28, 2007................................................... 89 6. Equitable Bank vs Sadac, G.R. No. 164772, June 8, 2006 ........................................................................................................ 90 7. Heirs of Sara Lee vs. Rey, G.R. No. 1499013, August 31, 2006 ............................................................................................... 91 8. Galaxi Steel Workers Union vs. NLRC, G.R. No. 165757, October 17, 2006, citing North Davao Mining ................................. 92 9. Sy vs. Metro Bank, G.R. No. 160618, November 2, 2006 ......................................................................................................... 93 10. King of Kings Transport vs. NLRC, G.R. No. 166208, June 29, 2007 ...................................................................................... 94 11. Johnson & Johnson v. Johnson Office & Sales Union, G.R. No. 172799, July 6, 2007 ........................................................... 96 12. Asian Terminal vs. NLRC, G.R. No. 158458, December 19, 2007, citing Standard Electric Mfg. vs. Standard Electric Employees Union, G.R. No. 166111, August 25, 2005 .................................................................................................................. 97 13. Smart Communications v. Astorga, G.R. No. 148142, January 28, 2008 ................................................................................ 98 14. Enriquez v. Bank of the Philippine Islands, G.R. No. 172812, February 12, 2008 ................................................................. 100 15. RB Michael Press vs. Galit, G.R. No. 153510, February 13, 2008 ......................................................................................... 102 16. Cosmos Bottling Corporation v. Nagrama, G.R. No 164403, March 4, 2008 ......................................................................... 104 17. School of the Holy Spirit of Q.C. vs. Taguiam, G.R. No. 165565, July 14, 2008 .................................................................... 105 18. Universal Staffing Services Inc. v. NLRC, G.R. No. 177576, July 21, 2008 ........................................................................... 106 19. Flight Attendants and Steward Association of the Philippines (FASAP) v. Philippine Airlines, G.R. No. 178083, G.R. No. 178083, July 22, 2008 .................................................................................................................................................................. 107 20. John Hancock Life Insurance Corp. vs. Davis, G.R. No. 169549, Sept. 3, 2008.................................................................... 111 21. Merin vs. NLRC, G.R. No. 171790, October 17, 2008 ........................................................................................................... 112

22. Yrasuegui vs. Phil Airlines, G.R. No. 168081, Oct. 17, 2008 ................................................................................................. 114 23. Sagales v. Rustans Commercial Corporation, G.R. No. 166554, November 27, 2008 .......................................................... 115 24. Garcia vs. PAL, G.R. No. 164856, Jan. 20, 2009, En Banc, citing Genuino vs. NLRC, G.R. No. 142732-33, December 4, 2007 ............................................................................................................................................................................................. 117 25. La Union Cement Workers Union et al., vs NLRC et al., G.R. No. 174621, January 30, 2009 .............................................. 118 26. Mendros, Jr. vs. Mitsubishi Motors Phils Corp., G.R. No. 169780, Feb. 16, 2009 ................................................................. 120 27. Rosa vs. Ambassador Hotel, G.R. No. 177059, March 13, 2009 ........................................................................................... 121 28. Motorola Phils. v. Ambrocio, G.R. No. 173279, March 30, 2009 ............................................................................................ 122 29. Perez et al., vs. Phil Telegraph & Telephone Company et al., G.R. No. 152048, April 7, 2009 ............................................. 123 30. Telecommunications Distributors Specialists Inc. et al., vs. Garriel, G.R. No. 174981, May 25, 2009, citing 2009 Perez ..... 124 31. Triumph International Philippines v. Apostol, G.R. No. 164423, June 16, 2009 ..................................................................... 125 32. Technological Institute of the Phils Teachers and Employees Organization vs. Court of Appeals, et al., G.R. No. 158703, June 26, 2009 .............................................................................................................................................................................. 127 33. Llamas v. Ocean Gateway Maritime and Management Services Inc., G.R. No. 179293, August 14, 2009 ........................... 128 34. Lowe Inc., v. CA, G.R. 164813 & 174590, August 14, 2009 .................................................................................................. 129 35. Estacio v. Pampanga I Electric Cooperative, G.R. No. 183196, August 19, 2009 ................................................................. 130 36. Maralit v. PNB, G.R. No. 163788, August 24, 2009 ............................................................................................................... 131 37. Quevedo v. Benguet Electric Cooperative, G.R. No. 168927, September 11, 2009 .............................................................. 133 38. Placido et al. v. NLRC, G.R. No. 180888, September 18, 2009 ............................................................................................. 134 39. Martinez v. B&B Fish Broker, G.R. No. 179985, September 18, 2009 ................................................................................... 135 40. Flight Attendants and Steward Association of the Phils vs. Phil Airlines, G.R. No. 178083, October 2, 2009, see July 22, 2008, main decision ..................................................................................................................................................................... 136 41. Eats-Cetera Food Services Outlet v. Letran, G.R. No. 179507, October 2, 2009 .................................................................. 137 42. Plantation Bay Resort and Spa, et al. vs. Dubrico, G.R. No. 182216, December 4, 2009 ..................................................... 139 43. Fulache v. ABS-CBN Broadcasting Corporation, G.R. No. 183810, January 21, 2010 .......................................................... 140 44. Ancheta vs. Destiny Financial Plans Inc. et al., G.R. No. 179702, Feb. 16, 2010 .................................................................. 141 45. Javellana, Jr. vs. Belen, G.R. Nos. 181913 & 182158, March 5, 2010................................................................................... 142 46. WPP Marketing Communications Inc., et al., vs. Galera, G.R. No. 169207, March 25, 2010 ................................................ 143 47. Mercado v. AMA Computer College, G.R. No. 183572, April 13, 2010 .................................................................................. 144 48. Pantoja vs. SCA Hygiene Products Corp., G.R. No. 163554, April 23, 2010 ......................................................................... 145 49. BPI v. NLRC, G.R. No. 179801, June 18, 2010 ..................................................................................................................... 146 50. Phil. Rural Reconstruction Movement vs. Pulgar, G.R. No. 169227, July 5, 2010 ................................................................. 147 51. Maribago Bluewater Beach Resort v. Dual, G.R. No. 180660, July 20, 2010 ........................................................................ 149 52. New Puerto Commercial vs. Lopez, G.R. No. 169999, July 26, 2010 .................................................................................... 151 53. Artificio vs. NLRC, G.R. No. 172988, July 26, 2010 ............................................................................................................... 152 54. Calipay vs. NLRC, G.R. No. 166411, August 3, 2010 ............................................................................................................ 154 55. Nacague v. Sulpicio Lines, G.R. No. 172589, August 8, 2010 ............................................................................................... 155 56. Century Canning Corp. vs. Ramil, G.R. No. 171630, August 8, 2010 .................................................................................... 156 57. D.M Consunji vs. Gobres, G.R. No. 169170, August 8, 2010 ................................................................................................ 158 58. Nagkaka-sang Lakas ng Manggagawa sa Keihin vs. Keihin Phils. Corp., G.R. No. 171115, August 9, 2010 ....................... 159 59. Garcia v. Molina, G.R. No. 157383, August 18, 2010 ............................................................................................................ 160 60. Escario v. NLRC, G.R. No. 160302, September 27, 2010 ..................................................................................................... 161 61. Simizu Phils Contractors v. Callanta, G.R. No. 165923, September 29, 2010 ....................................................................... 163 62. Solidbank Corporation v. Gamier, G.R. No. 159461, November 15, 2010 ............................................................................. 165 63. Coca-Cola Export Corp. v. Gacayan, G.R. No. 149433, December 15, 2010 ........................................................................ 166 64. Robinsons Galleria/Robinsons Supermarket v. Ranchez, G.R. No. 177937, January 19, 2011 ............................................ 168 65. Hospital Management Services v. HMSI-Medical Center Manila Employees Asso., G.R. No. 176287, January 31, 2011 ... 169 66. Culili v. Eastern Telecommunications Phils., G.R. No. 165381, February 9, 2011 ................................................................. 171 67. Plastimer Industrial Corp. v. Gopo, G.R. No. 183390, February 16, 2011 ............................................................................. 172 68. St. Marys Academy of Dipolog City vs. Palacio, G.R. No. 164913, September 8, 2010 ....................................................... 174 69. PLDT vs. Teves, G.R. No. 143511, November 15, 2010 ....................................................................................................... 175 70. University of the Immaculate Concepcion vs. NLRC, G.R. No. 181146, January 26, 2011 ................................................... 176 71. Simizu Phils Contractors v. Callanta, G.R. No. 165923, September 29, 2010 ....................................................................... 177 72. Manila Mining Corp. Employees Association-FFW vs. Manila Mining Corp., G.R. No. 178222-23, September 29, 2010...... 179 73. Lopez vs. Alturas Group of Companies, G.R. No. 191008, April 11, 2011 ............................................................................. 180

74. Apacible vs. Multimed Industries Inc., G.R. No. 178903, May 30, 2011................................................................................. 181 75. Barroga vs. Data Center College, G.R. No. 174158, June 27, 2011 ...................................................................................... 182 76. Lopez vs. Keppel Bank Phils., G.R. No. 176800, September 5, 2011.................................................................................... 183 77. St. Paul College Quezon City vs. Ancheta II, G.R. No. 169905, September 7, 2011 ............................................................. 184 78. Jumuad vs. Hi-Flyer Food, G.R. No. 187887, September 7, 2011 ......................................................................................... 185 79. Nissan Motor Phils. Angelo, G.R. No. 164181, September 14, 2011 ..................................................................................... 186 80. Phil. National Bank vs. Padao, G.R. No. 180849, November 16, 2011 .................................................................................. 187 81. Tamsons Enterprises Inc. vs. CA, G.R. No. 192881, November 16, 2011 ............................................................................. 188 82. Concepcion vs. Minex Import Corp., G.R. No. 153569, January 24, 2012 ............................................................................. 189 83. Morales vs. Harbour Centre Port Terminal Inc., G.R. No. 174208, January 25, 2012 ........................................................... 190 84. Mansion Printing Center vs. Bitara, G.R. No. 168120, January 25, 2012 .............................................................................. 191 85. Manila Electric Co. vs. Beltran, G.R. No. 173774, January 30, 2012 ..................................................................................... 192 86. Bank of Lubao vs. Manabat, G.R. No. 188722, February 1, 2012 ......................................................................................... 194 87. Canadian Opportunities Unlimited vs. Dalangin, G.R. No. 172223, February 6, 2012 ........................................................... 195 88. Manila Electric Co. vs. Gala, G.R. No. 191288 & 191304, March 7, 2012 ............................................................................. 196 89. Aro vs. NLRC, G.R. No. 174792, March 7, 2012.................................................................................................................... 197 90. Ymbong vs. ABS-CBN Broadcasting Corp., G.R. No. 184885, March 7, 2012 ...................................................................... 198 91. Blue Sky Trading Co. vs. Blas, G.R. No. 190559, March 7, 2012 .......................................................................................... 200 92. Internation management Services vs. Logarta, G.R. No. 163657, April 18, 2012 .................................................................. 201 93. Jiao vs. NLRC, G.R. No. 182331, April 18, 2012 ................................................................................................................... 203 94. Realda vs. New Age Graphics Inc., G.R. No. 192190, April 25, 2012 .................................................................................... 204 95. Kakampi and Its Members Panuelos vs. Kingspoint Express & Logistics, G.R. No. 194813, April 25, 2012 ......................... 205 SUSPENSION OF BUSINESS OPERATIONS................................................................................................................................. 207 1. JPL Marketing Promotion vs. Court of Appeals, G.R. No. 151966, July 8, 2005...................................................................... 207 2. Pido vs NLRC, G.R. No. 169812, February 23, 2007............................................................................................................... 207 3. Megaforce Security & Allied Services vs. Lactao, G.R. No. 160940, July 21, 2008 ................................................................. 208 4. National Mines and Allied Workers Union vs. Marcopper Mining Corp., G.R. No. 174641, Nov. 11, 2008 .............................. 210 5. Eagle Star Security Services Inc. vs. Mirando et al., G.R. No. 179512, July 30, 2009 ............................................................. 211 6. Nationwide Security & Allied Services v. Valderama, G.R. No. 186614, February 23, 2011 ................................................... 212 7. Nippon Housing Phils. vs. Leynes, G.R. No. 177816, August 3, 2011 ..................................................................................... 214 DISEASE AS A GROUND FOR TERMINATION .............................................................................................................................. 216 1. Sy vs. Court of Appeals, G.R. No. 142293, February 27, 2003................................................................................................ 216 2. Manly Express vs. Payong, G.R. No. 167462, October 25, 2005 ............................................................................................ 216 3. Duterte vs. Kingswood Trading Co., G.R. No. 160325, October 4, 2007 ................................................................................. 217 4. Villaruel vs. Yeo Han Guan, G.R. No. 169191, June 1, 2011 ................................................................................................... 218 OTHER CAUSES OF SEVERANCE OF EMPLOYMENT RELATION ............................................................................................. 220 1. Pantranco North Express vs. NLRC, 259 SCRA 161 [1996] .................................................................................................... 220 2. Phil. Airlines vs. Airline Pilots Asso. Of Phils., G.R. No. 143686, January 15, 2002 ................................................................ 221 3. Cainta Catholic School vs. Cainta Catholic School Employees Union, G.R. No. 151021, May 4, 2006 citing 1996 Pantranco North Express .............................................................................................................................................................................. 221 4. Jaculbe vs. Silliman University, G.R. No. 156934, March 16, 2007 ......................................................................................... 223 5. Globe Telecom vs. Crisologo, G.R. No. 17644, August 10, 2007 ............................................................................................ 226 6. BMG Records Phils et al., vs. Aparecio, et al., G.R. No. 153290, September 5, 2007, citing Phil Today vs. NLRC, 267 SCRA 202 [1996] .................................................................................................................................................................................... 227 7. Blue Angel Manpower and Security Services vs. CA, G.R. No. 161196, July 28, 2008 ........................................................... 228 8. Guerzon Jr et al vs. Pasig Industries Inc., et al., G.R. No. 170266, Sept. 12, 2008 ................................................................. 229 9. Suarez Jr. et al., vs. National Steel Corp., G.R. No. 150180, Oct. 17, 2008 ............................................................................ 230 10. Goodrich Mfg Corp vs. Ativo et al., G.R. No. 188002, Feb. 1, 2010 ....................................................................................... 231 11. Korean Air Co. Ltd. v. Yuson, G.R. No. 170369, June 16, 2010 ............................................................................................ 232 12. Cercado v. Uniprom Inc., G.R. No. 188154, October 13, 2010 .............................................................................................. 233 13. Bilbao vs. Saudi Arabian Airlines, G.R. No. 183915, December 14, 2011 ............................................................................. 234 14. San Miguel Properties vs. Gucaban, G.R. No. 153982, July 18, 2011 ................................................................................... 235 15. Skippers United Pacific vs. Doza, G.R. No. 175558, February 8, 2012 ................................................................................. 236

PRESCRIPTION OF CLAIMS .......................................................................................................................................................... 238 1. Ludo & Luym Corp. vs Saornido, G.R. No. 140960, January 20, 2003 .................................................................................... 238 2. Degamo vs. Avantgarde Shipping corp., G.R. no. 154460, November 22, 2005 ..................................................................... 239 3. Intercontinental Broadcasting Corp. vs. Panganiban, G.R. No. 151407, February 6, 2007 .................................................... 239 4. Far East Agricultural Supply vs. Lebatique, G.R. No. 162813, February 12, 2007 ................................................................. 240 5. Victory Liner, Inc. vs. Race, G.R. No. 164820, March 28, 2007 ............................................................................................... 241 6. J.K. Mercado & Sons Agricultural Enterprises vs. Sto. Tomas, G.R. No. 158084, August 29, 2008 ........................................ 242 7. Reyes vs. Nlrc, G.R. No. 180551, February 10, 2009 .............................................................................................................. 243 8. LWV Construction Corp. vs. Dupo, G.R. No. 172342, July 13, 2009 ....................................................................................... 244 9. PLDT v. Pingol, G.R. No. 182622, September 8, 2010 ............................................................................................................ 245 10. Medline Management Inc. vs. Roslinda, G.R. No. 168715, September 15, 2010 .................................................................. 247 11. University of East vs. University of East Employees Assoc., G.R. No. 179593, September 14, 2011 ................................... 248

LABOR RELATIONS
Atty. Jefferson M. Marquez
THE APPLICABLE LAWS GENERAL PRINCIPLES CASES: 1. Singer Sewing Machine vs. NLRC, 193 SCRA 271 Facts: Singer Machine Collectors Union-Baguio filed a petition for direct certification as the sole and exclusive bargaining agent of all collectors of Singer Sewing Machine. The company opposed the petition mainly because the union members are not employees but independent contractors as evidenced by the collection agency agreement which they signed. Med-Arbiter ruled that there exists an employee-employer relationship and granted the certification election which was affirmed by Sec. Drilon. The company files the present petition on the determination of the relationship. The union insists that the provisions of the Collection Agreement belie the companys position that the union members are independent contractors. Ruling: The present case calls for the application of the control test, which if not satisfied, would lead to the conclusion that no employee-employer relationship exists. If the union members are not employees, no right to organize for the purpose of bargaining or as a bargaining agent cannot be recognized. The following elements are generally considered in the determination of the relationship: the selection and engagement of the employee, payment of wages, power of dismissal and the power to control the employees conduct which is the most important element. The nature of the relationship between a company and its collecting agents depends on the circumstances of each particular relationship. Not all collecting agents are employees and neither are all collecting agents independent contractors. The agreement confirms the status of the collecting agents as independent contractor. The requirement that collection agents utilize only receipt forms and report forms issued by the company and that reports shall be submitted at least once a week is not necessarily an indication of control over the means by which the job collection is to be performed. Even if report requirements are to be called control measures, any control is only with respect to the end result of the collection since the requirements regulate the things to be done after the performance of the collection job or the rendition of service. The plain language of the agreement reveals that the designation as collection agent does not create an employment relationship and that the applicant is to be considered at all times as an independent contractor. The court finds that since private respondents are not employees of the company, they are not entitled to the constitutional right to form or join a labor organization for the purposes of collective bargaining. There is no constitutional and legal basis for their union to be granted their petition for direct certification.

2. Manila Golf Club vs. IAC, 237 SCRA 207 Facts: This is originally filed with the Social Security Commission (SSC) via petition of 17 persons who styled themselves as Caddies of Manila Golf and Country Club-PTCCEA for the coverage and availment of benefits of the Social Security Act as amended, PTCCEA (Philippine Technical, Clerical, Commercial Employees Association) a labor organization where which they claim for membership. The same time two other proceedings were filed and pending. These are certification election case filed by PTCCEA on behalf of the same caddies of Manila Golf and Country club which was in favor of the caddies and compulsory arbitration case involving PTCCEA and Manila Golf and Country Club which was dismissed and ruled that there was no employer-employee relationship between the caddies and the club. The question involved in the case is whether or not rendering caddying services for members of golf clubs and their guests in said clubs courses or premises are the employees of such clubs and therefore within the compulsory coverage of the Social Security System (SSS). Ruling: The Court does not agree that the facts logically point to the employer-employee relationship. In the very nature of things, caddies must submit to some supervision of their conduct while enjoying the privilege of pursuing their occupation within the premises and grounds of whatever club

Page 1

LABOR RELATIONS
Atty. Jefferson M. Marquez
they do work in. They work for the club to which they attach themselves on sufferance but, on the other hand, also without having to observe any working hours, free to leave anytime they please, to stay away for as long they like. These considerations clash frontally with the concept of employment. It can happen that a caddy who has rendered services to a player on one day may still find sufficient time to work elsewhere. Under such circumstances, the caddy may leave the premises and to go to such other place of work that he wishes. These are things beyond the control of the petitioner. The caddy (LLamar) is not an employee of petitioner Manila Golf and Country Club and the petitioner is under no obligation to report him for compulsory coverage of SSS.

3. Encyclopedia Britanica vs. NLRC, 264 SCRA 4 [1996] Facts: Limjoco was a Sales Divison of Encyclopaedia Britannica and was in charge of selling the products through some sales representatives. As compensation, he would receive commissions from the products sold by his agents. He was also allowed to use the petitioners name, goodwill and logo. It was agreed that office expenses would be deducted from Limjocos commissions. In 1974, Limjoco resigned to pursue his private business and filed a complaint against petitioner for alleged non-payment of separation pay and other benefits and also illegal deduction from sales commissions. Petitioner alleged that Limjoco was not an employee of the company but an independent dealer authorized to promote and sell its products and in return, received commissions therein. Petitioner also claims that it had no control and supervision over the complainant as to the manners and means he conducted his business operations. Limjoco maintained otherwise. He alleged he was hired by the petitioner and was assigned in the sales department. The Labor Arbiter ruled that Limjoco was an employee of the company. NLRC also affirmed the decision and opined that there was no evidence supporting allegation that Limjoco was an independent contractor or dealer. On appeal, petitioner assails that there was no employee-employer relationship. Ruling: There was no employee-employer relationship. In determining the relationship, the following elements must be present: selection and engagement of the employee, payment of wages, power of dismissal and power to control the employees conduct. The power of control is commonly regarded as the most crucial and determinative indicator of the presence or absence of an employee-employer relationship. Under the control test, an employee-employer relationship exists where the person for whom the services are performed reserves a right to control not only the end to be achieved, but also the manner and means to be employed in reaching that end. The issuance of guidelines by the petitioner was merely guidelines on company policies which sales managers follow and impose on their respective agents. Limjoco was not an employee of the company since he had the free rein in the means and methods for conducting the marketing operations. He was merely an agent or an independent dealer of the petitioner. He was free to conduct his work and he was free to engage in other means of livelihood. In ascertaining the employee-employer relationship, the factual circumstances must be considered. The element of control is absent where a person who works for another does so more or less at his own pleasure and is not subject to definite hours or conditions of work, and in turn is compensated in according to the result of his efforts and not the amount thereof. Hence, there was no employee-employer relationship.

4. Carungcong vs. Sunlife, 283 SCRA 319 Facts: Susan Carungcong began her career in the insurance industry in 1974 as an agent of Sun Life Assurance Company of Canada. She signed an Agent Agreement with Sun Life. In virtue of which she was designated the latters agent to solicit applications for its insurance and annuity policies. This contract was superseded some five years later when she signed two (2) new agreements. The first, denominated Career Agents or Unit Managers Agreement, dealt with such matters as the agents commissions, his obligations, limitations on his authority, and termination of the agreement by death, or by written notice with or without cause. The second was titled, Managers Supplementary Agreement. It explicitly described as a further agreement. Carungcong and Sun Life executed another Agreement named New Business Manager with the function generally to manage a New Business Office established. This latest Agreement stressed that the New Business Manager in performance of his

Page 2

LABOR RELATIONS
Atty. Jefferson M. Marquez
duties defined herein, shall be considered an independent contractor and not an employee of Sun Life, and that under no circumstance shall the New Business Manager and/or his employees be considered employees of Sun Life. Ms. Eleizer Sibayan, Manager of Sun Lifes Internal Audit Department, commenced an inquiry into the special fund availments of Carungcong and other New Business Managers. Respondent Lance Kemp, had been receiving reports of anomalies in relation thereto from unit managers and agents. Thereafter, on January 1990, Carungcong was confronted with and asked to explain the discrepancies set out in Sibayans report. She was given a letter signed by Metron V. Deveza, CLU, Director, Marketing, which advised of the termination of her relationship with Sun Life. Carungcong promptly instituted proceedings for vindication in the Arbitration Branch of the National Labor Relations Commissions on January 16, 1990. There she succeeded in obtaining a favorable judgment. Labor Arbiter found that there existed an employer-employee relationship between her and Sun Life. On appeal, the National Labor Relations Commission reversed the Arbiters judgment. It affirmed that no employment relationship existed between Carungcong and Sun Life. She contented that she was an employee subject to control and supervision by Sun Life. Ruling: Noteworthy is that this last agreement which emphasized, like the Career Agents or Unit Managers Agreement first signed by her, that in performance of her duties defined herein. Carungcong would be considered an independent contractor and not an employee of Sun Life, and that under no circumstance shall the New Business Manager and/or his employees be considered employees of Sun Life. Carungcong is an independent contractor. It was indicated in the very face of the contract. The rules and regulations of the company is not sufficient to establish an employer-employee relationship. It does not necessarily create any employer-employee relationship where the employers controls have to interfere in the methods and means by which employee would like employ to arrive at the desired results. Carungcong admitted that she was free to work as she pleases, at the place and time she felt convenient for her to do so. She was not paid to a fixed salary and was mainly paid by commissions depending on the volume of her performance. She was not an employee of Sun Life Co.

5. Ramos vs CA, 380 SCRA 467 Facts: Petitioner Erlinda Ramos was advised to undergo an operation for the removal of her stone in the gall bladder. She was referred to Dr. Hosaka, a surgeon, who agreed to do the operation. The operation was scheduled on June 17, 1985 in the De los Santos Medical Center. Erlinda was admitted to the medical center the day before the operation. On the following day, she was ready for operation as early as 7:30 am. Around 9:30, Dr. Hosaka has not yet arrived. By 10 am, Rogelio wanted to pull out his wife from the operating room. Dr. Hosaka finally arrived at 12:10 pm more than 3 hours of the scheduled operation. Dr. Guiterres tried to intubate Erlinda. The nail beds of Erlinda were bluish discoloration in her left hand. At 3 pm, Erlinda was being wheeled to the Intensive care Unit and stayed there for a month. Since the ill-fated operation, Erlinda remained in comatose condition until she died. The family of Ramos sued them for damages. One of the issues involved was that there was an employee-employer relationship that existed between the medical center and Drs. Hosaka and Guiterrez. Ruling: Private Hospitals hire, fire and exercise real control over their attending and visiting consultant staff. While consultants are not technically employees, the control exercised, the hiring and the right to terminate consultants fulfill the hallmarks of an employer-employee relationship with the exception of payment of wages. The control test is determining. In applying the four fold test, DLSMC cannot be considered an employer of the respondent doctors. It has been consistently held that in determining whether an employer-employee relationship exists between the parties, the following elements must be present: (1) selection and engagement of services; (2) payment of wages; (3) the power to hire and fire; and (4) the power to control not only the end to be achieved, but the means to be used in reaching such an end. The hospital does not hire consultants but it accredits and grants him the privilege of maintaining a clinic and/or admitting patients. It is the patient who pays the consultants. The hospital cannot dismiss the consultant but he may lose his privileges granted by the hospital. The hospitals obligation is limited to providing the patient with the preferred room accommodation and other things that will ensure that the doctors orders are carried out. The court finds that there is no employer-employee relationship between the doctors and the hospital.

6. Sonza vs. ABS-CBN, G.R. No. 138051, June 10, 2004

Page 3

LABOR RELATIONS
Atty. Jefferson M. Marquez
Facts: In May 1994, ABS-CBN signed an agreement with the Mel and Jay Management and Development Corporation (MJMDC). ABS-CBN was represented by its corporate officers while MJMDC was represented by Sonza, as President and general manager, and Tiangco as its EVP and treasurer. Referred to in the agreement as agent, MJMDC agreed to provide Sonzas services exclusively to ABS-CBN as talent for radio and television. ABS-CBN agreed to pay Sonza a monthly talent fee of P310, 000 for the first year and P317, 000 for the second and third year. On April 1996, Sonza wrote a letter to ABS-CBN where he irrevocably resigned in view of the recent events concerning his program and career. After the said letter, Sonza filed with the Department of Labor and Employment a complaint alleging that ABS-CBN did not pay his salaries, separation pay, service incentive pay,13th month pay, signing bonus, travel allowance and amounts under the Employees Stock Option Plan (ESOP). ABSCBN contended that no employee-employer relationship existed between the parties. However, ABS-CBN continued to remit Sonzas monthly talent fees but opened another account for the same purpose. The Labor Arbiter dismissed the complaint and found that there is no employee-employer relationship. NLRC affirmed the decision of the Labor Arbiter. CA also affirmed the decision of NLRC. Ruling: Case law has consistently held that the elements of an employee-employer relationship are selection and engagement of the employee, the payment of wages, the power of dismissal and the employers power to control the employee on the means and methods by which the work is accomplished. The last element, the so-called "control test", is the most important element. Sonzas services to co-host its television and radio programs are because of his peculiar talents, skills and celebrity status. Independent contractors often present themselves to possess unique skills, expertise or talent to distinguish them from ordinary employees. The specific selection and hiring of SONZA, because of his unique skills, talent and celebrity status not possessed by ordinary employees, is a circumstance indicative, but not conclusive, of an independent contractual relationship. All the talent fees and benefits paid to SONZA were the result of negotiations that led to the Agreement. For violation of any provision of the Agreement, either party may terminate their relationship. Applying the control test to the present case, we find that SONZA is not an employee but an independent contractor. The control test is the most important test our courts apply in distinguishing an employee from an independent contractor. This test is based on the extent of control the hirer exercises over a worker. The greater the supervision and control the hirer exercises, the more likely the worker is deemed an employee. The converse holds true as well the less control the hirer exercises, the more likely the worker is considered an independent contractor. To perform his work, SONZA only needed his skills and talent. How SONZA delivered his lines, appeared on television, and sounded on radio were outside ABS-CBNs control. ABS-CBN did not instruct SONZA how to perform his job. ABS-CBN merely reserved the right to modify the program format and airtime schedule "for more effective programming." ABS-CBNs sole concern was the quality of the shows and their standing in the ratings. Clearly, ABS-CBN did not exercise control over the means and methods of performance of Sonzas work. A radio broadcast specialist who works under minimal supervision is an independent contractor. Sonzas work as television and radio program host required special skills and talent, which SONZA admittedly possesses. ABS-CBN claims that there exists a prevailing practice in the broadcast and entertainment industries to treat talents like Sonza as independent contractors. The right of labor to security of tenure as guaranteed in the Constitution arises only if there is an employer-employee relationship under labor laws. Individuals with special skills, expertise or talent enjoy the freedom to offer their services as independent contractors. The right to life and livelihood guarantees this freedom to contract as independent contractors. The right of labor to security of tenure cannot operate to deprive an individual, possessed with special skills, expertise and talent, of his right to contract as an independent contractor.

7. Lazaro vs. Social Security Commission, 435 SCRA 472 [2004] Facts: Respondent Rosalina M. Laudato filed a petition before the SSC for social security coverage and remittance of unpaid monthly social security contributions against her three (3) employers. Among the respondents was herein petitioner Angelito L. Lazaro, proprietor of Royal Star Marketing (Royal Star), which is engaged in the business of selling home appliances. Lazaro denied that Laudato was an employee but instead claimed that she was an agent of the company. Lazaro also maintained that she was not mandated to work of definite work hours and thus not deemed to be a regular employee of Royal Star Marketing, the company of Lazaro. SSC promulgated a decision rendering that Laudato is a regular employee of Royal Star Marketing and entitled to social security contributions. Lazaro filed a petition for review before the CA where CA ruled that Laudato was an employee of Royal Star Marketing. This petition before the Court assails same arguments raised by Lazaro in SSC. She raised that Laudato was not an employee of Royal Star Marketing since Royal Star had no control over the activities of Laudato. For the purpose of determining whether the respondent is entitled to social security contributions, it

Page 4

LABOR RELATIONS
Atty. Jefferson M. Marquez
must be shown that Laudato was a regular employee of Royal Star Marketing. Ruling: It is an accepted doctrine that for the purposes of coverage under the Social Security Act, the determination of employer-employee relationship warrants the application of the control test, that is, whether the employer controls or has reserved the right to control the employee, not only as to the result of the work done, but also as to the means and methods by which the same is accomplished. The SSC, applying the control test found that Laudato was an employee of Royal Star. The Court agrees with the findings of the SSC and the CA. The fact that Laudato was paid by way of commission does not preclude the establishment of an employer-employee relationship. In the case of Grepalife v. Judico, the Court upheld the existence of an employer-employee relationship between the insurance company and its agents, despite the fact that the compensation that the agents on commission received was not paid by the company but by the investor or the person insured. The relevant factor remains, as stated earlier, whether the "employer" controls or has reserved the right to control the "employee" not only as to the result of the work to be done but also as to the means and methods by which the same is to be accomplished. Neither does it follow that a person who does not observe normal hours of work cannot be deemed an employee. In the case of Cosmopolitan Funeral Homes, Inc. v. Maalat, the employer similarly denied the existence of an employer-employee relationship, as the claimant according to it, was a supervisor on commission basis who did not observe normal hours of work. This Court declared that there was an employer-employee relationship, noting that [the] supervisor, although compensated on commission basis, [is] exempt from the observance of normal hours of work for his compensation is measured by the number of sales he makes.

8. Phil. Global Communications v. De Vera, 459 SCRA 260 [2005] Facts: Philippine Global Communications inc. is a corporation engaged in the business of communication services and allied activities while Ricardo de Vera is a physician by profession whom petitioner enlisted to attend to the medical needs of its employees. The controversy rose when petitioner terminated his engagement. In 1981, Dr. de Vera offered his services to petitioner. The parties agreed and formalized the respondents proposal in a document denominated as retainership contract which will be for a period of one year, subject to renewal and clearly stated that respondent will cover the retainership the company previously with Dr. Eulau. The agreement went until 1994, in the years 1995-1996, it was renewed verbally. The turning point of the parties relationship was when petitioner, thru a letter bearing the subject TERMINATION RETAINERSHIP CONTRACT, informed Dr. de Vera of its decision to discontinue the latters retainer contract because the management has decided that it would be more practical to provide medical services to its employees through accredited hospitals near the company premises. On January 1997, de Vera fileda complaint for illegal dismissal before the NLRC, alleging that he had been actually employed by the company as its company physician since 1991. The commission rendered decision in favor of Philcom and dismissed the complaint saying that de Vera was an independent contractor. On appeal to NLRC, it reversed the decision of the Labor Arbiter stating that de Vera is a regular employee and directed the company to reinstate him. Philcom appealed to the CA where it rendered decision deleting the award but reinstating de Vera. Philcom filed this petition involving the difference of a job contracting agreements from employee-employer relationship. Issue: Whether or not there is an employer-employee relationship between the parties. SC Ruling: The elements of an employer-employee relationship is wanting in this case. The record are replete with evidence showing that respondent had to bill petitioner for his monthly professional fees. It simply runs against the grain of common experience to imagine that an ordinary employee has yet to bill his employer to receive his salary. The power to terminate the parties relationship was mutually vested on both. Either may terminate the arrangement at will, with or without cause. Remarkably absent is the element of control whereby the employer has reserved the right to control the employee not only as to the result of the work done but also as to the means and methods by which the same is to be accomplished.

Page 5

LABOR RELATIONS
Atty. Jefferson M. Marquez
Petitioner had no control over the means and methods by which respondent went about performing his work at the company premises. In fine, the parties themselves practically agreed on every terms and conditions of the engagement, which thereby negates the element of control in their relationship.

9. ABS-CBN vs. Nazareno, G.R. No. 164156, Sept. 26, 2006 Facts: Petitioner ABS-CBN Broadcasting Corporation (ABS-CBN) is engaged in the broadcasting business and owns a network of television and radio stations, whose operations revolve around the broadcast, transmission, and relay of telecommunication signals. It sells and deals in or otherwise utilizes the airtime it generates from its radio and television operations. It has a franchise as a broadcasting company, and was likewise issued a license and authority to operate by the National Telecommunications Commission. Petitioner employed respondents Nazareno, Gerzon, Deiparine, and Lerasan as production assistants (PAs) on different dates. They were assigned at the news and public affairs, for various radio programs in the Cebu Broadcasting Station. On December 19, 1996, petitioner and the ABS-CBN Rank-and-File Employees executed a Collective Bargaining Agreement (CBA) to be effective during the period from December 11, 1996 to December 11, 1999. However, since petitioner refused to recognize PAs as part of the bargaining unit, respondents were not included to the CBA. On October 12, 2000, respondents filed a Complaint for Recognition of Regular Employment Status, Underpayment of Overtime Pay, Holiday Pay, Premium Pay, Service Incentive Pay, Sick Leave Pay, and 13th Month Pay with Damages against the petitioner before the NLRC. The Labor Arbiter rendered judgment in favor of the respondents, and declared that they were regular employees of petitioner as such, they were awarded monetary benefits. NLRC affirmed the decision of the Labor Arbiter. Petitioner filed a motion for reconsideration but CA dismissed it. The issue involved is whether the respondents were considered regular employees of ABS-CBN. Ruling: The respondents are regular employees of ABS-CBN. It was held that where a person has rendered at least one year of service, regardless of the nature of the activity performed, or where the work is continuous or intermittent, the employment is considered regular as long as the activity exists, the reason being that a customary appointment is not indispensable before one may be formally declared as having attained regular status. In Universal Robina Corporation v. Catapang, the Court states that the primary standard, therefore, of determining regular employment is the reasonable connection between the particular activity performed by the employee in relation to the usual trade or business of the employer. The test is whether the former is usually necessary or desirable in the usual business or trade of the employer. The connection can be determined by considering the nature of work performed and its relation to the scheme of the particular business or trade in its entirety. Also, if the employee has been performing the job for at least a year, even if the performance is not continuous and merely intermittent, the law deems repeated and continuing need for its performance as sufficient evidence of the necessity if not indispensability of that activity to the business. Hence, the employment is considered regular, but only with respect to such activity and while such activity exists. Additionally, respondents cannot be considered as project or program employees because no evidence was presented to show that the duration and scope of the project were determined or specified at the time of their engagement. In the case at bar, however, the employer-employee relationship between petitioner and respondents has been proven. In the selection and engagement of respondents, no peculiar or unique skill, talent or celebrity status was required from them because they were merely hired through petitioners personnel department just like any ordinary employee. Respondents did not have the power to bargain for huge talent fees, a circumstance negating independent contractual relationship. Respondents are highly dependent on the petitioner for continued work. The degree of control and supervision exercised by petitioner over respondents through its supervisors negates the allegation that respondents are independent contractors. The presumption is that when the work done is an integral part of the regular business of the employer and when the worker, relative to the employer, does not furnish an independent business or professional service, such work is a regular employment of such employee and not an independent contractor. As regular employees, respondents are entitled to the benefits granted to all other regular employees of petitioner under the CBA . Besides, only talent-artists were excluded from the CBA and not production assistants who are regular employees of the respondents. Moreover, under Article 1702 of the New Civil Code: In case of doubt, all labor legislation and all labor contracts shall be construed in favor of the safety and decent living of the laborer.

10. Francisco vs. NLRC, 500 SCRA 690 [2006] Facts:

Page 6

LABOR RELATIONS
Atty. Jefferson M. Marquez
Petitoner was hired by Kasei Corporation during the incorporation stage. She was designated as accountant and corporate secretary and was assigned to handle all the accounting needs of the company. She was also designated as Liason Officer to the City of Manila to secure permits for the operation of the company.In 1996, Petitioner was designated as Acting Manager. She was assigned to handle recruitment of all employees and perform management administration functions. In 2001, she was replaced by Liza Fuentes as Manager. Kasei Corporation reduced her salary to P2,500 per month which was until September. She asked for her salary but was informed that she was no longer connected to the company. She did not anymore report to work since she was not paid for her salary. She filed an action for constructive dismissal with the Labor Arbiter. The Labor Arbiter found that the petitioner was illegally dismissed. NLRC affirmed the decision while CA reversed it. The following issue is to be discussed, whether there was an employer-employee relationship. Ruling: The court held that in this jurisdiction, there has been no uniform test to determine the existence of an employer-employee relation. Generally, courts have relied on the so-called right of control test where the person for whom the services are performed reserves a right to control not only the end to be achieved but also the means to be used in reaching such end. In addition to the standard of right-of-control, the existing economic conditions prevailing between the parties, like the inclusion of the employee in the payrolls, can help in determining the existence of an employer-employee relationship. The better approach would therefore be to adopt a two-tiered test involving: (1) the putative employers power to control the employee with respect to the means and methods by which the work is to be accomplished; and (2) the underlying economic realities of the activity or relationship. In Sevilla v. Court of Appeals, the court observed the need to consider the existing economic conditions prevailing between the parties, in addition to the standard of right-of-control like the inclusion of the employee in the payrolls, to give a clearer picture in determining the existence of an employer-employee relationship based on an analysis of the totality of economic circumstances of the worker. Thus, the determination of the relationship between employer and employee depends upon the circumstances of the whole economic activity, such as: (1) the extent to which the services performed are an integral part of the employers business; (2) the extent of the workers investment in equipment and facilities; (3) the nature and degree of control exercised by the employer; (4) the workers opportunity for profit and loss; (5) the amount of initiative, skill, judgment or foresight required for the success of the claimed independent enterprise; (6) the permanency and duration of the relationship between the worker and the employer; and (7) the degree of dependency of the worker upon the employer for his continued employment in that line of business. The proper standard of economic dependence is whether the worker is dependent on the alleged employer for his continued employment in that line of business. By applying the control test, there is no doubt that petitioner is an employee of Kasei Corporation because she was under the direct control and supervision of Seiji Kamura, the corporations Technical Consultant. It is therefore apparent that petitioner is economically dependent on Respondent Corporation for her continued employment in the latters line of business. There can be no other conclusion that petitioner is an employee of respondent Kasei Corporation. She was selected and engaged by the company for compensation, and is economically dependent upon respondent for her continued employment in that line of business. Her main job function involved accounting and tax services rendered to Respondent Corporation on a regular basis over an indefinite period of engagement. Respondent Corporation hired and engaged petitioner for compensation, with the power to dismiss her for cause. More importantly, Respondent Corporation had the power to control petitioner with the means and methods by which the work is to be accomplished.

11. Nogales et. al., vs. Capitol Medical Center et al., G.R. No. 142625, December 19, 2006 Facts: Pregnant with her fourth child, Corazon Nogales, who was then 37 years old, was under the exclusive prenatal care of Dr. Oscar Estrada. While Corazon was on her last trimester of pregnancy, Dr. Estrada noted an increase in her blood pressure and development of leg edema indicating preeclampsia, which is a dangerous complication of pregnancy. Around midnight of 25 May 1976, Corazon started to experience mild labor pains prompting Corazon and Rogelio Nogales ("Spouses Nogales") to see Dr. Estrada at his home. After examining Corazon, Dr. Estrada advised her immediate admission to the Capitol Medical Center ("CMC"). Due to the inclement weather then, Dr. Espinola, who was fetched from his residence by an ambulance, arrived at the CMC about an hour later or at 9:00 a.m. He examined the patient and ordered some resuscitative measures to be administered. Despite Dr. Espinola's efforts, Corazon died at 9:15 a.m. The cause of death was "hemorrhage, post partum." Issue in this case is whether CMC is vicariously liable for the negligence of Dr. Estrada. Ruling:

Page 7

LABOR RELATIONS
Atty. Jefferson M. Marquez
The resolution of this issue rests, on the other hand, on the ascertainment of the relationship between Dr. Estrada and CMC. The Court also believes that a determination of the extent of liability of the other respondents is inevitable to finally and completely dispose of the present controversy. After a thorough examination of the voluminous records of this case, the Court finds no single evidence pointing to CMC's exercise of control over Dr. Estrada's treatment and management of Corazon's condition. It is undisputed that throughout Corazon's pregnancy, she was under the exclusive prenatal care of Dr. Estrada. At the time of Corazon's admission at CMC and during her delivery, it was Dr. Estrada, assisted by Dr. Villaflor, who attended to Corazon. There was no showing that CMC had a part in diagnosing Corazon's condition. While Dr. Estrada enjoyed staff privileges at CMC, such fact alone did not make him an employee of CMC. CMC merely allowed Dr. Estrada to use its facilities when Corazon was about to give birth, which CMC considered an emergency. Considering these circumstances, Dr. Estrada is not an employee of CMC, but an independent contractor. The question now is whether CMC is automatically exempt from liability considering that Dr. Estrada is an independent contractor-physician. In general, a hospital is not liable for the negligence of an independent contractor-physician. There is, however, an exception to this principle. The hospital may be liable if the physician is the "ostensible" agent of the hospital. This exception is also known as the "doctrine of apparent authority."In the instant case, CMC impliedly held out Dr. Estrada as a member of its medical staff. Through CMC's acts, CMC clothed Dr. Estrada with apparent authority thereby leading the Spouses Nogales to believe that Dr. Estrada was an employee or agent of CMC. CMC cannot now repudiate such authority. In the present case, there is no evidence of Nurse Dumlao's alleged failure to follow Dr. Estrada's specific instructions. Even assuming Nurse Dumlao defied Dr. Estrada's order, there is no showing that side-drip administration of hemacel proximately caused Corazon's death. No evidence linking Corazon's death and the alleged wrongful hemacel administration was introduced. Therefore, there is no basis to hold Nurse Dumlao liable for negligence. The Court finds respondent CMC vicariously liable for the negligence of Dr. Oscar Estrada.

12. Coca-Cola Bottlers Phils., vs. Dr. Climaco, G.R. No. 146881, February 15, 2007 Facts: Dr. Dean N. Climaco is a medical doctor who was hired by Coca-Cola Bottlers Phils., Inc. by virtue of a Retainer Agreement. The Retainer Agreement, which began on January 1, 1988, was renewed annually. The last one expired on December 31, 1993. Despite the non-renewal of the Retainer Agreement, respondent continued to perform his functions as company doctor to Coca-Cola until he received a letter dated March 9, 1995 from the company concluding their retainership agreement effective 30 days from receipt thereof. Dr. Climaco inquired from the management of the company whether it was agreeable to recognizing him as a regular employee. The management refused to do so. On February 24, 1994, respondent filed a Complaint before the NLRC, Bacolod City, seeking recognition as a regular employee of the company and prayed for the payment of all benefits of a regular employee. While the complaint was pending before the Labor Arbiter, respondent received a letter dated March 9, 1995 from Petitioner Company concluding their retainership agreement effective thirty (30) days from receipt thereof. Issue: Whether or not there exists an employer-employee relationship. Ruling: The Court, in determining the existence of an employer-employee relationship, has invariably adhered to the four-fold test: (1) the selection and engagement of the employee; (2) the payment of wages; (3) the power of dismissal; and (4) the power to control the employees conduct, or the so-called "control test," considered to be the most important element. No employer-employee relationship exists between the parties. Thecompany lacked the power of control over the performance by respondent of his duties. TheComprehensive Medical Plan, which contains the respondents objectives, duties and obligations, does not tell respondent "how to conduct his physical examination, how to immunize, or how to diagnose and treat his patients, employees of [petitioner] company, in each case." Neri v. National Labor Relations Commission It is admitted that FEBTC issued a job description which detailed her functions as a radio/telex operator. However, a cursory reading of the job description shows that what was sought to be controlled by FEBTC was actually the end result of the task, e.g., that the daily incoming and outgoing telegraphic transfer of funds received and relayed by her, respectively, tallies with that of the register. The guidelines were laid down merely to ensure that the desired end result was achieved. It did not, however, tell Neri how the radio/telex machine should be operated. Through the Comprehensive Medical Plan, provided guidelines merely to ensure that the end result was achieved, but did not control the means and methods by which respondent performed his assigned tasks. Likewise, the allegation of complainant that since he is on call at anytime of the day and night makes him a regular employee is off-tangent. Complainant does not dispute the fact that outside of the two (2) hours that he is required to be at respondent companys premises, he is not at all further required to just sit around in the premises and wait for an emergency to occur so as to enable him from using such hours for his own benefit and advantage. In fact, complainant maintains his own private clinic

Page 8

LABOR RELATIONS
Atty. Jefferson M. Marquez
attending to his private practice in the city, where he services his patients, bills them accordingly -- and if it is an employee of respondent company who is attended to by him for special treatment that needs hospitalization or operation, this is subject to a special billing. More often than not, an employee is required to stay in the employers workplace or proximately close thereto that he cannot utilize his time effectively and gainfully for his own purpose. Such is not the prevailing situation here. The Retainership Agreement granted to both parties the power to terminate their relationship upon giving a 30-day notice. Hence, Petitioner Company did not wield the sole power of dismissal or termination. Considering that there is no employer-employee relationship between the parties, the termination of the Retainership Agreement, which is in accordance with the provisions of the Agreement, does not constitute illegal dismissal of respondent. Consequently, there is no basis for the moral and exemplary damages granted by the Court of Appeals to respondent due to his alleged illegal dismissal.

13. Consolidated Broadcasting System vs. Oberio, G.R. No. 168424, June 8, 2007 Facts: Respondents alleged that they were employed as drama talents by DYWB-Bombo Radyo, a radio station owned and operated by petitioner Consolidated Broadcasting System, Inc. They reported for work daily for six days in a week and were required to record their drama production in advance. Some of them were employed by petitioner since 1974, while the latest one was hired in 1997. Their drama programs were aired not only in Bacolod City but also in the sister stations of DYWB in the Visayas and Mindanao areas. Sometime in August 1998, petitioner reduced the number of its drama productions from 14 to 11, but was opposed by respondents. After the negotiations failed, the latter sought the intervention of the Department of Labor and Employment (DOLE), which on November 12, 1998, conducted through its Regional Office, an inspection of DWYB station. The results thereof revealed that petitioner is guilty of violation of labor standard laws. Petitioner contended that respondents are not its employees and refused to submit the payroll and daily time records despite the subpoena duces tecum issued by the DOLE Regional Director. Petitioner further argued that the case should be referred to the NLRC because the Regional Director has no jurisdiction over the determination of the existence of employer-employee relationship which involves evidentiary matters that are not verifiable in the normal course of inspection. Vexed by the respondents' complaint, petitioner allegedly pressured and intimidated respondents. Respondents Oberio and Delta were suspended for minor lapses and the payment of their salaries were purportedly delayed. Eventually, on February 3, 1999, pending the outcome of the inspection case with the Regional Director, respondents were barred by petitioner from reporting for work; thus, the former claimed constructive dismissal. Issues: 1. Whether respondents were employees of petitioner. 2. Whether respondents dismissal was illegal. Ruling: 1. Yes, respondents employment with petitioner passed the "four-fold test" on employer-employee relations, namely: (1) the selection and engagement of the employee, or the power to hire; (2) the payment of wages; (3) the power to dismiss; and (4) the power to control the employee. Petitioner failed to controvert with substantial evidence the allegation of respondents that they were hired by the former on various dates from 1974 to 1997. If petitioner did not hire respondents and if it was the director alone who chose the talents, petitioner could have easily shown, being in possession of the records, a contract to such effect. However, petitioner merely relied on its contention that respondents were piece rate contractors who were paid by results.Note that under Policy Instruction No. 40, petitioner is obliged to execute the necessary contract specifying the nature of the work to be performed, rates of pay, and the programs in which they will work. Moreover, project or contractual employees are required to be apprised of the project they will undertake under a written contract. This was not complied with by the petitioner, justifying the reasonable conclusion that no such contracts exist and that respondents were in fact regular employees. Moreover, the engagement of respondents for a period ranging from 2 to 25 years and the fact that their drama programs were aired not only in Bacolod City but also in the sister stations of DYWB in the Visayas and Mindanao areas, undoubtedly show that their work is necessary and indispensable to the usual business or trade of petitioner. The test to determine whether employment is regular or not is the reasonable connection between the particular activity performed by the employee in relation to the usual business or trade of the employer. 2. Finally, we find that respondents were illegally dismissed. In labor cases, the employer has the burden of proving that the dismissal was for a just cause; failure to show this would necessarily mean that the dismissal was unjustified and, therefore, illegal. In this case, petitioner merely contended that it was respondents who ceased to report to work, and never presented any substantial evidence to support said allegation. Furthermore, if doubts exist between the evidence presented by the employer and the employee, the scales of justice must be tilted in favor of the latter -the employer must affirmatively show rationally adequate evidence that the dismissal was for a justifiable cause. It is a time-honored rule that in controversies between a laborer and his master, doubts reasonably arising from the evidence should be resolved in the former's favor. The policy is to extend the doctrine to a greater number of employees who can avail of the benefits under the law, which is in consonance with the avowed policy of the State to give maximum aid and protection of labor.

Page 9

LABOR RELATIONS
Atty. Jefferson M. Marquez

14. Dumpit-Morillo vs. CA, G.R. No. 164652, June 8, 2007, citing 2004 Sonza Facts: Associated Broadcasting Company (ABC) hired Thelma Dumpit-Murillo under a talent contract as a newscaster and co-anchor for BalitangBalita, an early evening news program. The contract was for a period of three months. After four years of repeated renewals, petitioners talent contract expired. Two weeks after the expiration of the last contract, petitioner sent a letter to Mr. Jose Javier, Vice President for News and Public Affairs of ABC, informing the latter that she was still interested in renewing her contract subject to a salary increase. Thereafter, petitioner stopped reporting for work. She sent a demand letter to ABC, demanding reinstatement, payment of unpaid wages and full backwages, payment of 13th month pay, vacation/sick/service incentive leaves and other monetary benefits due to a regular employee. ABC replied that a check covering petitioners talent fees had been processed and prepared, but that the other claims of petitioner had no basis in fact or in law. The Labor Arbiter dismissed the complaint for illegal constructive dismissal. NLRC reversed. Issue: Whether or not Murillo is an employee of Associated Broadcasting Company. Ruling: Thelma Dumpit-Murillo was a regular employee under contemplation of law. The practice of having fixed-term contracts in the industry does not automatically make all talent contracts valid and compliant with labor law. The assertion that a talent contract exists does not necessarily prevent a regular employment status. Further, the Sonza case is not applicable. In Sonza, the television station did not exercise control over the means and methods of the performance of Sonzas work. In the case at bar, ABC had control over the performance of petitioners work. Noteworthy too, is the comparatively low P28,000 monthly pay of petitioner vis the P300,000 a month salary of Sonza, that all the more bolsters the conclusion that petitioner was not in the same situation as Sonza. The duties of petitioner as enumerated in her employment contract indicate that ABC had control over the work of petitioner. Aside from control, ABC also dictated the work assignments and payment of petitioners wages. ABC also had power to dismiss her. All these being present, clearly, there existed an employment relationship between petitioner and ABC. Concerning regular employment, the requisites for regularity of employment have been met in the instant case. Petitioners work was necessary or desirable in the usual business or trade of the employer which includes, as a pre-condition for its enfranchisement, its participation in the governments news and public information dissemination. In addition, her work was continuous for a period of four years. This repeated engagement under contract of hire is indicative of the necessity and desirability of the petitioners work in private respondent ABCs business. As a regular employee, petitioner is entitled to security of tenure and can be dismissed only for just cause and after due compliance with procedural due process. Since private respondents did not observe due process in constructively dismissing the petitioner, there was an illegal dismissal.

15. Lopez vs. Bodega City, G.R. No. 155731, Sept. 3, 2007, citing 2004 Abante & 2005 Consulta Facts: Petitioner was the "lady keeper" of Bodega City tasked with manning its ladies' comfort room. In a letter signed by Yap dated February 10, 1995, petitioner was made to explain why the concessionaire agreement between her and respondents should not be terminated or suspended in view of an incident that happened on February 3, 1995, wherein petitioner was seen to have acted in a hostile manner against a lady customer of Bodega City who informed the management that she saw petitioner sleeping while on duty. In a subsequent letter dated February 25, 1995, Yap informed petitioner that because of the incident that happened on February 3, 1995, respondents had decided to terminate the concessionaire agreement between them. Issue: Whether or not employer-employee relationship exists Ruling: The Court applies the four-fold test expounded in Abante v. Lamadrid Bearing and Parts Corp.,to wit:

Page 10

LABOR RELATIONS
Atty. Jefferson M. Marquez
To ascertain the existence of an employer-employee relationship, jurisprudence has invariably applied the four-fold test, namely: (1) the manner of selection and engagement; (2) the payment of wages; (3) the presence or absence of the power of dismissal; and (4) the presence or absence of the power of control. Of these four, the last one is the most important. The so-called "control test" is commonly regarded as the most crucial and determinative indicator of the presence or absence of an employer-employee relationship. Under the control test, an employer-employee relationship exists where the person for whom the services are performed reserves the right to control not only the end achieved, but also the manner and means to be used in reaching that end. Petitioner failed to cite a single instance to prove that she was subject to the control of respondents insofar as the manner in which she should perform her job as a "lady keeper" was concerned. It is true that petitioner was required to follow rules and regulations prescribing appropriate conduct while within the premises of Bodega City. However, this was imposed upon petitioner as part of the terms and conditions in the concessionaire agreement. Petitioner is likewise estopped from denying the existence of the subject concessionaire agreement. She should not, after enjoying the benefits of the concessionaire agreement with respondents, be allowed to later disown the same through her allegation that she was an employee of the respondents when the said agreement was terminated by reason of her violation of the terms and conditions thereof. The principle of estoppel in pais applies wherein -- by one's acts, representations or admissions, or silence when one ought to speak out -- intentionally or through culpable negligence, induces another to believe certain facts to exist and to rightfully rely and act on such belief, so as to be prejudiced if the former is permitted to deny the existence of those facts. Petitioner insists that her ID card is sufficient proof of her employment. In Domasig v. National Labor Relations Commission, this Court held that the complainant's ID card and the cash vouchers covering his salaries for the months indicated therein were substantial evidence that he was an employee of respondents, especially in light of the fact that the latter failed to deny said evidence. This is not the situation in the present caseAs to the ID card, it is true that the words "EMPLOYEE'S NAME" appear printed below petitioner's name. However, she failed to dispute respondents' evidence consisting of Habitan's testimony, that he and the other "contractors" of Bodega City such as the singers and band performers, were also issued the same ID cards for the purpose of enabling them to enter the premises of Bodega City. Hence, going back to the element of control, the concessionaire agreement merely stated that petitioner shall maintain the cleanliness of the ladies' comfort room and observe courtesy guidelines that would help her obtain the results they wanted to achieve. There is nothing in the agreement which specifies the methods by which petitioner should achieve these results. Respondents did not indicate the manner in which she should go about in maintaining the cleanliness of the ladies' comfort room. Neither did respondents determine the means and methods by which petitioner could ensure the satisfaction of respondent company's customers. In fact, the last paragraph of the concessionaire agreement even allowed petitioner to engage persons to work with or assist her in the discharge of her functions. Moreover, petitioner was not subjected to definite hours or conditions of work. The fact that she was expected to maintain the cleanliness of respondent company's ladies' comfort room during Bodega City's operating hours does not indicate that her performance of her job was subject to the control of respondents as to make her an employee of the latter. Instead, the requirement that she had to render her services while Bodega City was open for business was dictated simply by the very nature of her undertaking, which was to give assistance to the users of the ladies' comfort room. Lastly, the Court finds that the elements of selection and engagement as well as the power of dismissal are not present in the instant case. It has been established that there has been no employer-employee relationship between respondents and petitioner. Their contractual relationship was governed by the concessionaire agreement embodied in the 1992 letter. Thus, petitioner was not dismissed by respondents. Instead, as shown by the letter of Yap to her dated February 15, 1995, their contractual relationship was terminated by reason of respondents' termination of the subject concessionaire agreement, which was in accordance with the provisions of the agreement in case of violation of its terms and conditions.

16. Calamba Medical Center vs. NLRC et al., G.R. No. 176484, Nov. 25, 2008

Facts: The Calamba Medical Center (petitioner), a privately-owned hospital, engaged the services of medical doctors-spouses Ronaldo Lanzanas (Dr. Lanzanas) and Merceditha Lanzanas (Dr. Merceditha) in March 1992 and August 1995, respectively, as part of its team of resident physicians. On March 7, 1998, Dr. Meluz Trinidad (Dr. Trinidad), also a resident physician at the hospital, inadvertently overheard a telephone conversation of respondent Dr. Lanzanas with a fellow employee, Diosdado Miscala, through an extension telephone line. Apparently, Dr. Lanzanas and Miscala were discussing the low "census" or admission of patients to the hospital. Dr. Desipeda whose attention was called to the above-said telephone conversation issued to Dr. Lanzanas a Memorandum of March 7, 1998. In the meantime, then Sec. Cresenciano Trajano of the Department of Labor and Employment (DOLE) certified the labor dispute to the NLRC for compulsory arbitration and issued on April 21, 1998 return-to-work Order to the striking union officers and employees of petitioner pending resolution of the labor dispute. Petitioner later sent Dr. Lanzanas a notice of termination which he received on April 25, 1998, indicating as grounds therefor his failure to report back to work despite the DOLE order and his supposed role in the striking union, thus: On April 23, 1998, you still did not report for work despite memorandum issued by

Page 11

LABOR RELATIONS
Atty. Jefferson M. Marquez
the CMC Medical Director implementing the Labor Secretary's ORDERYou are likewise aware that you were observed (re: signatories [sic] to the Saligang Batas of BMCMC-UWP) to be unlawfully participating as member in the rank-and-file union's concerted activities despite knowledge that your position in the hospital is managerial in nature (Nurses, Orderlies, and staff of the Emergency Room carry out your orders using your independent judgment) which participation is expressly prohibited by the New Labor Code For these reasons as grounds for termination, you are hereby terminated for cause from employment effective today, April 25, 1998, without prejudice to further action for revocation of your license before the Philippine [sic] Regulations [sic] Commission. Dr. Lanzanas thus amended his original complaint to include illegal dismissal. Issue: Whether or not employer-employee relationship exists Ruling: YES. Under the "control test," an employment relationship exists between a physician and a hospital if the hospital controls both the means and the details of the process by which the physician is to accomplish his task. As priorly stated, private respondents maintained specific workschedules, as determined by petitioner through its medical director, which consisted of 24-hour shifts totaling forty-eight hours each week and which were strictly to be observed under pain of administrative sanctions.That petitioner exercised control over respondents gains light from the undisputed fact that in the emergency room, the operating room, or any department or ward for that matter, respondents' work is monitored through its nursing supervisors, charge nurses and orderlies. Without the approval or consent of petitioner or its medical director, no operations can be undertaken in those areas. For control test to apply, it is not essential for the employer to actually supervise the performance of duties of the employee, it being enough that it has the right to wield the power. With respect to respondents' sharing in some hospital fees, this scheme does not sever the employment tie between them and petitioner as this merely mirrors additional form or another form of compensation or incentive similar to what commission-based employees receive as contemplated in Article 97 (f) of the Labor Code, thus: whether fixed or ascertained on a time, task, piece, or commission basis, or other method of calculating the same Respondents were in fact made subject to petitioner-hospital's Code of Ethics, the provisions of which cover administrative and disciplinary measures on negligence of duties, personnel conduct and behavior, and offenses against persons, property and the hospital's interest. More importantly, petitioner itself provided incontrovertible proof of the employment status of respondents, namely, the identification cards it issued them, the payslips and BIR W-2 (now 2316) Forms which reflect their status as employees, and the classification as "salary" of their remuneration. Moreover, it enrolled respondents in the SSS and Medicare (Philhealth) program. It bears noting at this juncture that mandatory coverage under the SSS Law is premised on the existence of an employer-employee relationship, except in cases of compulsory coverage of the self-employed. Finally, under Section 15, Rule X of Book III of the Implementing Rules of the Labor Code, an employer-employee relationship exists between the resident physicians and the training hospitals, unless there is a training agreement between them, and the training program is duly accredited or approved by the appropriate government agency. In respondents' case, they were not undergoing any specialization training. They were considered non-training general practitioners, assigned at the emergency rooms and ward sections. Turning now to the issue of dismissal, the Court upholds the appellate court's conclusion that private respondents were illegally dismissed. Dr. Lanzanas was neither a managerial nor supervisory employee but part of the rank-and-file. This is the import of the Secretary of Labor's Resolution of May 22, 1998 in OS A-05-15-98 which reads: In the motion to dismiss it filed before the Med-Arbiter, the employer (CMC) alleged that 24 members of petitioner are supervisors, namely Rolando Lanzonas. A close scrutiny of the job descriptions of the alleged supervisors narrated by the employer only proves that except for the contention that these employees allegedly supervise, they do not however recommend any managerial action. At most, their job is merely routinary in nature and consequently, they cannot be considered supervisory employees. They are not therefore barred from membership in the union of rank[-]and[-]file, which the petitioner [the union] is seeking to represent in the instant case. Participation in a strike and intransigence to a return-to-work order must, however, be duly proved in order to justify immediate dismissal in a "national interest" case. Mere membership in a labor union does not ipso facto mean participation in a strike. As for the case of Dr. Merceditha, her dismissal was worse, it having been effected without any just or authorized cause and without observance of due process. In fact, petitioner never proferred any valid cause for her dismissal except its view that "her marriage to [Dr. Lanzanas] has given rise to the presumption that her sympath[y] [is] with her husband; [and that when [Dr. Lanzanas] declared that he was going to boycott the scheduling of their workload by the medical doctor, he was presumed to be speaking for himself [and] for his wife Merceditha."

17. Escasinas et al., vs. Shangri-las Mactan Island Resort et al., G.R. No. 178827, March 4, 2009 Facts: Registered nurses Jeromie D. Escasinas and Evan Rigor Singco (petitioners) were engaged in 1999 and 1996, respectively, by Dr. Jessica Joyce R. Pepito (respondent doctor) to work in her clinic at respondent Shangri-las Mactan Island Resort (Shangri-la) in Cebu of which she was

Page 12

LABOR RELATIONS
Atty. Jefferson M. Marquez
a retained physician. In late 2002, petitioners filed with the National Labor Relations Commission (NLRC) Regional Arbitration Branch No. VII (NLRC-RAB No. VII) a complaint for regularization, underpayment of wages, non-payment of holiday pay, night shift differential and 13th month pay differential against respondents, claiming that they are regular employees of Shangri-la. Shangri-la claimed, however, that petitioners were not its employees but of respondent doctor whom it retained via Memorandum of Agreement (MOA) pursuant to Article 157 of the Labor Code, as amended. Respondent doctor for her part claimed that petitioners were already working for the previous retained physicians of Shangri-la before she was retained by Shangri-la; and that she maintained petitioners services upon their request. Issue: Whether or not employer-employee relationship exists Ruling: The resolution of the case hinges, in the main, on the correct interpretation of Art. 157 vis a vis Art. 280 and the provisions on permissible job contracting of the Labor Code, as amended. The Court holds that, contrary to petitioners postulation, Art. 157 does not require the engagement of full-time nurses as regular employees of a company employing not less than 50 workers. Thus, the Article provides: ART. 157. Emergency medical and dental services. It shall be the duty of every employer to furnish his employees in any locality with free medical and dental attendance and facilities consisting of: (b) The services of a full-time registered nurse, a part-time physician and dentist, and an emergency clinic, when the number of employees exceeds two hundred (200) but not more than three hundred (300); and Under the foregoing provision, Shangri-la, which employs more than 200 workers, is mandated to "furnish" its employees with the services of a full-time registered nurse, a part-time physician and dentist, and an emergency clinic which means that it should provide or make available such medical and allied services to its employees, not necessarily to hire or employ a service provider. As held in Philippine Global Communications vs. De Vera: x x x while it is true that the provision requires employers to engage the services of medical practitioners in certain establishments depending on the number of their employees, nothing is there in the law which says that medical practitioners so engaged be actually hired as employees, adding that the law, as written, only requires the employer "to retain", not employ, a part-time physician who needed to stay in the premises of the non-hazardous workplace for two (2) hours. The term "full-time" in Art. 157 cannot be construed as referring to the type of employment of the person engaged to provide the services, for Article 157 must not be read alongside Art. 280 in order to vest employer-employee relationship on the employer and the person so engaged. So De Vera teaches: x x x For, we take it that any agreement may provide that one party shall render services for and in behalf of another, no matter how necessary for the latters business, even without being hired as an employee. This set-up is precisely true in the case of an independent contractorship as well as in an agency agreement. Indeed, Article 280 of the Labor Code, quoted by the appellate court, is not the yardstick for determining the existence of an employment relationship. As it is, the provision merely distinguishes between two (2) kinds of employees, i.e., regular and casual. x x x The phrase "services of a full-time registered nurse" should thus be taken to refer to the kind of services that the nurse will render in the companys premises and to its employees, not the manner of his engagement. As to whether respondent doctor can be considered a legitimate independent contractor, the pertinent sections of DOLE Department Order No. 10, series of 1997, illuminate: Sec. 8. Job contracting. There is job contracting permissible under the Code if the following conditions are met: (1) The contractor carries on an independent business and undertakes the contract work on his own account under his own responsibility according to his own manner and method, free from the control and direction of his employer or principal in all matters connected with the performance of the work except as to the results thereof; and (2) The contractor has substantial capital or investment in the form of tools, equipment, machineries, work premises, and other materials which are necessary in the conduct of his business. Sec. 9. Labor-only contracting. (a) Any person who undertakes to supply workers to an employer shall be deemed to be engaged in labor-only contracting where such person: (1) Does not have substantial capital or investment in the form of tools, equipment, machineries, work premises and other materials; and (2) The workers recruited and placed by such persons are performing activities which are directly related to the principal business or operations of the employer in which workers are habitually employed. (b) Labor-only contracting as defined herein is hereby prohibited and the person acting as contractor shall be considered merely as an agent or intermediary of the employer who shall be responsible to the workers in the same manner and extent as if the latter were directly employed by him The existence of an independent and permissible contractor relationship is generally established by considering the following determinants: whether the contractor is carrying on an independent business; the nature and extent of the work; the skill required; the term and duration of the relationship; the right to assign the performance of a specified piece of work; the control and supervision of the work to another; the employer's

Page 13

LABOR RELATIONS
Atty. Jefferson M. Marquez
power with respect to the hiring, firing and payment of the contractor's workers; the control of the premises; the duty to supply the premises, tools, appliances, materials and labor; and the mode, manner and terms of payment. On the other hand, existence of an employer- employee relationship is established by the presence of the following determinants: (1) the selection and engagement of the workers; (2) power of dismissal; (3) the payment of wages by whatever means; and (4) the power to control the worker's conduct, with the latter assuming primacy in the overall consideration. Against the above-listed determinants, the Court holds that respondent doctor is a legitimate independent contractor. That Shangri-la provides the clinic premises and medical supplies for use of its employees and guests do not necessarily prove that respondent doctor lacks substantial capital and investment. Besides, the maintenance of a clinic and provision of medical services to its employees is required under Art. 157, which are not directly related to Shangri-las principal business operation of hotels and restaurants. As to payment of wages, respondent doctor is the one who underwrites the following: salaries, SSS contributions and other benefits of the staff; group life, group personal accident insurance and life/death insurance for the staff with minimum benefit payable at 12 times the employees last drawn salary, as well as value added taxes and withholding taxes, sourced from her P60,000.00 monthly retainer fee and 70% share of the service charges from Shangri-las guests who avail of the clinic services. It is unlikely that respondent doctor would report petitioners as workers, pay their SSS premium as well as their wages if they were not indeed her employees. With respect to the supervision and control of the nurses and clinic staff, it is not disputed that a document, "Clinic Policies and Employee Manual" claimed to have been prepared by respondent doctor exists, to which petitioners gave their conformity and in which they acknowledged their co-terminus employment status. It is thus presumed that said document, and not the employee manual being followed by Shangri-las regular workers, governs how they perform their respective tasks and responsibilities. Contrary to petitioners contention, the various office directives issued by Shangri-las officers do not imply that it is Shangri-las management and not respondent doctor who exercises control over them or that Shangri-la has control over how the doctor and the nurses perform their work. The letter addressed to respondent doctor dated February 7, 2003 from a certain Tata L. Reyes giving instructions regarding the replenishment of emergency kits is, at most, administrative in nature, related as it is to safety matters; while the letter dated May 17, 2004 from Shangri-las Assistant Financial Controller, Lotlot Dagat, forbidding the clinic from receiving cash payments from the resorts guests is a matter of financial policy in order to ensure proper sharing of the proceeds, considering that Shangri-la and respondent doctor share in the guests payments for medical services rendered. In fine, as Shangri-la does not control how the work should be performed by petitioners, it is not petitioners employer.

18. Tongko v. Manufacturer Life Insurance Co. (MANULIFE) Inc., et al., G.R. No 167622, January 25, 2011 Facts: Tongko was, initially an insurance agent of Manulife who was promoted to the role of a manager. The contractual relationship between Tongko and Manulife had two basic phases. The initial phase began on July 1, 1977under a Career Agents Agreement which regarded him as an independent contractor, not an employee. As an agent, his tasks were to canvass for applications for insurance products and collect money due to the Company. The second phase started in 1983 when Tongko was named Unit Manager. In 1990, he became a Branch Manager. In 1996, Tongko became a Regional Sales Manager, where he earned commissions, persistency income and management overrides. Since the beginning, Tongko consistently declared himself self-employed in his income tax returns. However, in 2001, Manulife instituted manpower development programs which directed the managers to increase the number of agents to at least 1,000 strong for a start. It was found that Tongkos region was the lowest performer in terms of recruiting in 2000. Subsequently, Tongko received another letter, dated December 18, 2001, terminating his services. Tongko then filed an illegal dismissal complaint with the NLRC Arbitration Branch. He alleged the existence of an employment relationship. In support of this he asserted that as Unit Manager, he was paid an annual over-rider, a travel and entertainment allowance in addition to his overriding commissions. He was tasked with numerous administrative functions and supervisory authority over Manulifes employees. He was required to follow at least three codes of conduct. On the other hand, Manulife contended that what existed between them was a mere agency relationship. Decisions of the Judicial Tribunals LA: No employer-employee relationship existed between the parties. NLRC: It found the existence of an employer-employee relationship. There was illegal dismissal. CA: It reverted to the labor arbiters decision that no employer-employee relationship existed between them. SC: In reversing the CA ruling, it declared that an employment relationship existed between them. First, there exists the possibility of an insurance agent becoming an employee of an insurance company if evidence shows that the company promulgated rules or regulations that effectively controlled or restricted an insurance agents choice of methods or the methods themselves in selling insurance. Second, Manulife had the power of control over Tongko, sufficient to characterize him as an employee, as shown by the fact that he complied with 3 different codes of conduct and that he performed administrative duties. Also, Tongko was tasked to recruit some agents in addition to his other administrative functions.

Page 14

LABOR RELATIONS
Atty. Jefferson M. Marquez
Hence, a Motion for Reconsideration was filed by Manulife and was granted by the SC. Issue: Whether or not there exists an employer-employee relationship. SC Ruling: Rules regarding the desired results (e.g., the required volume to continue to qualify as a company agent & legal/ ethical rules to be followed) are built-in elements of control specific to an insurance agency and should not and cannot be read as elements of control that attend an employment relationship governed by the Labor Code. Based on decided cases, a determination of the presence of the Labor Code element of control was made on the basis of the stipulations of the subsequent contracts. In this case, while Tongko was later on designated unit manager in 1983, Branch Manager in 1990, and Regional Sales Manager in 1996, no formal contract regarding these undertakings appears in the records of the case. Any such contract or agreement, had there been any, could have at the very least provided the bases for properly ascertaining the juridical relationship established between the parties. For this reason, we can take judicial notice that as a matter of Insurance Code-based business practice, an agency relationship prevails in the insurance industry for the purpose of selling insurance. Significantly, evidence shows that Tongkos role as an insurance agent never changed during his relationship with Manulife. Tongko essentially remained an agent, but moved up in this role through Manulifes recognition that he could use other agents approved by Manulife but operating under his guidance. For want of a better term, Tongko perhaps could be labeled as a "lead agent" who guided under his wing other Manulife agents. Evidence indicates that Tongko consistently clung to the view that he was an independent agent since he invariably declared himself a business or self-employed person in his income tax returns. The concept of estoppel a legal and equitable concept necessarily must come into play. Tongkos previous admissions in several years of tax returns as an independent agent, as against his belated claim that he was all along an employee, are too diametrically opposed to be simply dismissed or ignored. There was, indeed, lack of evidence on record showing that Manulife ever exercised means-and-manner control, even to a limited extent, over Tongko during his ascent in Manulifes sales ladder. The reality is, prior to the directives sent by De Dios, Manulife had practically left Tongko alone not only in doing the business of selling insurance, but also in guiding the agents under his wing. In addition, the mere presentation of codes or of rules and regulations is not per se indicative of labor law control. The codes of conduct do not intrude into the insurance agents means and manner of conducting their sales and only control them as to the desired results. Guidelines indicative of labor law "control," based on the case of Insular Life, should not merely relate to the mutually desirable result intended by the contractual relationship; they must have the nature of dictating the means or methods to be employed in attaining the result, or of fixing the methodology and of binding or restricting the party hired to the use of these means. Hence, the failure of Tongko to comply with the guidelines & directives of Manulife is recruiting more agents, as a ground for termination of Tongkos agency, is a matter that the labor tribunals cannot rule upon in the absence of an employer-employee relationship. Jurisdiction over the matter belongs to the courts applying the laws of insurance, agency and contracts. SC: Tongko is just an AGENT. In effect, the SC is telling us that, first, there must be an evidence of a contract that shows that the relationship has been converted from contract of agency to that of employment, which is absent in the case at bar. Secondly, adherence to a code of conduct is not, per se, indicative of control when it merely controls the desired results and not the means and the manner by which agents are to conduct their sales. The directive of De Dios to Tongko (in increasing the number of agents) was merely suggestive. Hence, not indicative of control

19. Caong, Jr. v. Begualos, G.R. No. 179428, January 26, 2011 Facts: Petitioners Primo E. Caong, Jr. (Caong), Alexander J. Tresquio (Tresquio), and Loriano D. Daluyon (Daluyon) were employed by respondent Avelino Regualos under a boundary agreement, as drivers of his jeepneys. In November 2001, they filed separate

Page 15

LABOR RELATIONS
Atty. Jefferson M. Marquez
complaintshttp://sc.judiciary.gov.ph/jurisprudence/2011/january2011/179428.htm - _ftn2 for illegal dismissal against respondent who barred them from driving the vehicles due to deficiencies in their boundary payments. Issue: Whether or not the policy of suspending drivers pending payment of arrears in their boundary obligations is reasonable. Ruling: It is already settled that the relationship between jeepney owners/operators and jeepney drivers under the boundary system is that of employeremployee and not of lessor-lessee. The fact that the drivers do not receive fixed wages but only get the amount in excess of the so-called boundary that they pay to the owner/operator is not sufficient to negate the relationship between them as employer and employee. Petitioners suspension cannot be categorized as dismissal, considering that there was no intent on the part of respondent to sever the employer-employee relationship between him and petitioners. In fact, it was made clear that petitioners could put an end to the suspension if they only pay their recent arrears. As it was, the suspension dragged on for years because of petitioners stubborn refusal to pay. It would have been different if petitioners complied with the condition and respondent still refused to readmit them to work. Then there would have been a clear act of dismissal. But such was not the case. Instead of paying, petitioners even filed a complaint for illegal dismissal against respondent. Respondents policy of suspending drivers who fail to remit the full amount of the boundary was fair and reasonable under the circumstances. Respondent explained that he noticed that his drivers were getting lax in remitting their boundary payments and, in fact, herein petitioners had already incurred a considerable amount of arrears. He had to put a stop to it as he also relied on these boundary payments to raise the full amount of his monthly amortizations on the jeepneys. Demonstrating their obstinacy, petitioners, on the days immediately following the implementation of the policy, incurred deficiencies in their boundary remittances. It is acknowledged that an employer has free rein and enjoys a wide latitude of discretion to regulate all aspects of employment, including the prerogative to instill discipline on his employees and to impose penalties, including dismissal, if warranted, upon erring employees. This is a management prerogative. Indeed, the manner in which management conducts its own affairs to achieve its purpose is within the managements discretion. The only limitation on the exercise of management prerogative is that the policies, rules, and regulations on work-related activities of the employees must always be fair and reasonable, and the corresponding penalties, when prescribed, commensurate to the offense involved and to the degree of the infraction. A company policy must be implemented in such manner as will accord social justice and compassion to the employee. In case of noncompliance with the company policy, the employer must consider the surrounding circumstances and the reasons why the employee failed to comply. When the circumstances merit the relaxation of the application of the policy, then its noncompliance must be excused. In the present case, petitioners merely alleged that there were only few passengers during the dates in question. Such excuse is not acceptable without any proof or, at least, an explanation as to why passengers were scarce at that time. It is simply a bare allegation, not worthy of belief. We also find the excuse unbelievable considering that petitioners incurred the shortages on separate days, and it appears that only petitioners failed to remit the full boundary payment on said dates.

20. Atok Big Wedge Company vs. Gison, G.R. No. 169510, August 8, 2011 Facts: The respondent in this case, Jesus P. Gison, was engaged as part-time consultant of the petitioner, Atok Big Wedge Company thorugh its then Asst. VP and Acting Resident Manager, Rutillo A. Torres. As a consultant on retainer basis, the former assisted the petitioners retained legal counsel with matters pertaining to the prosecution of cases against illegal surface occupants within the area covered by the companys mineral claims. He also tasked to perform liason work with government agencies which he said his expertise. Respondent is not required to report to its office on a regular basis, except when occassionally requested by the management to discuss the matters which needs of his expertise as a consultant. He is paid a retainer fee of 3,000Php a month and delivered to him either in his residence or in a local restaurant. They have also executed a retainer agreement however was misplaced and can no longer be found. This kind of arrangement continued on for the next 11 years. Since respondent was getting old, he requested petitioner to cause his registration with the Social Security System but petitioner did not accede to his request considering the former only a retainer/consultant.

Page 16

LABOR RELATIONS
Atty. Jefferson M. Marquez
Respondent herein, filed a complaint with SSS against petitioners refusal to cause his registration with the SSS. The Resident Manager of the petitioner issued then a Memorandum advising respondent that within 30 days from receipt thereof, petitioners services as a retainer/consultant will be terminated since his services are no longer necessary. As a result, respondent filed a complaint for illegal dismissal, unfair labor practice, underpayment of wages, non-payment of 13th Month pay, vacation pay and sick leave with the NLRC, Regional Arbitration Branch and Cordillera Administrative Region against the petitioner. The Labor Arbiter rendered a decision in favor of the petitioner ruling that there is no employer-employee relationship and dismissed the complaint for lack of merit. An appeal was made before the NLRC but same was dismissed and affirmed the decision of the Labor Arbiter. A petition for review was filed under Rule 65 before the Court of Appeals. The Court of Appeals annuled and has set aside the decision of NLRC. The CA opined that, both the Labor Arbiter and NLRC overlooked Article 280 of the Labor Code, which distinguishes between the two kinds of employees, i.e., regular and casual employees. The respondent is deemed a regular employee of the petitioner after the lapse of one year from his employment. Considering also that the respondent had been performing services for the petitioner for the last 11 years entitling him to the rights and privileges of a regular employee. The CA added that although there was an agreement between the parties that the employment of the respondent will be only temporary, it clearly disregarded the same by repeatedly giving petitioner several tasks to perform. Moreover, although the respondent may have waived his right to attain a regular status when he agreed to perform these tasks on a temporary employment status, still it was the law that recognized and considered him a regular employee after his first year of rendering service to petitioner. As such, the waiver is ineffective. Petitioner herein posits that CA erred in applying Article 280 of the Labor Code in determining whether there exists an employer-employee relationship. Petitioner contends that where the existence of an employer-employee relationship is in dispute, Article 280 of the Labor Code is inapplicable. The said article only set the distinction between a casual employee from a regular employee for purposes of determining the rights of an employee to be entitled to certain benefits. Issue: Whether or not CA erred in applying Article 280? Ruling: Well-entrenched is the doctrine that the existence of an employer-employee relationship is ultimately a question of fact and that the findings thereon by the Labor Arbiter and the NLRC shall be accorded not only respect but even finality when supported by substantial evidence. Being a question of fact, the determination whether such a relationship exists between petitioner and respondent was well within the province of the Labor Arbiter and the NLRC. Being supported by substantial evidence, such determination should have been accorded great weight by the CA in resolving the issue. To ascertain the existence of an employer-employee relationship jurisprudence has invariably adhered to the four-fold test, to wit: (1) the selection and engagement of the employee; (2) the payment of wages; (3) the power of dismissal; and (4) the power to control the employee's conduct, or the so-called "control test." The so-called "control test" is commonly regarded as the most crucial and determinative indicator of the presence or absence of an employer-employee relationship Applying the aforementioned test, an employer-employee relationship is apparently absent in the case at bar. Among other things, respondent was not required to report everyday during regular office hours of petitioner. Respondent's monthly retainer fees were paid to him either at his residence or a local restaurant. More importantly, petitioner did not prescribe the manner in which respondent would accomplish any of the tasks in which his expertise as a liaison officer was needed; respondent was left alone and given the freedom to accomplish the tasks using his own means and method. Respondent was assigned tasks to perform, but petitioner did not control the manner and methods by which respondent performed these tasks. Verily, the absence of the element of control on the part of the petitioner engenders a conclusion that he is not an employee of the petitioner. Moreover, the absence of the parties' retainership agreement notwithstanding, respondent clearly admitted that petitioner hired him in a limited capacity only and that there will be no employer-employee relationship between them. Respondent was well aware of the agreement that he was hired merely as a liaison or consultant of the petitioner and he agreed to perform tasks for the petitioner on a temporary employment status only. However, respondent anchors his claim that he became a regular employee of the petitioner based on his contention that the "temporary" aspect of his job and its "limited" nature could not have lasted for eleven years unless some time during that period, he became a regular employee of the petitioner by continually performing services for the company. Respondent is not an employee, much more a regular employee of petitioner. The appellate court's premise that regular employees are those who perform activities which are desirable and necessary for the business of the employer is not determinative in this case. In fact, any agreement may provide that one party shall render services for and in behalf of another, no matter how necessary for the latter's business, even without being hired as an employee. Hence,respondent's length of service and petitioner's repeated act of assigning respondent some tasks to be performed did not result to respondent's entitlement to the rights and privileges of a regular employee. Furthermore, despite the fact that petitioner made use of the services of respondent for eleven years, he still cannot be considered as a regular employee of petitioner. Article 280 of the Labor Code, in which the lower court used to buttress its findings that respondent became a regular employee of the petitioner, is not applicable in the case at bar. Indeed, the Court has ruled that said provision is not the yardstick for determining the existence of an employment relationship because it merely distinguishes between two kinds of employees, i.e., regular employees and

Page 17

LABOR RELATIONS
Atty. Jefferson M. Marquez
casual employees, for purposes of determining the right of an employee to certain benefits, to join or form a union, or to security of tenure; it does not apply where the existence of an employment relationship is in dispute.It is, therefore, erroneous on the part of the Court of Appeals to rely on Article 280 in determining whether an employer-employee relationship exists between respondent and the petitioner. Considering that there is no employer-employee relationship between the parties, the termination of respondent's services by the petitioner after due notice did not constitute illegal dismissal warranting his reinstatement and the payment of full backwages, allowances and other benefits.

21. Semblante vs. CA, G.R. No. 196426, August 15, 2011 Facts: Petitioners Marticio Semblante and Dubrick Pilar worked in the Gallera de Mandaue owned by the respondents-spouses Vicente and Maria Luisa Loot. The petitioners rendered their services as the official massiador and sentenciador in 1993. As the masiador, Semblante calls and takes the bets from the gamecock owners and other bettors and orders the start of the cockfight. He also distributes the winnings after deducting the arriba, or the commission for the cockpit. Meanwhile, as the sentenciador, Pilar oversees the proper gaffing of fighting cocks, determines the fighting cocks' physical condition and capabilities to continue the cockfight, and eventually declares the result of the cockfight. As masiador and sentenciador, Semblante receives PhP2,000 per week or a total of PhP8,000 per month, while Pilar gets PhP3,500 a week or PhP14,000 per month. They work every Tuesday, Wednesday, Saturday, and Sunday every week, excluding monthly derbies and cockfights held on special holidays. Their working days start at 1:00 p.m. and last until 12:00 midnight, or until the early hours of the morning depending on the needs of the cockpit. Petitioners had both been issued employees' identification cards that they wear every time they report for duty. However on November 14,1993, petitioners were denied entry into the cockpit upon the instructions of respondents and were informed of the termination of their employment effective that date. Respondents denied that petitioners were their employees and alleged that they were associates of respondents independent contractor, Tomas Vega. They claimed that petitioners have no regular working time or day and they are free to decide for themselves whether to report for work or not on any cockfighting day. And the identification card issued was only to free them from the normal entrance fees and to differentiate them from the general public. The Labor Arbiter found that there exist an employer-employee relationship between the petitioner and the respondents because the latter performed the works necessary and indispensable to the usual trade or business of the respondents for a number of years. It has ruled that petitioners were illegally dismissed and are entitled to their backwages and separation pay. However, the NLRC reversed the Labor Arbiters decision. It held that respondents having no power on the selection and engagement of petitioners and that no separate individual contract with respondents was ever executed by petitioners. In its appeal to the CA, the latter ruled in favor for the respondents and held that referees and bet-takers in a cockfight need to have the kind of expertise that is characteristic of the game to interpret messages conveyed by mere gestures. Hence, petitioners are akin to independent contractors who possess unique skills , expertise and talent to distinguish them from ordinary employees. Further, petitioners were not provided by tools and instrumentalities they needed to perform their work. They only need their unique skills and talents in the performance of their job as masiador and sentenciador. Issue: Whether or not the dismissal of the petitioners is illegal on the ground that that they are regular employees of the respondents? Ruling: Respondents had no part in petitioners' selection and management; petitioners' compensation was paid out of the arriba (which is a percentage deducted from the total bets), not by petitioners; and petitioners performed their functions as masiador and sentenciador free from the direction and control of respondents. In the conduct of their work, petitioners relied mainly on their "expertise that is characteristic of the cockfight gambling," and were never given by respondents any tool needed for the performance of their work. Respondents, not being petitioners' employers, could never have dismissed, legally or illegally, petitioners, since respondents were without power or prerogative to do so in the first place. The rule on the posting of an appeal bond cannot defeat the substantive rights of respondents to be free from an unwarranted burden of answering for an illegal dismissal for which they were never responsible.

Page 18

LABOR RELATIONS
Atty. Jefferson M. Marquez
22. Bernarte vs. Phil. Basketball Assoc., G.R. No. 192084, September 14, 2011

Facts: Complainants (Jose Mel Bernarte and Renato Guevarra) aver that they were invited to join the PBA as referees. During the leadership of Commissioner Emilio Bernardino, they were made to sign contracts on a year-to-year basis. During the term of Commissioner Eala, however, changes were made on the terms of their employment. Complainant Bernarte, for instance, was not made to sign a contract during the first conference of the All-Filipino Cup which was from February 23, 2003 to June 2003. It was only during the second conference when he was made to sign a one and a half month contract for the period July 1 to August 5, 2003. On January 15, 2004, Bernarte received a letter from the Office of the Commissioner advising him that his contract would not be renewed citing his unsatisfactory performance on and off the court. It was a total shock for Bernarte who was awarded Referee of the year in 2003. He felt that the dismissal was caused by his refusal to fix a game upon order of Ernie De Leon. On the other hand, complainant Guevarra alleges that he was invited to join the PBA pool of referees in February 2001. On March 1, 2001, he signed a contract as trainee. Beginning 2002, he signed a yearly contract as Regular Class C referee. On May 6, 2003, respondent Martinez issued a memorandum to Guevarra expressing dissatisfaction over his questioning on the assignment of referees officiating out-of-town games. Beginning February 2004, he was no longer made to sign a contract. Respondents aver, on the other hand, that complainants entered into two contracts of retainer with the PBA in the year 2003. The first contract was for the period January 1, 2003 to July 15, 2003; and the second was for September 1 to December 2003. After the lapse of the latter period, PBA decided not to renew their contracts. Complainants were not illegally dismissed because they were not employees of the PBA. Their respective contracts of retainer were simply not renewed. PBA had the prerogative of whether or not to renew their contracts, which they knew were fixed. Both the Labor Arbiter and NLRC decided that the petitioners were employees whose dismissals by respondents were illegal. However, the Court of Appeals overturned the decisions of the NLRC and Labor Arbiter on the ground that the petitioner is an independent contractor since respondents did not exercise any form of control over the means and methods by which petitioner performed his work as a basketball referee. Issue: Whether petitioner is an employee of respondents, which in turn determines whether petitioner was illegally dismissed. Ruling The Supreme Court affirmed the assailed decision of the Court of Appeals. To determine the existence of an employer-employee relationship, case law has consistently applied the four-fold test, to wit: (a) the selection and engagement of the employee; (b) the payment of wages; (c) the power of dismissal; and (d) the employer's power to control the employee on the means and methods by which the work is accomplished. The so-called "control test" is the most important indicator of the presence or absence of an employer-employee relationship. In this case, PBA admits repeatedly engaging petitioner's services, as shown in the retainer contracts. PBA pays petitioner a retainer fee, exclusive of per diem or allowances, as stipulated in the retainer contract. PBA can terminate the retainer contract for petitioner's violation of its terms and conditions. However, respondents argue that the all-important element of control is lacking in this case, making petitioner an independent contractor and not an employee of respondents. We agree with respondents that once in the playing court, the referees exercise their own independent judgment, based on the rules of the game, as to when and how a call or decision is to be made. The referees decide whether an infraction was committed, and the PBA cannot overrule them once the decision is made on the playing court. The referees are the only, absolute, and final authority on the playing court. Respondents or any of the PBA officers cannot and do not determine which calls to make or not to make and cannot control the referee when he

Page 19

LABOR RELATIONS
Atty. Jefferson M. Marquez
blows the whistle because such authority exclusively belongs to the referees. The very nature of petitioner's job of officiating a professional basketball game undoubtedly calls for freedom of control by respondents. Moreover, the following circumstances indicate that petitioner is an independent contractor: (1) the referees are required to report for work only when PBA games are scheduled, which is three times a week spread over an average of only 105 playing days a year, and they officiate games at an average of two hours per game; and (2) the only deductions from the fees received by the referees are withholding taxes. In other words, unlike regular employees who ordinarily report for work eight hours per day for five days a week, petitioner is required to report for work only when PBA games are scheduled or three times a week at two hours per game. In addition, there are no deductions for contributions to the Social Security System, Philhealth or Pag-Ibig, which are the usual deductions from employees' salaries. These undisputed circumstances buttress the fact that petitioner is an independent contractor, and not an employee of respondents. Furthermore, the applicable foreign case law declares that a referee is an independent contractor, whose special skills and independent judgment is required specifically for such position and cannot possibly be controlled by the hiring party. In addition, the fact that PBA repeatedly hired petitioner does not by itself prove that petitioner is an employee of the former. For a hired party to be considered an employee, the hiring party must have control over the means and methods by which the hired party is to perform his work, which is absent in this case. The continuous rehiring by PBA of petitioner simply signifies the renewal of the contract between PBA and petitioner, and highlights the satisfactory services rendered by petitioner warranting such contract renewal. Conversely, if PBA decides to discontinue petitioner's services at the end of the term fixed in the contract, whether for unsatisfactory services, or violation of the terms and conditions of the contract, or for whatever other reason, the same merely results in the non-renewal of the contract, as in the present case. The non-renewal of the contract between the parties does not constitute illegal dismissal of petitioner by respondents.

23. Lirio vs. Genovia, G.R. No. 169757, November 23, 2011 Facts: Respondent Wilmer D. Genovia filed a complaint against petitioner Cesar Lirio and/or Celkor Ad Sonicmix Recording Studio for illegal dismissal, non-payment of commission and award of moral and exemplary damages. Respondent Genovia alleged in his position paper that on August 15, 2001, he was hired as studio manager by petitioner Lirio, owner of Celkor Ad Sonicmix Recording Studio (Celkor). He was employed to manage and operate Celkor and to promote and sell the recording studio's services to music enthusiasts and other prospective clients. He received a monthly salary of P7,000.00. They also agreed that he was entitled to an additional commission of P100.00 per hour as recording technician whenever a client uses the studio for recording, editing or any related work. He was made to report for work from Monday to Friday from 9:00 a.m. to 6 p.m. On Saturdays, he was required to work half-day only, but most of the time, he still rendered eight hours of work or more. All the employees of petitioner, including respondent, rendered overtime work almost everyday, but petitioner never kept a daily time record to avoid paying the employees overtime pay. He also alleged that petitioner approached him and told him about his project to produce an album for his daughter, Celine Mei Lirio. Petitioner asked respondent to compose and arrange songs for Celine and promised that he (Lirio) would draft a contract to assure respondent of his compensation for such services. As agreed upon, the additional services that respondent would render included composing and arranging musical scores only, while the technical aspect in producing the album, such as digital editing, mixing and sound engineering would be performed by respondent in his capacity as studio manager for which he was paid on a monthly basis. Petitioner instructed respondent that his work on the album as composer and arranger would only be done during his spare time, since his other work as studio manager was the priority. Respondent then started working on the album. After the album was completed and released, respondent again reminded petitioner about the contract on his compensation as composer and arranger of the album. Petitioner told respondent that since he was practically a nobody and had proven nothing yet in the music industry, respondent did not deserve a high compensation, and he should be thankful that he was given a job to feed his family. Petitioner informed respondent that he was entitled only to 20% of the net profit, and not of the gross sales of the album, and that the salaries he received and would continue to receive as studio manager of Celkor would be deducted from the said 20% net profit share. Respondent objected and insisted that he be properly compensated. On March 14, 2002, petitioner verbally terminated respondents services, and he was instructed not to report for work. Respondent asserts that he was illegally dismissed as he was terminated without any valid grounds, and no hearing was conducted before he was terminated, in violation of his constitutional right to due process. Having worked for more than six months, he was already a regular

Page 20

LABOR RELATIONS
Atty. Jefferson M. Marquez
employee. Although he was a so called studio manager, he had no managerial powers, but was merely an ordinary employee. Respondent prayed for his reinstatement without loss of seniority rights, or, in the alternative, that he be paid separation pay, backwages and overtime pay; and that he be awarded unpaid commission for services rendered as a studio technician as well as moral and exemplary damages. Respondents evidence consisted of the Payroll dated July 31, 2001 to March 15, 2002, which was certified correct by petitioner, and Petty Cash Voucher evidencing receipt of payroll payments by respondent from Celkor. In defense, petitioner stated in his Position Paper that respondent was not hired as studio manager, composer, technician or as an employee in any other capacity of Celkor. Respondent could not have been hired as a studio manager, since the recording studio has no personnel except petitioner. According to petitioner, respondent had no track record as a composer, and he was not known in the field of music. Nevertheless, after some discussion, respondent verbally agreed with petitioner to co-produce the album. Petitioner asserted that his relationship with respondent is one of an informal partnership and that he had no control over the time and manner by which respondent composed or arranged the songs, except on the result thereof. Respondent reported to the recording studio between 10:00 a.m. and 12:00 noon. Hence, petitioner contended that no employer-employee relationship existed between him and the respondent, and there was no illegal dismissal to speak of. The Labor Arbiter rendered a decision finding that an employer-employee relationship existed between petitioner and respondent, and that respondent was illegally dismissed. However, the NLRC reversed and set aside the decision of the Labor Arbiter on the ground that respondent failed to prove his employment tale with substantial evidence. It held that respondent failed to proved with substantial evidence that he was selected and engaged by petitioner, that petitioner had the power to dismiss him, and that they had the power to control him not only as to the result of his work, but also as to the means and methods of accomplishing his work. The Court of Appeals rendered a decision reversing and setting aside the resolution of the NLRC, and reinstating the decision of the Labor Arbiter. Hence, petitioner Lirio filed this petition. Issue: Whether respondent is an employee of the petitioner, which in turn determines whether respondent was illegally dismissed. Ruling: The Supreme Court affirmed the assailed decision of the Court of Appeals. Before a case for illegal dismissal can prosper, it must first be established that an employer-employee relationship existed between petitioner and respondent. The elements to determine the existence of an employment relationship are: (a) the selection and engagement of the employee; (b) the payment of wages; (c) the power of dismissal; and (d) the employers power to control the employees conduct. The most important element is the employers control of the employees conduct, not only as to the result of the work to be done, but also as to the means and methods to accomplish it. It is settled that no particular form of evidence is required to prove the existence of an employer-employee relationship. Any competent and relevant evidence to prove the relationship may be admitted. In this case, the documentary evidence presented by respondent to prove that he was an employee of petitioner are as follows: (a) a document denominated as "payroll" (dated July 31, 2001 to March 15, 2002) certified correct by petitioner, which showed that respondent received a monthly salary of P7,000.00 (P3,500.00 every 15th of the month and another P3,500.00 every 30th of the month) with the corresponding deductions due to absences incurred by respondent; and (2) copies of petty cash vouchers, showing the amounts he received and signed for in the payrolls.

Page 21

LABOR RELATIONS
Atty. Jefferson M. Marquez
The said documents showed that petitioner hired respondent as an employee and he was paid monthly wages of P7,000.00. Petitioner wielded the power to dismiss as respondent stated that he was verbally dismissed by petitioner, and respondent, thereafter, filed an action for illegal dismissal against petitioner. The power of control refers merely to the existence of the power. It is not essential for the employer to actually supervise the performance of duties of the employee, as it is sufficient that the former has a right to wield the power. Nevertheless, petitioner stated in his Position Paper that it was agreed that he would help and teach respondent how to use the studio equipment. In such case, petitioner certainly had the power to check on the progress and work of respondent. On the other hand, petitioner failed to prove that his relationship with respondent was one of partnership. Such claim was not supported by any written agreement. The Court notes that in the payroll dated July 31, 2001 to March 15, 2002, there were deductions from the wages of respondent for his absence from work, which negates petitioners claim that the wages paid were advances for respondents work in the partnership. The Court agrees with the Court of Appeals that the evidence presented by the parties showed that an employer-employee relationship existed between petitioner and respondent. In termination cases, the burden is upon the employer to show by substantial evidence that the termination was for lawful cause and validly made.Article 277 (b) of the Labor Code puts the burden of proving that the dismissal of an employee was for a valid or authorized cause on the employer, without distinction whether the employer admits or does not admit the dismissal. For an employees dismissal to be valid, (a) the dismissal must be for a valid cause, and (b) the employee must be afforded due process. Procedural due process requires the employer to furnish an employee with two written notices before the latter is dismissed: (1) the notice to apprise the employee of the particular acts or omissions for which his sought, which is the equivalent of a charge; and (2) the notice informing the employee of his dismissal, to be issued after the employee has been given reasonable opportunity to answer and to be heard on his defense. Petitioner failed to comply with these legal requirements; hence, the Court of Appeals correctly affirmed the Labor Arbiters finding that respondent was illegally dismissed, and entitled to the payment of backwages, and separation pay in lieu of reinstatement.

24. Jao vs. BCC Product Sales Inc., G.R. No. 163700, April 18, 2012 Facts: Petitioner maintained that respondent BCC Product Sales Inc. (BCC) and its President, Terrance Ty, employed him as comptroller starting from September 1995 with a monthly salary of P20,000.00 to handle the financial aspect of BCCs business. On October 19,1995, the security guards of BCC, acting upon the instruction of Ty, barred him from entering the premises of BCC where he then worked. His attempts to report to work in November and December 12, 1995 were frustrated because he continued to be barred from entering the premises of BCC. He then filed a complaint for illegal dismissal, reinstatement with full backwages, non-payment of wages, damages and attorneys fees. Respondents countered that petitioner was not their employee but the employee of Sobien Food Corporation (SFC), the major creditor and supplier of BCC; and that SFC had posted him as its comptroller in BCC to oversee BCCs finances and business operations and to look after SFCs interests or investments in BCC. Issue: Whether or not an employer-employee relationship existed between petitioner Jao and BCC Ruling: The Supreme Court speaking through Justice Bersamin declared that the court cannot side with petitioner. In determining the presence or absence of an employer-employee relationship, the Court has consistently looked for the following incidents, to wit: (a) the selection and engagement of the employee; (b) the payment of wages; (c) the power of dismissal; and (d) the employers power to control the employee on the means and methods by which the work is accomplished. The last element, the so-called control test, is the most important element. Hereunder are some of the circumstances and incidents occurring while petitioner was supposedly employed by BCC that debunked his claim against respondents. It can be deduced from the March 1996 affidavit of petitioner that respondents challenged his authority to deliver some 158 checks to SFC. Considering that he contested respondents challenge by pointing to the existing arrangements between BCC and SFC, it should be clear that respondents did not exercise the power of control over him, because he thereby acted for the benefit and in the interest of SFC more than of BCC.

Page 22

LABOR RELATIONS
Atty. Jefferson M. Marquez
RIGHT TO SECURITY OF TENURE CASES: 1. ALU-TUCP vs. NLRC, 234 SCRA 678 [1994] Facts: National Steel Corporation (NSC) employed petitioners in connection with its Five Year Expansion Program. It undertook this program with the end in view of expanding the volume and increasing the kinds of products that it may offer for sale to the public. Petitioners were then terminated. They filed a complaint for unfair labor practice, regularization and monetary benefits. Their contention was that they should be considered regular employees because their jobs are necessary, desirable and work related to NSCs main business which is steel making and that they have rendered service for more than six years. Issue: Whether or not petitioners were properly characterized as regular employees rather than project employees. Ruling: Petitioners are project employees. Project employees are those employed for a fixed for a specific project or undertaking the completion or termination of which has been determined at the time of the engagement of the employee. On the other hand, regular employees are legally entitled to remain in the service of their employer until that service is terminated by one or another of the recognized modes or termination of service under the Labor Code. The principal test for determining whether an employee is properly characterized as project employees is whether or not the project employees were carrying out a specific project or undertaking, the duration and the scope of which were specified at the time the employees were engaged for that project. There are two types of project activities. First is that a project could refer to a particular job or undertaking that is within the regular or usual business of the employer company, but which is distinct and separate and identifiable as such, from the other undertakings of the company. Such job or undertaking begins and ends at determined or determinable times. Second is a particular job or undertaking that is not within the regular business of the corporation. Such a job or undertaking must also be identifiably separate and distinct from the ordinary or regular business operations of the employer. It must also begin and end at determined or determinable times. The case at bar falls on the second type of project activity. The carrying out of the Five Year Expansion Program constitutes a distinct undertaking identifiable from the ordinary business and activity of NSC. Each component project, of course, begins and ends at specified times which had already been determined by the time petitioners were engaged. During the time petitioners rendered services to NSC, their work was limited to one or another of the specific component projects which made up the Five Year Expansion Program. They were not hired or assigned to any other purpose. The services of these project employees may be lawfully terminated at the completion of the project. It is dependent and coterminous with the completion or termination of the specific undertaking or activity for which the employee was hired which has been pre-determined at the time of the engagement. Furthermore, the length of service of a project employee is not the controlling test of employment of tenure. The simple fact that the employment of petitioners as project employees had gone beyond one year does not detract from or legally dissolve their status as project employees. Whichever type of project employment is found in a particular case, a common basic requisite is that the designation of named employees as "project employees" and their assignment to a specific project, are effected and implemented in good faith, and not merely as a means of evading otherwise applicable requirements of labor laws.

2. Cosmos Bottling Corp., vs NLRC, 255 SCRA 358 [1996] Facts: Gil C. Castro was employed by Cosmos Bottling Corporation for a specific period. Having satisfactorily served the company for two (2) terms, Castro was recommended for reemployment with the companys Maintenance Team for the Davao Project, he was re-hired and assigned to the Maintenance Division of the Davao Project tasked to install the private respondents annex plant machines in its Davao plant. Castros employment was terminated due to the completion of the special project. Cosmos Bottling Corporation in valid exercise of its management prerogative terminated the services of some 228 regular employees by reason of retrenchment. For obvious reasons, Castro was not among the list of those regular employees whose services were terminated by reason of retrenchment or those who voluntarily resigned. Castro filed a

Page 23

LABOR RELATIONS
Atty. Jefferson M. Marquez
complaint for illegal dismissal against Cosmos Bottling Corporation with the Labor Arbiter contending that being a regular employee, he could not be dismissed without a just and valid cause. The company alleged that Castro was a mere project employee whose employment was coterminous with the project for which he was hired. Issue: Whether or not private respondent Gil C. Castro is a regular employee or was a mere project employee of petitioner Cosmos Bottling Corporation. Ruling: Private respondent being a project employee, or to use the correct term, seasonal employee, considering that his employment was limited to the installation and dismantling of petitioners annex plant machines after which there was no more work to do, his employment legally ended upon completion of the project. That being so, the termination of his employment cannot and should not constitute an illegal dismissal. Neither should it constitute retrenchment as private respondent was a seasonal employee whose services were already terminated on May 21, 1990 prior to the termination of the other regular employees of Cosmos by reason of retrenchment.

3. Purefoods v. NLRC 283 SCRA 136 [1997] Facts: The private respondents (numbering 906) were hired by petitioner Pure Foods Corporation to work for a fixed period of five months at its tuna cannery plant. After the expiration of their respective contracts of employment, their services were terminated. They thenexecuted a Release and Quitclaim stating that they had no claim whatsoever against Pure Foods. The private respondents filed before the NLRC Sub-Regional Arbitration Branch, a complaint for illegal dismissal against the petitioner and its plant manager. Pure Foods Corp submits that the private respondents are now estopped from questioning their separation from petitioners employ in view of their express conformity with the five-month duration of their employment contracts. Besides, they fell within the exception provided in Article 280 of the Labor Code which reads: [E]xcept where the employment has been fixed for a specific project or undertaking the completion or termination of which has been determined at the time of the engagement of the employee. Moreover, the first paragraph of the said article must be read and interpreted in conjunction with the proviso in the second paragraph, which reads: Provided that any employee who has rendered at least one year of service, whether such service is continuous or broken, shall be considered a regular employee with respect to the activity in which he is employed.... In the instant case, the private respondents were employed for a period of five months only. In any event, private respondents' prayer for reinstatement is well within the purview of the Release and Quitclaim they had executed wherein they unconditionally released the petitioner from any and all other claims which might have arisen from their past employment with the petitioner. The private respondents, on the other hand, argue that contracts with a specific period of employment may be given legal effect provided, however, that they are not intended to circumvent the constitutional guarantee on security of tenure. They submit that the practice of the petitioner in hiring workers to work for a fixed duration of five months only to replace them with other workers of the same employment duration was apparently to prevent the regularization of these so-called casuals, which is a clear circumvention of the law on security of tenure. Issue: Whether employees hired for a definite period and whose services are necessary and desirable in the usual business or trade of the employer are regular employees. Ruling: We find the petition devoid of merit. Article 280 of the Labor Code defines regular and casual employment as follows: ART. 280. Regular and Casual Employment.-- The provisions of written agreement to the contrary notwithstanding and regardless of the oral argument of the parties, an employment shall be deemed to be regular where the employee has been engaged to perform activities which are usually necessary or desirable in the usual business or trade of the employer, except where the employment has been fixed for a specific project or undertaking the completion or termination of which has been determined at the time of the engagement of the employee or where the work or services to be performed is seasonal in nature and the employment is for the duration of the season.

Page 24

LABOR RELATIONS
Atty. Jefferson M. Marquez
An employment shall be deemed to be casual if it is not covered by the preceding paragraph; Provided, That, any employee who has rendered at least one year of service, whether such service is continuous or broken, shall be considered a regular employee with respect to the activity in which he is employed and his employment shall continue while such activity exists. Thus, the two kinds of regular employees are (1) those who are engaged to perform activities which are necessary or desirable in the usual business or trade of the employer; and (2) those casual employees who have rendered at least one year of service, whether continuous or broken, with respect to the activity in which they are employed. In the instant case, the private respondents activities consisted in the receiving, skinning, loining, packing, and casing-up of tuna fish which were then exported by the petitioner. Indisputably, they were performing activities which were necessary and desirable in petitioners business or trade. xxx Contrary to petitioner's submission, the private respondents could not be regarded as having been hired for a specific project or undertaking. The term specific project or undertaking under Article 280 of the Labor Code contemplates an activity which is not commonly or habitually performed or such type of work which is not done on a daily basis but only for a specific duration of time or until completion; the services employed are then necessary and desirable in the employers usual business only for the period of time it takes to complete the project. The fact that the petitioner repeatedly and continuously hired workers to do the same kind of work as that performed by those whose contracts had expired negates petitioners contention that those workers were hired for a specific project or undertaking only. xxx Where from the circumstances it is apparent that the periods have been imposed to preclude acquisition of tenurial security by the employee, they should be struck down or disregarded as contrary to public policy and morals. xxx criteria under which term employment cannot be said to be in circumvention of the law on security of tenure: 1) The fixed period of employment was knowingly and voluntarily agreed upon by the parties without any force, duress, or improper pressure being brought to bear upon the employee and absent any other circumstances vitiating his consent; or 2) It satisfactorily appears that the employer and the employee dealt with each other on more or less equal terms with no moral dominance exercised by the former or the latter. None of these criteria had been met in the present case. This scheme of the petitioner was apparently designed to prevent the private respondents and the other casual employees from attaining the status of a regular employee. It was a clear circumvention of the employees right to security of tenure and to other benefits like minimum wage, cost-of-living allowance, sick leave, holiday pay, and 13th month pay. Indeed, the petitioner succeeded in evading the application of labor laws. Also, it saved itself from the trouble or burden of establishing a just cause for terminating employees by the simple expedient of refusing to renew the employment contracts. The five-month period specified in private respondents employment contracts having been imposed precisely to circumvent the constitutional guarantee on security of tenure should, therefore, be struck down or disregarded as contrary to public policy or morals

4. Phil. Fruit and Vegetable Industries v. NLRC, 310 SCRA 680 [1999] Facts: Private respondent Philippine Fruit and Vegetable Workers Union-Tupas Local Chapter, for and in behalf of 127 of its members, filed a complaint for unfair labor practice and/or illegal dismissal with damages against Petitioner Corporation. They alleged that the dismissals were due to complainants' involvement in union activities and were without just cause.

Page 25

LABOR RELATIONS
Atty. Jefferson M. Marquez
Labor Arbiter rendered judgment that Philippine Fruit & Vegetable Industries, Inc (PFVII) were indeed guilty of Illegal Dismissal. On appeal, NLRC set aside the Labor Arbiters decision and remanded the said case to the Arbitration Branch for further proceedings. Arbitration Branch rendered a decision finding PFVII liable for illegal dismissal. On appeal, NLRC affirmed the Arbitration Branchs decision but modified the awards of attorneys fees. PFVII filed a motion for reconsideration which was denied by NLRC Issue: Whether or not the complainants were illegal dismissed by PFVII One of PFVIIs contentions is that the complainants are seasonal workers. According to them, its operation starts only in February with the processing of tomatoes into tomato paste and ceases by the end of the same month when the supply is consumed. It then resumes operations at the end of April or early May, depending on the availability of supply with the processing of mangoes into purees and ceases operation in June. The severance of complainants' employment from petitioner corporation was a necessary consequence of the nature of seasonal employment; and since complainants are seasonal workers as defined by the Labor Code, they cannot invoke any tenurial benefit Ruling: By the very nature of things in a business enterprise like PFVII, the services of the complainants are, indeed, more than six (6) months a year. The company did not confine itself just to the processing of tomatoes and mangoes. It also processed guyabano, calamansi, papaya, pineapple, etc. Besides, they have the office of administrative functions, cleaning and upkeeping of machines and other duties and tasks to keep up a big food processing corporation. Considering, therefore, that under Article 280 of the Labor Code "the provisions of written agreement to the contrary notwithstanding and considering further that the tasks which complainants performed were usually necessary and desirable in the employers usual business or trade, Supreme Court held that complainants are regular seasonal employees, thus, entitled to security of tenure. The Labor Code provides an employment shall be deemed to be regular where the employee has been engaged to perform activities that are usually necessary or desirable in the usual business or trade of the employers, except where the employment has been fixed for a specific project. An employment shall be deemed to be casual if it is not covered by the preceding paragraph; provided, that, any employee who has rendered at least one year of service whether such service is continuous or broken, shall be considered a regular employee with respect to the activity in which he is employed and his employment shall continue while such actually exists. In the case at bar, the work of complainants as seeders, operators, sorters, slicers, janitors, drivers, truck helpers, mechanics and office personnel is without doubt necessary in the usual business of a food processing company like petitioner PFVII. It should be noted that complainants' employment has not been fixed for a specific project or undertaking the completion or termination of which has been determined at the time of their appointment or hiring. Neither is their employment seasonal in nature. While it may be true that some phases of petitioner company's processing operations is dependent on the supply of fruits for a particular season, the other equally important aspects of its business, such as manufacturing and marketing are not seasonal. The fact is that large-scale food processing companies such as Petitioner Company continue to operate and do business throughout the year even if the availability of fruits and vegetables is seasonal. PETITION DENIED. COMPLAINANTS ARE REGULAR EMPLOYEES BY VIRTUE OF THE FACT THAT THEY PERFORMED FUNCTIONS WHICH ARE NECESSARY AND DESIRABLE IN THE USUAL BUSINESS OF PFVII.

5. Philips Semiconductor vs. Fardiquela, G.R. No. 141717, April 14, 2004 Facts: The petitioner Philips Semiconductors (Phils.), Inc. is a domestic corporation engaged in the production and assembly of semiconductors such

Page 26

LABOR RELATIONS
Atty. Jefferson M. Marquez
as power devices, RF modules, CATV modules, RF and metal transistors and glass diods. It caters to domestic and foreign corporations that manufacture computers, telecommunications equipment and cars. Aside from contractual employees, the petitioner employed 1,029 regular workers. The employees were subjected to periodic performance appraisal based on output, quality, attendance and work attitude. One was required to obtain a performance rating of at least 3.0 for the period covered by the performance appraisal to maintain good standing as an employee. On May 8, 1992, respondent Eloisa Fadriquela executed a Contract of Employment with the petitioner in which she was hired as a production operator with a daily salary of P118. Her initial contract was for a period of three months up to August 8, 1992, but was extended for two months when she garnered a performance rating of 3.15. Her contract was again renewed for two months or up to December 16, 1992, when she received a performance rating of 3.8.After the expiration of her third contract, it was extended anew, for three months, that is, from January 4, 1993 to April 4, 1993. After garnering a performance rating of 3.4, the respondents contract was extended for another three months, that is, from April 5, 1993 to June 4, 1993. She, however, incurred five absences in the month of April, three absences in the month of May and four absences in the month of June. Line supervisor Shirley F. Velayo asked the respondent why she incurred the said absences, but the latter failed to explain her side. The respondent was warned that if she offered no valid justification for her absences, Velayo would have no other recourse but to recommend the non-renewal of her contract. The respondent still failed to respond, as a consequence of which her performance rating declined to 2.8. Velayo recommended to the petitioner that the respondents employment be terminated due to habitual absenteeism, in accordance with the Company Rules and Regulations. Thus, the respondents contract of employment was no longer renewed. Issues and Rulings: (a) whether or not the respondent was still a contractual employee of the petitioner as of June 4, 1993; The two kinds of regular employees under the law are (1) those engaged to perform activities which are necessary or desirable in the usual business or trade of the employer; and (2) those casual employees who have rendered at least one year of service, whether continuous or broken, with respect to the activities in which they are employed. The primary standard to determine a regular employment is the reasonable connection between the particular activity performed by the employee in relation to the business or trade of the employer. The test is whether the former is usually necessary or desirable in the usual business or trade of the employer. If the employee has been performing the job for at least one year, even if the performance is not continuous or merely intermittent, the law deems the repeated and continuing need for its performance as sufficient evidence of the necessity, if not indispensability of that activity to the business of the employer. Hence, the employment is also considered regular, but only with respect to such activity and while such activity exists.[22] The law does not provide the qualification that the employee must first be issued a regular appointment or must be declared as such before he can acquire a regular employee status.In this case, the respondent was employed by the petitioner on May 8, 1992 as production operator. She was assigned to wirebuilding at the transistor division. There is no dispute that the work of the respondent was necessary or desirable in the business or trade of the petitioner. She remained under the employ of the petitioner without any interruption since May 8, 1992 to June 4, 1993 or for one (1) year and twenty-eight (28) days. The original contract of employment had been extended or renewed for four times, to the same position, with the same chores. Such a continuing need for the services of the respondent is sufficient evidence of the necessity and indispensability of her services to the petitioners business. By operation of law, then, the respondent had attained the regular status of her employment with the petitioner, and is thus entitled to security of tenure as provided for in Article 279 of the Labor Code which reads: (b) whether or not the petitioner dismissed the respondent from her employment; if so, whether or not she was accorded the requisite notice and investigation prior to her dismissal; and On the second and third issues, we agree with the appellate court that the respondent was dismissed by the petitioner without the requisite notice and without any formal investigation. Given the factual milieu in this case, the respondents dismissal from employment for incurring five (5) absences in April 1993, three (3) absences in May 1993 and four (4) absences in June 1993, even if true, is too harsh a penalty. We do agree that an employee may be dismissed for violation of reasonable regulations/rules promulgated by the employer. Dismissal is the ultimate penalty that can be meted to an employee. Where a penalty less punitive would suffice, whatever missteps may have been committed by the worker ought not to be visited with a consequence so severe such as dismissal from employment. For, the Constitution guarantees the right of workers to security of tenure. The misery and pain attendant to the loss of jobs then could be avoided if there be acceptance of the view that under certain circumstances of the case the workers should not be deprived of their means of livelihood. (c)whether or not the respondent is entitled to reinstatement and full payment of backwages as well as attorneys fees. The court concludes that petitioners dismissal is illegal because, first, she was dismissed in the absence of a just cause, and second, she was not afforded procedural due process. In pursuance of Article 279 of the Labor Code, we deem it proper to order the reinstatement of petitioner to her former job and the payment of her full backwages.Also, having been compelled to come to court to protect her rights, we grant petitioners prayer for attorneys fees.

Page 27

LABOR RELATIONS
Atty. Jefferson M. Marquez
6. Alcira vs. NLRC, G.R. No. 149859, June 9, 2004 Facts: Middleby Philippines Corp. hired petitioner as engineering support services supervisor on a probationary basis for six months. On 20 November 1996, a senior officer of Middleby withheld his time card and did not allow him to work. Alcira filed with the NLRC a complaint for illegal dismissal on the contention that he had become a regular employee when he was illegally dismissed. In their defense, respondents claim that, during petitioners probationary employment, he showed poor performance in his assigned tasks, incurred ten absences, was late several times and violated company rules on the wearing of uniform. Since he failed to meet company standards, petitioners application to become a regular employee was disapproved and his employment was terminated. Issues and Rulings: (1) Whether petitioner was allowed to work beyond his probationary period and was therefore already a regular employee at the time of his alleged dismissal. Petitioner insists that he already attained the status of a regular employee when he was dismissed on November 20, 1996 because, having started work on May 20, 1996, the six-month probationary period ended on November 16, 1996. According to petitioners computation, since Article 13 of the Civil Code provides that one month is composed of thirty days, six months total one hundred eighty days. As the appointment provided that petitioners status was probationary (6 mos.) without any specific date of termination, the 180 th day fell on November 16, 1996. Thus, when he was dismissed on November 20, 1996, he was already a regular employee. Petitioners contention is incorrect. Our computation of the 6-month probationary period is reckoned from the date of appointment up to the same calendar date of the 6th month following. In short, since the number of days in each particular month was irrelevant, petitioner was still a probationary employee when respondent Middleby opted not to regularize him on November 20, 1996. (2) Whether respondent Middleby informed petitioner of the standards for regularization at the start of his employment. Section 6 (d) of Rule 1 of the Implementing Rules of Book VI of the Labor Code (Department Order No. 10, Series of 1997) provides that: In all cases of probationary employment, the employer shall make known to the employee the standards under which he will qualify as a regular employee at the time of his engagement. Where no standards are made known to the employee at that time, he shall be deemed a regular employee. We hold that respondent Middleby substantially notified petitioner of the standards to qualify as a regular employee when it apprised him, at the start of his employment, that it would evaluate his supervisory skills after five months. an employer is deemed to substantially comply with the rule on notification of standards if he apprises the employee that he will be subjected to a performance evaluation on a particular date after his hiring. We agree with the labor arbiter when he ruled that: In the instant case, petitioner cannot successfully say that he was never informed by private respondent of the standards that he must satisfy in order to be converted into regular status. This runs counter to the agreement between the parties that after five months of service the petitioners performance would be evaluated. It is only but natural that the evaluation should be made vis--vis the performance standards for the job. Private respondent Trifona Mamaradlo speaks of such standard in her affidavit referring to the fact that petitioner did not perform well in his assigned work and his attitude was below par compared to the companys standard required of him. (3) Whether petitioner was illegally dismissed when respondent Middleby opted not to renew his contract on the last day of his probationary employment. It is settled that even if probationary employees do not enjoy permanent status, they are accorded the constitutional protection of security of tenure. This means they may only be terminated for just cause or when they otherwise fail to qualify as regular employees in accordance with reasonable standards made known to them by the employer at the time of their engagement. But this constitutional protection ends on the expiration of the probationary period. On that date, the parties are free to either renew or terminate their contract of employment. This development has rendered moot the question of whether there was a just cause for the dismissal of the petitioners. Middleby exercised its option not to renew the contract when it informed petitioner on the last day of his probationary employment that it did not intend to grant him a regular status. Although we can regard petitioners severance from work as dismissal, the same cannot be deemed illegal. As found by the labor arbiter, the NLRC and the Court of Appeals, petitioner (1) incurred ten absences (2) was tardy several times (3) failed to wear the proper uniform many times and (4) showed inferior supervisory skills. Petitioner failed to satisfactorily refute these substantiated allegations. Taking all this in its entirety, respondent Middleby was clearly justified to end its employment relationship with petitioner.

Page 28

LABOR RELATIONS
Atty. Jefferson M. Marquez
7. Mitsubishi Motors Phils. vs. Chrysler Phil Labor Union, G.R. No. 148738, June 29, 2004 Facts: Nelson Paras first worked for Mitsubishi Motors Philippines Corporation (MMPC) as a shuttle bus driver from March 19, 1976 to June 16, 1982, when he resigned to work abroad. After working in Saudi Arabia from 1982 to 1993, he was re-hired as a welder-fabricator at the MMPC tooling shop from October 3, 1994 to October 31, 1994. On October 29, 1994, his contract was renewed from November 1, 1994 up to March 3, 1995. Paras was later re-hired on a probationary basis as a manufacturing trainee at the Plant Engineering Maintenance Department sometime in May of 1996. He and the new and re-hired employees were then given an orientation about the companys history, corporate philosophy, organizational structure, and company rules and regulations, including the company standards for regularization, code of conduct and companyprovided benefits. On May 27, 1996, Paras started reporting for work. He was assigned at the paint ovens, air make-up and conveyors. As part of the MMPCs policy, Paras was evaluated by his immediate supervisors after six (6) months, and received an average rating. Later, the supervisors informed Paras that based on his performance rating, he would be regularized. However, the Department and Division Managers reviewed the performance evaluation made on Paras. They unanimously agreed, along with Paras immediate supervisors, that his performance was unsatisfactory. As a consequence, Paras was not considered for regularization. On November 26, 1996, he received a Notice of Termination dated November 25, 1996, informing him that his services were terminated effective the said date since he failed to meet the required company standards for regularization. CPLU, a legitimate labor organization and the duly certified bargaining agent of the hourly-paid regular rank and file employees of MMPC in which Paras was a member, argued that Paras was dismissed on his one hundred eighty third (183rd) day of employment, or three (3) days after the expiration of the probationary period of six (6) months, hence, he was already a regular employee on the date of the termination of his probationary employment. Meanwhile, MMPC argued that under Article 13 of the New Civil Code, Paras probationary employment which commenced on May 27, 1996 would expire on November 27, 1996. Since he received the notice of termination of his employment on November 25, 1996, the same should be considered to have been served within the six-month probationary period. Issue: Whether or not Paras was already a regular employee when he was terminated. Ruling: Yes. Indeed, an employer, in the exercise of its management prerogative, may hire an employee on a probationary basis in order to determine his fitness to perform work. Under Article 281 of the Labor Code, the employer must inform the employee of the standards for which his employment may be considered for regularization. Such probationary period, unless covered by an apprenticeship agreement, shall not exceed six (6) months from the date the employee started working. The employees services may be terminated for just cause or for his failure to qualify as a regular employee based on reasonable standards made known to him. Respondent Paras was employed as a management trainee on a probationary basis. During the orientation conducted on May 15, 1996, he was apprised of the standards upon which his regularization would be based. He reported for work on May 27, 1996. As per the companys policy, the probationary period was from three (3) months to a maximum of six (6) months. Applying Article 13 of the Civil Code, the probationary period of six (6) months consists of one hundred eighty (180) days.This is in conformity with paragraph one, Article 13 of the Civil Code, which provides that the months which are not designated by their names shall be understood as consisting of thirty (30) days each. The number of months in the probationary period, six (6), should then be multiplied by the number of days within a month, thirty (30); hence, the period of one hundred eighty (180) days. As clearly provided for in the last paragraph of Article 13, in computing a period, the first day shall be excluded and the last day included. Thus, the one hundred eighty (180) days commenced on May 27, 1996, and ended on November 23, 1996. The termination letter dated November 25, 1996 was served on respondent Paras only at 3:00 a.m. of November 26, 1996. He was, by then, already a regular employee of the petitioner under Article 281 of the Labor Code.

8. Pangilinan vs. General Milling Co., G.R. No. 149329, July 2, 2004 Facts: The respondent General Milling Corporation is a domestic corporation engaged in the production and sale of livestock and poultry. It is, likewise, the distributor of dressed chicken to various restaurants and establishments nationwide. As such, it employs hundreds of employees, some on a regular basis and others on a casual basis, as "emergency workers." The petitioners were employed by the respondent on different dates as emergency workers at its poultry plant under separate "temporary/casual contracts of employment" for a period of five months. Most of them worked as chicken dressers, while the others served as packers or helpers.

Page 29

LABOR RELATIONS
Atty. Jefferson M. Marquez
Upon the expiration of their respective contracts, their services were terminated. They later filed separate complaints for illegal dismissal and non-payment of holiday pay, 13th month pay, night-shift differential and service incentive leave pay against the respondent. The petitioners alleged that their work as chicken dressers was necessary and desirable in the usual business of the respondent. They stressed that based on the nature of their work, they were regular employees of the respondent; hence, could not be dismissed from their employment unless for just cause and after due notice. Issue: Whether or not petitioners are regular employees and, thus, cannot be dismissed without just cause and the required due process. Ruling: Article 280 of the Labor Code comprehends three kinds of employees: (a) regular employees or those whose work is necessary or desirable to the usual business of the employer; (b) project employees or those whose employment has been fixed for a specific project or undertaking the completion or termination of which has been determined at the time of the engagement of the employee or where the work or services to be performed is seasonal in nature and the employment is for the duration of the season; and, (c) casual employees or those who are neither regular nor project employees. A regular employee is one who is engaged to perform activities which are necessary and desirable in the usual business or trade of the employer as against those which are undertaken for a specific project or are seasonal. There are two separate instances whereby it can be determined that an employment is regular: (1) if the particular activity performed by the employee is necessary or desirable in the usual business or trade of the employer; and, (2) if the employee has been performing the job for at least a year. IT IS CLEARLY STIPULATED THAT THE CONDITION OF THE EMPLOYMENT CONTRACT SHALL BE AS FOLLOWS: 1. This employment contract shall be on a DAY-TO-DAY BASIS and shall not extend beyond the period specified therein; 2. The employee aforementioned may be laid off or separated from the Firm, EVEN BEFORE THE EXPIRY DATE OF THIS CONTRACT, if his/her services are no longer needed, or if such services are found to be unsatisfactory, or if she/he has violated any of the established rules and regulations of the Company; 3. In any case, the period of employment shall not go beyond the duration of the work or purpose for which the aforementioned employee has been engaged; 4. That the employee hereby agrees to work in any work shift schedule that may be assigned to him by the Firm during the period of this contract; and This Temporary/Casual Employment contract, unless sooner terminated for any of the causes above-cited, shall then automatically cease on its expiry date, without the necessity of any prior notice to the employee concerned. The records reveal that the stipulations in the employment contracts were knowingly and voluntarily agreed to by the petitioners without force, duress or improper pressure, or any circumstances that vitiated their consent. Similarly, nothing therein shows that these contracts were used as a subterfuge by the respondent GMC to evade the provisions of Articles 279 and 280 of the Labor Code. The petitioners were hired as "emergency workers" and assigned as chicken dressers, packers and helpers at the Cainta Processing Plant. The respondent GMC is a domestic corporation engaged in the production and sale of livestock and poultry, and is a distributor of dressed chicken. While the petitioners' employment as chicken dressers is necessary and desirable in the usual business of the respondent, they were employed on a mere temporary basis, since their employment was limited to a fixed period. As such, they cannot be said to be regular employees, but are merely "contractual employees." Consequently, there was no illegal dismissal when the petitioners' services were terminated by reason of the expiration of their contracts. Lack of notice of termination is of no consequence, because when the contract specifies the period of its duration, it terminates on the expiration of such period. A contract for employment for a definite period terminates by its own term at the end of such period.

9. Ravago vs. Esso Eastern Marine Ltd., G.R. No. 158324, March 14, 2005

Facts: Roberto Ravago was hired by Trans-Global to work as a seaman on board various Esso vessels. On February 13, 1970, Ravago commenced his duty as S/N wiper on board the Esso Bataan under a contract that lasted until February 10, 1971. Thereafter, he was assigned to work in different Esso vessels where he was designated diverse tasks, such as oiler, then assistant engineer. He was employed under a total of 34 separate and unconnected contracts, each for a fixed period, by three different companies, namely, Esso Tankers, Inc. (ETI), EEM and Esso International Shipping (Bahamas) Co., Ltd. (EIS), Singapore Branch. Ravago worked with Esso vessels until August 22, 1992, a period spanning more than 22 years. Shortly after completing his latest contract with EIS, Ravago was granted a vacation leave with pay. One the night, a stray bullet hit Ravago on the left leg while he was waiting for a bus ride in Cubao, Quezon City. He fractured his left proximal tibia and was

Page 30

LABOR RELATIONS
Atty. Jefferson M. Marquez
hospitalized at the Philippine Orthopedic Hospital. As a result of his injury, Ravago's doctor opined that he would not be able to cope with the job of a seaman and suggested that he be given a desk job. Ravago's left leg had become apparently shorter, making him walk with a limp. For this reason, the company physician, Dr. Virginia G. Manzo, found him to have lost his dexterity, making him unfit to work once again as a seaman. Consequently, instead of rehiring Ravago, EIS paid him his Career Employment Incentive Plan (CEIP) and his final tax refund for 1992. After deducting his Social Security System and medical contributions, EIS remitted the net amount of P162,232.65, following Ravago's execution of a Deed of Quitclaim and/or Release. However, Ravago filed a complaint for illegal dismissal with prayer for reinstatement, backwages, damages and attorney's fees against Trans-Global and EIS with the Philippine Overseas Employment Administration Adjudication Office. Respondents denied that Ravago was dismissed without notice and just cause. Rather, his services were no longer engaged in view of the disability he suffered which rendered him unfit to work as a seafarer. They averred that Ravago was a contractual employee and was hired under 34 separate contracts by different companies. In his position paper, Ravago insisted that he was fit to resume pre-injury activities as evidenced by the certification issued by Dr. Marciano Foronda M.D., one of his attending physicians at the Philippine Orthopedic Hospital, that "at present, fracture of tibia has completely healed and patient is fit to resume pre-injury activities anytime." Ravago, likewise, asserted that he was not a mere contractual employee because the respondents regularly and continuously rehired him for 23 years and, for his continuous service, was awarded a CEIP payment upon his termination from employment. The Court of Appeals decided that the employment status of seafarers has been established with finality by the Court's reconsideration of its decision in Millares v. National Labor Relations Commission, wherein it was ruled that seamen are contractual employees. According to the CA, the fact that Ravago was not rehired upon the completion of his contract did not result in his illegal dismissal; hence, he was not entitled to reinstatement or payment of separation pay. Issue: Whether or not the Court of Appeals gravely erred, blatantly disregarded the constitutional mandate on protection to Filipino overseas workers, and countenanced unwarranted discrimination when it ruled that petitioner cannot become a regular employee. Ruling: In a catena of cases, this Court has consistently ruled that seafarers are contractual, not regular, employees. In Brent School, Inc. v. Zamora, the Court ruled that seamen and overseas contract workers are not covered by the term "regular employment" as defined in Article 280 of the Labor Code. As a Filipino seaman, petitioner is governed by the Rules and Regulations Governing Overseas Employment and the said Rules do not provide for separation or termination pay. The Standard Employment Contract governing the Employment of All Filipino Seamen on Board Ocean-Going Vessels of the POEA, particularly in Part I, Sec. C, specifically provides that the contract of seamen shall be for a fixed period. And in no case should the contract of seamen be longer than 12 months. It reads: Section C. Duration of Contract The period of employment shall be for a fixed period but in no case to exceed 12 months and shall be stated in the Crew Contract. Any extension of the Contract period shall be subject to the mutual consent of the parties. Their employment is governed by the contracts they sign every time they are rehired and their employment is terminated when the contract expires. Their employment is contractually fixed for a certain period of time. They fall under the exception of Article 280 whose employment has been fixed for a specific project or undertaking the completion or termination of which has been determined at the time of engagement of the employee or where the work or services to be performed is seasonal in nature and the employment is for the duration of the season.

10. Hacienda Bino/Hortencia Stark vs. Cuenca, G.R. No. 150478, April 15, 2005, citing 2003 Hacienda Fatima Facts: Hortencia L. Starke, herein petitioner, is the owner and operator of the Hacienda Bino. During the off milling season of 1996 he issued an Order or Notice which stated, that all Hacienda Employees who signed in favor of CARP are expressing their desire to get out of employment on their own volition and wherefore, only those who did not sign for CARP will be given employment by the hacienda. Herein respondents are employees of the hacienda performing various works, such as cultivation, planting of cane points, fertilization, watering, weeding, harvesting and loading of harvested sugarcanes to cargo trucks are those who signed in favor of CARP. They allege that they are regular and permanent workers of the hacienda and that they were dismissed without just and lawful cause. They further alleged that they were dismissed because they applied as beneficiaries under the Comprehensive Agrarian Reform Program (CARP) over the land owned by petitioner

Page 31

LABOR RELATIONS
Atty. Jefferson M. Marquez
Starke. Petitioner Starke alleged that in there was little work in the plantation as it was off-season; and so, on account of the seasonal nature of the work, she issued the order giving preference to those who supported the re-classification. She pointed out that when the milling season began, the work was plentiful again and she issued notices to all workers, including the respondents, informing them of the availability of work. However, the respondents refused to report back to work. Issue: Whether or not the respondents are regular employee? Ruling: It held that the ruling in Mercado, Sr. v. NLRC does not operate to abandon the settled doctrine that sugar workers are considered regular and permanent farm workers of a sugar plantation owner, considering that there are facts peculiar in that case which are not present in the case at bar. In the Mercado case, the farm laborers worked only for a definite period for a farm owner since the area of the land was comparatively small, after which they offer their services to other farm owners. In this case, the area of the hacienda, which is 236 hectares, simply does not allow for the respondents to work for a definite period only. The petitioners did not present any evidence that the respondents were required to perform certain phases of agricultural work for a definite period of time. Although the petitioners assert that the respondents made their services available to the neighboring haciendas, the records do not, however, support such assertion. The primary standard for determining regular employment is the reasonable connection between the particular activity performed by the employee in relation to the usual trade or business of the employer. There is no doubt that the respondents were performing work necessary and desirable in the usual trade or business of an employer. Hence, they can properly be classified as regular employees. For respondents to be excluded from those classified as regular employees, it is not enough that they perform work or services that are seasonal in nature. They must have been employed only for the duration of one season. While the records sufficiently show that the respondents work in the hacienda was seasonal in nature, there was, however, no proof that they were hired for the duration of one season only. In fact, the payrolls, submitted in evidence by the petitioners, show that they availed the services of the respondents since 1991. Absent any proof to the contrary, the general rule of regular employment should, therefore, stand. It bears stressing that the employer has the burden of proving the lawfulness of his employees dismissal.

11. Phil Global Communication v. De Vera, G.R. No. 157214, June 7, 2005 Facts: Petitioner Philippine Global Communications, Inc. (PhilCom), is a corporation engaged in the business of communication services and allied activities, while respondent Ricardo De Vera is a physician by profession whom petitioner enlisted to attend to the medical needs of its employees. On 15 May 1981, De Vera, via a letter offered his services to the petitioner, therein proposing his plan of works required of a practitioner in industrial medicine, to include [CERTAIN TASKS]. The parties agreed and formalized respondents proposal in a document denominated as RETAINERSHIP CONTRACTwhich will be for a period of one year subject to renewal, and that respondents retainer fee will be at P4,000.00 a month. Said contract was renewed yearly. The retainership arrangement went on from 1981 to 1994 with changes in the retainers fee. However, for the years 1995 and 1996, renewal of the contract was only made verbally. In December 1996 Philcom, thru a letter bearing on the subject boldly written as TERMINATION RETAINERSHIP CONTRACT, informed De Vera of its decision to discontinue the latters retainers contract with the Company effective at the close of business hours of December 31, 1996 because management has decided that it would be more practical to provide medical services to its employees through accredited hospitals near the company premises. De Vera filed a complaint for illegal dismissal before the National Labor Relations Commission (NLRC), alleging that he had been actually employed by Philcom as its company physician since 1981 and was dismissed without due process. He averred that he was designated as a company physician on retainer basis for reasons allegedly known only to Philcom. He likewise professed that since he was not conversant with labor laws, he did not give much attention to the designation as anyway he worked on a full-time basis and was paid a basic monthly salary plus fringe benefits, like any other regular employees of Philcom. Labor Arbiter dismissed De Veras complaint for lack of merit, on the rationale that as a retained physician under a valid contract mutually agreed upon by the parties, De Vera was an independent contractor and that he was not dismissed but rather his contract with [PHILCOM] ended when said contract was not renewed after December 31, 1996. NLRC reversed on a finding that - De Vera is Philcoms regular employee and accordingly directed the company to reinstate him to his former position without loss of seniority rights and privileges and with full backwages from the date of his dismissal until actual reinstatement. Philcom then went to the Court of Appeals which rendered a decision, modifying that of the NLRC by deleting the award of traveling allowance,

Page 32

LABOR RELATIONS
Atty. Jefferson M. Marquez
and ordering payment of separation pay to De Vera in lieu of reinstatement. Issue: Whether the case is one of a LEGITIMATE JOB CONTRACTING AGREEMENT or an EMPLOYER-EMPLOYEE RELATIONSHIP. Ruling: The primordial issue of whether an employer-employee relationship exists between petitioner and respondent, the existence of which is, in itself, a question of fact well within the province of the NLRC. Nonetheless, given the reality that the NLRCs findings are at odds with those of the labor arbiter, the Court is constrained to look deeper into the attendant circumstances obtaining in this case, as appearing on record. APPLYING THE FOUR-FOLD TEST TO THIS CASE, it was respondent himself who sets the parameters of what his duties would be in offering his services to petitioner, as borne by no less than his own letter which was substantially the basis of the labor arbiters finding that there existed no employer-employee relationship between petitioner and respondent in addition to the following factual settings: [AS TO SELECTION OR HIRING] The fact that the complainant was not considered an employee was recognized by the complainant himself in a signed LETTER to the respondent. The tenor of this letter indicates that the complainant was proposing to extend his time with the respondent and seeking additional compensation for said extension. This shows that the respondent PHILCOM did not have control over the schedule of the complainant as it [is] the complainant who is proposing his own schedule and asking to be paid for the same. This is proof that the complainant understood that his relationship with the respondent PHILCOM was a retained physician and not as an employee. If he were an employee he could not negotiate as to his hours of work. The complainant is a Doctor of Medicine, and presumably, a well-educated person. The retainer fee is indicated on the RETAINERSHIP CONTRACT, and the duration of the contract for one year is also clearly indicated in paragraph 5 of the Retainership Contract. The complainant cannot claim that he was unaware that the contract was good only for one year, as he signed the same without any objections. The complainant also accepted its renewal every year thereafter until 1994. As a literate person and educated person, the complainant cannot claim that he does not know what contract he signed and that it was renewed on a year to year basis. [AS TO WAGES] from the time he started to work with petitioner, he never was included in its payroll; was never deducted any contribution for remittance to the Social Security System (SSS); and was in fact subjected by petitioner to the ten (10%) percent withholding tax for his professional fee, in accordance with the National Internal Revenue Code, matters which are simply inconsistent with an employer-employee relationship. But he never raised those issues. An ordinary employee would consider the SSS payments important and thus make sure they would be paid. The complainant never bothered to ask the respondent to remit his SSS contributions. This clearly shows that the complainant never considered himself an employee of PHILCOM and thus, respondent need not remit anything to the SSS in favor of the complainant. Clearly, the elements of an employer-employee relationship are wanting in this case. the records are replete with evidence showing that respondent had to bill petitioner for his monthly professional fees. It simply runs against the grain of common experience to imagine that an ordinary employee has yet to bill his employer to receive his salary. [AS TO DISMISSAL] the power to terminate the parties relationship was mutually vested on both. Either may terminate the arrangement at will, with or without cause. Finally, remarkably absent from the parties arrangement is the ELEMENT OF CONTROL, whereby the employer has reserved the right to control the employee not only as to the result of the work done but also as to the means and methods by which the same is to be accomplished. Here, petitioner had no control over the means and methods by which respondent went about performing his work at the company premises. He could even embark in the private practice of his profession, not to mention the fact that respondents work hours and the additional compensation therefor were negotiated upon by the parties. In fine, the parties themselves practically agreed on every terms and conditions of respondents engagement, which thereby negates the element of control in their relationship. For sure, respondent has never cited even a single instance when petitioner interfered with his work.

Page 33

LABOR RELATIONS
Atty. Jefferson M. Marquez

ARTICLE 280 OF THE LABOR CODE (sic) provides: The provisions of written agreement to the contrary notwithstanding and regardless of the oral agreements of the parties, an employment shall be deemed to be REGULAR where the employee has been engaged to perform in the usual business or trade of the employer, EXCEPT where the employment has been fixed for a specific project or undertaking the completion or termination of which has been determined at the time of the engagement of the employee or where the work or services to be performed is seasonal in nature and the employment is for the duration of the season. An employment shall be deemed to be CASUAL if it is not covered by the preceding paragraph: Provided, That, any employee who has rendered at least one (1) year of service, whether such is continuous or broken, shall be considered a REGULAR with respect to the activity in which he is employed and his employment shall continue while such activity exists. Parenthetically, the position of company physician, in the case of petitioner, is usually necessary and desirable because the need for medical attention of employees cannot be foreseen, hence, it is necessary to have a physician at hand. In fact, the importance and desirability of a physician in a company premises is recognized by ART. 157 OF THE LABOR CODE, which requires the presence of a physician depending on the number of employees and in the case at bench, in petitioners case, as found by public respondent, petitioner employs more than 500 employees. The appellate courts premise that REGULAR EMPLOYEES are those who perform activities which are desirable and necessary for the business of the employer is not determinative in this case. For, we take it that any agreement may provide that one party shall render services for and in behalf of another, no matter how necessary for the latters business, even without being hired as an employee. This set-up is precisely true in the case of an INDEPENDENT CONTRACTORSHIP as well as in an AGENCY AGREEMENT. ARTICLE 280 OF THE LABOR CODE, is not the yardstick for determining the existence of an employment relationship. As it is, the provision merely distinguishes between two (2) kinds of employees, i.e., regular and casual. It does not apply where, as here, the very existence of an employment relationship is in dispute. ART. 157 OF THE LABOR CODE while it is true that the provision requires employers to engage the services of medical practitioners in certain establishments depending on the number of their employees, nothing is there in the law which says that medical practitioners so engaged be actually hired as employees, adding that the law, as written, only requires the employer to retain, not employ, a part-time physician who needed to stay in the premises of the non-hazardous workplace for two (2) hours. what applies here is the last paragraph of Article 157 which, to stress, provides that the employer may engage the services of a physician and dentist on retained basis, subject to such regulations as the Secretary of Labor may prescribe. The successive retainership agreements of the parties definitely hue to the very statutory provision relied upon by respondent. ARTICLE 157 OF THE LABOR CODE clearly and unequivocally allows employers in non-hazardous establishments to engage on retained basis the service of a dentist or physician. Nowhere does the law provide that the physician or dentist so engaged thereby becomes a regular employee. The very phrase that they may be engaged on retained basis, revolts against the idea that this engagement gives rise to an employer-employee relationship. With the recognition of the fact that petitioner consistently engaged the services of respondent on a retainer basis, as shown by their various retainership contracts, so can petitioner put an end, with or without cause, to their retainership agreement as therein provided. [MOOT AND ACADEMIC] We note, however, that - even as the contracts entered into by the parties invariably provide for a 60-DAY NOTICE REQUIREMENT prior to termination, the same was not complied with by petitioner when it terminated the verbally-renewed retainership agreement. Be that as it may, the record shows, and this is admitted by both parties, that execution of the NLRC decision had already been made at the

Page 34

LABOR RELATIONS
Atty. Jefferson M. Marquez
NLRC despite the pendency of the present recourse. For sure, accounts of petitioner had already been garnished and released to respondent despite the previous Status Quo Order[29] issued by this Court. To all intents and purposes, therefore, the 60-day notice requirement has become moot and academic if not waived by the respondent himself. WHEREFORE, the petition is GRANTED and the challenged decision of the Court of Appeals REVERSED and SET ASIDE. The 21 December 1998 decision of the labor arbiter is REINSTATED.

12. Integrated Contractor and Plumbing Works, Inc. vs. National Labor Relations Commission and Glen Solon, G.R. No. 152427. August 9, 2005 Facts: Petitioner is a plumbing contractor. Its business depends on the number and frequency of the projects it is able to contract with its clients. On February 23, 1998, while private respondent was about to log out from work, he was informed by the warehouseman that the main office had instructed them to tell him it was his last day of work as he had been terminated. When private respondent went to the petitioner's office on February 24, 1998 to verify his status, he found out that indeed, he had been terminated. He filed a complaint alleging that he was illegally dismissed without just cause and without due process. the Labor Arbiter ruled that private respondent was a regular employee and could only be removed for cause. Petitioner was ordered to reinstate private respondent to his former position with full backwages from the time his salary was withheld until his actual reinstatement, and pay him service incentive leave pay, and 13th month pay for three years. Petitioner further filed a motion for reconsideration which was denied. It filed an appeal before the CA but it was subsequently dismissed for lack of merit. Issue: Whether the respondent is a project employee of the petitioner or a regular employee. HELD No. He was considered as a regular employee. We held in Tomas Lao Construction v. NLRC 12 that the principal test in determining whether an employee is a "project employee" or "regular employee," is, whether he is assigned to carry out a "specific project or undertaking," the duration (and scope) of which are specified at the time the employee is engaged in the project. 13 "Project" refers to a particular job or undertaking that is within the regular or usual business of the employer, but which is distinct and separate and identifiable from the undertakings of the company. Such job or undertaking begins and ends at determined or determinable times. A review of private respondent's work assignments patently showed he belonged to a work pool tapped from where workers are and assigned whenever their services were needed. In a work pool, the workers do not receive salaries and are free to seek other employment during temporary breaks in the business. They are like regular seasonal workers insofar as the effect of temporary cessation of work is concerned. This arrangement is beneficial to both the employer and employee for it prevents the unjust situation of "coddling labor at the expense of capital" and at the same time enables the workers to attain the status of regular employees. 15 Nonetheless, the pattern of re-hiring and the recurring need for his services are sufficient evidence of the necessity and indispensability of such services to petitioner's business or trade. In Maraguinot, Jr. v. NLRC we ruled that once a project or work pool employee has been: (1) continuously, as opposed to intermittently, re-hired by the same employer for the same tasks or nature of tasks; and (2) these tasks are vital, necessary and indispensable to the usual business or trade of the employer, then the employee must be deemed a regular employee. The test to determine whether employment is regular or not is the reasonable connection between the particular activity performed by the employee in relation to the usual business or trade of the employer. Also, if the employee has been performing the job for at least one year, even if the performance is not continuous or merely intermittent, the law deems the repeated and continuing need for its performance as sufficient evidence of the necessity, if not indispensability of that activity to the business. Thus, we held that where the employment of project employees is extended long after the supposed project has been finished, the employees are removed from the scope of project employees and are considered regular employees. While length of time may not be the controlling test for project employment, it is vital in determining if the employee was hired for a specific undertaking or tasked to perform functions vital, necessary and indispensable to the usual business or trade of the employer. Here, private

Page 35

LABOR RELATIONS
Atty. Jefferson M. Marquez
respondent had been a project employee several times over. His employment ceased to be coterminous with specific projects when he was repeatedly re-hired due to the demands of petitioner's business. 20 Where from the circumstances it is apparent that periods have been imposed to preclude the acquisition of tenurial security by the employee, they should be struck down as contrary to public policy, morals, good customs or public order. Further, Policy Instructions No. 20 requires employers to submit a report of an employee's termination to the nearest public employment office every time his employment was terminated due to a completion of a project. The failure of the employer to file termination reports is an indication that the employee is not a project employee. 22 Department Order No. 19 superseding Policy Instructions No. 20 also expressly provides that the report of termination is one of the indications of project employment. 23 In the case at bar, there was only one list of terminated workers submitted to the Department of Labor and Employment. 24 If private respondent was a project employee, petitioner should have submitted a termination report for every completion of a project to which the former was assigned.

13. Lacuesta vs. Ateneo de Manila, G.R. No. 152777, December 9, 2005 Facts: Respondent Ateneo de Manila University (Ateneo) hired, on a contractual basis, petitioner Lolita R. Lacuesta as a part-time lecturer in its English Department for the second semester of school year 1988-1989. She was re-hired, still on a contractual basis, for the first and second semesters of school year 1989-1990. On July 13, 1990, the petitioner was first appointed as full-time instructor on probation, in the same department effective June 1, 1990 until March 31, 1991. Thereafter, her contract as faculty on probation was renewed effective April 1, 1991 until March 31, 1992. She was again hired for a third year effective April 1, 1992 until March 31, 1993. During these three years she was on probation status. Respondent Dr. Leovino Ma. Garcia, Dean of Ateneos Graduate School and College of Arts and Sciences, notified petitioner that her contract would no longer be renewed because she did not integrate well with the English Department.Petitioner filed a complaint for illegal dismissal with prayer for reinstatement, back wages, and moral and exemplary damages. She contends that Articles 280 and 281 of the Labor Code, not the Manual of Regulations for Private Schools, is the applicable law to determine whether or not an employee in an educational institution has acquired regular or permanent status. She argues that (1) under Article 281, probationary employment shall not exceed six (6) months from date of employment unless a longer period had been stipulated by an apprenticeship agreement; (2) under Article 280, if the apprenticeship agreement stipulates a period longer than one year and the employee rendered at least one year of service, whether continuous or broken, the employee shall be considered as regular employee with respect to the activity in which he is employed while such activity exists; and (3) it is with more reason that petitioner be made regular since she had rendered services as part-time and full-time English teacher for four and a half years, services which are necessary and desirable to the usual business of Ateneo. Issues: 1. Whether or not the Court of Appeals erred in ruling that it is the Manual of Regulations For Private Schools, not the Labor Code, that determines the acquisition of regular or permanent status of faculty members in an educational institution; 2. Whether or not after completing the three-year probation with an above-average performance, petitioner already acquired permanent status. Rulings: 1. The Manual of Regulations for Private Schools, and not the Labor Code, determines whether or not a faculty member in an educational institution has attained regular or permanent status. Under Policy Instructions No. 11 issued by the Department of Labor and Employment, the probationary employment of professors, instructors and teachers shall be subject to the standards established by the Department of Education and Culture. Section 93of the 1992 Manual of Regulations for Private Schools provides that full-time teachers who have satisfactorily completed their probationary period shall be considered regular or permanent. Moreover, for those teaching in the tertiary level, the probationary period shall not be more than six consecutive regular semesters of satisfactory service.The requisites to acquire permanent employment, or security of tenure, are (1) the teacher is a full-time teacher; (2) the teacher must have rendered three consecutive years of service; and (3) such service must have been satisfactory. 2. A part-time teacher cannot acquire permanent status. Only when one has served as a full-time teacher can he acquire permanent or regular status. The petitioner was a part-time lecturer before she was appointed as a full-time instructor on probation. As a part-time lecturer, her employment as such had ended when her contract expired. Thus, the three semesters she served as part-time lecturer could not be credited to her in computing the number of years she has served to qualify her for permanent status. And completing the probation period does not automatically qualify her to become a permanent employee of the university. Petitioner could only qualify to become a permanent employee

Page 36

LABOR RELATIONS
Atty. Jefferson M. Marquez
upon fulfilling the reasonable standards for permanent employment as faculty member. Consistent with academic freedom and constitutional autonomy, an institution of higher learning has the prerogative to provide standards for its teachers and determine whether these standards have been met.At the end of the probation period, the decision to re-hire an employee on probation, belongs to the university as the employer alone.

14. Poseidon Fishing/Terry De Jesus v. NLRC, G.R. No. 168052, February 20. 2006 Facts: Petitioner Poseidon Fishing is a fishing company engaged in the deep-sea fishing industry with Terry de Jesus as the manager. Jimmy S. Estoquia was employed as Chief Mate in January 1988 and after five years. The contract with Eustoqia per the "Kasunduan", there was a provision stating that he was being employed only on a por viaje basis and that his employment would be terminated at the end of the trip for which he was being hired. He was promoted to Boat Captain but was later demoted to Radio Operator. As a Radio Operator, he monitored the daily activities in their office and recorded in the duty logbook the names of the callers and time of their calls. On 3 July 2000, Estoquia failed to record a 7:25 a.m. call in one of the logbooks. When he reviewed the two logbooks, he noticed that he was not able to record the said call in one of the logbooks so he immediately recorded the 7:25 a.m. call after the 7:30 a.m. entry. In the morning of 4 July 2000, petitioner detected the error in the entry in the logbook. Estoquia was asked to prepare an incident report to explain the reason for the said oversight. On the same day, Poseidons secretary summoned Estoquia to get his separation pay Estoquia filed a complaint for illegal dismissal with the Labor Arbiter. Poseidon and Terry de Jesus asserted that Estoquia was a contractual or a casual employee employed only on a"por viaje" or per trip basis and that his employment would be terminated at the end of the trip for which he was being hired. Issue:S: WON Eustoqia was a regular employee WON deep -sea fishing is a seasonal industry WON Eustoqia was illegally dismissed Ruling: Yes, Eustoquia was a regular employee. Article 280 draws a line between regular and casual employment. The provision enumerates two (2) kinds of employees, the regular employees and the casual employees. The regular employees consist of the following: 1) those engaged to perform activities which are usually necessary or desirable in the usual business or trade of the employer; and 2) those who have rendered at least one year of service whether such service is continuous or broken. In a span of 12 years, Eustoquia worked for petitioner first as a Chief Mate, then Boat Captain, and later as Radio Operator. His job was directly related to the deep-sea fishing business of petitioner Poseidon. His work was, therefore, necessary and important to the business of his employer. Such being the scenario involved, Eustoquia is considered a regular employee. There is nothing in the contract that says complainant is a casual, seasonal or a project worker. The date July 1 to 31, 1998 under the heading "Pagdating" had been placed there merely to indicate the possible date of arrival of the vessel and is not an indication of the status of employment of the crew of the vessel. The test to determine whether employment is regular or not is the reasonable connection between the particular activity performed by the employee in relation to the usual business or trade of the employer. And, if the employee has been performing the job for at least one year, even if the performance is not continuous or merely intermittent, the law deems the repeated and continuing need for its performance as sufficient evidence of the necessity, if not indispensability of that activity to the business. In the case at bar, the act of hiring and re-hiring in various capacities is a mere gambit employed by petitioner to thwart the tenurial protection of private respondent. Such pattern of re-hiring and the recurring need for his services are testament to the necessity and indispensability of such services to petitioners business or trade. No, the activity of catching fish is a continuous process and could hardly be considered as seasonal in nature.

Page 37

LABOR RELATIONS
Atty. Jefferson M. Marquez
Project employees is defined as those workers hired: (1) for a specific project or undertaking, and (2) the completion or termination of such project has been determined at the time of the engagement of the employee. The principal test for determining whether particular employees are "project employees" as distinguished from "regular employees," is whether or not the "project employees" were assigned to carry out a "specific project or undertaking," the duration and scope of which were specified at the time the employees were engaged for that project. In this case, Eustoquia was never informed that he will be assigned to a "specific project or undertaking at the time of their engagement. Once a project or work pool employee has been: (1) continuously, as opposed to intermittently, re-hired by the same employer for the same tasks or nature of tasks; and (2) these tasks are vital, necessary and indispensable to the usual business or trade of the employer, then the employee must be deemed a regular employee. Eustoquias functions were usually necessary or desirable in the usual business or trade of petitioner fishing company and he was hired continuously for 12 years for the same nature of tasks. Hence, he was of regular employee. Yes, Eustoqia was illegally dismissed. There is no sufficient evidence on record to prove Eustoqias negligence, gross or simple, in the performance of his duties to warrant a reduction of six months salary and be summarily dismissed. At best, the simple negligence is punishable only with admonition or suspension for a day or two. His dismissal was without valid cause and where illegal dismissal is proven, the worker is entitled to back wages and other similar benefits without deductions or conditions.

15. Abesco Construction vs. Ramirez, G.R. No. 141168, April 10, 2006 Facts: Petitioner company was engaged in a construction business where respondents where hired on different dates from 1976 to 1992 either as laborers, road roller operators, painters or drivers. Respondents filed two separate complaints for illegal dismissal against the company and its General Manager before Labor Arbiter. The complaints included claims for non-payment of 13th month pay, 5-days service incentive leave pay, premium pay for holidays, rest days, and moral and exemplary damages. Petitioners alleged that respondents were only project employees whose employment was coterminous with the project they are assigned. They were not regular employees, who would enjoy security of tenure and entitlement of separation pay. LA declared judgment declaring respondents as regular employees belonged to a workpool and where hired and rehired over a period of 18 years and petitioners are guilty of illegal dismissal. Petitioner appealed to NLRC which affirmed LAs decision. Petitioner then file petition for review to CA alleging that they were not guilty of illegal dismissal since respondents services were merely put on hold until the resumption of their business operations. CA dismissed the petition for petitioner is barred from raising a new defense at this stage of the case. Issue: Whether or not respondents are project employees or regular employees. Ruling: Respondents are regular employees. Duration as well as particular work/service to be performed must be defined in an Employment Agreement and is made clear to the employees at the time of hiring. Petitioners failed to comply with this requirement. Petitioners inconsistent and conflicting position on their true relation with the respondents made it all the more evident that the latter were indeed their regular employees. Petitioner failed to adhere the two-notice rule: (1) a notice informing them of the particular acts for which they are being dismissed and; (2) a notice advising them of the decision to terminate the employment. Respondents were never given such notices. Petition is denied.

16. Cebu Metal Corp., vs. Saliling, G.R. No. 154463, September 5, 2006

Page 38

LABOR RELATIONS
Atty. Jefferson M. Marquez
Facts: Cebu Metal Corporation is a corporation engaged in buying and selling of scrap iron. In the Bacolod branch it has (3) regular employees holding such positions as Officer-in-Charge, a scaler, and a yardman, whose salaries are paid directly by its main office in Cebu. The complainants, Gregorio Saliling, Elias Bolido, Manuel Alquiza, Benjie Amparado are the one who undertakes pakiao work in the unloading of scrap iron. The Bacolod buying station is mainly a stockyard where scrap metal delivered by its suppliers are stockpiled. The supply of scrap metal is not steady as it depends upon many factors, such as availability of supplies, price, competition and demand among others. There are weeks were there are no delivery while there are weeks were a quite of number of trucks are delivered to the stockyard. The arrivals of these trucks and the deliveries of scrap metal iron are not regular and the schedules of deliveries to the stockyard are not known before hand by the respondent Cebu Metal Corporation. These trucks have their own driver and truck boys employed by the different suppliers. Sometimes, these trucks do not have any truck boys, and in these instances, the corporation hires the services of people for the unloading of the scrap metal from these trucks. It is for this reason that the unloaders hired by the respondent to unload are basically seasonal workers. They are hired whenever there are trucks of suppliers do not have any accompanying truck boys. Whoever is available and whoever are willing to help unload on a particular occasion are hired to unload. Usually, there is a leader for a particular group who is tasked to unload the scrap metal from a particular truck. It is this leader who distributes the individual takes of each member of the particular group unloading the scrap metal from a particular work The complainants maintained that they are hired by Cebu Metal Corparation as employees and filed on January 10, 1997 a complaint with the regional arbitration in Bacolod City for underpayment of wages and non-payment of the following benefits 1. 13th month pay; 2. holiday pay; 3. service incentive leave pay. On March 6, 1998 includes the claim for illegal dismissal because they were dismissed after the filing of the complaint. The Labor Arbiter rendered a decision in favor of the complainants. Aggrieved, Cebu Metal Corporation filed an appeal with the NLRC. The NLRC reversed and set aside the decision of the Labor Arbiter and held that the complainants were not regular employees, thus, they could not have been illegally dismissed. The order of the reversal was based on the Commissions finding that the petty cash vouchers submitted by Cebu Metal Corporation confirmed the fact that unloaders were paid on pakiao or task basis at Php 15.00 per metric ton. The Commission further rationalized that with the irregular nature of the work involved in the stoppage and resumption of which depended solely on the availability or supply of scrap metal, it necessarily follows that after the job of unloading was completed and unloaders are paid the contract price, the latters working relationship with Cebu Metal Corporation legally ended. They were then free to offer their services to others. The complainants challenged the decision of the NLRC with the Court of Appeals, and it rendered the decision annulling the decision of the NLRC and reinstated the decision made by the Labor Arbiter. Hence, this petition. Issue: Whether or not the complainant respondents are regular employees. Ruling: The Supreme Court ruled there can be no illegal dismissal to speak of. Besides, the complainants cannot claim regularity in the hiring every time a truck comes loaded with scrap metal. This is confirmed in the Petty Cash Vouchers which are in the names of different leaders who are apportion the amount earned among its members. And, quite telling is the fact that not every truck delivery of scrap metal requires the services of respondent complainants when particular truck is accompanied by its own unloader. And whenever required, respondent complainants were not always the ones contracted to undertake the unloading of the trucks since the work was offered to whomever were available at a given time. It should be remembered that the Philippine Constitution, while inexorably committed towards the protection of the working class from exploitation and unfair treatment, nevertheless mandates the policy of social justice so as to strike a balance between an avowed predilection for labor, on the one hand, and the maintenance of the legal rights of capital, the proverbial hen that lays the golden egg, on the other. Indeed we should not be mindful of the legal norm that justice is in every case for the deserving, to be dispensed with in the light of established facts, the applicable law, an existing jurisprudence.

17. Liganza v. RBL Shipyard Corp., G.R. No. 159682, October 17, 2006 Facts: After working as a carpenter for respondent since August 1991, petitioner's employment was terminated on 30 October 1999. Such event prompted petitioner to file a complaint for illegal dismissal, alleging that on said date he was verbally informed that he was already terminated from employment and barred from entering the premises. On the same occasion, he was told to look for another job. Thus, he claimed that he was unceremoniously terminated from employment without any valid or authorized cause. On the other hand, respondent insisted that petitioner was a mere project employee who was terminated upon completion of the project for which he was hired.

Page 39

LABOR RELATIONS
Atty. Jefferson M. Marquez
Issue: 1. Whether or not the Court of Appeals seriously erred in concluding that "petitioner is a project employee, not a regular employee? Ruling: Petitioner is a regular employee. A project employee is one whose "employment has been fixed for a specific project or undertaking, the completion or termination of which has been determined at the time of the engagement of the employee or where the work or service to be performed is seasonal in nature and the employment is for the duration of the season." Before an employee hired on a per project basis can be dismissed, a report must be made to the nearest employment office of the termination of the services of the workers every time it completed a project, pursuant to Policy Instruction No. 20. While the appropriate evidence to show that a person is a project employee is the employment contract specifying the project and the duration of such project, the existence of such contract is not always conclusive of the nature of one's employment. In the instant case, respondent seeks to prove the status of petitioner's employment through four (4) employment contracts covering a period of only two (2) years to declare petitioner as a project employee. Respondent failed to present the contracts purportedly covering petitioner's employment from 1991 to July 1997, spanning six (6) years of the total eight (8) years of his employment. To explain its failure in this regard, respondent claims that the records and contracts covering said period were destroyed by rains and flashfloods that hit the company's office. Such contention is clearly unconvincing. Even assuming that petitioner is a project employee, respondent failed to prove that his termination was for a just and valid cause. While it is true that the employment contract states that the contract ends upon a specific date, or upon completion of the project, respondent failed to prove that the last project was indeed completed so as to justify petitioner's termination from employment. In termination cases, the burden of proof rests on the employer to show that the dismissal is for a just cause. Respondent could have easily proved that the project or phase for which petitioner was hired has already been completed. A certificate from the owner of the vessel serviced by the company, pictures perhaps, of the work accomplished, and other proof of completion could have been procured by respondent. However, all that we have is respondent's self-serving assertion that the project has been completed. This Court has held that an employment ceases to be co-terminous with specific projects when the employee is continuously rehired due to the demands of employer's business and re-engaged for many more projects without interruption. In Maraguinot, Jr. v. NLRC (Second Division), the Court ruled that "once a project or work pool employee has been: (1) continuously, as opposed to intermittently, rehired by the same employer for the same tasks or nature of tasks; and (2) these tasks are vital, necessary and indispensable to the usual business or trade of the employer, then the employee must be deemed a regular employee, pursuant to Article 280 of the Labor Code and jurisprudence." Petitioner, as carpenter, was tasked to "make and repair cabinet, flooring, quarters, ceiling, windows, doors, kitchen and other parts of the vessel that needs to be repaired." As such, petitioner's work was necessary or desirable to respondent's business. Assuming, without granting that petitioner was initially hired for specific projects or undertakings, the repeated re-hiring and continuing need for his services for over eight (8) years have undeniably made him a regular employee. Respondent capitalizes on our ruling in D.M. Consunji, Inc. v. NLRC27 which reiterates the rule that the length of service of a project employee is not the controlling test of employment tenure but whether or not "the employment has been fixed for a specific project or undertaking the completion or termination of which has been determined at the time of the engagement of the employee." Surely, length of time is not the controlling test for project employment. Nevertheless, it is vital in determining if the employee was hired for a specific undertaking or tasked to perform functions vital, necessary and indispensable to the usual business or trade of the employer. In the case at bar, respondent had been a project employee several times over. His employment ceased to be coterminous with specific projects when he was repeatedly re-hired due to the demands of petitioner's business. Where from the circumstances it is apparent that periods have been imposed to preclude the acquisition of tenurial security by the employee, they should be struck down as contrary to public policy, morals, good customs or public order. All considered, there are serious doubts in the evidence on record that petitioner is a project employee, or that he was terminated for just cause. These doubts shall be resolved in favor of petitioner, in line with the policy of the law to afford protection to labor and construe doubts in favor of labor.

Page 40

LABOR RELATIONS
Atty. Jefferson M. Marquez
WHEREFORE, the petition is GRANTED and the challenged decision of the Court of Appeals is REVERSED and SET ASIDE. The 22 February 2001 Decision of the Labor Arbiter is REINSTATED.

18. Fabeza v. San Miguel Corp., G.R. No. 150658, February 9, 2007

Facts: Petitioners were hired by respondent San Miguel Corporation (SMC) as "Relief Salesmen" for the Greater Manila Area (GMA) under separate but almost similarly worded "Contracts of Employment With Fixed Period." After having entered into successive contracts of the same nature with SMC, the services of petitioners, as well as de Lara and Alovera, were terminated after SMC no longer agreed to forge another contract with them. SMC and its co-respondent Arman Hicarte, who was its Human Resources Manager, claimed that the hiring of petitioners was not intended to be permanent, as the same was merely occasioned by the need to fill in a vacuum arising from SMCs gradual transition to a new system of selling and delivering its products. While some of the qualified regular salesmen were readily upgraded to the position of Accounts Specialist, respondents claimed that SMC still had to sell its beer products using the conventional routing system during the transition stage, thus giving rise to the need for temporary employees; and the members of the regular Route Crew then existing were required to undergo a training program to determine whether they possessed or could be trained for the necessary attitude and aptitude required of an Accounts Specialist, hence, the hiring of petitioners and others for a fixed period, co-terminus with the completion of the transition period and Training Program for all prospective Accounts Specialists The petitioners alleged that they were illegally dismissed by SMC. The Labor Arbiter held petitioners were illegally dismissed. The Decision of the Labor Arbiter was affirmed on appeal by the NLRC. Court of Appeals reversed the decision of the Labor Arbiter and of the NLRC Issue: WON the termination of the petitioners is valid. Ruling: Article 280 of the Labor Code defines regular employment as follows: ART. 280. Regular and casual employment. The provisions of written agreement to the contrary notwithstanding and regardless of the oral agreement of the parties, an employment shall be deemed to be regular where the employee has been engaged to perform activities which are usually necessary or desirable in the usual business or trade of the employer, except where the employment has been fixed for a specific project or undertaking the completion or termination of which has been determined at the time of the engagement of the employee or where the work or services to be performed is seasonal in nature and the employment is for the duration of the season. In Pure Foods Corp. v. NLRC, Supreme Court held that under the above-quoted provision, there are two kinds of regular employees, namely: (1) those who are engaged to perform activities which are necessary or desirable in the usual business or trade of the employer, and (2) those casual employees who have rendered at least one year of service, whether continuous or broken, with respect to the activity in which they are employed. In Brent School, Inc. v. Zamora, the Supreme Court laid out that Article 280 of the Labor Code appears to prevent circumvention of the employees right to be secure in his tenure, the clause in said article indiscriminately and completely ruling out all written or oral agreements conflicting with the concept of regular employment as defined therein should be construed to refer to the substantive evil that the Code itself has singled out: agreements entered into precisely to circumvent security of tenure. It should have no application to instances where a fixed period of employment was agreed upon knowingly and voluntarily by the parties, without any force, duress or improper pressure being brought to bear upon the employee and absent any other circumstances vitiating his consent, or where it satisfactorily appears that the employer and employee dealt with each other on more or less equal terms with no moral dominance whatever being exercised by the former over the latter.

Page 41

LABOR RELATIONS
Atty. Jefferson M. Marquez
Since respondents attribute the termination of petitioners employment to the expiration of their respective contracts, a determination of whether petitioners were hired as project or seasonal employees, or as fixed-term employees without any force, duress or improper pressure having been exerted against them is in order. If petitioners fall under any of these categories, then indeed their termination follows from the expiration of their contracts. The NLRC had sufficient basis to believe that the shift of SMC to the Pre-Selling System was not the real basis for the forging of fixed-term contracts of employment with petitioners and that the periods were fixed only as a means to preclude petitioners from acquiring security of tenure. That petitioners themselves insisted on the one-year fixed-term is not even alleged by respondents. In fact, the sustained desire of each of the petitioners to enter into another employment contract upon the termination of the earlier ones clearly indicates their interest in continuing to work for SMC. Moreover, respondents have not established that the engagement of petitioners services, which is not in the nature of a project employment, required a definite date of termination as a sine qua non.

19. Soriano vs. NLRC, G.R. No. 165594, April 23, 2007, citing 2005 Filipina Pre-fabricated Bldg. System (Filisystem) Facts: Petitioner and certain individuals namely Sergio Benjamin (Benjamin), Maximino Gonzales (Gonzales), and Noel Apostol (Apostol) were employed by the respondent as Switchman Helpers in its Tondo Exchange Office (TEO). After participating in several trainings and seminars, petitioner, Benjamin, and Gonzales were promoted as Switchmen. Apostol, on the other hand, was elevated to the position of Frameman. One of their duties as Switchmen and Frameman was the manual operation and maintenance of the Electronic Mechanical Device (EMD) of the TEO. In November 1995, respondent PLDT implemented a company-wide redundancy program. Subsequently, the respondent PLDT gave separate letters dated 15 July 1996 to petitioner, Benjamin, Gonzales, and Apostol informing them that their respective positions were deemed redundant due to the above-cited reasons and that their services will be terminated on 16 August 1996.10 They requested the respondent PLDT for transfer to some vacant positions but their requests were denied since all positions were already filled up. Hence, on 16 August 1996, respondent PLDT dismissed the four from employment. Ruling: The Labor Arbiter, the NLRC, and the Court of Appeals all found that substantial evidence supports the absence of illegal dismissal in the present case. Article 283 of the Labor Code provides that an employer may dismiss from work an employee by reason of redundancy. The same provision also states the procedural requirements for the validity of the dismissal. It is clear that the foregoing documentary evidence constituted substantial evidence to support the findings of Labor Arbiter Lustria and the NLRC that petitioners employment was terminated by respondent PLDT due to a valid or legal redundancy program since substantial evidence merely refers to that amount of evidence which a reasonable mind might accept as adequate to support a conclusion. The records show that respondent PLDT had sufficiently established the existence of redundancy in the position of Switchman. It is evident from the foregoing facts that respondent PLDTs utilization of high technology equipment in its operation such as computers and digital switches necessarily resulted in the reduction of the demand for the services of a Switchman since computers and digital switches can aptly perform the function of several Switchmen. Indubitably, the position of Switchman has become redundant. As to whether Lazam was competent to testify on the effects of respondent PLDTs adoption of new technology vis--vis the petitioners position of Switchman, the records show that Lazam was highly qualified to do so. He is a licensed electrical engineer and has been employed by the respondent PLDT since 1971. He was a Senior Manager for Switching Division in several offices of the respondent PLDT, and had attended multiple training programs on Electronic Switching Systems in progressive countries. He was also a training instructor of Switchmen in the respondents office. The fact that respondent PLDT hired contractual employees after implementing its redundancy program does not necessarily negate the existence of redundancy. As amply stated by the respondent PLDT, such hiring was intended solely for winding up operations using the old system. Since the respondent PLDT determined that petitioners services are no longer necessary either as a Switchman or in any other position, and such determination was made in good faith and in furtherance of its business interest, the petitioners contention that he should be the last switchman to be laid-off by reason of his qualifications and outstanding work must fail. WHEREFORE, the petition is DENIED. SO ORDERED.

20. Caseres vs. Universal Robina Sugar Milling Corp., et al., G.R. No. 159343, September 28, 2007

Page 42

LABOR RELATIONS
Atty. Jefferson M. Marquez
Facts: Universal Robina Sugar Milling Corporation (respondent) is a corporation engaged in the cane sugar milling business. Pedy Caseres (petitioner Caseres) started working for respondent in 1989, while Andito Pael (petitioner Pael) in 1993. At the start of their respective employments, they were made to sign a Contract of Employment for Specific Project or Undertaking. Petitioners' contracts were renewed from time to time; until May 1999 when they were informed that their contracts will not be renewed anymore. Petitioners filed a complaint for illegal dismissal, regularization, incentive leave pay, 13th month pay, damages and attorneys fees. Issue: Whether or not the petitioners are seasonal/project/term employees and not regular employees of respondents Ruling: Article 280 of the Labor Code provides: ART. 280. Regular and Casual Employees. The provision of written agreement to the contrary notwithstanding and regardless of the oral agreement of the parties, an employment shall be deemed to be regular where the employee has been engaged to perform activities which are usually necessary or desirable in the usual business or trade of the employer, except where the employment has been fixed for a specific project or undertaking the completion or termination of which has been determined at the time of the engagement of the employee or where the work or services to be performed is seasonal in nature and the employment is for the duration of the season. An employment shall be deemed to be casual if it is not covered by the preceding paragraph: Provided, That, any employee who has rendered at least one year of service, whether such service is continuous or broken, shall be considered a regular employee with respect to the activity in which he is employed and his employment shall continue while such actually exists. The foregoing provision provides for three kinds of employees: (a) regular employees or those who have been engaged to perform activities which are usually necessary or desirable in the usual business or trade of the employer; (b) project employees or those whose employment has been fixed for a specific project or undertaking, the completion or termination of which has been determined at the time of the engagement of the employee or where the work or services to be performed is seasonal in nature and the employment is for the duration of the season; and (c) casual employees or those who are neither regular nor project employees. The principal test for determining whether an employee is a project employee or a regular employee is whether the employment has been fixed for a specific project or undertaking, the completion or termination of which has been determined at the time of the engagement of the employee. A project employee is one whose employment has been fixed for a specific project or undertaking, the completion or termination of which has been determined at the time of the engagement of the employee or where the work or service to be performed is seasonal in nature and the employment is for the duration of the season. A true project employee should be assigned to a project which begins and ends at determined or determinable times, and be informed thereof at the time of hiring. Petitioners contend that respondent's repeated hiring of their services qualifies them to the status of regular employees. On this score, the LA ruled: This is further buttressed by the fact that the relationship between complainants and the respondent URSUMCO, would clearly reveal that the very nature of the terms and conditions of their hiring would show that complainants were required to perform phases of special projects which are not related to the main operation of the respondent for a definite period, after which their services are available to any farm owner. The NLRC, agreeing with the LA, further ruled that: In the case at bar, We note that complainants never bothered to deny that they voluntarily, knowingly and willfully executed the contracts of employment. Neither was there any showing that respondents exercised moral dominance on the complainants, x x x it is clear that the contracts of employment are valid and binding on the complainants. The execution of these contracts in the case at bar is necessitated by the peculiar nature of the work in the sugar industry which has an off milling season. The very nature of the terms and conditions of complainants' hiring reveals that they were required to perform phases of special projects for a definite period after, their services are available to other farm owners. This is so because the planting of sugar does not entail a whole year operation, and utility works are comparatively small during the off-milling season. Finally, the CA noted: Petitioner Pedy Caseres first applied with private respondent URSUMCO on January 9, 1989 as a worker assisting the crane operator at the transloading station. Upon application, Caseres was interviewed and made to understand that his employment would be co-terminus with the phase of work to which he would be then assigned, that is until February 5, 1989 and thereafter he would be free to seek employment elsewhere. Caseres agreed and signed the contract of employment for specific project or undertaking. After an absence of more than five (5) months, Caseres re-applied with respondent as a seasonal project worker assisting in the general underchassis reconditioning to transport units on July 17, 1989. Like his first assignment, Caseres was made to understand that his services would be co-terminus with the work to which he

Page 43

LABOR RELATIONS
Atty. Jefferson M. Marquez
would be then assigned that is from July 17, 1989 to July 20, 1989 and that thereafter he is free to seek employment elsewhere to which Caseres agreed and readily signed the contract of employment for specific project or undertaking issued to him. Thereafter Caseres voluntarily signed several other employment contracts for various undertakings with a determinable period. As in the first contract, Caseres' services were co-terminus with the work to which he was assigned, and that thereafter, he was free to seek employment with other sugar millers or elsewhere. The nature and terms and conditions of employment of petitioner Andito Pael were the same as that of his co-petitioner Caseres. It must be noted that there were intervals in petitioners' respective employment contracts, and that their work depended on the availability of such contracts or projects. Consequently, the employment of URSUMCO's work force was not permanent but co-terminous with the projects to which the employees were assigned and from whose payrolls they were paid (Palomares vs. NLRC, 277 SCRA 439). Petitioners' repeated and successive re-employment on the basis of a contract of employment for more than one year cannot and does not make them regular employees. Length of service is not the controlling determinant of the employment tenure of a project employee (Rada vs. NLRC, 205 SCRA 69). It should be stressed that contracts for project employment are valid under the law. In Villa v. National Labor Relations Commission,[16] the Court stated that: by entering into such contract, an employee is deemed to understand that his employment is coterminous with the project. He may not expect to be employed continuously beyond the completion of the project. It is of judicial notice that project employees engaged for manual services or those for special skills like those of carpenters or masons, are, as a rule, unschooled. However, this fact alone is not a valid reason for bestowing special treatment on them or for invalidating a contract of employment. Project employment contracts are not lopsided agreements in favor of only one party thereto. The employers interest is equally important as that of the employees for theirs is the interest that propels economic activity. While it may be true that it is the employer who drafts project employment contracts with its business interest as overriding consideration, such contracts do not, of necessity, prejudice the employee. Neither is the employee left helpless by a prejudicial employment contract. After all, under the law, the interest of the worker is paramount. The fact that petitioners were constantly re-hired does not ipso facto establish that they became regular employees. Their respective contracts with respondent show that there were intervals in their employment. In petitioner Caseres's case, while his employment lasted from August 1989 to May 1999, the duration of his employment ranged from one day to several months at a time, and such successive employments were not continuous. With regard to petitioner Pael, his employment never lasted for more than a month at a time. These support the conclusion that they were indeed project employees, and since their work depended on the availability of such contracts or projects, necessarily the employment of respondents work force was not permanent but co-terminous with the projects to which they were assigned and from whose payrolls they were paid. As ruled in Palomares v. National Labor Relations Commission, it would be extremely burdensome for their employer to retain them as permanent employees and pay them wages even if there were no projects to work on. Moreover, even if petitioners were repeatedly and successively re-hired, still it did not qualify them as regular employees, as length of service is not the controlling determinant of the employment tenure of a project employee, but whether the employment has been fixed for a specific project or undertaking, its completion has been determined at the time of the engagement of the employee. Further, the proviso in Article 280, stating that an employee who has rendered service for at least one (1) year shall be considered a regular employee, pertains to casual employees and not to project employees. Accordingly, petitioners cannot complain of illegal dismissal inasmuch as the completion of the contract or phase thereof for which they have been engaged automatically terminates their employment.

21. Pier 8 Arrastre & Stevedoring Services, Inc. vs Boclot, G.R. No. 173849, September 28, 2007 Facts: Petitioner Pier 8 Arrastre and Stevedoring Services, Inc. (PASSI) is a domestic corporation engaged in the business of providing arrastre and stevedoring services[5] at Pier 8 in the Manila North Harbor. PASSI has been rendering arrastre and stevedoring services at the port area since 1974 and employs stevedores who assist in the loading and unloading of cargoes to and from the vessels. Petitioner Eliodoro C. Cruz is its Vice-President and General Manager. Respondent Jeff B. Boclot was hired by PASSI to perform the functions of a stevedore starting 20 September 1999. The facts show that respondent rendered actual services to PASSI during the following periods: Period Duration September - December 1999 (4 months) 21 days January - April 2000 (4 months) 20 days March - December 2001 (10 months) 85 days January - December 2002 (12 months) 70.5 days January - June 2003 (6 months) 32 days Total 36 months 228.5 days On 15 April 2000, the Philippine Ports Authority (PPA) seized the facilities and took over the operations of PASSI through its Special Takeover Unit, absorbing PASSI workers as well as their relievers. By virtue of a Decision dated 9 January 2001 of the Court of Appeals, petitioners were

Page 44

LABOR RELATIONS
Atty. Jefferson M. Marquez
able to regain control of their arrastre and stevedoring operations at Pier 8 on 12 March 2001. On 9 May 2003, respondent filed a Complaint with the Labor Arbiter of the NLRC, claiming regularization; payment of service incentive leave and 13th month pays; moral, exemplary and actual damages; and attorneys fees. Respondent alleged that he was hired by PASSI in October 1999 and was issued company ID No. 304, a PPA Pass and SSS documents. In fact, respondent contended that he became a regular employee by April 2000, since it was his sixth continuous month in service in PASSIs regular course of business. He argued on the basis of Articles 280[9] and 281 of the Labor Code. He maintains that under paragraph 2 of Article 280, he should be deemed a regular employee having rendered at least one year of service with the company. According to respondent, he remained a casual employee from the time he was first hired to perform the services of a stevedore. Thus, respondent claimed he was denied the rights and privileges of a regular employee, including those granted under the Collective Bargaining Agreement (CBA) such as wage increase; medical, dental and hospitalization benefits; vacation and sick leaves; uniforms, Christmas gifts, productivity bonus, accident insurance, special separation pays, and others. In the instant petition, petitioners are vehemently denying that respondent has become PASSI regular employee. Petitioners insist that respondent was hired as a mere stevedore and, thus, could not become a regular stevedore. Petitioners presented a list of the days when respondents services as stevedore were engaged, to support its claim that respondent is a reliever. Petitioners aver that the employment of the stevedores is governed by a system of rotation. Based on this system of rotation, the work available to reliever stevedores is dependent on the actual stevedoring and arrastre requirements at a current given time. Petitioners posit that respondent, as a reliever stevedore, is a mere extra worker whose work is dependent on the absence of regular stevedores during any given shift. During rotation proper as petitioners term it, all regular employees are first called and given work before any reliever is assigned. Petitioners assert that while the regular stevedores work an average of 4 days a week (or 16 days a month), respondent performed services for a total of 228.5 days (or only for an average of 6.34 days a month) from September 1999 to June 2003. In defense of the Court of Appeals ruling grounded on Articles 280 and 281 of the Labor Code, petitioners maintain that the foregoing provisions are inapplicable on the postulation that respondent is neither a probationary nor a casual employee. For the same reasons, petitioners argue that Article XXV of the CBA cannot be used to support respondents contention that he is a regular employee since the CBA provision he invokes refers to all incumbent probationary or casual employees and workers in the company and not to respondent who is neither a casual nor a probationary employee. Ruling: De Leon v. National Labor Relations Commission succinctly explains the delineation of the foregoing employee classification, to wit: The primary standard, therefore, of determining a regular employment is the reasonable connection between the particular activity performed by the employee in relation to the usual business or trade of the employer. The test is whether the former is usually necessary or desirable in the usual business or trade of the employer. The connection can be determined by considering the nature of the work performed and its relation to the scheme of the particular business or trade in its entirety. Also, if the employee has been performing the job for at least one year, even if the performance is not continuous or merely intermittent, the law deems the repeated and continuing need for its performance as sufficient evidence of the necessity if not indispensability of that activity to the business. Hence, the employment is also considered regular, but only with respect to such activity and while such activity exists. (Emphasis supplied.) PASSI is engaged in providing stevedoring and arrastre services in the port area in Manila. Stevedoring, dock and arrastre operations include, but are not limited to, the opening and closing of a vessels hatches; discharging of cargoes from ship to truck or dock, lighters and barges, and vice-versa; movement of cargoes inside vessels, warehouses, terminals and docks; and other related work. In line with this, petitioners hire stevedores who assist in the loading and unloading of cargoes to and from the vessels. Petitioners concede that whenever respondent worked as a reliever stevedore due to the absence of a regular stevedore, he performed tasks that are usually necessary and desirable to their business. Petitioners, however, contend that this in itself does not make him a regular stevedore, postulating that the hiring of respondent as a reliever is akin to a situation in which a worker goes on vacation leave, sick leave, maternity leave or paternity leave; and the employer is constrained to hire another worker from outside the establishment to ensure the smooth flow of its operations. Based on the circumstances of the instant case, this Court agrees. It takes judicial notice that it is an industry practice in port services to hire reliever stevedores in order to ensure smooth-flowing 24-hour stevedoring and arrastre operations in the port area. No doubt, serving as a stevedore, respondent performs tasks necessary or desirable to the usual business of petitioners. However, it should be deemed part of the nature of his work that he can only work as a stevedore in the absence of the employee regularly employed for the very same function. Bearing in mind that respondent performed services from September 1999 until June 2003 for a period of only 228.5 days in 36 months, or roughly an average of 6.34 days a month; while a regular stevedore working for petitioners, on the other hand, renders service for an average of 16 days a month, demonstrates that respondents employment is subject to the availability of work, depending on the absences of the regular stevedores. Moreover, respondent does not contest that he was well aware that he would only be given work when there are absent or unavailable employees. Respondent also does not allege, nor is there any showing, that he was disallowed or prevented from offering his services to other cargo handlers in the other piers at the North Harbor other than petitioners. As aforestated, the situation of respondent is akin to that of a seasonal or project or term employee, albeit on a daily basis. Anent petitioners contention that respondent is neither a probationary nor a casual employee, this Court again refers to Article 280 of the Labor Code. The second paragraph thereof stipulates in unequivocal terms that all other employees who do not fall under the definitions in the first

Page 45

LABOR RELATIONS
Atty. Jefferson M. Marquez
paragraph of regular, project and seasonal employees, are deemed casual employees.[25] Not qualifying under any of the kinds of employees covered by the first paragraph of Article 280 of the Labor Code, then respondent is a casual employee under the second paragraph of the same provision. The same provision, however, provides that a casual employee can be considered as regular employee if said casual employee has rendered at least one year of service regardless of the fact that such service may be continuous or broken. Section 3, Rule V, Book II of the Implementing Rules and Regulations of the Labor Code clearly defines the term at least one year of service to mean service within 12 months, whether continuous or broken, reckoned from the date the employee started working, including authorized absences and paid regular holidays, unless the working days in the establishment as a matter of practice or policy, or that provided in the employment contract, is less than 12 months, in which case said period shall be considered one year. If the employee has been performing the job for at least one year, even if the performance is not continuous or merely intermittent, the law deems the repeated and continuing need for its performance as sufficient evidence of the necessity, if not indispensability, of that activity to the business of the employer. Applying the foregoing, respondent, who has performed actual stevedoring services for petitioners only for an accumulated period of 228.5 days does not fall under the classification of a casual turned regular employee after rendering at least one year of service, whether continuous or intermittent. NONETHELESS, this Court still finds respondent to be a regular employee on the basis of pertinent provisions under the CBA between PASSI and its Workers union, which was effective from 4 March 1998 to 3 March 2003: The Company agrees to convert to regular status all incumbent probationary or casual employees and workers in the Company who have served the Company for an accumulated service term of employment of not less than six (6) months from his original date of hiring. The probationary period for all future workers or employees shall be the following: (a) All skilled workers such as crane operator, mechanic, carpenter, winchman, signalman and checkers shall become regular after three (3) months continuous employment; (b) All semi-skilled personnel shall become regular after four (4) months of continuous employment; (c) All non-skilled personnel shall be regular after six (6) months continuous employment. Petitioners were crucified on this argument raised by respondent. The union which negotiated the existing CBA is the sole and exclusive bargaining representative of all the stevedores, dock workers, gang bosses, rank and file employees working at Pier 8, and its offices. The NLRC ruled that respondents reliance on the CBA to show that he has become a regular employee is misplaced for the reason that the CBA applies only to regular workers of the company. Respondent assents that he is not a member of the union, as he was not recognized by PASSI as its regular employee, but this Court notes that PASSI adopts a union-shop agreement, culling from Article II of the CBA which stipulates: The Union and the Company (PASSI) hereby agree to adopt the Union Shop as a condition of employment to the position (sic) covered by this Agreement. Under a union-shop agreement, although nonmembers may be hired, an employee is required to become a union member after a certain period, in order to retain employment. This requirement applies to present and future employees.[36] The same article of the CBA stipulates that employment in PASSI cannot be obtained without prior membership in the union. Apropos, applying the foregoing provisions of the CBA, respondent should be considered a regular employee after six months of accumulated service. It is clearly stipulated therein that petitioners shall agree to convert to regular status all incumbent probationary or casual employees and workers in PASSI who have served PASSI for an accumulated service term of employment of not less than six months from the original date of hiring. Having rendered 228.5 days, or eight months of service to petitioners since 1999, then respondent is entitled to regularization by virtue of the said CBA provisions. In light of the foregoing, petitioners must accord respondent the status of a regular employee.

22. Pacquing vs. Coca-Cola Bottlers Phils., Inc., G.R. No. 157966, January 31, 2008, citing Magsalin vs. National Organization of Workingmen, G.R. No. 148492, May 9, 2003 Facts: Eddie Pacquing, Roderick Centeno, Juanito M. Guerra, Claro Dupilad, Jr., Louie Centeno, David Reblora, Raymundo Andrade (petitioners) were sales route helpers or cargadores-pahinantes of Coca-Cola Bottlers Philippines, Inc., (respondent), with the length of employment as follows: Name Date Hired Date Dismissed Eddie P. Pacquing Roderick Centeno Juanito M. Guerra Claro Dupilad, Jr. David R. Reblora June 14, 1987 November 15, 1985 June 16, 1980 March 1, 1992 September 15, 1988 January 30, 1988 January 15, 1995 February 20, 1995 June 30, 1995 December 15, 1995

Page 46

LABOR RELATIONS
Atty. Jefferson M. Marquez
Louie Centeno Raymundo Andrade September 15, 1988 January 15, 1988 March 15, 1996 October 15, 1995

Petitioners were part of a complement of three personnel comprised of a driver, a salesman and a regular route helper, for every delivery truck. They worked exclusively at respondent's plants, sales offices, and company premises. On October 22, 1996, petitioners filed a Complaint against respondent for unfair labor practice and illegal dismissal with claims for regularization, recovery of benefits under the Collective Bargaining Agreement (CBA), moral and exemplary damages, and attorney's fees. In their Position Paper, petitioners alleged that they should be declared regular employees of respondent since the nature of their work as cargadores-pahinantes was necessary or desirable to respondent's usual business and was directly related to respondent's business and trade. In its Position Paper, respondent denied liability to petitioners and countered that petitioners were temporary workers who were engaged for a five-month period to act as substitutes for an absent regular employee. Issue: What is their status as employees? Ruling: The pivotal question of whether respondent's sales route helpers or cargadores or pahinantes are regular workers of respondent has already been resolved in Magsalin v. National Organization of Working Men, thus: The basic law on the case is Article 280 of the Labor Code. Its pertinent provisions read: Art. 280. Regular and Casual Employment. The provisions of written agreement to the contrary notwithstanding and regardless of the oral agreement of the parties, an employment shall be deemed to be regular where the employee has been engaged to perform activities which are usually necessary or desirable in the usual business or trade of the employer, except where the employment has been fixed for a specific project or undertaking the completion or termination of which has been determined at the time of the engagement of the employee or where the work or services to be performed is seasonal in nature and the employment is for the duration of the season. An employment shall be deemed to be casual if it is not covered by the preceding paragraph: Provided, That, any employee who has rendered at least one year of service, whether such service is continuous or broken, shall be considered a regular employee with respect to the activity in which he is employed and his employment shall continue while such activity exists. Coca-Cola Bottlers Phils., Inc., is one of the leading and largest manufacturers of softdrinks in the country. Respondent workers have long been in the service of petitioner company. Respondent workers, when hired, would go with route salesmen on board delivery trucks and undertake the laborious task of loading and unloading softdrink products of Petitioner Company to its various delivery points. Even while the language of law might have been more definitive, the clarity of its spirit and intent, i.e., to ensure a regular workers security of tenure, however, can hardly be doubted. In determining whether an employment should be considered regular or non-regular, the applicable test is the reasonable connection between the particular activity performed by the employee in relation to the usual business or trade of the employer. The standard, supplied by the law itself, is whether the work undertaken is necessary or desirable in the usual business or trade of the employer, a fact that can be assessed by looking into the nature of the services rendered and its relation to the general scheme under which the business or trade is pursued in the usual course. It is distinguished from a specific undertaking that is divorced from the normal activities required in carrying on the particular business or trade. But, although the work to be performed is only for a specific project or seasonal, where a person thus engaged has been performing the job for at least one year, even if the performance is not continuous or is merely intermittent, the law deems the repeated and continuing need for its performance as being sufficient to indicate the necessity or desirability of that activity to the business or trade of the employer. The employment of such person is also then deemed to be regular with respect to such activity and while such activity exists. The argument of petitioner that its usual business or trade is softdrink manufacturing and that the work assigned to respondent workers as sales route helpers so involves merely post production activities, one which is not indispensable in the manufacture of its products, scarcely can be persuasive. If, as so argued by petitioner company, only those whose work are directly involved in the production of softdrinks may be held performing functions necessary and desirable in its usual business or trade, there would have then been no need for it to even maintain regular truck sales route helpers. The nature of the work performed must be viewed from a perspective of the business or trade in its entirety and not on a confined scope. The repeated rehiring of respondent workers and the continuing need for their services clearly attest to the necessity or desirability of their services in the regular conduct of the business or trade of petitioner company. The Court of Appeals has found each of respondents to have worked for at least one year with petitioner company. While this Court, in Brent School, Inc. vs. Zamora,has upheld the legality of a fixed-term employment, it has done so, however, with a stern admonition that where from the circumstances it is apparent that the period has been imposed to preclude the acquisition of tenurial security by the employee, then it should be struck down as being contrary to law, morals, good customs, public order and public policy. The pernicious practice of having employees, workers and laborers, engaged for a fixed period of few months, short of the normal six-month probationary period of employment, and, thereafter, to be hired on a day-to-day basis, mocks the law. Any obvious

Page 47

LABOR RELATIONS
Atty. Jefferson M. Marquez
circumvention of the law cannot be countenanced. The fact that respondent workers have agreed to be employed on such basis and to forego the protection given to them on their security of tenure, demonstrate nothing more than the serious problem of impoverishment of so many of our people and the resulting unevenness between labor and capital. A contract of employment is impressed with public interest. The provisions of applicable statutes are deemed written into the contract, and the parties are not at liberty to insulate themselves and their relationships from the impact of labor laws and regulations by simply contracting with each other.

23. Cocomangas Hotel Beach Resort v. Visca, G.R. No. 167045, August 29, 2008 Facts: The present controversy stemmed from five individual complaints for illegal dismissal filed on by Federico F. Visca (Visca), Johnny G. Barredo, Ronald Q. Tibus, Richard G. Visca and Raffie G. Visca (respondents) against Cocomangas Hotel Beach Resort and/or its owner-manager, Susan Munro (petitioners) before Sub-Regional Arbitration Branch No. VI of the National Labor Relations Commission (NLRC) in Kalibo, Aklan. In their consolidated Position Paper respondents alleged that they were regular employees of petitioners, with designations and dates of employment as follows: Name Designation Date Employed Federico F. Visa Foreman Johnny G. Barredo Carpenter Ronald Q. Tibus Mason Richard G. Visca Carpenter Raffie G. Visca Mason/Carpenter tasked with the maintenance and repair of the resort facilities. October 1, 1987 April 23, 1993 November 9, 1996 April 1998 March 27, 1993

Maria Nida Iigo-Taala, the Front Desk Officer/Sales Manager, informed them not to report for work since the ongoing constructions and repairs would be temporarily suspended because they caused irritation and annoyance to the resort's guests; as instructed, they did not report for work the succeeding days. John Munro, husband of petitioner Susan Munro, subsequently visited respondent foreman Visca and informed him that the work suspension was due to budgetary constraints. When respondent Visca later discovered that four new workers were hired to do respondents' tasks, he confronted petitioner Munro who explained that respondents' resumption of work was not possible due to budgetary constraints. Hence, they filed their individual complaints for illegal dismissal. In addition to reinstatement with payment of full backwages, respondents prayed for payment of premium pay for rest day, service incentive leave pay, 13th month pay, and cost-of-living allowance, plus moral and exemplary damages and attorney's fees. In their Position Paper, petitioners denied any employer-employee relationship with respondents and countered that respondent Visca was an independent contractor who was called upon from time to time when some repairs in the resort facilities were needed and the other respondents were selected and hired by him. Labor Arbiter: dismissed the complaint, holding that respondent Visca was an independent contractor and the other respondents were hired by him to help him with his contracted works at the resort; that there was no illegal dismissal but completion of projects; that respondents were project workers, not regular employees. NLRC: set aside the Decision of the LA and ordering the payment to respondents of backwages, 13th month pay and service incentive leave pay for three years, in addition to 10% attorney's fees. The NLRC held that respondents were regular employees of petitioners since all the factors determinative of employer-employee relationship were present and the work done by respondents was clearly related to petitioners' resort business. It took into account the following: (a) respondent Visca was reported by petitioners as an employee in the Quarterly Social Security System (SSS) report; (b) all of the respondents were certified to by petitioner Munro as workers and even commended for their satisfactory performance; (c) respondents were paid their holiday and overtime pay; and (d) respondents had been continuously in petitioners' employ from three to twelve years and were all paid by daily wage given weekly. Petitioners then filed a Motion for Reconsideration, arguing that respondents were project employees.

Page 48

LABOR RELATIONS
Atty. Jefferson M. Marquez
NLRC (Motion for Reconsideration): Acting upon the petitioners motion, the NLRC made a complete turnabout from its original decision and issued a Resolution dismissing the complaint, holding that respondents were not regular employees but project employees, hired for a short period of time to do some repair jobs in petitioners' resort business. Nonetheless, it ordered payment of P10,000.00 to each complainant as financial assistance. CA: The CA held respondents were regular employees, not project workers, since in the years that petitioners repeatedly hired respondents' services, the former failed to set, even once, specific periods when the employment relationship would be terminated; that the repeated hiring of respondents established that the services rendered by them were necessary and desirable to petitioners' resort business; at the least, respondents were regular seasonal employees, hired depending on the tourist season and when the need arose in maintaining petitioners' resort for the benefit of guests. In addition, the CA awarded respondents P50,000.00 as damages, since their termination was attended by bad faith, in that petitioners not only gave respondents the run-around but also blatantly hired others to take respondents' place despite their claim that the so-called temporary stoppage of work was due to budgetary constraints. Issue: Whether respondents are regular employees or project employees. Ruling: Respondents are regular employees and not project employees. The petitioners were ambivalent in categorizing respondents. In their Position Paper filed before the LA, petitioners classified respondent Visca as an independent contractor and the other respondents as his employees; while in their Motion for Reconsideration before the NLRC, petitioners treated respondents as project employees. Further, petitioners' position in their Motion for Reconsideration before the NLRC runs contrary to their earlier submission in their Position Paper before the LA. While initially advancing the absence of an employer-employee relationship, petitioners on appeal, sang a different tune, so to speak, essentially invoking the termination of the period of their employer-employee relationship. The NLRC should not have considered the new theory offered by the petitioners in their Motion for Reconsideration. It is a matter of law that when a party adopts a particular theory and the case is tried and decided upon that theory in the court below, he will not be permitted to change his theory on appeal. To permit a party to change his theory on appeal will be unfair to the adverse party. At any rate, after a careful examination of the records, the Court finds that the CA did not err in finding that respondents were regular employees, not project employees. A project employee is one whose "employment has been fixed for a specific project or undertaking, the completion or termination of which has been determined at the time of the engagement of the employee or where the work or service to be performed is seasonal in nature and the employment is for the duration of the season." Before an employee hired on a per-project basis can be dismissed, a report must be made to the nearest employment office, of the termination of the services of the workers every time completes a project, pursuant to Policy Instruction No. 20. In the present case, respondents cannot be classified as project employees, since they worked continuously for petitioners from three to twelve years without any mention of a "project" to which they were specifically assigned. While they had designations as "foreman," "carpenter" and "mason," they performed work other than carpentry or masonry. They were tasked with the maintenance and repair of the furniture, motor boats, cottages, and windbreakers and other resort facilities. There is likewise no evidence of the project employment contracts covering respondents' alleged periods of employment. More importantly, there is no evidence that petitioners reported the termination of respondents' supposed project employment to the DOLE as project employees. Department Order No. 19, as well as the old Policy Instructions No. 20, requires employers to submit a report of an employee's termination to the nearest public employment office every time his employment is terminated due to a completion of a project. Petitioners' failure to file termination reports is an indication that the respondents were not project employees but regular employees. This Court has held that an employment ceases to be coterminous with specific projects when the employee is continuously rehired due to the demands of employer's business and re-engaged for many more projects without interruption. The Court is not persuaded by petitioners' submission that respondents' services are not necessary or desirable to the usual trade or business of

Page 49

LABOR RELATIONS
Atty. Jefferson M. Marquez
the resort. The repeated and continuing need for their services is sufficient evidence of the necessity, if not indispensability, of their services to petitioners' resort business. In Maraguinot, Jr. v. National Labor Relations Commission, the Court ruled that "once a project or work pool employee has been: (1) continuously, as opposed to intermittently, rehired by the same employer for the same tasks or nature of tasks; and (2) these tasks are vital, necessary and indispensable to the usual business or trade of the employer, then the employee must be deemed a regular employee, pursuant to Article 280 of the Labor Code and jurisprudence. That respondents were regular employees is further bolstered by the following evidence: (a) the SSS Quarterly Summary of Contribution Payments listing respondents as employees of petitioners; (b) the Service Record Certificates stating that respondents were employees of petitioners for periods ranging from three to twelve years and all have given "very satisfactory performance"; (c) petty cash vouchers showing payment of respondents' salaries and holiday and overtime pays. Thus, substantial evidence supported the CA finding that respondents were regular employees. Being regular employees, they were entitled to security of tenure, and their services may not be terminated except for causes provided by law. Article 279 of the Labor Code, as amended, provides that an illegally dismissed employee shall be entitled to reinstatement, full backwages, inclusive of allowances, and to his other benefits or their monetary equivalent computed from the time his compensation was withheld from him up to the time of his actual reinstatement. The Court notes that the NLRC, in its earlier Decision which was affirmed by the CA, computed the award for backwages from May 8, 1999 to July 31, 2002 only. It is evident that respondents' backwages should not be limited to said period. The backwages due respondents must be computed from the time they were unjustly dismissed until actual reinstatement to their former positions. Thus, until petitioners implement the reinstatement aspect, its obligation to respondents, insofar as accrued backwages and other benefits are concerned, continues to accumulate.

24. Price, et al., v Innodata Phils., G.R. No. 178505, September 30, 2008 Facts: INNODATA had since ceased operations due to business losses in June 2002. Petitioners Cherry J. Price, Stephanie G. Domingo, and Lolita Arbilera were employed as formatters by INNODATA. The parties executed an employment contract denominated as a Contract of Employment for a Fixed Period, stipulating that the contract shall be effective from FEB. 16, 1999 to FEB. 16, 2000 a period of ONE YEAR. On 16 February 2000, the HRAD Manager of INNODATA wrote petitioners informing them of their last day of work, at the end of the close of business hours onFebruary 16, 2000. According to INNODATA, petitioners employment already ceased due to the end of their contract. On 22 May 2000, petitioners filed a Complaint for illegal dismissal and damages against respondents. Petitioners claimed that they should be considered regular employees since their positions as formatters were necessary and desirable to the usual business of INNODATA as an encoding, conversion and data processing company. Petitioners finally argued that they could not be considered project employees considering that their employment was not coterminous with any project or undertaking, the termination of which was predetermined. Respondents asserted that petitioners were not illegally dismissed, for their employment was terminated due to the expiration of their terms of employment. The Labor Arbiter issued its Decision finding petitioners complaint for illegal dismissal and damages meritorious. Respondent INNODATA appealed the Labor Arbiters Decision to the NLRC. The NLRC reversed the Labor Arbiters Decision dated 17 October 2000, and absolved INNODATA of the charge of illegal dismissal. On 25 September 2006, the Court of Appeals promulgated its Decision sustaining the ruling of the NLRC that petitioners were not illegally dismissed. Hence, this petition. Issues: Whether petitioners were illegally dismissed by respondents Whether petitioners were hired by INNODATA under valid fixed-term employment contracts Ruling:

Page 50

LABOR RELATIONS
Atty. Jefferson M. Marquez
The Court finds merit in the present Petition. There were no valid fixed-term contracts and petitioners were regular employees of the INNODATA who could not be dismissed except for just or authorized cause. The employment status of a person is defined and prescribed by law and not by what the parties say it should be. Equally important to consider is that a contract of employment is impressed with public interest such that labor contracts must yield to the common good. Thus, provisions of applicable statutes are deemed written into the contract, and the parties are not at liberty to insulate themselves and their relationships from the impact of labor laws and regulations by simply contracting with each other. Regular employment has been defined by Article 280 of the Labor Code, as amended, which reads: Art. 280. Regular and Casual Employment. The provisions of written agreement to the contrary notwithstanding and regardless of the oral agreement of the parties, an employment shall be deemed to be regular where the employee has been engaged to perform activities which are usually necessary or desirable in the usual business or trade of the employer, except where the employment has been fixed for a specific project or undertaking the completion or termination of which has been determined at the time of engagement of the employee or where the work or services to be performed is seasonal in nature and employment is for the duration of the season. An employment shall be deemed to be casual if it is not covered by the preceding paragraph. Provided, That, any employee who has rendered at least one year of service, whether such service is continuous or broken, shall be considered a regular employee with respect to the activity in which he is employed and his employment shall continue while such activity exists. (Underscoring ours). Based on the afore-quoted provision, the following employees are accorded regular status: (1) those who are engaged to perform activities which are necessary or desirable in the usual business or trade of the employer, regardless of the length of their employment; and (2) those who were initially hired as casual employees, but have rendered at least one year of service, whether continuous or broken, with respect to the activity in which they are employed. Undoubtedly, petitioners belong to the first type of regular employees. Under Article 280 of the Labor Code, the applicable test to determine whether an employment should be considered regular or non-regular is the reasonable connection between the particular activity performed by the employee in relation to the usual business or trade of the employer. In the case at bar, petitioners were employed by INNODATA on 17 February 1999 as formatters. The primary business of INNODATA is data encoding, and the formatting of the data entered into the computers is an essential part of the process of data encoding. Formatting organizes the data encoded, making it easier to understand for the clients and/or the intended end users thereof. Undeniably, the work performed by petitioners was necessary or desirable in the business or trade of INNODATA. However, it is also true that while certain forms of employment require the performance of usual or desirable functions and exceed one year, these do not necessarily result in regular employment under Article 280 of the Labor Code. Under the Civil Code, fixed-term employment contracts are not limited, as they are under the present Labor Code, to those by nature seasonal or for specific projects with predetermined dates of completion; they also include those to which the parties by free choice have assigned a specific date of termination. The decisive determinant in term employment is the day certain agreed upon by the parties for the commencement and termination of their employment relationship, a day certain being understood to be that which must necessarily come, although it may not be known when. Seasonal employment and employment for a particular project are instances of employment in which a period, where not expressly set down, is necessarily implied. While this Court has recognized the validity of fixed-term employment contracts, it has consistently held that this is the exception rather than the general rule. More importantly, a fixed-term employment is valid only under certain circumstances. In Brent, the very same case invoked by respondents, the Court identified several circumstances wherein a fixed-term is an essential and natural appurtenance, to wit: Some familiar examples may be cited of employment contracts which may be neither for seasonal work nor for specific projects, but to which a fixed term is an essential and natural appurtenance: overseas employment contracts, for one, to which, whatever the nature of the engagement, the concept of regular employment with all that it implies does not appear ever to have been applied, Article 280 of the Labor Code notwithstanding; also appointments to the positions of dean, assistant dean, college secretary, principal, and other administrative offices in educational institutions, which are by practice or tradition rotated among the faculty members, and where fixed terms are a necessity without which no reasonable rotation would be possible. Similarly, despite the provisions of Article 280, Policy Instructions No. 8 of the Minister of Labor implicitly recognize that certain company officials may be elected for what would amount to fixed periods, at the expiration of which they would have to stand down, in providing that these officials, "x x may lose their jobs as president, executive vice-president or vice president, etc. because the stockholders or the board of directors for one reason or another did not reelect them." After considering petitioners contracts in their entirety, as well as the circumstances surrounding petitioners employment at INNODATA, the Court

Page 51

LABOR RELATIONS
Atty. Jefferson M. Marquez
is convinced that the terms fixed therein were meant only to circumvent petitioners right to security of tenure and are, therefore, invalid. The contracts of employment submitted by respondents are highly suspect for not only being ambiguous, but also for appearing to be tampered with.They later on admitted in their Memorandum filed with this Court that petitioners were originally hired on 16 February 1999 but the project for which they were employed was completed before the expiration of one year. Petitioners were merely rehired on 6 September 1999 for a new project. While respondents submitted employment contracts with 6 September 1999 as beginning date of effectivity, it is obvious that in one of them, the original beginning date of effectivity, 16 February 1999, was merely crossed out and replaced with 6 September 1999. Such modification and denial by respondents as to the real beginning date of petitioners employment contracts render the said contracts ambiguous. The contracts themselves state that they would be effective until 16 February 2000 for a period of one year. If the contracts took effect only on 6 September 1999, then its period of effectivity would obviously be less than one year, or for a period of only about five months. Obviously, respondents wanted to make it appear that petitioners worked for INNODATA for a period of less than one year. The only reason the Court can discern from such a move on respondents part is so that they can preclude petitioners from acquiring regular status based on their employment for one year. Nonetheless, the Court emphasizes that it has already found that petitioners should be considered regular employees of INNODATA by the nature of the work they performed as formatters, which was necessary in the business or trade of INNODATA.Hence, the total period of their employment becomes irrelevant. Further attempting to exonerate itself from any liability for illegal dismissal, INNODATA contends that petitioners were project employees whose employment ceased at the end of a specific project or undertaking. This contention is specious and devoid of merit. Scrutinizing petitioners employment contracts with INNODATA, however, failed to reveal any mention therein of what specific project or undertaking petitioners were hired for. Although the contracts made general references to a project, such project was neither named nor described at all therein. The conclusion by the Court of Appeals that petitioners were hired for the Earthweb project is not supported by any evidence on record. The one-year period for which petitioners were hired was simply fixed in the employment contracts without reference or connection to the period required for the completion of a project. More importantly, there is also a dearth of evidence that such project or undertaking had already been completed or terminated to justify the dismissal of petitioners. In fact, petitioners alleged - and respondents failed to dispute that petitioners did not work on just one project, but continuously worked for a series of projects for various clients of INNODATA. Under Section 3, Article XVI of the Constitution, it is the policy of the State to assure the workers of security of tenure and free them from the bondage of uncertainty of tenure woven by some employers into their contracts of employment. This was exactly the purpose of the legislators in drafting Article 280 of the Labor Code to prevent the circumvention by unscrupulous employers of the employees right to be secure in his tenure by indiscriminately and completely ruling out all written and oral agreements inconsistent with the concept of regular employment. cralaIn all, respondents insistence that it can legally dismiss petitioners on the ground that their term of employment has expired is untenable. To reiterate, petitioners, being regular employees of INNODATA, are entitled to security of tenure. In the words of Article 279 of the Labor Code: ART. 279. Security of Tenure. In cases of regular employment, the employer shall not terminate the services of an employee except for a just cause or when authorized by this Title. An employee who is unjustly dismissed from work shall be entitled to reinstatement without loss of seniority rights and other privileges and to his full backwages, inclusive of allowances, and to his other benefits or their monetary equivalent computed from the time his compensation was withheld from him up to the time of his actual reinstatement. The Petition for Review on Certiorari is GRANTED.

25. Agusan del Norte Electric Cooperative v. Cagampang, G.R. No. 167627, October 10, 2008 Facts: Respondents Joel Cagampang and Glenn Garzon started working as linemen for petitioner Agusan del Norte Electric Cooperative, Inc. (ANECO) on October 1, 1990, under an employment contract which was for a period not exceeding three months. They were both allegedly required to work eight hours a day and sometimes on Sundays, getting a daily salary of P122.00. When the contract expired, the two were laid-off for one to five days and then ordered to report back to work but on the basis of job orders. After several renewals of their job contracts in the form of job orders for similar employment periods of about three months each, the said contracts eventually expired on April 31, 1998 and July 30, 1999. Respondents' contracts were no longer renewed, resulting in their loss of employment. Thus, on January 11, 2001, respondents filed an illegal dismissal case against petitioners before the LA. They prayed for payment of backwages, salary differential, allowances, premium for alleged work during holidays and rest days, service incentive leave, and separation pay. LA ruled that there was illegal dismissal. However, NLRC reversed the LAs decision. CA however set aside NLRCs decision.

Page 52

LABOR RELATIONS
Atty. Jefferson M. Marquez
Issue: WON Cagampang and Garzon are regular employees/workers of the petitioner. Ruling: SC denied the petition. It held that respondents Cagampang and Garzon are deemed regular workers, and as such were illegally dismissed. There is no dispute that the respondents' work as linemen was necessary or desirable in the usual business of ANECO. Additionally, the respondents have been performing the job for at least one year. The law deems the repeated and continuing need for its performance as sufficient evidence of the necessity, if not indispensability, of that activity to the business. As held in Integrated Contractor and Plumbing Works, Inc. v. National Labor Relations Commission: [11] The test to determine whether employment is regular or not is the reasonable connection between the particular activity performed by the employee in relation to the usual business or trade of the employer. Also, if the employee has been performing the job for at least one year, even if the performance is not continuous or merely intermittent, the law deems the repeated and continuing need for its performance as sufficient evidence of the necessity, if not indispensability of that activity to the business. Thus, we held that where the employment of project employees is extended long after the supposed project has been finished, the employees are removed from the scope of project employees and are considered regular employees. While length of time may not be the controlling test for project employment, it is vital in determining if the employee was hired for a specific undertaking or tasked to perform functions vital, necessary and indispensable to the usual business or trade of the employer. Here, private respondent had been a project employee several times over. His employment ceased to be coterminous with specific projects when he was repeatedly re-hired due to the demands of petitioner's business. Where from the circumstances it is apparent that periods have been imposed to preclude the acquisition of tenurial security by the employee, they should be struck down as contrary to public policy, morals, good customs or public order. [12] Respondents in the present case being regular employees, ANECO as the employer had the burden of proof to show that the respondents' termination was for a just cause. Unfortunately, however, what petitioners did was merely to refuse, without justifiable reason, to renew respondents' work contracts for the performance of what would otherwise be regular jobs in relation to the trade or business of the former. [13]Such conduct dismally falls short of the requirements of our labor laws regarding dismissals. No twin notices of termination were issued to the employees, hence the employer did not observe due process in dismissing them from their employment. Their dismissals were patently illegal. Employer is burdened to prove just cause for terminating the employment of its employee with clear and convincing evidence. The weakness of the employee's defense should not operate to relieve nor discharge the employer of its burden to prove its charges pursuant to the guaranty of tenure granted by the Constitution to employees under the Labor Code. The case of the employer must stand or fall on its own merits.

26. William Uy Construction et. al vs. Trinidad, GR No. 183250, March 10, 2010 Facts: Respondent Jorge R. Trinidad filed a complaint for illegal dismissal and unpaid benefits against petitioner William Uy Construction Corporation. Trinidad claimed that he had been working with the latter company for 16 years since 1988 as driver of its service vehicle, dump truck, and transit mixer. He had signed several employment contracts with the company that identified him as a project employee although he had always been assigned to work on one project after another with some intervals. Respondent Trinidad further alleged that petitioner company terminated him from work after it shut down operations because of lack of projects. He learned later, however, that although it opened up a project in Batangas, it did not hire him back for that project. Petitioner company counteredhttp://sc.judiciary.gov.ph/jurisprudence/2010/march2010/183250.htm - _ftn1 that it was in the construction business. By the nature of such business, it had to hire and engage the services of project construction workers, including respondent Trinidad, whose employments had to be co-terminous with the completion of specific company projects. For this reason, every time the company employed Trinidad, he had to execute an employment contract with it, called Appointment as Project Worker. Petitioner company stressed that employment intervals or gaps were inherent in the construction business. In compliance with labor rules, the company submitted an establishment termination report to the Department of Labor and Employment (DOLE). The Labor Arbiter rendered a decision, dismissing respondent Trinidads complaint

Page 53

LABOR RELATIONS
Atty. Jefferson M. Marquez
for unjust dismissal. The Labor Arbiter, however, ordered petitioner company to pay Trinidad P1,500.00 in unpaid service incentive leave, taking into consideration the three-year prescriptive period for money claims.http://sc.judiciary.gov.ph/jurisprudence/2010/march2010/183250.htm - _ftn2 The Labor Arbiter held that, since Trinidad was a project employee and since his company submitted the appropriate establishment termination report to DOLE, his loss of work cannot be regarded as unjust dismissal. Issue: Whether or not petitioner companys repeated rehiring of respondent Trinidad over several years as project employee for its various projects automatically entitled him to the status of a regular employee. Ruling: The test for distinguishing a project employee from a regular employee is whether or not he has been assigned to carry out a specific project or undertaking, with the duration and scope of his engagement specified at the time his service is contracted.http://sc.judiciary.gov.ph/jurisprudence/2010/march2010/183250.htm - _ftn5 Here, it is not disputed that petitioner company contracted respondent Trinidads service by specific projects with the duration of his work clearly set out in his employment contracts.http://sc.judiciary.gov.ph/jurisprudence/2010/march2010/183250.htm - _ftn6 He remained a project employee regardless of the number of years and the various projects he worked for the company.http://sc.judiciary.gov.ph/jurisprudence/2010/march2010/183250.htm - _ftn7 Generally, length of service provides a fair yardstick for determining when an employee initially hired on a temporary basis becomes a permanent one, entitled to the security and benefits of regularization. But this standard will not be fair, if applied to the construction industry, simply because construction firms cannot guarantee work and funding for its payrolls beyond the life of each project. And getting projects is not a matter of course. Construction companies have no control over the decisions and resources of project proponents or owners. There is no construction company that does not wish it has such control but the reality, understood by construction workers, is that work depended on decisions and developments over which construction companies have no say. Respondent Trinidads series of employments with petitioner company were co-terminous with its projects. When its Boni Serrano-Katipunan Interchange Project was finished, Trinidads employment ended with it. He was not dismissed. His employment contract simply ended with the project for which he had signed up. His employment history belies the claim that he continuously worked for the company. Intervals or gaps separated one contract from another.http://sc.judiciary.gov.ph/jurisprudence/2010/march2010/183250.htm - _ftn9 Petitioner company needed only to show the last status of Trinidads employment, namely, that of a project employee under a contract that had ended and the companys compliance with the reporting requirement for the termination of that employment. Indeed, both the Labor Arbiter and the NLRC were satisfied that the fact of petitioner companys compliance with DOLE Order 19 had been proved in this case. WHEREFORE, the Court GRANTS the petition, and REINSTATES the decision of the National Labor Relations Commission which affirmed the decision of the Labor Arbiter.

27. Dacuital vs. L.M. Camus Engineering Corp.,G.R. No. 176748, September 1, 2010 Facts: Petitioners (LMCEC Employees) filed a complaint for illegal dismissal and non-payment of monetary benefits against respondent LM Camus Engineering Corp. before the National Labor Relations Commission (NLRC). The employees alleged that they were illegally dismissed from employment and that their employer failed to pay them their holiday pay, premium pay for holiday, rest day, service incentive leave pay, and 13th month pay during the existence and duration of their employment. They also averred that they were not provided with sick and vacation leaves. Respondents denied that petitioners were illegally dismissed from employment. They claimed that petitioners were project employees and, upon the completion of each project, they were served notices of project completion. They clarified that the termination of petitioners employment was due to the completion of the projects for which they were hired. Petitioners, however, countered that they were regular employees as they had been engaged to perform activities which are usually necessary or desirable in the usual business or trade of LMCEC. They denied that they were project or contractual employees because their employment was continuous and uninterrupted for more than one (1) year. Finally, they maintained that they were part of a work pool from which LMCEC drew its workers for its various projects. The Labor Arbiter rendered a decision declaring the dismissal of the complainant-employees as ILLEGAL and the complainants are entitled to reinstatement without back wages. The NLRC modified the decision of the Labor Arbiter and ordered the reinstatement of the complainants with

Page 54

LABOR RELATIONS
Atty. Jefferson M. Marquez
limited backwages. The respondents appealed the decision to the Court of Appeals and the appellate court held that the complainants are PROJECT EMPLOYEES and hence, there was no illegal dismissal. ISSUE: Whether or not the Court of Appeals is correct in concluding that the petitioners are PROJECT EMPLOYEES and that their dismissal from employment was legal RULING: The Supreme Court speaking through Justice Nachura answered in the NEGATIVE. Article 280 of the Labor Code distinguishes a "project employee" from a "regular employee" in this wise: Article 280. Regular and casual employment.The provisions of written agreement to the contrary notwithstanding and regardless of the oral agreement of the parties, an employment shall be deemed to be regular where the employee has been engaged to perform activities which are usually necessary or desirable in the usual business or trade of the employer, except where the employment has been fixed for a specific project or undertaking the completion or termination of which has been determined at the time of the engagement of the employee or where the work or services to be performed is seasonal in nature and the employment is for the duration of the season...xxx The principal test used to determine whether employees are project employees is whether or not the employees were assigned to carry out a specific project or undertaking, the duration or scope of which was specified at the time the employees were engaged for that project. Even though the absence of a written contract does not by itself grant regular status to petitioners, such a contract is evidence that petitioners were informed of the duration and scope of their work and their status as project employees. In this case, where no other evidence was offered, the absence of the employment contracts raises a serious question of whether the employees were properly informed at the onset of their employment of their status as project employees. While it is true that respondents presented the employment contract of Dacuital, the contract does not show that he was informed of the nature, as well as the duration of his employment. In fact, the duration of the project for which he was allegedly hired was not specified in the contract. Hence, the Dismissal of the petitioners are declared ILLEGAL.

28. Millenium Erectors Corp. vs. Magallanes, G.R. No. 184362, November 15, 2010 Facts: Respondent Virgilio Magallanes started working in 1988 as a utility man for Laurencito Tiu (Tiu), Chief Executive Officer of Millennium Erectors Corporation (petitioner), Tiu's family, and Kenneth Construction Corporation. He was assigned to different construction projects undertaken by petitioner in Metro Manila, the last of which was for a building in Libis, Quezon City. In July of 2004 he was told not to report for work anymore allegedly due to old age, prompting him to file on August 6, 2004 an illegal dismissal complaint 1 before the Labor Arbiter. Issue: Whether or not Magallanes dismissal violates security of tenure. Arguments: MEC Respondent was a project employee whom it hired for a building project in Libis on January 30 and which was in near completion on August 3, 2004, when services were terminated. Said all DOLE requirements were complied. Petitioner moved for reconsideration of the NLRC decision, contending that respondent's motion for reconsideration which it treated as an appeal was not perfected, it having been belatedly filed; that there was no statement of the date of receipt of the appealed decision; and that it lacked verification and copies thereof were not furnished the adverse parties RULING: 1. A project employee is one whose "employment has been fixed for a specific project or undertaking, the completion or termination of which has been determined at the time of the engagement of the employee or where the work or service to be performed is seasonal in nature and the employment is for the duration of the season." As the Court has consistently held, the service of project employees are coterminus [sic] with the project and may be terminated upon the end or completion of that project or project phase for which they were hired. Regular employees, in contrast, enjoy security of tenure and are entitled to

Page 55

LABOR RELATIONS
Atty. Jefferson M. Marquez
hold on to their work or position until their services are terminated by any of the modes recognized under the Labor Code. (emphasis and underscoring supplied) Assuming arguendo that petitioner hired respondent initially on a per project basis, his continued rehiring, as shown by the sample payrolls converted his status to that of a regular employee 2. In labor cases, rules of procedure should not be applied in a very rigid and technical sense. Technicalities should not be permitted to stand in the way of equitably and completely resolving the rights and obligations of the parties. Where the ends of substantial justice shall be better served, the application of technical rules of procedure may be relaxed. As to the defective verification in the appeal memorandum before the NLRC, the same liberality applies. After all, the requirement regarding verification of a pleading is formal, not jurisdictional. WHEREFORE, the petition is DENIED.

29. EXODUS INTERNATIONAL CONSTRUCTION CORPORATION vs. GUILLERMO BISCOCHO et. al.G.R. No. 166109, February 23, 2011 Facts: Exodus International Construction Corporation obtained a contract from Dutch Boy Philippines, Inc. for the painting of the Imperial Sky Garden located in Binondo, Manila. Dutch Boy awarded another contract to Exodus for the painting of Pacific Plaza, Towers in Fort Bonifacio, Taguig City. In the furtherance of its business, Exodus hired respondents as painters on different dates. On November 27, 2000, respondents filed a complaint for illegal dismissal and non-payment of holiday pay, service incentive leave pay, 13th month pay and night-shift differential pay. Petitioners denied respondents' allegations. As regards Gregorio, petitioners averred that he absented himself from work and applied as a painter with SAEI-EEI which is the general building contractor of Pacific Plaza Towers. Since then, he never reported back to work. Guillermo absented himself from work without leave. When he reported for work the following day, he was reprimanded so he worked only halfday and thereafter was unheard of until the filing of the instant complaint. Fernando, Ferdinand, and Miguel were caught eating during working hours for which they were reprimanded by their foreman. Since then they no longer reported for work. The Labor Arbiter exonerated Exodus from the charge of illegal dismissal as respondents chose not to report for work. Since there is neither illegal dismissal nor abandonment of job, respondents were ordered be reinstated but without any backwages. Issues: WON respondents were illegally dismissed for abandonment of work WON they are regular employees, thus entitled to reinstatement Ruling: (1) No. There was no dismissal, much less illegal, and there was also no abandonment of job to speak of. As found by the Labor Arbiter, there was no evidence that respondents were dismissed nor were they prevented from returning to their work. It was only respondents' unsubstantiated conclusion that they were dismissed. As a matter of fact, respondents could not name the particular person who effected their dismissal and under what particular circumstances. Absent any showing of an overt or positive act proving that petitioners had dismissed respondents, the latters' claim of illegal dismissal cannot be sustained. Indeed, a cursory examination of the records reveal no illegal dismissal to speak of. The Labor Arbiter is also correct in ruling that there was no abandonment on the part of respondents that would justify their dismissal from their employment. Abandonment is the deliberate and unjustified refusal of an employee to resume his employment. It is a settled rule that mere absence or failure to report for work is not enough to amount to abandonment of work. To constitute abandonment of work, two elements must concur: the employee must have failed to report for work or must have been absent without valid or justifiable reason and there must have been a clear intention on the part of the employee to sever the employer-employee relationship manifested by some overt act.

Page 56

LABOR RELATIONS
Atty. Jefferson M. Marquez
It is the employer who has the burden of proof to show a deliberate and unjustified refusal of the employee to resume his employment without any intention of returning." It is therefore incumbent upon petitioners to ascertain the respondents' interest or non-interest in the continuance of their employment. However, petitioners failed to do so. Petitioners posit that the reinstatement of respondents to their former positions, which were no longer existing, is impossible, highly unfair and unjust. The project was already completed by petitioners, having completed their tasks, their positions automatically ceased to exist. Consequently, there were no more positions where they can be reinstated as painters. (2) Respondents are regular employees of petitioners. It is clear from the records that when one project is completed, respondents were automatically transferred to the next project awarded to petitioners. There was no employment agreement given to respondents which clearly spelled out the duration of their employment, the specific work to be performed and that such is made clear to them at the time of hiring. It is now too late for petitioners to claim that respondents are project employees whose employment is coterminous with each project or phase of the project to which they are assigned. Nonetheless, assuming that respondents were initially hired as project employees, a project employee may acquire the status of a regular employee. The evidence on record shows that respondents were employed and assigned continuously to the various projects of petitioners. As painters, they performed activities which were necessary and desirable in the usual business of petitioners, who are engaged in subcontracting jobs for painting of residential units, condominium and commercial buildings. As regular employees, respondents are entitled to be reinstated without loss of seniority rights. Respondents are also entitled to their money claims such as the payment of holiday pay, service incentive leave pay, and 13th month pay. However, they cannot be entitled to backwages. In cases where there is no evidence of dismissal, the remedy is reinstatement but without backwages.

30. Leyte Geothermal Power Progressive Employees Union v. Phil National Oil Co., G.R. No. 176351, March 30, 2011 Facts: [Respondent Philippine National Oil Corporation]-Energy Development Corporation [PNOC-EDC] is a government-owned and controlled corporation engaged in exploration, development, utilization, generation and distribution of energy resources like geothermal energy. Petitioner is a legitimate labor organization, duly registered with the Department of Labor and Employment (DOLE) Regional Office No. VIII, Tacloban City. Among [respondent's] geothermal projects is the Leyte Geothermal Power Project located at the Greater Tongonan Geothermal Reservation in Leyte. The said Project is composed of the Tongonan 1 Geothermal Project (T1GP) and the Leyte Geothermal Production Field Project (LGPF) which provide the power and electricity needed not only in the provinces and cities of Central and Eastern Visayas (Region VII and VIII), but also in the island of Luzon as well. Thus, the [respondent] hired and employed hundreds of employees on a contractual basis, whereby, their employment was only good up to the completion or termination of the project and would automatically expire upon the completion of such project. Majority of the employees hired by [respondent] in its Leyte Geothermal Power Projects had become members of petitioner. In view of that circumstance, the petitioner demands from the [respondent] for recognition of it as the collective bargaining agent of said employees and for a CBA negotiation with it. However, the [respondent] did not heed such demands of the petitioner. Sometime in 1998 when the project was about to be completed, the [respondent] proceeded to serve Notices of Termination of Employment upon the employees who are members of the petitioner. On December 28, 1998, the petitioner filed a Notice of Strike with DOLE against the [respondent] on the ground of purported commission by the latter of unfair labor practice for "refusal to bargain collectively, union busting and mass termination." On the same day, the petitioner declared a strike and staged such strike. To avert any work stoppage, then Secretary of Labor Bienvenido E. Laguesma intervened and issued the Order, dated January 4, 1999, certifying the labor dispute to the NLRC for compulsory arbitration. Accordingly, all the striking workers were directed to return to work within twelve (12) hours from receipt of the Order and for the [respondent] to accept them back under the same terms and conditions of employment prior to the strike. Further, the parties were directed to cease and desist from committing any act that would exacerbate the situation. However, despite earnest efforts on the part of the Secretary of Labor and Employment to settle the dispute amicably, the petitioner remained adamant and unreasonable in its position, causing the failure of the negotiation towards a peaceful compromise. In effect, the petitioner did not abide by [the] assumption order issued by the Secretary of Labor. Consequently, on January 15, 1999, the [respondent] filed a Complaint for Strike Illegality, Declaration of Loss of Employment and Damages at the NLRC-RAB VIII in Tacloban City and at the same time, filed a Petition for Cancellation of Petitioner's Certificate of Registration with DOLE, Regional Office No. VIII. The two cases were later on consolidated pursuant to the New NLRC Rules of Procedure. The consolidated case was docketed as NLRC Certified Case No. V-

Page 57

LABOR RELATIONS
Atty. Jefferson M. Marquez
02-99 (NCMB-RAB VIII-NS-12-0190-98; RAB Case No. VIII-1-0019-99). The said certified case was indorsed to the NLRC 4th Division in Cebu City on June 21, 1999 for the proper disposition thereof. Issue:S Whether the officers and members of petitioner Union are project employees of respondent; and Whether the officers and members of petitioner Union engaged in an illegal strike. Ruling: On the first issue, petitioner Union contends that its officers and members performed activities that were usually necessary and desirable to respondent's usual business. In fact, petitioner Union reiterates that its officers and members were assigned to the Construction Department of respondent as carpenters and masons, and to other jobs pursuant to civil works, which are usually necessary and desirable to the department. Petitioner Union likewise points out that there was no interval in the employment contract of its officers and members, who were all employees of respondent, which lack of interval, for petitioner Union, "manifests that the `undertaking' is usually necessary and desirable to the usual trade or business of the employer." We cannot subscribe to the view taken by petitioner Union. The distinction between a regular and a project employment is provided in Article 280, paragraph 1, of the Labor Code: ART. 280. Regular and Casual Employment.-- The provisions of written agreement to the contrary notwithstanding and regardless of the oral agreement of the parties, an employment shall be deemed to be regular where the employee has been engaged to perform activities which are usually necessary or desirable in the usual business or trade of the employer, except where the employment has been fixed for a specific project or undertaking the completion or termination of which has been determined at the time of the engagement of the employee or where the work or service to be performed is seasonal in nature and the employment is for the duration of the season. An employment shall be deemed to be casual if it is not covered by the preceding paragraph: Provided, That, any employee who has rendered at least one year of service, whether such service is continuous or broken, shall be considered a regular employee with respect to the activity in which he is employed and his employment shall continue while such actually exists. The foregoing contemplates four (4) kinds of employees: (a) regular employees or those who have been "engaged to perform activities which are usually necessary or desirable in the usual business or trade of the employer"; (b) project employees or those "whose employment has been fixed for a specific project or undertaking[,] the completion or termination of which has been determined at the time of the engagement of the employee"; (c) seasonal employees or those who work or perform services which are seasonal in nature, and the employment is for the duration of the season; and (d) casual employees or those who are not regular, project, or seasonal employees. Jurisprudence has added a fifth kind-- a fixed-term employee. Article 280 of the Labor Code, as worded, establishes that the nature of the employment is determined by law, regardless of any contract expressing otherwise. The supremacy of the law over the nomenclature of the contract and the stipulations contained therein is to bring to life the policy enshrined in the Constitution to "afford full protection to labor." Thus, labor contracts are placed on a higher plane than ordinary contracts; these are imbued with public interest and therefore subject to the police power of the State. However, notwithstanding the foregoing iterations, project employment contracts which fix the employment for a specific project or undertaking remain valid under the law: x x x By entering into such a contract, an employee is deemed to understand that his employment is coterminous with the project. He may not expect to be employed continuously beyond the completion of the project. It is of judicial notice that project employees engaged for manual services or those for special skills like those of carpenters or masons, are, as a rule, unschooled. However, this fact alone is not a valid reason for bestowing special treatment on them or for invalidating a contract of employment. Project employment contracts are not lopsided agreements in favor of only one party thereto. The employer's interest is equally important as that of the employee[s'] for theirs is the interest that propels economic activity. While it may be true that it is the employer who drafts project employment contracts with its business interest as overriding consideration, such contracts do not, of necessity, prejudice the employee. Neither is the employee left helpless by a prejudicial employment contract. After all, under the law, the interest of the worker is paramount. In the case at bar, the records reveal that the officers and the members of petitioner Union signed employment contracts indicating the specific project or phase of work for which they were hired, with a fixed period of employment. The NLRC correctly disposed of this issue: A deeper examination also shows that [the individual members of petitioner Union] indeed signed and accepted the [employment contracts] freely and voluntarily. No evidence was presented by [petitioner] Union to prove improper pressure or undue influence when they entered, perfected and consummated [the employment] contracts. In fact, it was clearly established in the course of the trial of this case, as explained by no less than the President of [petitioner] Union, that the contracts of employment were read, comprehended, and voluntarily accepted by them. x x x. As clearly shown by [petitioner] Union's own admission, both parties had executed the contracts freely and voluntarily without force, duress or

Page 58

LABOR RELATIONS
Atty. Jefferson M. Marquez
acts tending to vitiate the worker[s'] consent. Thus, we see no reason not to honor and give effect to the terms and conditions stipulated therein. x x x. Thus, we are hard pressed to find cause to disturb the findings of the NLRC which are supported by substantial evidence. It is well-settled in jurisprudence that factual findings of administrative or quasi-judicial bodies, which are deemed to have acquired expertise in matters within their respective jurisdictions, are generally accorded not only respect but even finality, and bind the Court when supported by substantial evidence. Rule 133, Section 5 defines substantial evidence as "that amount of relevant evidence which a reasonable mind might accept as adequate to justify a conclusion." Consistent therewith is the doctrine that this Court is not a trier of facts, and this is strictly adhered to in labor cases. We may take cognizance of and resolve factual issues, only when the findings of fact and conclusions of law of the Labor Arbiter or the NLRC are inconsistent with those of the CA. In the case at bar, both the NLRC and the CA were one in the conclusion that the officers and the members of petitioner Union were project employees. Nonetheless, petitioner Union insists that they were regular employees since they performed work which was usually necessary or desirable to the usual business or trade of the Construction Department of respondent. Policy Instruction No. 12 of the Department of Labor and Employment discloses that the concept of regular and casual employees was designed to put an end to casual employment in regular jobs, which has been abused by many employers to prevent so - called casuals from enjoying the benefits of regular employees or to prevent casuals from joining unions. The same instructions show that the proviso in the second paragraph of Art. 280 was not designed to stifle small-scale businesses nor to oppress agricultural land owners to further the interests of laborers, whether agricultural or industrial. What it seeks to eliminate are abuses of employers against their employees and not, as petitioners would have us believe, to prevent small-scale businesses from engaging in legitimate methods to realize profit. Hence, the proviso is applicable only to the employees who are deemed "casuals" but not to the "project" employees nor the regular employees treated in paragraph one of Art. 280. Clearly, therefore, petitioners being project employees, or, to use the correct term, seasonal employees, their employment legally ends upon completion of the project or the [end of the] season. The termination of their employment cannot and should not constitute an illegal dismissal.

31. St. Paul College Quezon City vs. Ancheta II, G.R. No. 169905, September 7, 2011 Facts: Remigio Michael Ancheta was a full-time probationary teacher in the School Year 1996-1997 which was renewed in the following SY 1997-1998. His wife, Cynthia was hired as a part time teacher of the Mass Communication Department in the second semester of SY 1996-1997 and her appointment was renewed for SY 1997-1998. On February 13, 1998, respondents signified their intentions to renew their contracts for SY 1998-1999. They were later sent two letters informing them that the school is extending to them new contracts for SY 1998-1999. Thereafter, a letter was written to Remigio Michael, enumerating the departmental and instructional policies that spouses failed to comply with, such as the late submission of final grades, failure to submit final test questions to the Program Coordinator, the giving of tests in the essay form instead of the multiple choice format as mandated by the school, failure to report to work on time; the high number of students with failing grades in the classes that they handled, and not being open to suggestions to improve themselves as teachers, among others. Thereafter, Sr. Bernadette (Department Coordinator) endorsed the immediate termination of the teaching services of the spouses. Respondent spouses were given an opportunity to comment on the letter-recommendation. Subsequently however, they received their respective letters of termination. Thus, spouses filed a Complaint for illegal dismissal. St. Paul contends that it did not extend the contracts of respondent spouses. Although, it has sent letters to the spouses informing them that the school is extending to them new contracts for the coming school year, the letters do not constitute as actual employment contracts but merely offers to teach on the said school year. Issues: WON respondents were considered regular employees WON they were illegally dismissed

Page 59

LABOR RELATIONS
Atty. Jefferson M. Marquez
Ruling: (1) Employment on probationary status of teaching personnel is that they are not governed purely by the Labor Code. The Labor Code is supplemented with respect to the period of probation by special rules found in the Manual of Regulations for Private Schools. On the matter of probationary period, Section 92 of these regulations provides: Section 92.Probationary Period. Subject in all instances to compliance with the Department and school requirements, the probationary period for academic personnel shall not be more than three (3) consecutive years of satisfactory service for those in the elementary and secondary levels, six (6) consecutive regular semesters of satisfactory service for those in the tertiary level, and nine (9) consecutive trimesters of satisfactory service for those in the tertiary level where collegiate courses are offered on a trimester basis. A probationary employee or probationer is one who is on trial for an employer, during which the latter determines whether or not he is qualified for permanent employment. The probationary employment is intended to afford the employer an opportunity to observe the fitness of a probationary employee while at work, and to ascertain whether he will become an efficient and productive employee. While the employer observes the fitness, propriety and efficiency of a probationer to ascertain whether he is qualified for permanent employment, the probationer, on the other hand, seeks to prove to the employer that he has the qualifications to meet the reasonable standards for permanent employment. Thus, the word probationary, as used to describe the period of employment, implies the purpose of the term or period, not its length. The common practice is for the employer and the teacher to enter into a contract, effective for one school year. At the end of the school year, the employer has the option not to renew the contract, particularly considering the teacher's performance. If the contract is not renewed, the employment relationship terminates. If the contract is renewed, usually for another school year, the probationary employment continues. Again, at the end of that period, the parties may opt to renew or not to renew the contract. If renewed, this second renewal of the contract for another school year would then be the last year since it would be the third school year of probationary employment. At the end of this third year, the employer may now decide whether to extend a permanent appointment to the employee, primarily on the basis of the employee having met the reasonable standards of competence and efficiency set by the employer. For the entire duration of this three-year period, the teacher remains under probation. Upon the expiration of his contract of employment, being simply on probation, he cannot automatically claim security of tenure and compel the employer to renew his employment contract. (2) No. Section 91 of the Manual of Regulations for Private Schools, states that: Section 91.Employment Contract. Every contract of employment shall specify the designation, qualification, salary rate, the period and nature of service and its date of effectivity, and such other terms and condition of employment as may be consistent with laws and rules, regulations and standards of the school. A copy of the contract shall be furnished the personnel concerned. It is important that the contract of probationary employment specify the period or term of its effectivity. The failure to stipulate its precise duration could lead to the inference that the contract is binding for the full three-year probationary period. Therefore, the letters sent by petitioner Sr. Bernadette, which were void of any specifics cannot be considered as contracts. The closest they can resemble to are that of informal correspondence among the said individuals. As such, petitioner school has the right not to renew the contracts of the respondents, the old ones having been expired at the end of their terms. Assuming, arguendo, that the employment contracts between the school and the spouses were renewed, this Court finds that there was a valid and just cause for their dismissal. The Labor Code commands that before an employer may legally dismiss an employee from the service, the requirement of substantial and procedural due process must be complied with. Under the requirement of substantial due process, the grounds for termination of employment must be based on just or authorized causes. Of the charges against Remigio Michael, his spouse also shared the same defenses and admissions as to the charges against her. The plain admissions of the charges against them were the considerations taken into account by the petitioner school in their decision not to renew the respondent spouses' employment contracts. This is a right of the school that is mandated by law and jurisprudence. It is the prerogative of the school to set high standards of efficiency for its teachers since quality education is a mandate of the Constitution. As long as the standards fixed are reasonable and not arbitrary, courts are not at liberty to set them aside. Schools cannot be required to adopt standards which barely satisfy criteria set for government recognition. The same academic freedom grants the school the autonomy to decide for itself the terms and conditions for hiring its teacher, subject of course to the overarching limitations under the Labor Code. The authority to hire is likewise covered and protected by its management prerogative the right of an employer to regulate all aspects of employment, such as hiring, the freedom to prescribe work assignments, working methods, process to be followed, regulation regarding transfer of employees, supervision of their work, lay-off and discipline, and dismissal and recall of workers.

Page 60

LABOR RELATIONS
Atty. Jefferson M. Marquez
32. Lynvil Fishing Enterprises vs. Ariola, G.R. No. 181974, February 1, 2012 Facts: Petitioner Lynvil Fishing Enterprises, Inc. (Lynvil) is engaged in deep-sea fishing. Respondents services were engaged in various capacities: Andres G. Ariola, captain; Jessie D. Alcovendas, chief mate; Jimmy B. Calinao, chief engineer; Ismael G. Nubla, cook; Elorde Baez, oiler; and Leopoldo G. Sebullen, bodegero. On Aug. 1, 1998, Lynvil received a report from Ramonito Clarido, one of its employees, that on July 31, 1998, he witnessed that while on board the company vessel Analyn VIII, respondents conspired with one another and stole eight tubs of pampano and tangigue fish and delivered them to another vessel. Petitioner filed a criminal complaint against respondents before the office of the City Prosecutor of Malabon City which found probable cause for indictment of respondents for the crime of qualified theft. Relying on the finding and Nasipit Lumber Company v. NLRC, 257 Phil. 937 (1989), Lynvil asserted there was sufficient basis for valid termination of employment of respondents based on serious misconduct and/or loss of trust and confidence. Issues: Whether a finding of the city prosecutor of probable cause to indict employees of qualified theft is sufficient basis for valid termination for serious misconduct and/or loss of trust or confidence? Whether the employees were validly terminated? Ruling: On the first issue, the Supreme Court ruled in the negative. We ruled that proof beyond reasonable doubt of an employees misconduct is not required when loss of confidence is the ground for dismissal. It is sufficient if the employer has some basis to lose confidence or that the employer has reasonable ground to believe or to entertain the moral conviction that the employee concerned is responsible for the misconduct and that the nature of his participation therein rendered him absolutely unworthy of the trust and confidence demanded by his position. Lynvil cannot argue that since the Office of the Prosecutor found probable cause for theft the Labor Arbiter must follow the finding as a valid reason for the termination of respondents employment. The proof required for purposes that differ from one and the other are likewise different. On the second question, the Court stated that nonetheless, even without reliance on the prosecutors finding, we find that there was valid cause for respondents dismissal. Just cause is required for a valid dismissal. The Labor Code provides that an employer may terminate an employment based on fraud or willful breach of the trust reposed on the employee. Such breach is considered willful if it is done intentionally, knowingly, and purposely, without justifiable excuse, as distinguished from an act done carelessly, thoughtlessly, heedlessly or inadvertently. It must also be based on substantial evidence and not on the employers whims or caprices or suspicions otherwise, the employee would eternally remain at the mercy of the employer. Loss of confidence must not be indiscriminately used as a shield by the employer against a claim that the dismissal of an employee was arbitrary. And, in order to constitute a just cause for dismissal, the act complained of must be work-related and shows that the employee concerned is unfit to continue working for the employer. In addition, loss of confidence as a just cause for termination of employment is premised on the fact that the employee concerned holds a position of responsibility, trust and confidence or that the employee concerned is entrusted with confidence with respect to delicate matters, such as the handling or care and protection of the property and assets of the employer. The betrayal of this trust is the essence of the offense for which an employee is penalized. Breach of trust is present in this case. However, Lynvil contends that it cannot be guilty of illegal dismissal because the private respondents were employed under a fixed-term contract which expired at the end of the voyage. Contrarily, the private respondents (employees) contend that they became regular employees by reason of their continuous hiring and performance of tasks necessary and desirable in the usual trade and business of Lynvil. Jurisprudence, laid two conditions for the validity of a fixed-contract agreement between the employer and employee: first, the fixed period of employment was knowingly and voluntarily agreed upon by the parties without any force, duress, or improper pressure being brought to bear upon the employee and absent any other circumstances vitiating his consent; or second, it satisfactorily appears that the employer and the employee dealt with each other on more or less equal terms with no moral dominance exercised by the former or the latter. In the context of the facts that: (1) the respondents were doing tasks necessarily to Lynvils fishing business with positions ranging from captain

Page 61

LABOR RELATIONS
Atty. Jefferson M. Marquez
of the vessel to bodegero; (2) after the end of a trip, they will again be hired for another trip with new contracts; and (3) this arrangement continued for more than ten years, the clear intention is to go around the security of tenure of the respondents as regular employees. And respondents are so by the express provisions of the second paragraph of Article 280, thus: xxx Provided, That any employee who has rendered at least one year of service, whether such service is continuous or broken, shall be considered a regular employee with respect to the activity in which he is employed and his employment shall continue while such activity exists. Having found that respondents are regular employees who may be, however, dismissed for cause as we have so found in this case, there is a need to look into the procedural requirement of due process in Section 2, Rule XXIII, Book V of the Rules Implementing the Labor Code. It is required that the employer furnish the employee with two written notices: (1) a written notice served on the employee specifying the ground or grounds for termination, and giving to said employee reasonable opportunity within which to explain his side; and (2) a written notice of termination served on the employee indicating that upon due consideration of all the circumstances, grounds have been established to justify his termination. In this case, it is clear that the employees were not given the final written notices of dismissal. The Court ruled that since employees were dismissed for just cause, they were not entitle to separation pay and backwages. However, they were to be granted nominal damages for failure of the employer to comply with statutory due process.

33. D.M. Consunji Inc. vs. Jamin, G.R. No. 192514, April 18, 2012 citing Maraguinot Facts: On December 17, 1968, petitioner D.M. Consunji, Inc. (DMCI), a construction company, hired respondent Estelito L. Jamin as a laborer. Sometime in 1975, Jamin became a helper carpenter. Since his initial hiring, Jamins employment contract had been renewed a number of times. On March 20, 1999, his work at DMCI was terminated due to the completion of the SM Manila project. This termination marked the end of his employment with DMCI as he was not rehired again. On April 5, 1999, Jamin filed a complaint for illegal dismissal, with several money claims (including attorneys fees), against DMCI and its President/General Manager, David M. Consunji. Jamin alleged that DMCI terminated his employment without a just and authorized cause at a time when he was already 55 years old and had no independent source of livelihood. He claimed that he rendered service to DMCI continuously for almost 31 years. In addition to the schedule of projects (where he was assigned) submitted by DMCI to the labor arbiter, he alleged that he worked for three other DMCI projects: Twin Towers, Ritz Towers, from July 29, 1980 to June 12, 1982; New Istana Project, B.S.B. Brunei, from June 23, 1982 to February 16, 1984; and New Istana Project, B.S.B. Brunei, from January 24, 1986 to May 25, 1986. DMCI denied liability. It argued that it hired Jamin on a project-to-project basis, from the start of his engagement in 1968 until the completion of its SM Manila project on March 20, 1999 where Jamin last worked. With the completion of the project, it terminated Jamins employment. It alleged that it submitted a report to the Department of Labor and Employment (DOLE) everytime it terminated Jamins services. ISSUE Whether there was violation of security of tenure. RULING Jamin worked for DMCI for almost 31 years, initially as a laborer and, for the most part, as a carpenter. Through all those years, DMCI treated him as a project employee, so that he never obtained tenure. On the surface and at first glance, DMCI appears to be correct. Jamin entered into a contract of employment (actually an appointment paper to which he signified his conformity) with DMCI either as a field worker, a temporary worker, a casual employee, or a project employee everytime DMCI needed his services and a termination of employment paper was served on him upon completion of every project or phase of the project where he worked. The CA pierced the cover of Jamins project employment contract and declared him a regular employee who had been dismissed without cause and without notice. To reiterate, the CAs findings were based on: (1) Jamins repeated and successive engagements in DMCIs construction projects, and (2) Jamins performance of activities necessary or desirable in DMCIs usual trade or business. We agree with the CA. In Liganza v. RBL Shipyard Corporation, the Court held that "[a]ssuming, without granting[,] that [the] petitioner was initially hired for specific projects or undertakings, the repeated re-hiring and continuing need for his services for over eight (8) years have undeniably made him a regular employee." We find the Liganza ruling squarely applicable to this case, considering that for almost 31 years, DMCI had repeatedly, continuously and successively engaged Jamins services since he was hired on December 17, 1968 or for a total of 38

Page 62

LABOR RELATIONS
Atty. Jefferson M. Marquez
times 35 as shown by the schedule of projects submitted by DMCI to the labor arbiter and three more projects or engagements added by Jamin, which he claimed DMCI intentionally did not include in its schedule so as to make it appear that there were wide gaps in his engagements. We reviewed Jamins employment contracts as the CA did and we noted that while the contracts indeed show that Jamin had been engaged as a project employee, there was an almost unbroken string of Jamins rehiring from December 17, 1968 up to the termination of his employment on March 20, 1999. While the history of Jamins employment (schedule of projects) relied upon by DMCI shows a gap of almost four years in his employment for the period between July 28, 1980 (the supposed completion date of the Midtown Plaza project) and June 13, 1984 (the start of the IRRI Dorm IV project), the gap was caused by the companys omission of the three projects. For not disclosing that there had been other projects where DMCI engaged his services, Jamin accuses the company of suppressing vital evidence that supports his contention that he rendered service in the companys construction projects continuously and repeatedly for more than three decades. The non-disclosure might not have constituted suppression of evidence it could just have been overlooked by the company but the oversight is unfair to Jamin as the non-inclusion of the three projects gives the impression that there were substantial gaps not only of several months but years in his employment with DMCI. To reiterate, Jamins employment history with DMCI stands out for his continuous, repeated and successive rehiring in the companys construction projects. In all the 38 projects where DMCI engaged Jamins services, the tasks he performed as a carpenter were indisputably necessary and desirable in DMCIs construction business. He might not have been a member of a work pool as DMCI insisted that it does not maintain a work pool, but his continuous rehiring and the nature of his work unmistakably made him a regular employee. In Maraguinot, Jr. v. NLRC, the Court held that once a project or work pool employee has been: (1) continuously, as opposed to intermittently, rehired by the same employer for the same tasks or nature of tasks; and (2) these tasks are vital, necessary and indispensable to the usual business or trade of the employer, then the employee must be deemed a regular employee. Further, as we stressed in Liganza, "[r]espondent capitalizes on our ruling in D.M. Consunji, Inc. v. NLRC which reiterates the rule that the length of service of a project employee is not the controlling test of employment tenure but whether or not the employment has been fixed for a specific project or undertaking the completion or termination of which has been determined at the time of the engagement of the employee." "Surely, length of time is not the controlling test for project employment. Nevertheless, it is vital in determining if the employee was hired for a specific undertaking or tasked to perform functions vital, necessary and indispensable to the usual business or trade of the employer. Here, [private] respondent had been a project employee several times over. His employment ceased to be coterminous with specific projects when he was repeatedly re-hired due to the demands of petitioners business." Without doubt, Jamins case fits squarely into the employment situation just quoted.

Page 63

LABOR RELATIONS
Atty. Jefferson M. Marquez
MANAGEMENT PREROGATIVE

1. Dosch vs. NLRC, 123 SCRA 296 [1983] Facts: Helmut Dosch, an American citizen, married to a Filipina, was the resident Manager of Northwest Airlines, Inc. in the Philippines. He has to his credit eleven (11) years of continuous service with the company, including nine (9) years as Northwest Manager with station at Manila. He received an inter-office communication from R.C. Jenkins, Northwest's Vice President for Orient Region based in Tokyo, promoting him to the position of Director of International Sales and transferring him to Northwest's General Office in Minneapolis, U.S.A., effective immediately. Dosch in his letter, expressed appreciation for the promotion and at the same time regretted that "for personal reasons and reasons involving his family, he is unable to accept a transfer from the Philippines and that he would, therefore, prefer to remain in his position, of Manager-Philippines until such time that his services in that capacity are no longer required by the company. Petitioner tried to resume his duties as Manager after an authorized vacation but the Vice-President for the Orient Region of Northwest advised petitioner that in view of his letter, his status as an employee of the company ceased on the close of business and the company therefore considers his letter to be a resignation without notice. Issues: Does the employers letter constitute a transfer as a valid exercise of a management prerogative? Assuming arguendo that the communication or letter of Mr. Jenkins was basically a transfer, under the particular and peculiar facts obtaining in the case at bar, does Dosch's inability or refusal to be transferred a valid cause for dismissal? Rulings: No, Jenkins letter is a letter directing the promotion of Dosch from his position as Philippine manager to Director of International Sales in Minneapolis, U.S.A. It is not merely a transfer order alone but as the Solicitor General correctly observes, "it is more in the nature of a promotion that a transfer, the latter being merely incidental to such promotion." The inter-office communication of Vice President Jenkins is captioned "Transfer" but it is basically and essentially a promotion for the nature of an instrument is characterized not by the title given to it but by its body and contents. The communication informed the petitioner that effective August 18, 1975, he was to be promoted to the position of Director of International Sales, and his compensation would be upgraded and the payroll accordingly adjusted. Petitioner was, therefore, advanced to a higher position and rank and his salary was increased and that is a promotion. It has been held that promotion denotes a scalar ascent of an officer or an employee to another position, higher either in rank or salary. In the Millares case above, the Supreme Court, speaking thru Acting Chief Justice J.B.L. Reyes, distinguished between transfer and promotion as follows: "A transfer is a movement from one position to another of equivalent rank, level or salary, without break in the service. Promotion, on the other hand, is the advancement from one position to another with an increase in duties and responsibilities as authorized by law, and usually accompanied by an increase in salary. Whereas, promotion denotes a scalar ascent of a senior officer or employee to another position, higher either in rank or salary, transfer refers to lateral movement from one position to another, of equivalent rank, level or salary." There is no law that compels an employee to accept a promotion, as a promotion is in the nature of a gift or a reward, which a person has a right to refuse. When petitioner refused to accept his promotion to Director of International Sales, he was exercising a right and he cannot be punished for it as qui jure suo utitur neminem laedit. He who uses his own legal right injures no one. No, assuming for the sake of argument that the communication or letter of Mr. Jenkins was basically a transfer, under the particular and peculiar facts obtaining in the case at bar, petitioner's inability or his refusal to be transferred was not a valid cause for dismissal. While it may be true that the right to transfer or reassign an employee is an employer's exclusive right and the prerogative of management, such right is not absolute. The right of an employer to freely select or discharge his employee is limited by the paramount police power for the relations between capital and labor are not merely contractual but impressed with public interest (Article 1700, New Civil Code). And neither capital nor labor shall act oppressively against each other (Article 1701, New Civil Code). There can be no dispute that the constitutional guarantee of security of tenure mandated under Section 9, Article 2, 1973 Constitution applies to all employees and laborers, whether in the government service or in the private sector. The fact that petitioner is a managerial employee does not by itself exclude him from the protection of the constitutional guarantee of security of tenure. Even a manager in a private concern has the right to be secure in his position, to decline a promotion where, although the promotion carries an increase in his salary and rank but results in his transfer to a new place of assignment or station and away from his family. Such an order constitutes removal without just cause and is illegal. Indeed, the outright dismissal of petitioner from his position as ManagerPhilippines of Northwest Airlines is much too severe, considering the length of service that petitioner has rendered for eleven (11) fruitful and loyal years, a strong and vital factor that must be taken into account in labor law determinations which this Court, speaking thru Chief Justice Fernando in Meracap vs. International Ceramics Manufacturing Co., Inc., L-48235-36, July 30, 1979, 92 SCRA 412 emphasized should not only be secundum rationem but also secundum caritatem, to wit: "It would imply at the very least that where a penalty less punitive would suffice,

Page 64

LABOR RELATIONS
Atty. Jefferson M. Marquez
whatever missteps may be committed by labor ought not to be visited with a consequence so severe. It is not only because of the law's concern for the workingman. There is, in addition his family to consider. Unemployment brings untold hardships and sorrows on those dependent on the wage-earner. The misery and pain attendant on the loss of jobs then could be avoided if there be acceptance of the view that under all the circumstances of this case, petitioners should not be deprived of their means of livelihood. Nor is this to condone what had been done by them. For all this to condone what had been done by them. For all this while, since private respondent considered them separated from the service, they had not been paid. For the strictly juridical standpoint, it cannot be too strongly stressed, to follow Davis in his masterly work, Discretionary Justice, that where a decision may be made to rest on informed judgment rather than rigid rules, all the equities of the case must be accorded their due weight. Finally, labor law determinations, to quote from Bultmann, should be not only secundum rationem but also secundum caritatem." (This excerpt was cited in Almira vs. B.F. Goodrich Philippines, Inc., 58 SCRA 120, 131.)

2. PT&T v. Court of Appeals, G.R. No. 152057, September 23, 2003 Facts: The petitioner is a domestic corporation engaged in the business of providing telegraph and communication services thru its branches all over the country. It employed various employees, among whom were herein private respondents. Sometime in 1997, after conducting a series of studies regarding the profitability of its retail operations, its existing branches and the number of employees, the petitioner came up with a Relocation and Restructuring Program designed to (a) sustain its (PT&Ts) retail operations; (b) decongest surplus workforce in some branches, to promote efficiency and productivity; (c) lower expenses incidental to hiring and training new personnel; and (d) avoid retrenchment of employees occupying redundant positions. On August 11, 1997, private respondents received separate letters from the petitioner, giving them the option to choose the branch to which they could be transferred. Thereafter, the private respondents and other petitioners employees were directed to relocate to their new PT&T Branches. The affected employees were directed to report to their respective relocation assignments in a Letter dated September 16, 1997. The petitioner offered benefits/allowances to those employees who would agree to be transferred under its new program. The private respondents rejected the petitioners offer. Their main concern is the proximity of their transfer; that it would be burdensome for them to leave their families and relocate to those areas. Dissatisfied with this explanation, the petitioner considered the private respondents refusal as insubordination and willful disobedience to a lawful order; hence, the private respondents were dismissed from work. They forthwith filed their respective complaints against the petitioner before the appropriate sub-regional branches of the NLRC. Subsequently, the private respondents bargaining agent, PT&T Workers Union-NAFLU-KMU, filed a complaint against the petitioner for illegal dismissal and unfair labor practice before the arbitration branch of the NLRC. In their position paper, the complainants (herein private respondents) declared that their refusal to transfer could not possibly give rise to a valid dismissal on the ground of willful disobedience, as their transfer was prejudicial and inconvenient; thus unreasonable. For its part, the petitioner (respondent therein) alleged that the private respondents transfers were made in the lawful exercise of its management prerogative and were done in good faith. The transfers were aimed at decongesting surplus employees and detailing them to a more demanding branch. Issue: WON the transfer was tantamount to a promotion. And if it is, WON private respondents committed insubordination in refusing such promotion/transfer which would then justify PT&Ts act of dismissing them in exercise of management prerogative. Ruling: The petition is denied due course. YES, the transfer was indeed in a nature of a promotion. Hence, there can be no insubordination in refusing such promotion and subsequently, there can be no valid justification in dismissing the private respondents. In its position with the labor arbiter, the petitioner adverted that when the private respondents were transferred, they were also promoted, thus: Clearly, the transfer of the complainants is not unreasonable nor does it involve demotion in rank. They are being moved to branches where the complainants will function with maximum benefit to the company and they were in fact promoted not demoted from a lower job-grade to a higher job-grade and receive even higher salaries than before. Thus, transfer of the complainants would not also result in diminution in pay benefit and privilege since the salaries of the complainant would be receiving a bigger salary if not the same salary plus additional special relocation

Page 65

LABOR RELATIONS
Atty. Jefferson M. Marquez
package. Although the increase in the pay is not significant this however would be translated into an increase rather than decrease in their salary because the complainants who were transferred from the city to the province would greatly benefit because it is of judicial notice that the cost of living in the province is much lower than in the city. This would mean a higher purchasing power of the same salary previously being received by the complainants. Indeed, the increase in the respondents responsibility can be ascertained from the scalar ascent of their job grades. With or without a corresponding increase in salary, the respective transfer of the private respondents was in fact promotions, following the ruling enunciated in Homeowners Savings and Loan Association, Inc. v. NLRC: [P]romotion, as we defined in Millares v, Subido, is the advancement from one position to another with an increase in duties and responsibilities as authorized by law, and usually accompanied by an increase in salary. Apparently, the indispensable element for there to be a promotion is that there must be an advancement from one position to another or an upward vertical movement of the employees rank or position. Any increase in salary should only be considered incidental but never determinative of whether or not a promotion is bestowed upon an employee. This can be likened to the upgrading of salaries of government employees without conferring upon the, the concomitant elevation to the higher positions. The admissions of the petitioner are conclusive on it. An employee cannot be promoted, even if merely as a result of a transfer, without his consent. A transfer that results in promotion or demotion, advancement or reduction or a transfer that aims to lure the employee away from his permanent position cannot be done without the employees consent. There is no law that compels an employee to accept a promotion for the reason that a promotion is in the nature of a gift or reward, which a person has a right to refuse. Hence, the exercise by the private respondents of their right cannot be considered in law as insubordination, or willful disobedience of a lawful order of the employer. As such, there was no valid cause for the private respondents dismissal. As the questioned dismissal is not based on any of the just or valid grounds under Article 282 of the Labor Code, the NLRC correctly ordered the private respondents reinstatement without loss of seniority rights and the payment of backwages from the time of their dismissal up to their actual reinstatement. IN LIGHT OF THE ALL THE FOREGOING, the Decision of the Court of Appeals dated June 15, 2001 is hereby AFFIRMED. SO ORDERED.

3. Mendoza vs. Rural Bank of Lucban, G.R. No. 155421, July 7, 2004 Facts: On April 25, 1999, the Board of Directors of the Rural Bank of Lucban, Inc., issued Board Resolution Nos. 99-52 and 99-53, which read: Board Res. No. 99-52 RESOLVED AS IT IS HEREBY RESOLVED that in line with the policy of the bank to familiarize bank employees with the various phases of bank operations and further strengthen the existing internal control system[,] all officers and employees are subject to reshuffle of assignments. Moreover, this resolution does not preclude the transfer of assignment of bank officers and employees from the branch office to the head office and vice-versa. Petitioner Elmer Mendoza expressed his opinion on the reshuffle in an undated letter addressed to Daya, bank board chairman, that the reshuffling deemed to be a demotion without any legal basis and is a blatant harassment on from the employer as a prelude petitioners termination in due time. That it resulted to unfair labor practice. Daya replied that it was never the intention (of management) to downgrade petitioners position, and that the reshuffle will also afford management an effective tool in providing the bank a sound internal control system/check and balance and a basis in evaluating the performance of each employee. Petitioner availed 30 days in total leave of absence and on June 24, 1999 petitioner filed a Complaint for illegal dismissal, underpayment, separation pay and damages against the Rural Bank of Lucban and/or its president, Alejo B. Daya; and its Tayabas branch manager, Briccio V. Cada. Issue:

Page 66

LABOR RELATIONS
Atty. Jefferson M. Marquez
Whether or not the reshuffling or transfer is deemed to be a demotion on petitioners position. Ruling: Management Prerogative to Transfer Employees. Jurisprudence recognizes the exercise of management prerogatives. For this reason, courts often decline to interfere in legitimate business decisions of employers. Indeed, labor laws discourage interference in employers judgments concerning the conduct of their business. The law must protect not only the welfare of employees, but also the right of employers. In the pursuit of its legitimate business interest, management has the prerogative to transfer or assign employees from one office or area of operation to another -- provided there is no demotion in rank or diminution of salary, benefits, and other privileges; and the action is not motivated by discrimination, made in bad faith, or effected as a form of punishment or demotion without sufficient cause. This privilege is inherent in the right of employers to control and manage their enterprise effectively. The right of employees to security of tenure does not give them vested rights to their positions to the extent of depriving management of its prerogative to change their assignments or to transfer them. Petitioners Transfer Lawful. Petitioners transfer was made in pursuit of respondents policy to familiarize bank employees with the various phases of bank operations and further strengthen the existing internal control system of all officers and employees. We have previously held that employees may be transferred -- based on their qualifications, aptitudes and competencies -- to positions in which they can function with maximum benefit to the company. There appears no justification for denying an employer the right to transfer employees to expand their competence and maximize their full potential for the advancement of the establishment. Petitioner was not singled out; other employees were also reassigned without their express consent. Neither was there any demotion in the rank of petitioner; or any diminution of his salary, privileges and other benefits. This fact is clear in respondents Board Resolutions, the April 30, 1999 letter of Bank President Daya to Branch Manager Cada, and the May 10, 1999 letter of Daya to petitioner.

4. Duncan Assn. of Detailman-PTFWO vs Glaxo Wellcome Phils. G.R. 162994 Facts: Petitioner Tecson was hired by respondent Glaxo Wellcome Phils. as medical representative. Tecson signed a contract of employment, which stipulates among others, that he agrees to disclose existing or future relationship with co-employees and employees of competing companies that should such relationship poses a conflict of interest, to resign from the company. Despite repeated warnings, Tecson and Bettsy, an employee of a competing company, got married. Glaxo transferred Tecson to Butuan, but he defied such orders and continued acting as medical representative in Camarines area. The National Conciliation and Mediation board rendered as valid the policy and the right to transfer. Issue: Whether or not the policy constitutes a prohibition against marriage. Ruling: No. Glaxos policy prohibiting an employee from having a relationship is a valid exercise of management prerogatives as relationships of that nature might compromise the interests of the company. Glaxo has a right to guard its trade secrets, manufacturing formulas, marketing strategies and other confidential programs and information for competitors. The right to protect its economic interests is recognized by the constitution which recognizes the right of enterprises to adopt and enforce such a policy to protect its right to reasonable returns on investments and for expansion and growth. Indeed, while our laws endeavor to give life to the constitutional policy on social justice and the protection of labor, it does not mean that every labor dispute will be decided in favor of the workers. The law also recognized that management has rights which are also entitled to respect and enforcement in the interest of fair play. The challenged company policy does not violate the equal protection clause of the constitution as such clause is addressed only to the state or those acting under color of its authority. The policy being questioned is not a policy against marriage. An employee of the company remains free to marry anyone of his or her choosing. The policy is aimed at restricting a personal prerogative that belongs only to the individual. However, an employees personal decision does not detract the employer from exercising management prerogatives to ensure maximum profit and business success.

5. Norkis Trading Co., vs. NLRC, G.R. No. 168159, August 19, 2005 Facts:

Page 67

LABOR RELATIONS
Atty. Jefferson M. Marquez
Petitioner Norkis Trading Co., Inc. employed private respondent Ma. Arlene C. Gnilo on March 8, 1990, initially trained as administration and finance officer assigned to the companys branch at Calamba, Laguna. On January 24, 2001, she was promoted as Acting Senior Branch Control Officer for Bicol Region. As such, she was instructed by her immediate superior to confirm transactions pertaining to collections and deposits of BCO Marivic Faura at Polangui. In a memorandum dated May 22, 2002, private respondent was informed about a recent company audit which disclosed that she had disregarded the detailed instructions of her superior and failed to perform her duties as a Senior Branch Control Officer. An investigation by the companys Internal Audit Group ensued and private respondent was formally charged with Negligence Resulting to Material Loss. After the hearing of the IAP was concluded, private respondent made a written Request for Re-assignment to be assigned as Cashier of the Naga Branch which is vacant and considering that she is a resident of Naga City and a mother of three growing kids. On July 29, 2002, she reiterated this request to be assigned anew in Naga City while waiting for the resolution of her case. In August and September 2002, private respondent also requested to be furnished a copy of the minutes and audit report of the IAP investigation. The company did not accede to her requests and she continued reporting at the main office performing whatever work assigned to her, such as monitoring of collections at Cubao Branch. For the period March 18-April 1, 2003, the company withheld the Transportation and Travel Allowance (TNT) being received by private respondent amounting to P7,555.00, prompting her to formally protest her questionable assignment at the Home Office in Mandaluyong City which she insisted is against her appointment as Senior BCO for Bicol Region and Samar. She wrote a letter to the management criticizing them for wanting to ease her out of the company due to a labor case filed by her husband, who also worked at Norkis for more than 13 years, and such withdrawal of her travel allowances is calculated to cause suffering on her part. She expressed that the situation has become unbearable for her so that she is forced to report back to Naga City effective March 24, 2003, there being no written order issued by the management for her to stay in the main office. Upon returning to Naga City, however, private respondent learned that the management instructed to deny her entry to the branch premises and access to company records. She was ordered to report back to the main office and to explain why no disciplinary action should be taken against her for abandonment of work, which under existing company policy, carries the penalty of dismissal. Private respondent, however, maintained her position that she could no longer report to the Home Office after the company withdrew her monthly TNT. She asserted that considering her difficult situation, she had no choice but to stick to her appointment as Senior BCO-Bicol Region and Samar there being no superseding memo changing her assignment. On April 14, 2003, private respondent received a memo from the IAP for an investigation on the charges of abandonment of work, insubordination and refusal to report back to the place of work, and directing her to attend a hearing set on April 16, 2003 at the main office, which she failed to attend because the company did not act on her request to allow her cash advances to defray her travel expenses. Her salary then was withheld, prompting her to file a case with the NLRC on April 21, 2003 for the reason that the situation had become unbearable for her tantamount to constructive dismissal, with claims for nonpayment of salaries, service incentive leave pay, 13th month pay, and praying for reinstatement with full back wages, and moral and exemplary damages, and attorneys fees. On April 30, 2003, the company terminated her services effective May 2, 2003. The Labor Arbiter found Norkis guilty of illegal dismissal and the NLRC and the Court of Appeals, on appeal, affirmed the decision. Petitioner contends that its acts were legitimate exercises of the corporations management prerogative and that the private respondent was guilty of insubordination and willful disobedience justifying her the termination. Issues: What are the scope and the limitations on the exercise of management prerogatives, particularly on the transfer of employees? What is the test to determine the validity of the transfer of employees? Ruling: Concededly, employers are allowed, under the broad concept of management prerogative, to regulate all aspects of personnel administration including hiring, work assignments, working methods, time, place and manner of work, tools to be used, processes to be followed, supervision of workers, working regulations, transfer of employees, work supervision, lay-off of workers, and the dismissal and recall of workers. It is the employers prerogative, based on its assessment and perception of its employees qualifications, aptitudes, and competence, to move them around in the various areas of its business operations in order to ascertain where they will function with maximum benefit to the company. An employees right to security of tenure does not give him such a vested right in his position as would deprive the company of its prerogative to change his assignment or transfer him where he will be most useful. When his transfer is not unreasonable, nor inconvenient, nor prejudicial to him, and it does not involve a demotion in rank or a diminution of his salaries, benefits, and other privileges, the employee may not complain that it amounts to a constructive dismissal. The managements right to transfer or re-assign its personnel, however, is not absolute as it is subject to limitations imposed by law, collective bargaining agreements, and general principles of fair play and justice. The managerial prerogative to transfer personnel must be exercised without grave abuse of discretion, bearing in mind the basic elements of justice and fair play. Having the right should not be confused with the manner in which that right is exercised. Thus, it cannot be used as a subterfuge by the employer to rid himself of an undesirable worker. In particular, the employer must be able to show that the transfer is not unreasonable, inconvenient or prejudicial to the employee; nor does it involve a demotion in rank or a diminution of his salaries, privileges and other benefits. Should the employer fail to overcome this burden of proof, the employees transfer shall be tantamount to constructive dismissal.

Page 68

LABOR RELATIONS
Atty. Jefferson M. Marquez

6. PLDT vs. Paquio, G.R. No. 152689, October 12, 2005 Facts: Petitioner Philippine Long Distance Telephone Company, Inc. (PLDT) has 27 Exchanges in its Greater Metro Manila (GMM) Network. Alfredo S. Paguio was the Head of the Garnet Exchange who sent a letter to his immediate supervisor and Asst. VP criticizing the PLDT criteria for performance rating as unfair because they depended on manpower after receiving its appraisal rating. He also suggested that the criteria failed to recognize that exchanges with new plants could easily meet the objectives of GMM compared to those with old plants. Despite Paguios criticism, Garnet Exchange, the oldest plant in GMM, obtained the top rating in the GMM. Nevertheless, Paguio reiterated his letter to Santos and objected to the performance rating as it was based only on the attainment of objectives, without considering other relevant factors. Two years later on June 1996, PLDT rebalanced the manpower of the East Center. Paguio wrote Santos and requested reconsideration of the manpower rebalancing, claiming it was unfair to Garnet Exchange because as the oldest exchange in the East Center, it was disallowed to use contractors for new installations and was not made beneficiary of the cut-over bonus. He was then was reassigned as Head for Special Assignment at the Office of the GMM East Center and asked to turn over his functions as Garnet Exchange Head to Tessie Go. Believing that his transfer was a disciplinary action, Paguio requested the first VP for a formal hearing of the charges against him and asked that his reassignment be deferred. He also filed a complaint against his supervisor for grave abuse of authority and manipulation of the East Center performance. Findings were that the memo was in order as it was based on the finding that Paguio was not a team player and cannot accept decisions of management, which is short of insubordination. He was then advised to transfer to any group in the company that may avail of his services. Likewise, another memo informed Paguio that his transfer was not in the nature of a disciplinary action that required investigation and that he agreed with the reasons of the transfer. Aggrieved, Paguio files a complaint for illegal dismissal with prayer for reinstatement and damages which was later amended to illegal demotion with prayer for reversion to old position, damages and attorneys fees. Issue: In brief, the petitioner asks this Court to resolve now the legality of Paguios transfer. Ruling: PLDT alleges that the NLRC ruling would allow a change of cause of action since the complaint alleged illegal demotion while the decision involved illegal transfer. Prefatorily, we note from the records that there has been no change of cause of action from illegal demotion to illegal transfer. Illegal demotion is a type of illegal transfer. Moreover, it is familiar and fundamental doctrine that it is not the title of the action but the allegations in the pleading that determines the nature of the action. An employer is free to regulate, according to his own discretion and judgment, all aspects of employment, including the transfer of employees. It is the employers prerogative, based on its assessment and perception of its employees qualifications, aptitudes, and competence, to deploy its employees in the various areas of its business operations in order to ascertain where they will function with maximum benefit to the company. An employees right to security of tenure does not give him such a vested right in his position as would deprive the company of its prerogative to change his assignment or transfer him where he will be most useful. Nonetheless, there are limits to the management prerogative. While it may be conceded that management is in the best position to know its operational needs, the exercise of management prerogative cannot be utilized to circumvent the law and public policy on labor and social justice. That prerogative accorded management should not defeat the very purpose for which our labor laws exist: to balance the conflicting interests of labor and management. By its very nature, management prerogative must be exercised always with the principles of fair play and justice. In particular, the employer must be able to show that the transfer is not unreasonable, inconvenient or prejudicial to the employee; nor does it involve a demotion in rank or a diminution of his salaries, privileges and other benefits. The employer bears the burden of proving that the transfer of the employee has complied with the foregoing test. In the present case, we see no credible reason for Paguios transfer except his criticisms of the companys performance evaluation methods. Based on the undisputed facts, Garnet Exchange was doing well and excelled in the performance rating. In the same way, Paguios performance was consistently rated as outstanding. There was also no proof that Paguio refused to comply with any management policy. Patently, his transfer could not be due to poor performance. Neither was it because he was needed in the new post for the new assignment was functionless and it was nothing but a title. Paguios transfer could only be caused by the managements negative reception of his comments. It is prejudicial to Paguio because it left him out for a possible promotion as he was assigned to a functionless position with neither office nor staff. Hence, transfer was not valid.

Page 69

LABOR RELATIONS
Atty. Jefferson M. Marquez
7. Star Paper Corp., vs. Simbol, G.R. No. 164774, April 12, 2006 Facts: According to the respondents, Simbol and Comia allege that they did not resign voluntarily; they were compelled to resign in view of an illegal company policy. As to respondent Estrella, she alleges that she had a relationship with co-worker Zuiga who misrepresented himself as a married but separated man. After he got her pregnant, she discovered that he was not separated. Thus, she severed her relationship with him to avoid dismissal due to the company policy. On November 30, 1999, she met an accident and was advised by the doctor at the Orthopedic Hospital to recuperate for twenty-one (21) days. She returned to work on December 21, 1999 but she found out that her name was on-hold at the gate. She was denied entry. She was directed to proceed to the personnel office where one of the staff handed her a memorandum. The memorandum stated that she was being dismissed for immoral conduct. She refused to sign the memorandum because she was on leave for twenty-one (21) days and has not been given a chance to explain. The management asked her to write an explanation. However, after submission of the explanation, she was nonetheless dismissed by the company. Due to her urgent need for money, she later submitted a letter of resignation in exchange for her thirteenth month pay. Respondents later filed a complaint for unfair labor practice, constructive dismissal, separation pay and attorneys fees. They averred that the aforementioned company policy is illegal and contravenes Article 136 of the Labor Code. Issue: Whether or not the 1995 Policy/Regulation of the company is violative of the Constitutional rights towards marriage and the family of employees and of article 136 of the Labor Code. Ruling: The Supreme Court held that The 1987 Constitution under Article II, Section 18; Article XIII, Section 3 state our policy towards the protection of labor under the following provisions. The Civil Code likewise protects labor with the following provisions such as articles 1700 and 1702. The Labor Code is the most comprehensive piece of legislation protecting labor. The case at bar involves Article 136 of the Labor Code which provides: Art. 136. It shall be unlawful for an employer to require as a condition of employment or continuation of employment that a woman employee shall not get married, or to stipulate expressly or tacitly that upon getting married a woman employee shall be deemed resigned or separated, or to actually dismiss, discharge, discriminate or otherwise prejudice a woman employee merely by reason of her marriage. In denying the contention of the petitioner company, the SC applied the two factors to justify a bona fide occupational qualification: Since the finding of a bona fide occupational qualification justifies an employers no-spouse rule, the exception is interpreted strictly and narrowly. There must be a compelling business necessity for which no alternative exists other than the discriminatory practice. To justify a bona fide occupational qualification, the employer must prove two factors: (1) that the employment qualification is reasonably related to the essential operation of the job involved; and, (2) that there is a factual basis for believing that all or substantially all persons meeting the qualification would be unable to properly perform the duties of the job. The requirement that a company policy must be reasonable under the circumstances to qualify as a valid exercise of management prerogative was also at issue in the 1997 case of Philippine Telegraph and Telephone Company v. NLRC. In said case, the employee was dismissed in violation of petitioners policy of disqualifying from work any woman worker who contracts marriage. We held that the company policy violates the right against discrimination afforded all women workers under Article 136 of the Labor Code, but established a permissible exception, viz.: A requirement that a woman employee must remain unmarried could be justified as a bona fide occupational qualification, or BFOQ, where the particular requirements of the job would justify the same, but not on the ground of a general principle, such as the desirability of spreading work in the workplace. A requirement of that nature would be valid provided it reflects an inherent quality reasonably necessary for satisfactory job performance. The cases of Duncan and PT&T instruct us that the requirement of reasonableness must be clearly established to uphold the questioned employment policy. The employer has the burden to prove the existence of a reasonable business necessity. The burden was successfully discharged in Duncan but not in PT&T. The SC does not find a reasonable business necessity in the case at bar.

Page 70

LABOR RELATIONS
Atty. Jefferson M. Marquez
Petitioners sole contention that the company did not just want to have two (2) or more of its employees related between the third degree by affinity and/or consanguinity is lame. That the second paragraph was meant to give teeth to the first paragraph of the questioned rule is evidently not the valid reasonable business necessity required by the law. It is significant to note that in the case at bar, respondents were hired after they were found fit for the job, but were asked to resign when they married a co-employee. Petitioners failed to show how the marriage of Simbol, then a Sheeting Machine Operator, to Alma Dayrit, then an employee of the Repacking Section, could be detrimental to its business operations. Neither did petitioners explain how this detriment will happen in the case of Wilfreda Comia, then a Production Helper in the Selecting Department, who married Howard Comia, then a helper in the cutter-machine. The policy is premised on the mere fear that employees married to each other will be less efficient. If we uphold the questioned rule without valid justification, the employer can create policies based on an unproven presumption of a perceived danger at the expense of an employees right to security of tenure. Petitioners contend that their policy will apply only when one employee marries a co-employee, but they are free to marry persons other than coemployees. The questioned policy may not facially violate Article 136 of the Labor Code but it creates a disproportionate effect and under the disparate impact theory, the only way it could pass judicial scrutiny is a showing that it is reasonable despite the discriminatory, albeit disproportionate, effect. The failure of petitioners to prove a legitimate business concern in imposing the questioned policy cannot prejudice the employees right to be free from arbitrary discrimination based upon stereotypes of married persons working together in one company. Decision of the CA affirmed.

8. Rivera vs. Solidbank, G.R. No. 163269, April 19, 2006 Facts: Rivera started working with Solidbank Corporation as an audit clerk since July 1, 1977. Then promoted as credit investigator, senior clerk, assistant accountant, and finally as assistant manager. Prior to his retirement, he became the Manager of the banks Credit Investigation and Appraisal Division of the Consumer's Banking Group. In the meantime, Rivera and his brother-in-law put up a poultry business in Cavite. In December 1994, Solidbank offered two retirement programs to its employees: (a) the Ordinary Retirement Program (ORP), under which an employee would receive 85% of his monthly basic salary multiplied by the number of years in service; and (b) the Special Retirement Program (SRP), under which a retiring employee would receive 250% of the gross monthly salary multiplied by the number of years in service. Rivera decided to devote his time and attention to his poultry business in Cavite and applied for retirement under the SRP. Solidbank approved the application and confirmed his separation from Solidbank on February 25, 1995. However, Solidbank required Rivera to sign an undated Release, Waiver and Quitclaim, which was notarized on March 1, 1995. He acknowledged receipt of the net proceeds of his separation and retirement benefits and promised that "he would not, at any time, in any manner whatsoever, directly or indirectly engage in any unlawful activity prejudicial to the interest of Solidbank, its parent, affiliate or subsidiary companies, their stockholders, officers, directors, agents or employees, and their successors-in-interest and will not disclose any information concerning the business of Solidbank, its manner or operation, its plans, processes, or data of any kind." He also signed in an Undertaking upon which he promised that "not to seek employment with a competitor bank or financial institution within one (1) year from February 28, 1995, and that any breach of the Undertaking or the provisions of the Release, Waiver and Quitclaim would entitle Solidbank to a cause of action against him before the appropriate courts of law. But on May 1, 1995, Rivera got employed with Equitable Banking Corporation (Equitable) as Manager of its Credit Investigation and Appraisal Division of its Consumers' Banking Group. Upon learning this, Solidbank wrote a letter dated May 18, 1995, informing Rivera that he had violated the Undertaking and demanded the return of all the monetary benefits he received in consideration of the SRP within five (5) days from receipt; otherwise, appropriate legal action would be taken against him. Issue: Whether the employment ban incorporated in the Undertaking which petitioner executed upon his retirement is unreasonable, oppressive, hence, contrary to public policy. Ruling: The petition is meritorious. There is no dispute between the parties that, in consideration for his availment of the SRP, petitioner executed the Release, Waiver and Quitclaim, and the Undertaking as supplement thereto, and that he received retirement pay amounting to P963,619.28 from respondent. We agree with petitioner's contention that the issue as to whether the post-retirement competitive employment ban incorporated in the Undertaking is against public policy is a genuine issue of fact, requiring the parties to present evidence to support their respective claims. The well-entrenched doctrine is that the law does not relieve a party from the effects of an unwise, foolish or disastrous contract, entered into

Page 71

LABOR RELATIONS
Atty. Jefferson M. Marquez
with full awareness of what he was doing and entered into and carried out in good faith. Such a contract will not be discarded even if there was a mistake of law or fact. Courts have no jurisdiction to look into the wisdom of the contract entered into by and between the parties or to render a decision different therefrom. They have no power to relieve parties from obligation voluntarily assailed, simply because their contracts turned out to be disastrous deals. On the other hand, retirement plans, in light of the constitutional mandate of affording full protection to labor, must be liberally construed in favor of the employee, it being the general rule that pension or retirement plans formulated by the employer are to be construed against it. Retirement benefits, after all, are intended to help the employee enjoy the remaining years of his life, releasing him from the burden of worrying for his financial support, and are a form of reward for being loyal to the employer. Undeniably, petitioner retired under the SRP and received P963,619.28 from respondent. However, petitioner is not proscribed, by waiver or estoppel, from assailing the post-retirement competitive employment ban since under Article 1409 of the New Civil Code, those contracts whose cause, object or purpose is contrary to law, morals, good customs, public order or public policy are inexistent or void from the beginning. Estoppel cannot give validity to an act that is prohibited by law or one that is against public policy. Respondent, as employer, is burdened to establish that a restrictive covenant barring an employee from accepting a competitive employment after retirement or resignation is not an unreasonable or oppressive, or in undue or unreasonable restraint of trade, thus, unenforceable for being repugnant to public policy. Courts should carefully scrutinize all contracts limiting a man's natural right to follow any trade or profession anywhere he pleases and in any lawful manner. But it is just as important to protect the enjoyment of an establishment in trade or profession, which its employer has built up by his own honest application to every day duty and the faithful performance of the tasks which every day imposes upon the ordinary man. What one creates by his own labor is his. Public policy does not intend that another than the producer shall reap the fruits of labor; rather, it gives to him who labors the right by every legitimate means to protect the fruits of his labor and secure the enjoyment of them to himself. 56 Freedom to contract must not be unreasonably abridged. Neither must the right to protect by reasonable restrictions that which a man by industry, skill and good judgment has built up, be denied. Consideration must be given to the employee's right to earn a living and to his ability to determine with certainty the area within which his employment ban is restituted. A provision on territorial limitation is necessary to guide an employee of what constitutes as violation of a restrictive covenant and whether the geographic scope is co-extensive with that in which the employer is doing business. In considering a territorial restriction, the facts and circumstances surrounding the case must be considered. Thus, in determining whether the contract is reasonable or not, the trial court should consider the following factors: (a) whether the covenant protects a legitimate business interest of the employer; (b) whether the covenant creates an undue burden on the employee; (c) whether the covenant is injurious to the public welfare; (d) whether the time and territorial limitations contained in the covenant are reasonable; and (e) whether the restraint is reasonable from the standpoint of public policy. We are not impervious of the distinction between restrictive covenants barring an employee to accept a post-employment competitive employment or restraint on trade in employment contracts and restraints on post-retirement competitive employment in pension and retirement plans either incorporated in employment contracts or in collective bargaining agreements between the employer and the union of employees, or separate from said contracts or collective bargaining agreements which provide that an employee who accepts post retirement competitive employment will forfeit retirement and other benefits or will be obliged to restitute the same to the employer. A post-retirement competitive employment restriction is designed to protect the employer against competition by former employees who may retire and obtain retirement or pension benefits and, at the same time, engage in competitive employment.

9. Tiu v. Platinum Plans, Inc., G.R. No. 163512, February 28, 2007 Facts: Platinum Plans Philippines, Inc. is a domestic corporation engaged in the pre-need industry. From 1987 to 1989, petitioner Daisy B. Tiu was its Division Marketing Director. Platinum Inc. rehired Tiu as Senior Assistant Vice-President and Territorial Operations Head in charge of its Hongkong and Asean operations. The parties executed a contract of employment valid for five years. On September 16, 1995, petitioner stopped reporting for work. In November 1995, she became the Vice-President for Sales of Professional Pension Plans, Inc., a corporation engaged also in the pre-need industry. Platinum Plans sued Tiu for damages before the RTC of Pasig City, Branch 261. alleging, among others, that petitioners employment with Professional Pension Plans, Inc. violated the non-involvement clause in her contract of employment: The EMPLOYEE further undertakes that during his/her engagement with EMPLOYER and in case of separation from the Company, whether voluntary or for cause, he/she shall not, for the next TWO (2) years thereafter, engage in or be involved with any corporation, association or entity, whether directly or indirectly, engaged in the same business or belonging to the same pre-need industry as the EMPLOYER. Any breach of the foregoing provision shall render the EMPLOYEE liable to the EMPLOYER in the amount of One Hundred Thousand Pesos (P100,000.00) for and as liquidated damages.

Page 72

LABOR RELATIONS
Atty. Jefferson M. Marquez
Tiu countered that the non-involvement clause was unenforceable for being against public order or public policy: First, the restraint imposed was much greater than what was necessary to afford respondent a fair and reasonable protection. Petitioner contended that the transfer to a rival company was an accepted practice in the pre-need industry. Court of Appeals and trial court ruled against Tiu. It reasoned that petitioner entered into the contract on her own will and volition. Thus, she bound herself to fulfill not only what was expressly stipulated in the contract, but also all its consequences that were not against good faith, usage, and law. The appellate court also ruled that the stipulation prohibiting non-employment for two years was valid and enforceable considering the nature of respondents business. Issue: WON the non-involvement clause is valid. Ruling: The petition is DENIED for lack of merit. Supreme Court held that said that such clause was unreasonable restraint of trade and therefore against public policy. However, a noninvolvement clause is not necessarily void for being in restraint of trade as long as there are reasonable limitations as to time, trade, and place. Nevertheless, in this case, the non-involvement clause has a time limit: two years from the time petitioners employment with respondent ends. It is also limited as to trade, since it only prohibits petitioner from engaging in any pre-need business akin to respondents. Since petitioner was the Senior Assistant Vice-President and Territorial Operations Head in charge of respondents Hongkong and Asean operations, she had been privy to confidential and highly sensitive marketing strategies of respondents business. To allow her to engage in a rival business soon after she leaves would make respondents trade secrets vulnerable especially in a highly competitive marketing environment. In sum, we find the non-involvement clause not contrary to public welfare and not greater than is necessary to afford a fair and reasonable protection to respondent.13 In any event, Article 1306 of the Civil Code provides that parties to a contract may establish such stipulations, clauses, terms and conditions as they may deem convenient, provided they are not contrary to law, morals, good customs, public order, or public policy. Article 115914 of the same Code also provides that obligations arising from contracts have the force of law between the contracting parties and should be complied with in good faith. Courts cannot stipulate for the parties nor amend their agreement where the same does not contravene law, morals, good customs, public order or public policy, for to do so would be to alter the real intent of the parties, and would run contrary to the function of the courts to give force and effect thereto.15 Not being contrary to public policy, the non-involvement clause, which petitioner and respondent freely agreed upon, has the force of law between them, and thus, should be complied with in good faith. Thus, as held by the trial court and the Court of Appeals, petitioner is bound to pay respondent P100,000 as liquidated damages. While we have equitably reduced liquidated damages in certain cases, we cannot do so in this case, since it appears that even from the start, petitioner had not shown the least intention to fulfill the non-involvement clause in good faith.

10. Duldulao vs. Court of Appeals, G.R. No. 164893, March 1, 2007 Facts: Petitioner Constancia P. Duldulao was hired by respondent Baguio Colleges Foundation (BCF) as secretary/clerk-typist and assigned to the College of Law sometime in June of 1987. In August 1996, a certain law student filed a complaint against petitioner for alleged irregularities in the performance of her work. Petitioner was told to submit her answer to the complaint and given several extensions within which to do so. Despite the extensions, she failed to submit her answer. On 1 October 1996, Dean Honorato V. Aquino of the College of Law informed respondents President, Atty. Edilberto B. Tenefrancia, of petitioners failure to file her answer and recommended the assignment of petitioner outside the College of Law, not only because of such failure to answer but also her having admitted fraternizing with students of the College. On the same day, respondents Vice President for Administration, Leonardo S. dela Cruz, issued a Department Order to Mrs. Duldulao informing her of her transfer to the Office of the Principals of the High School and Elementary Departments. On 21 January 1997, the Administrative Investigating Committee found the Department Order appropriate since it was intended to prevent the controversy between petitioner and the

Page 73

LABOR RELATIONS
Atty. Jefferson M. Marquez
complaining student from adversely affecting a harmonious relationship within the College of Law among all its constituents. On 17 February 1997, petitioner filed a complaint for constructive dismissal with prayer for moral and exemplary damages and attorneys fees before the NLRC Regional Arbitration Branch-Cordillera Administrative Region. She stated that aside from being tainted with procedural lapses in violation of her right to due process, the transfer also amounted to her demotion in rank. The NLRC dismissed the complaint for lack of merit which decision was affirmed by the Court of Appeals. Issue: Whether petitioners transfer as secretary/clerk-typist from the College of Law to the High School and Elementary Departments amounts to constructive dismissal. Ruling: There was no constructive dismissal. There is constructive dismissal if an act of clear discrimination, insensibility, or disdain by an employer becomes so unbearable on the part of the employee that it would foreclose any choice by him except to forego his continued employment. It exists where there is cessation of work because "continued employment is rendered impossible, unreasonable or unlikely, as an offer involving a demotion in rank and a diminution in pay." The factual milieu in this case is different. It is the employers prerogative, based on its assessment and perception of its employees qualifications, aptitudes, and competence, to move them around in the various areas of its business operations in order to ascertain where they will function with maximum benefit to the company. An employees right to security of tenure does not give him such a vested right in his position as would deprive the company of its prerogative to change his assignment or transfer him where he will be most useful. When his transfer is not unreasonable, nor inconvenient, nor prejudicial to him, and it does not involve a demotion in rank or a diminution of his salaries, benefits, and other privileges, the employee may not complain that it amounts to a constructive dismissal. The transfer of petitioner does not amount to a demotion in rank and status. Petitioner was a secretary/clerk-typist of the College of Law. As such secretary/clerk-typist, she would only have to perform the same duties in the Office of the Principals of the High School and Elementary Departments. Petitioner was not denied due process. Reassignments made by management pending investigation of irregularities allegedly committed by an employee fall within the ambit of management prerogative. The transfer, while incidental to the pending charges against petitioner, was not meant to be a penalty, but rather a preventive measure to avoid further damage to the College of Law. The purpose of reassignments is no different from that of preventive suspension which management could validly impose as a measure of protection of the companys property pending investigation of any malfeasance or misfeasance committed by the employee.

11. Almario v. Philippine Airlines, G.R. No. 170928, September 11, 2007 Facts: Vicente S. Almario (Almario), was hired by respondent, Philippine Airlines, Inc. (PAL), as a Boeing 747 Systems Engineer. He successfully bid for the higher position of Airbus 300 (A-300) First Officer. Since said higher position required additional training, he underwent, at PALs expense, more than five months of training consisting of ground schooling in Manila and flight simulation in Melbourne, Australia. After completing the training course, Almario served as A-300 First Officer of PAL, but after eight months of service, he tendered his resignation, for personal reasons. PAL sent Almario a letter informing that his proposed resignation will make him reimburse the training costs plus damages as he is required to render 3 years of service because the company invested heavily on his professional training. Almario denied the existence of any agreement with PAL and pointed out that the CBA between PAL and Airlines Pilot Association carried no such agreement. RTC rendered judgment in favor of Almario and CA reversed the decision, hence this petition. Issue: WON Almario should reimburse the training cost. Ruling: Yes. PAL invested for the training of Almario to enable him to acquire a higher level of skill, proficiency, or technical competence so that he could efficiently discharge the position of A-300 First Officer. Given that, PAL expected to recover the training costs by availing of Almarios services for at least three years. The expectation of PAL was not fully realized, however, due to Almarios resignation after only eight months of service following the completion of his training course. He cannot, therefore, refuse to reimburse the costs of training without violating the principle of unjust enrichment. The pertinent provision of the CBA and its rationale aside, contrary to Almarios claim, Article 22 of the Civil Code applies.

Page 74

LABOR RELATIONS
Atty. Jefferson M. Marquez
Art. 22. Every person who through an act of performance by another, or any other means, acquires or comes into possession of something at the expense of the latter without just or legal ground, shall return the same to him,

12. San Miguel Corp. v. Pontillas, G.R. No. 155178, May 07, 2008 Facts: On 24 October 1980, San Miguel Corporation (petitioner) employed Angel C. Pontillas (respondent) as a daily wage company guard. In 1984 respondent became a monthly-paid employee which entitled him to yearly increases in salary. Respondent alleged that his yearly salary increases were only a percentage of what the other security guards received. On 19 October 1993, respondent filed an action for recovery of damages due to discrimination under Article 100 4 of the Labor Code, as well as for recovery of salary differential and backwages, against petitioner. Manager, issued a Memorandum ordering, among others, the transfer of responsibility of the Oro Verde Warehouse to the newly-organized VisMin Logistics Operations effecting the formal transfer of responsibility of the security personnel and equipment in the Oro Verde Warehouse. Simultaneously, the manager gave the same information to his Supervising Security Guards for them to relay the information to the company security guards. Petitioner alleged that respondent was properly notified of the transfer but he refused to receive Respondent continued to report at Oro Verde Warehouse. He alleged that he was not properly notified of the transfer and that he did not receive any written order Petitioner alleged that respondent was properly notified of the transfer but he refused to receive Petitioner also alleged that respondent was given notices of Guard Detail separately dated. but he still refused to report for duty at the VisMin Logistics Operations. In a letter, petitioner informed respondent that an administrative investigation would be conducted on relative to his alleged offenses of Insubordination or Wilful Disobedience in Carrying out Reasonable Instructions of his superior. , respondent filed an amended complaint against petitioner for illegal dismissal and payment of backwages, termination pay, moral and exemplary damages, and attorney's fees. LA: Ruled in favour of the company and against Pontillas. The Labor Arbiter recognized the management prerogative to transfer its employees from one station to another. The Labor Arbiter found nothing prejudicial, unjust, or unreasonable to petitioner's decision to merge the functions of the Materials Management of the Mandaue Brewery and the Physical Distribution Group which resulted to the forming of the VisMin Logistics Operations. The Labor Arbiter further ruled that petitioner did not violate Article 100 of the Labor Code. Labor Arbiter ruled that respondent was accorded due process before his termination from the service. He was investigated with the assistance of counsel, and he was able to confront petitioner's witnesses and present evidence in his favor. NLRC: Set aside the Labor Arbiter's Decision. The NLRC ruled that respondent was not informed of his transfer from Oro Verde Warehouse to VisMin Logistics Operations. The notices allegedly sent to respondent did not indicate any receipt from respondent. NLRC further ruled that respondent was a victim of discrimination. The NLRC declared that petitioner failed to justify why respondent was not entitled to the full rate of salary increases enjoyed by other security guards. CA: Court of Appeals affirmed with modification the NLRC's Decision. Court of Appeals ruled that under Article 282(a) of the Labor Code, as amended, an employer may terminate an employment for serious misconduct or willful disobedience by the employee of the lawful orders of his employer or his representative in connection with his work. The Court of Appeals ruled that there was no sufficient evidence that would show that respondent's failure to report to his new superior was willful and characterized by a perverse and wrongful attitude. Issue: Legality of respondent's dismissal from employment. Ruling: Respondent was dismissed for a just cause. An employer may terminate an employment for serious misconduct or wilful disobedience by the employee of the lawful orders of his employer or representative in connection with his work. Wilful disobedience requires the concurrence of two elements: (1) the employee's assailed conduct must have been wilful, that is, characterized by a wrongful and perverse attitude; and (2) the order violated must have been reasonable, lawful, made known to the employee, and must pertain to the duties which he had been engaged to discharge. Employer exercises the prerogative to transfer an employee for valid reasons and according to the requirements of its business, provided the transfer does not result in demotion in rank or diminution of the employee's salary, benefits, and other privileges. In this case, we found that the order of transfer was reasonable and lawful considering the integration of Oro Verde Warehouse with VisMin Logistics Operations. Respondent was properly informed of the transfer

Page 75

LABOR RELATIONS
Atty. Jefferson M. Marquez
but he refused to receive the notices on the pretext that he was wary because of his pending case against petitioner. REINSTATE Decision of the Labor Arbiter.

13. Bisig Manggagawa sa Tryco vs. NLRC, G.R. No. 151309, Oct. 15, 2008 Facts: Petitioners are employees of Tryco Pharmaceuticals Corporation, maker of veterinary medicines and products. Tryco and the petitioners signed a Memorandum of Agreement (MOA), providing for a compressed workweek schedule to be implemented in the company effective May 20, 1996. The MOA was entered into pursuant to Department of Labor and Employment Department Order (D.O.) No. 21, Series of 1990, Guidelines on the Implementation of Compressed Workweek. As provided in the MOA, 8:00 a.m. to 6:12 p.m., from Monday to Friday, shall be considered as the regular working hours, and no overtime pay shall be due and payable to the employee for work rendered during those hours. The MOA specifically stated that the employee waives the right to claim overtime pay for work rendered after 5:00 p.m. until 6:12 p.m. from Monday to Friday considering that the compressed workweek schedule is adopted in lieu of the regular workweek schedule which also consists of 46 hours. However, should an employee be permitted or required to work beyond 6:12 p.m., such employee shall be entitled to overtime pay. Tryco informed the Bureau of Working Conditions of the Department of Labor and Employment of the implementation of a compressed workweek in the company. In January 1997, BMT and Tryco negotiated for the renewal of their collective bargaining agreement (CBA) but failed to arrive at a new agreement. Meantime, Tryco received the Letter dated March 26, 1997 from the Bureau of Animal Industry of the Department of Agriculture reminding it that its production should be conducted in San Rafael, Bulacan, not in Caloocan City since its operating permit was licensed there. Accordingly, Tryco issued a Memorandum dated April 7, 1997 which directed petitioners to report to the company's plant site in Bulacan. BMT opposed the transfer of its members to San Rafael, Bulacan, contending that it constitutes unfair labor practice. In protest, BMT declared a strike on May 26, 1997. Petitioners then filed their complaints to the labor arbiter alleging that Tryco negotiated in bad faith and unfair labor practice of Tryco by transferring the members of the union in order to paralyze it and that therefore it amounted to constructive dismissal. Issue: Was there constructive dismissal due to the transfer of the petitioners from Caloocan City to San Rafael Bulacan? Ruling: The petition has no merit. Findings of fact of labor officials, who are deemed to have acquired expertise in matters within their respective jurisdiction, are generally accorded not only respect but even finality, and bind us when supported by substantial evidence.This is particularly true when the findings of the Labor Arbiter, the NLRC and the CA are in absolute agreement. In this case, the Labor Arbiter, the NLRC, and the CA uniformly agreed that the petitioners were not constructively dismissed. Tryco's decision to transfer its production activities to San Rafael, Bulacan, regardless of whether it was made pursuant to the letter of the Bureau of Animal Industry, was within the scope of its inherent right to control and manage its enterprise effectively. While the law is solicitous of the welfare of employees, it must also protect the right of an employer to exercise what are clearly management prerogatives. The free will of management to conduct its own business affairs to achieve its purpose cannot be denied. This prerogative extends to the management's right to regulate, according to its own discretion and judgment, all aspects of employment, including the freedom to transfer and reassign employees according to the requirements of its business.Management's prerogative of transferring and reassigning employees from one area of operation to another in order to meet the requirements of the business is, therefore, generally not constitutive of constructive dismissal. Thus, the consequent transfer of Tryco's personnel, assigned to the Production Department was well within the scope of its management prerogative. When the transfer is not unreasonable, or inconvenient, or prejudicial to the employee, and it does not involve a demotion in rank or diminution of salaries, benefits, and other privileges, the employee may not complain that it amounts to a constructive dismissal. However, the employer has the burden of proving that the transfer of an employee is for valid and legitimate grounds. The employer must show that the transfer is not unreasonable, inconvenient, or prejudicial to the employee; nor does it involve a demotion in rank or a diminution of his salaries, privileges and other benefits. Indisputably, in the instant case, the transfer orders do not entail a demotion in rank or diminution of salaries, benefits and other privileges of the petitioners. Petitioners, therefore, anchor their objection solely on the ground that it would cause them great inconvenience since they are all residents of Metro Manila and they would incur additional expenses to travel daily from Manila to Bulacan.

Page 76

LABOR RELATIONS
Atty. Jefferson M. Marquez
The Court has previously declared that mere incidental inconvenience is not sufficient to warrant a claim of constructive dismissal. Objection to a transfer that is grounded solely upon the personal inconvenience or hardship that will be caused to the employee by reason of the transfer is not a valid reason to disobey an order of transfer. Hence petition was denied for lack of merit.

14. Coca-Cola Bottlers Philippines, Inc. v. Del Villar, G.R. No. 163091, October 6, 2010 Facts: Coca-Cola hired respondent Angel U. del Villar (Del Villar) on May 1, 1990 as Physical Distribution Fleet Manager with a job grade of S-7 and monthly salary of P50,000.00, aside from the use of a company car, gasoline allowance, and annual foreign travel, among other benefits. In 1992, as part of the reorganization of the Company, Del Villar became the Transportation Services Manager, under the Business Logistic Directorate, headed by Director Edgardo I. San Juan (San Juan). As Transportation Services Manager, Del Villar prepares the budget for the vehicles of the Company nationwide. Del Villar submitted a Report to the Company President, detailing an alleged fraudulent scheme undertaken by certain Company officials in conspiracy with local truck manufacturers, overpricing the trucks purchased by the Company by as much as P70,000.00 each. Del Villar implicated San Juan and Jose L. Pineda, Jr., among other Company officials, as part of the conspiracy. Pineda then served as the Executive Assistant in the Business Logistic Directorate in charge of the Refrigeration Services of the Company. Seven months after the submission of his Report on the fraudulent scheme of several company officials, Del Villar received a Memorandum from San Juan, informing him that (1) he was designated as Staff Assistant to the Corporate Purchasing and Materials Control Manager, with a job grade of NS-VII; (2) with Del Villars new assignment, he ceased to be entitled to the benefits accruing to an S-7 position under existing company rules and policies; and (3) Del Villar was to turn over the vehicle assigned to him as Transportation Services Manager to Pineda by July 10, 1996. Although as the Staff Assistant of the Corporate Purchasing and Materials Control Manager, Del Villar continued to receive the same salary as Transportation Services Manager, but his car and other privileges were withdrawn and he spent his time at his new post sitting "at a desk with no meaningful work whatsoever." Del Villar believed that he was demoted by the Company to force him to resign. Unable to endure any further the harassment, Del Villar filed with the Arbitration Branch of the NLRC on November 11, 1996 a complaint against the Company for illegal demotion and forfeiture of company privileges. According to Coca-Cola [Del Villar] was not outrightly dismissed; instead, he was removed from his former position as Transportation Services Manager, and demoted to Staff Assistant to the Corporate Purchasing and Materials Control Manager. The Company embarked on a reorganization of the Business Logistic Directorate. As a result, the functions related to Refrigeration were assigned to the Transportation Services Manager, which was renamed the Transportation and Refrigeration Services Manager. The Company failed to appear, despite due notice, at the scheduled preliminary conference before the NLRC Arbitration Branch. The Company reasoned that in appointing Del Villar as the Staff Assistant of the Corporate Purchasing and Materials Control Manager, from his former position as Transportation Services Manager, the Company was merely exercising its inherent management prerogative to transfer an employee from one position to another. They contended that Del Villar had no vested right to the privileges he previously enjoyed as Transportation Services Manager. Since the various programs will affect some of its employees, in good faith the Company has initiated a special program called "Project New Start". This program is intended to assist employees whose positions will be declared redundant with the implementation of new distribution systems, utilization of improved operational processes and functional re-organizations. Labor Arbiter rendered a Decision in Del Villars favor. The Labor Arbiter held that the allegations in Del Villars complaint sufficiently presented a cause of action against the Company. Del Villar appears to have been singled out or discriminated upon due to his having reported the 1996 truck scam, and his present isolation can be seen as a punishment for acting in a righteous and forthright manner. Otherwise, as a "Staff Assistant" [Del Villar] should have been given some meaningful or responsible work appurtenant to the job designation. NLRC reversed the Labor Arbiter, reasoning that:virtualaw Contrary to the Labor Arbiters pronouncement that [the Company] should have rebutted allegations of bad faith and malice, we are more inclined to apply the presumption of good faith. Mere conclusions of fact and law should not be used as bases for an automatic finding of bad faith. As it

Page 77

LABOR RELATIONS
Atty. Jefferson M. Marquez
is, we do not even see any disclosure of the scam and his alleged demotion. If indeed the so-called "great grandmother of Coca cola scams of 1996" were true, the logical consequence of such disclosure is for the president of the company to dismiss the erring employees and officers for their highly irregular acts and not to penalize [Del Villar] for making such disclosure. This is amply supported by the fact that the [the Company] conducted a thorough investigation of the reported scam and even obtained the services of an independent auditor to determine whether the alleged anomalous transactions were actually irregular and/or questionable. This manifests that [Del Villars] disclosure was taken seriously contrary to his claims of discrimination. Accordingly, it cannot be said that the act of the [Company] was retaliatory or penal in nature nor tainted with bad faith and/or malice. Otherwise, [the Company] would not have given grave attention to the disclosure of [Del Villar]. A company cannot, however, be reasonably expected to provide the same benefits to an employee whose position for example, requires that he stays in the office during working hours. Benefits, privileges and perquisites that attach to a certain position do not provide sufficient bases for determining the superiority or inferiority of the position so held. Issue: Whether or not Company, in transferring Del Villar from the position of Transportation Services Manager to Staff Assistant to the Corporate Purchasing and Materials Control Manager, validly exercised its management prerogative or committed constructive dismissal, or demotion? Whether or not there has been redundancy in the position held by Del Villar that justified the company from the act of taking the position from him? Ruling: In the pursuit of its legitimate business interest, management has the prerogative to transfer or assign employees from one office or area of operation to another provided there is no demotion in rank or diminution of salary, benefits, and other privileges; and the action is not motivated by discrimination, made in bad faith, or effected as a form of punishment or demotion without sufficient cause. The right of employees to security of tenure does not give them vested rights to their positions to the extent of depriving management of its prerogative to change their assignments or to transfer them.virtuallawlibrary Managerial prerogatives, however, are subject to limitations provided by law, collective bargaining agreements, and general principles of fair play and justice. But, like other rights, there are limits thereto. The managerial prerogative to transfer personnel must be exercised without grave abuse of discretion, bearing in mind the basic elements of justice and fair play. Having the right should not be confused with the manner in which that right is exercised. Thus, it cannot be used as a subterfuge by the employer to rid himself of an undesirable worker. In particular, the employer must be able to show that the transfer is not unreasonable, inconvenient or prejudicial to the employee; nor does it involve a demotion in rank or a diminution of his salaries, privileges and other benefits. Should the employer fail to overcome this burden of proof, the employees transfer shall be tantamount to constructive dismissal, which has been defined as a quitting because continued employment is rendered impossible, unreasonable or unlikely; as an offer involving a demotion in rank and diminution in pay. Likewise, constructive dismissal exists when an act of clear discrimination, insensibility or disdain by an employer has become so unbearable to the employee leaving him with no option but to forego with his continued employment.uallawlibrary After a careful scrutiny of the records, we agree with the Labor Arbiter and the Court of Appeals that the Company failed to discharge this burden of proof. The Company and its officials attempt to justify the transfer of Del Villar by alleging his unsatisfactory performance as Transportation Services Manager. The Company disclosed that: [Del Villar] displayed an utterly woeful performance. He was unable to submit basic data as to type and brand of vehicles with highest/lowest maintenance cost as requested. [Del Villar] could not even update the records of his office. He could not work with minimum or no supervision. His activities needed to be closely and constantly monitored by his superiors. [Del Villar] lacked initiative and had to be constantly reminded of what to do. He merited a mediocre grade of 2 in a scale of one (1) to five (5), the latter number being the highest grade We are unconvinced. The dismal performance evaluations of Del Villar were prepared by San Juan and Pineda after Del Villar already implicated his two superiors in his Report dated January 4, 1996 in an alleged fraudulent scheme against the Company. More importantly, we give weight to the following instances establishing that Del Villar was not merely transferred from the position of Transportation Services Manager to the position of Staff Assistant to the Corporate Purchasing and Materials Control Manager; he was evidently demoted. A transfer is a movement from one position to another which is of equivalent rank, level or salary, without break in service. Promotion, on the other hand, is the advancement from one position to another with an increase in duties and responsibilities as authorized by law, and usually accompanied by an increase in salary. Conversely, demotion involves a situation where an employee is relegated to a subordinate or less

Page 78

LABOR RELATIONS
Atty. Jefferson M. Marquez
important position constituting a reduction to a lower grade or rank, with a corresponding decrease in duties and responsibilities, and usually accompanied by a decrease in salary. Del Villars demotion is readily apparent in his new designation. Formerly, he was the Transportation Services Manager; then he was made a Staff Assistant a subordinate to another manager, particularly, the Corporate Purchasing and Materials Control Manager. Second, the two posts are not of the same weight in terms of duties and responsibilities. Del Villars position as Transportation Services Manager involved a high degree of responsibility, he being in charge of preparing the budget for all of the vehicles of the Company nationwide. As Staff Assistant of the Corporate Purchasing and Materials Control Manager, Del Villar contended that he was not assigned any meaningful work at all. Third, while Del Villars transfer did not result in the reduction of his salary, there was a diminution in his benefits. The Company admits that as Staff Assistant of the Corporate Purchasing and Materials Control Manager, Del Villar could no longer enjoy the use of a company car, gasoline allowance, and annual foreign travel, which Del Villar previously enjoyed as Transportation Services Manager. Fourth, it was not bad enough that Del Villar was demoted, but he was even placed by the Company under the control and supervision of Pineda as the latters Staff Assistant. To recall, Pineda was one of the Company officials who Del Villar accused of defrauding the Company in his Report dated January 4, 1996. It is not too difficult to imagine that the working relations between Del Villar, the accuser, and Pineda, the accused, had been strained and hostile. The situation would be more oppressive for Del Villar because of his subordinate position vis--vis Pineda. Fifth, all the foregoing caused Del Villar inconvenience and prejudice, so unbearable for him that he was constrained to seek remedy from the NLRC. The Labor Arbiter was correct in his observation that had Del Villar resigned immediately after his "transfer," he could be said to have been constructively dismissed. There is constructive dismissal when there is a demotion in rank and/or diminution in pay; or when a clear discrimination, insensibility or disdain by an employer becomes unbearable to the employee. Eventually, however, the Company actually terminated Del Villars services effective May 31, 1998, as his position was no longer necessary or was considered redundant due to the reorganization of the Business Logistic Directorate. Redundancy is one of the authorized causes for the dismissal of an employee. It is governed by Article 283 of the Labor Code, which reads: ART. 283. Closure of establishment and reduction of personnel. The employer may also terminate the employment of any employee due to the installation of labor-saving devices, redundancy, retrenchment to prevent losses or the closing or cessation of operation of the establishment or undertaking unless the closing is for the purpose of circumventing the provisions of this Title, by serving a written notice on the workers and the Department of Labor and Employment at least one (1) month before the intended date thereof. In case of termination due to the installation of labor-saving devices or redundancy, the worker affected thereby shall be entitled to a separation pay equivalent to at least his one (1) month pay or to at least one (1) month pay for every year of service, whichever is higher. In case of retrenchment to prevent losses and in cases of closures or cessation of operations of establishment or undertaking not due to serious business losses or financial reverses, the separation pay shall be equivalent to one (1) month pay or to at least one-half (1/2) month pay for every year of service, whichever is higher. A fraction of at least six (6) months shall be considered one (1) whole year. Redundancy, for purposes of the Labor Code, exists where the services of an employee are in excess of what is reasonably demanded by the actual requirements of the enterprise. Succinctly put, a position is redundant where it is superfluous, and superfluity of a position or positions may be the outcome of a number of factors, such as overhiring of workers, decreased volume of business, or dropping of a particular product line or service activity previously manufactured or undertaken by the enterprise. The determination that the employee's services are no longer necessary or sustainable and, therefore, properly terminable for being redundant is an exercise of business judgment of the employer. The wisdom or soundness of this judgment is not subject to discretionary review of the Labor Arbiter and the NLRC, provided there is no violation of law and no showing that it was prompted by an arbitrary or malicious act. In other words, it is not enough for a company to merely declare that it has become overmanned. It must produce adequate proof of such redundancy to justify the dismissal of the affected employees. Coca Cola presented no other evidence , Neither did the Company present proof that it had complied with the procedural requirement in Article 283 of prior notice to the Department of Labor and Employment (DOLE) of the termination of Del Villars employment due to redundancy one month prior to May 31, 1998. Del Villars poor employee performance is irrelevant as regards the issue on redundancy. Redundancy arises because there is no more need for the employees position in relation to the whole business organization, and not because the employee unsatisfactorily performed the duties and responsibilities required by his position. An employee who is illegally dismissed is entitled to the twin reliefs of full backwages and reinstatement. If reinstatement is not viable, separation

Page 79

LABOR RELATIONS
Atty. Jefferson M. Marquez
pay is awarded to the employee. In awarding separation pay to an illegally dismissed employee, in lieu of reinstatement, the amount to be awarded shall be equivalent to one month salary for every year of service. Under Republic Act No. 6715, employees who are illegally dismissed are entitled to full backwages, inclusive of allowances and other benefits or their monetary equivalent, computed from the time their actual compensation was withheld from them up to the time of their actual reinstatement but if reinstatement is no longer possible, the backwages shall be computed from the time of their illegal termination up to the finality of the decision. We note that Del Villars reinstatement is no longer possible because the position he previously occupied no longer exists, per San Juans Affidavit dated October 15, 1998. Also, Del Villar had already received his separation pay sometime in October 1998.

15. Manila Electric Co. vs. Lim, G.R. No. 184769, October 5, 2010 Facts: Rosario G. Lim (respondent), also known as Cherry Lim, is an administrative clerk at the Manila Electric Company (MERALCO). On June 4, 2008, an anonymous letter was posted at the door of the Metering Office of the Administration building of MERALCO Plaridel, Bulacan Sector, at which respondent is assigned, denouncing respondent. The letter reads: Cherry Lim: MATAPOS MONG LAMUNIN LAHAT NG BIYAYA NG MERALCO, NGAYON NAMAN AY GUSTO MONG PALAMON ANG BUONG KUMPANYA SA MGA BUWAYA NG GOBYERNO. KAPAL NG MUKHA MO, LUMAYAS KA RITO, WALANG UTANG NA LOOB Resource Staffing, directed the transfer of respondent to MERALCO's Alabang Sector in Muntinlupa as "A/F OTMS Clerk," effective July 18, 2008 in light of the receipt of ". . . reports that there were accusations and threats directed against her from unknown individuals and which could possibly compromise her safety and security." Respondent, by letter of July 10, 2008 addressed to petitioner Ruben A. Sapitula, Vice-President and Head of MERALCO's Human Resource Administration, appealed her transfer and requested for a dialogue so she could voice her concerns and misgivings on the matter, claiming that the "punitive" nature of the transfer amounted to a denial of due process. Citing the grueling travel from her residence in Pampanga to Alabang and back entails, and violation of the provisions on job security of their Collective Bargaining Agreement (CBA). Respondent filed a petition for the issuance of a writ of habeas data against petitioners before the Regional Trial Court (RTC) of Bulacan. By respondent's allegation, petitioners' unlawful act and omission consisting of their continued failure and refusal to provide her with details or information about the alleged report which MERALCO purportedly received concerning threats to her safety and security amount to a violation of her right to privacy in life, liberty and security, correctible by habeas data. Respondent is essentially questioning the transfer of her place of work by her employer and the terms and conditions of her employment which arise from an employer-employee relationship over which the NLRC and the Labor Arbiters under Article 217 of the Labor Code have jurisdiction. Petitioners moved for the dismissal of the petition and recall of the TRO on the grounds that, inter alia, resort to a petition for writ of habeas data was not in order; and the RTC lacked jurisdiction over the case which properly belongs to the National Labor Relations Commission (NLRC). Issue: Whether or not, RTC has jurisdiction. Ruling: Respondent's plea that she be spared from complying with MERALCO's Memorandum directing her reassignment to the Alabang Sector, under the guise of a quest for information or data allegedly in possession of petitioners, does not fall within the province of a writ of habeas data. It is evident that respondent's reservations on the real reasons for her transfer a legitimate concern respecting the terms and conditions of one's employment are what prompted her to adopt the extraordinary remedy of habeas data. Jurisdiction over such concerns is inarguably lodged by law with the NLRC and the Labor Arbiters.

16. Bello vs. Bonifacio Security Services, G.R. No. 188086, August 3, 2011 Facts:

Page 80

LABOR RELATIONS
Atty. Jefferson M. Marquez
Respondent Bonifacio Security Services, Inc. (BSSI) is a domestic private corporation engaged in the business of providing security services. In July 2001, the BSSI hired Bello as a roving traffic marshal to manage traffic and to conduct security and safety-related operations in the Bonifacio Global City (BGC). In August 2001, Bello was posted at the Negros Navigation Company in Pier 2, North Harbor, to supervise sectoral operations. In November 2001, he was assigned at BGC as assistant detachment commander. After a week, he was transferred to Pacific Plaza Towers as assistant detachment commander and later as detachment commander. In June 2002, he was assigned at Pier 2, North Harbor as assistant detachment commander, but later reassigned to BGC. In August 2002, the BSSI hired a new operations manager, resulting in the reorganization of posts. In October 2002, Bello was assigned as roving traffic marshal at the BGC. On October 25, 2002, he filed an indefinite leave of absence when his new assignment took effect. On November 5, 2002, Bello filed a complaint against the BSSI and its General Manager, respondent Samuel Tomas, with the National Labor Relations Commission (NLRC), claiming that he had been constructively dismissed when he was demoted from a detachment commander to a mere traffic marshal. He alleged that he received a series of promotions from 2001 to 2002, from traffic marshal to supervisor, to assistant detachment commander, and to detachment commander. The BSSI denied Bello's claim of constructive dismissal, arguing that no promotion took place; Bello's designation as assistant detachment commander or detachment commander was not an employment position but a duty-related assignment; Bello abandoned his job when he went on an indefinite leave of absence and did not report for work. Labor Arbiter Cresencio G. Ramos, Jr. found that Bello was illegally dismissed, noting that the BSSI failed to adduce evidence that Bello abandoned his employment. In its March 26, 2008 resolution, the NLRC affirmed the labor arbiter's decision, finding that Bello had been constructively dismissed when he was demoted to the rank-and-file position of traffic marshal after occupying the supervisory position of assistant detachment commander and detachment commander. The CA nullified the NLRC resolutions, finding the records bereft of evidence substantiating the labor arbiter's and the NLRC's conclusions that Bello had been constructively dismissed. It noted that Bello offered no evidence to prove that there was a series of promotions that would justify his claim of subsequent demotion. The CA denied the BSSI's motion for reconsideration, paving the way for the present petition. Issue: Whether or not, Bello was illegally dismissed. Ruling: We find no reason to disturb the CA conclusion that there was no constructive dismissal. Case law defines constructive dismissal as a cessation of work because continued employment has been rendered impossible, unreasonable, or unlikely, as when there is a demotion in rank or diminution in pay, or both, or when a clear discrimination, insensibility, or disdain by an employer becomes unbearable to the employee. Other than his bare and self-serving allegations, Bello has not offered any evidence that he was promoted in a span of four months since his employment as traffic marshal in July 2001 to a detachment commander in November 2001. During his six-month probationary period of employment, it is highly improbable that Bello would be promoted after just a month of employment, from a traffic marshal in July 2001 to supervisor in August 2001, and three months later to assistant detachment commander and to detachment commander in November 2001. At most, the BSSI merely changed his assignment or transferred him to the post where his service would be most beneficial to its clients. The management's prerogative of transferring and reassigning employees from one area of operation to another in order to meet the requirements of the business is generally not constitutive of constructive dismissal. We see this to be the case in the present dispute so that the consequent reassignment of Bello to a traffic marshal post was well within the scope of the BSSI's management prerogative.

17. Alert Security and Investigation Agency vs. Pasawilan, G.R. No. 182397, September 14, 2011 Facts: Respondents Saidali Pasawilan, Wilfredo Verceles and Melchor Bulusan were all employed by petitioner Alert Security and Investigation Agency, Inc. (Alert Security) as security guards beginning March 31, 1996, January 14, 1997, and January 24, 1997, respectively. They were paid 165.00 pesos a day as regular employees, and assigned at the Department of Science and Technology (DOST) pursuant to a security service contract between the DOST and Alert Security.

Page 81

LABOR RELATIONS
Atty. Jefferson M. Marquez
Respondents aver that because they were underpaid, they filed a complaint for money claims against Alert Security and its president and general manager, petitioner Manuel D. Dasig, before Labor Arbiter Ariel C. Santos. As a result of their complaint, they were relieved from their posts in the DOST and were not given new assignments despite the lapse of six months. On January 26, 1999, they filed a joint complaint for illegal dismissal against petitioners. Petitioners, on the other hand, deny that they dismissed the respondents. Petitioners presented "Duty Detail Orders" that Alert Security issued to show that respondents were in fact assigned to LRTA. Respondents, however, failed to report at the LRTA and instead kept loitering at the DOST and tried to convince other security guards to file complaints against Alert Security. Thus, on August 3, 1998, Alert Security filed a "termination report" with the Department of Labor and Employment relative to the termination of the respondents. Issue: Whether respondents were illegally dismissed Rulings: We rule in the affirmative. As a rule, employment cannot be terminated by an employer without any just or authorized cause. No less than the 1987 Constitution in Section 3, Article 13 guarantees security of tenure for workers and because of this, an employee may only be terminated for just or authorized causes that must comply with the due process requirements mandated by law. Hence, employers are barred from arbitrarily removing their workers whenever and however they want. The law sets the valid grounds for termination as well as the proper procedure to take when terminating the services of an employee. Although we recognize the right of employers to shape their own work force, this management prerogative must not curtail the basic right of employees to security of tenure. There must be a valid and lawful reason for terminating the employment of a worker. Otherwise, it is illegal and would be dealt with by the courts accordingly. The Labor Code, as amended, enumerates several just and authorized causes for a valid termination of employment. An employee asserting his right and asking for minimum wage is not among those causes. Dismissing an employee on this ground amounts to retaliation by management for an employees legitimate grievance without due process. Such stroke of retribution has no place in Philippine Labor Laws. On the element of the failure of the employee to report for work, we also cannot accept the allegations of petitioners that respondents unjustifiably refused to report for duty in their new posts. A careful review of the records reveals that there is no showing that respondents were notified of their new assignments. Granting that the "Duty Detail Orders" were indeed issued, they served no purpose unless the intended recipients of the orders are informed of such. The employer cannot simply conclude that an employee is ipso facto notified of a transfer when there is no evidence to indicate that the employee had knowledge of the transfer order. Hence, the failure of an employee to report for work at the new location cannot be taken against him as an element of abandonment. We acknowledge and recognize the right of an employer to transfer employees in the interest of the service. This exercise is a management prerogative which is a lawful right of an employer. However, like all rights, there are limitations to the right to transfer employees. As ruled in the case of Blue Dairy Corporation v. NLRC: x x x The managerial prerogative to transfer personnel must be exercised without grave abuse of discretion, bearing in mind the basic elements of justice and fair play. Having the right should not be confused with the manner in which that right is exercised. Thus, it cannot be used as a subterfuge by the employer to rid himself of an undesirable worker. In particular, the employer must be able to show that the transfer is not unreasonable, inconvenient or prejudicial to the employee; nor does it involve a demotion in rank or a diminution of his salaries, privileges and other benefits. x x x In addition to these tests for a valid transfer, there should be proper and effective notice to the employee concerned. It is the employers burden to show that the employee was duly notified of the transfer. Verily, an employer cannot reasonably expect an employee to report for work in a new location without first informing said employee of the transfer. Petitioners insistence on the sufficiency of mere issuance of the transfer order is indicative of bad faith on their part.

18. Manila Pavilion Hotel vs. Delada, G.R. No. 189947, January 25, 2012 Facts: Delada was the Union President of the Manila Pavilion Supervisors Association at MPH. He was originally assigned as Head Waiter of

Page 82

LABOR RELATIONS
Atty. Jefferson M. Marquez
Rotisserie, a fine-dining restaurant operated by petitioner. Pursuant to a supervisory personnel reorganization program, MPH reassigned him as Head Waiter of Seasons Coffee Shop, another restaurant operated by petitioner at the same hotel. Respondent declined the inter-outlet transfer and instead asked for a grievance meeting on the matter, pursuant to their Collective Bargaining Agreement (CBA). He also requested his retention as Head Waiter of Rotisserie while the grievance procedure was ongoing. MPH replied and told respondent to report to his new assignment for the time being, without prejudice to the resolution of the grievance involving the transfer. He adamantly refused to assume his new post at the Seasons Coffee Shop and instead continued to report to his previous assignment at Rotisserie. Thus, MPH sent him several memoranda on various dates, requiring him to explain in writing why he should not be penalized for the following offenses: serious misconduct; willful disobedience of the lawful orders of the employer; gross insubordination; gross and habitual neglect of duties; and willful breach of trust. Despite the notices from MPH, Delada persistently rebuffed orders for him to report to his new assignment. According to him, since the grievance machinery under their CBA had already been initiated, his transfer must be held in abeyance. Thus, on 9 May 2007, MPH initiated administrative proceedings against him. Issue: Whether MPH retained the authority to continue with the administrative case against Delada for insubordination and willful disobedience of the transfer order. Rulings: Accordingly, we rule in this case that MPH did not lose its authority to discipline respondent for his continued refusal to report to his new assignment. In relation to this point, we recall our Decision in Allied Banking Corporation v. Court of Appeals. In Allied Banking Corporation, employer Allied Bank reassigned respondent Galanida from its Cebu City branch to its Bacolod and Tagbilaran branches. He refused to follow the transfer order and instead filed a Complaint before the Labor Arbiter for constructive dismissal. While the case was pending, Allied Bank insisted that he report to his new assignment. When he continued to refuse, it directed him to explain in writing why no disciplinary action should be meted out to him. Due to his continued refusal to report to his new assignment, Allied Bank eventually terminated his services. When the issue of whether he could validly refuse to obey the transfer orders was brought before this Court, we ruled thus: The refusal to obey a valid transfer order constitutes willful disobedience of a lawful order of an employer. Employees may object to, negotiate and seek redress against employers for rules or orders that they regard as unjust or illegal. However, until and unless these rules or orders are declared illegal or improper by competent authority, the employees ignore or disobey them at their peril. For Galanidas continued refusal to obey Allied Bank's transfer orders, we hold that the bank dismissed Galanida for just cause in accordance with Article 282(a) of the Labor Code. Galanida is thus not entitled to reinstatement or to separation pay. (Emphasis supplied, citations omitted). It is important to note what the PVA said on Deladas defiance of the transfer order: In fact, Delada cannot hide under the legal cloak of the grievance machinery of the CBA or the voluntary arbitration proceedings to disobey a valid order of transfer from the management of the hotel. While it is true that Deladas transfer to Seasons is the subject of the grievance machinery in accordance with the provisions of their CBA, Delada is expected to comply first with the said lawful directive while awaiting the results of the decision in the grievance proceedings. This issue falls squarely in the case of Allied Banking Corporation vs. Court of Appeals x x x. Pursuant to Allied Banking, unless the order of MPH is rendered invalid, there is a presumption of the validity of that order. Since the PVA eventually ruled that the transfer order was a valid exercise of management prerogative, we hereby reverse the Decision and the Resolution of the CA affirming the Decision of the PVA in this respect. MPH had the authority to continue with the administrative proceedings for insubordination and willful disobedience against Delada and to impose on him the penalty of suspension. As a consequence, petitioner is not liable to pay back wages and other benefits for the period corresponding to the penalty of 90-day suspension.

Page 83

LABOR RELATIONS
Atty. Jefferson M. Marquez
TERMINATION OF EMPLOYMENT

1. Retuya v. NLRC, G.R. No. 148848, August 5, 2003, citing Bustamante Facts: Private respondent, Insular Builders, Inc., is a family-owned corporation managed and operated principally by Antonio Murillo, father, and his son, Rodolfo Murillo. It is engaged in the construction business. Petitioners, on the other hand, were workers who have rendered services in various corporations of private respondents, namely Mindanao Integrated Builders, Inc., Sta. Clara Plywood, Inc., Insular Builders, Inc. and Queen City Builders, Inc. Early 1993, at the height of the feud between private respondents Antonio Murillo and Rodolfo Murillo, the former discharged the latter from his position as manager of Insular Builders, Inc. and assumed control of the company. Petitioners found themselves in the middle of the crossfire and were told to temporarily stop working. Later, or on July 26, 1993, private respondent Antonio Murillo dismissed petitioners and reported the matter to the Department of Labor and Employment (DOLE). Petitioners were however made to continue their work, rendering the same services, in the same place, locality and at the same office but under a different company, the Queen City Builders, Inc., managed and controlled by private respondent Rodolfo Murillo. On August 3, 1993, petitioners filed with the NLRC, Regional Arbitration Branch No. X, Davao City, a complaint for illegal dismissal, non-payment of wages, 13th month pay, and retirement pay as regards petitioner Abdon Dayson. Petitioners averred that they were terminated from employment on July 26, 1993 without prior notice and also in absence of any valid cause. They alleged that their termination was an off-shoot of the supposed personal rift and disagreements between private respondents Antonio Murillo and Rodolfo Murillo. Issue: Whether petitioners are entitled to full back wages and separation pay in accordance with Article 279 of the Labor Code HELD Bustamante v. NLRC held that illegally dismissed employees were entitled to full back wages that should not be diminished or reduced by the amount they had earned from another employment during the period of their illegal dismissal. While litigating, employees must still earn a living. Furthermore, as penalty for their illegal dismissal, their employers must pay them full back wages. This rule has been uniformly applied in subsequent cases. In the present case, petitioners were dismissed because of a "change of management." They were not given any prior written notice, but simply told that their services were terminated on the day they stopped working for Insular Builders, Inc. Under the circumstances, the CA was correct in upholding the labor arbiter's finding that they had been illegally dismissed. Having been illegally dismissed, petitioners should be awarded back wages in accordance with Bustamante v. NLRC. The fact that they worked for a sister company immediately after being dismissed from Insular Builders, Inc. should not preclude such award. Bustamante v. NLRC held that illegally dismissed employees were entitled to full back wages that should not be diminished or reduced by the amount they had earned from another employment during the period of their illegal dismissal. While litigating, employees must still earn a living. Furthermore, as penalty for their illegal dismissal, their employers must pay them full back wages. This rule has been uniformly applied in subsequent cases. In the present case, petitioners were dismissed because of a "change of management." They were not given any prior written notice, but simply told that their services were terminated on the day they stopped working for Insular Builders, Inc. Under the circumstances, the CA was correct in upholding the labor arbiter's finding that they had been illegally dismissed. Having been illegally dismissed, petitioners should be awarded back wages in accordance with Bustamante v. NLRC. The fact that they worked for a sister company immediately after being dismissed from Insular Builders, Inc. should not preclude such award. While it may be true that petitioners continued to work in the same place and office as in their previous employment, it is equally true that they had in fact been illegally dismissed by their previous employer. Thus, they lost their former work status and benefits in a manner violative of the law. Be it noted that, without their consent, their employment was changed. They became new employees of the latter firm and, as such, were

Page 84

LABOR RELATIONS
Atty. Jefferson M. Marquez
deprived of seniority and other employment benefits they had when they were still with their former employer.

2. Agabon vs. NLRC, G.R. No. 158693, November 17, 2004 Facts: Riviera Home Improvements, Inc. is engaged in the business of selling and installing ornamental and construction materials. It employed petitioners Virgilio Agabon and Jenny Agabon as gypsum board and cornice installers on January 2, 1992 until February 23, 1999 when they were dismissed for abandonment of work. Petitioners then filed a complaint for illegal dismissal and payment of money claims and on December 28, 1999. The Labor Arbiter: declared the dismissals illegal and ordered private respondent to pay the monetary claims. NLRC: reversed the Labor Arbiter because it found that the petitioners had abandoned their work, and were not entitled to backwages and separation pay. The other money claims awarded by the Labor Arbiter were also denied for lack of evidence. CA: reversed the decision of the NLRC. Hence this petition. Issue: Whether petitioners were illegally dismissed. Ruling: Negative. To dismiss an employee, the law requires not only the existence of a just and valid cause but also enjoins the employer to give the employee the opportunity to be heard and to defend himself. Article 282 of the Labor Code enumerates the just causes for termination by the employer: (a) serious misconduct or willful disobedience by the employee of the lawful orders of his employer or the latters representative in connection with the employees work; (b) gross and habitual neglect by the employee of his duties; (c) fraud or willful breach by the employee of the trust reposed in him by his employer or his duly authorized representative; (d) commission of a crime or offense by the employee against the person of his employer or any immediate member of his family or his duly authorized representative; and (e) other causes analogous to the foregoing. Abandonment is the deliberate and unjustified refusal of an employee to resume his employment. It is a form of neglect of duty, hence, a just cause for termination of employment by the employer. For a valid finding of abandonment, these two factors should be present: (1) the failure to report for work or absence without valid or justifiable reason; and (2) a clear intention to sever employer-employee relationship, with the second as the more determinative factor which is manifested by overt acts from which it may be deduced that the employees has no more intention to work. The intent to discontinue the employment must be shown by clear proof that it was deliberate and unjustified. In February 1999, petitioners were frequently absent having subcontracted for an installation work for another company. Subcontracting for another company clearly showed the intention to sever the employer-employee relationship with private respondent. This was not the first time they did this. In January 1996, they did not report for work because they were working for another company. Private respondent at that time warned petitioners that they would be dismissed if this happened again. Petitioners disregarded the warning and exhibited a clear intention to sever their employer-employee relationship. The record of an employee is a relevant consideration in determining the penalty that should be meted out to him. The law imposes many obligations on the employer such as providing just compensation to workers, observance of the procedural requirements of notice and hearing in the termination of employment. On the other hand, the law also recognizes the right of the employer to expect from its workers not only good performance, adequate work and diligence, but also good conduct and loyalty. The employer may not be compelled to continue to employ such persons whose continuance in the service will patently be inimical to his interests. After establishing that the terminations were for a just and valid cause, we now determine if the procedures for dismissal were observed. The procedure for terminating an employee is found in Book VI, Rule I, Section 2(d) of the Omnibus Rules Implementing the Labor Code: Standards of due process: requirements of notice. In all cases of termination of employment, the following standards of due process shall be substantially observed: I. For termination of employment based on just causes as defined in Article 282 of the Code: (a) A written notice served on the employee specifying the ground or grounds for termination, and giving to said employee reasonable opportunity within which to explain his side; (b) A hearing or conference during which the employee concerned, with the assistance of counsel if the employee so desires, is given opportunity to respond to the charge, present his evidence or rebut the evidence presented against him; and (c) A written notice of termination served on the employee indicating that upon due consideration of all the circumstances, grounds have been

Page 85

LABOR RELATIONS
Atty. Jefferson M. Marquez
established to justify his termination. In case of termination, the foregoing notices shall be served on the employees last known address. Dismissals based on just causes contemplate acts or omissions attributable to the employee while dismissals based on authorized causes involve grounds under the Labor Code which allow the employer to terminate employees. A termination for an authorized cause requires payment of separation pay. When the termination of employment is declared illegal, reinstatement and full backwages are mandated under Article 279. If reinstatement is no longer possible where the dismissal was unjust, separation pay may be granted. Procedurally, (1) if the dismissal is based on a just cause under Article 282, the employer must give the employee two written notices and a hearing or opportunity to be heard if requested by the employee before terminating the employment: a notice specifying the grounds for which dismissal is sought a hearing or an opportunity to be heard and after hearing or opportunity to be heard, a notice of the decision to dismiss; and (2) if the dismissal is based on authorized causes under Articles 283 and 284, the employer must give the employee and the Department of Labor and Employment written notices 30 days prior to the effectivity of his separation. From the foregoing rules four possible situations may be derived: (1) the dismissal is for a just cause under Article 282 of the Labor Code, for an authorized cause under Article 283, or for health reasons under Article 284, and due process was observed; (2) the dismissal is without just or authorized cause but due process was observed; (3) the dismissal is without just or authorized cause and there was no due process; and (4) the dismissal is for just or authorized cause but due process was not observed. In the first situation, the dismissal is undoubtedly valid and the employer will not suffer any liability. In the second and third situations where the dismissals are illegal, Article 279 mandates that the employee is entitled to reinstatement without loss of seniority rights and other privileges and full backwages, inclusive of allowances, and other benefits or their monetary equivalent computed from the time the compensation was not paid up to the time of actual reinstatement. In the fourth situation, the dismissal should be upheld. While the procedural infirmity cannot be cured, it should not invalidate the dismissal. However, the employer should be held liable for non-compliance with the procedural requirements of due process. The present case squarely falls under the fourth situation. The dismissal should be upheld because it was established that the petitioners abandoned their jobs to work for another company. Private respondent, however, did not follow the notice requirements and instead argued that sending notices to the last known addresses would have been useless because they did not reside there anymore. Unfortunately for the private respondent, this is not a valid excuse because the law mandates the twin notice requirements to the employees last known address. Thus, it should be held liable for non-compliance with the procedural requirements of due process. To be sure, the Due Process Clause in Article III, Section 1 of the Constitution embodies a system of rights based on moral principles so deeply imbedded in the traditions and feelings of our people as to be deemed fundamental to a civilized society as conceived by our entire history. Due process is that which comports with the deepest notions of what is fair and right and just. It is a constitutional restraint on the legislative as well as on the executive and judicial powers of the government provided by the Bill of Rights. Due process under the Labor Code, like Constitutional due process, has two aspects: substantive, i.e., the valid and authorized causes of employment termination under the Labor Code; and procedural, i.e., the manner of dismissal. Procedural due process requirements for dismissal are found in the Implementing Rules of P.D. 442, as amended, otherwise known as the Labor Code of the Philippines in Book VI, Rule I, Sec. 2, as amended by Department Order Nos. 9 and 10. Breaches of these due process requirements violate the Labor Code. Therefore statutory due process should be differentiated from failure to comply with constitutional due process. Constitutional due process protects the individual from the government and assures him of his rights in criminal, civil or administrative proceedings; while statutory due process found in the Labor Code and Implementing Rules protects employees from being unjustly terminated without just cause after notice and hearing. The employer should not be compelled to continue employing a person who is admittedly guilty of misfeasance or malfeasance and whose continued employment is patently inimical to the employer. The law protecting the rights of the laborer authorizes neither oppression nor self-destruction of the employer. It must be stressed that in the present case, the petitioners committed a grave offense, i.e., abandonment, which, if the requirements of due process were complied with, would undoubtedly result in a valid dismissal. An employee who is clearly guilty of conduct violative of Article 282 should not be protected by the Social Justice Clause of the Constitution. Social justice, as the term suggests, should be used only to correct an injustice. As the eminent Justice Jose P. Laurel observed, social justice must be founded on the recognition of the necessity of interdependence among diverse units of a society and of the protection that should be equally and evenly extended to all groups as a combined force in our social and economic life, consistent with the fundamental and paramount objective of the state of promoting the health, comfort, and quiet of all persons, and of bringing about the greatest good to the greatest number. Private respondent claims that the Court of Appeals erred in holding that it failed to pay petitioners holiday pay, service incentive leave pay and 13th month pay. We are not persuaded. We affirm the ruling of the appellate court on petitioners money claims. Private respondent is liable for petitioners holiday pay, service incentive leave pay and 13th month pay without deductions. As a general rule, one who pleads payment has the burden of proving it. Even where the employee must allege non-payment, the general rule is that the burden rests on the employer to prove payment, rather than on the employee to prove non-payment. The reason for the rule is that the pertinent personnel files, payrolls, records, remittances and other similar documents which will show that overtime, differentials, service incentive leave and other claims of workers have been paid are not in the possession of the worker but in the custody and absolute control of the employer. In the case at bar, if private respondent indeed paid petitioners holiday pay and service incentive leave pay, it could have easily presented documentary proofs of such monetary benefits to disprove the claims of the petitioners. But it did not, except with respect to the 13th month pay wherein it presented cash vouchers showing payments of the benefit in the years disputed. Allegations by private respondent that it does not operate during holidays and that it allows its employees 10 days leave with pay, other than being self-serving, do not constitute proof of payment. Consequently, it failed to discharge the onus probandi thereby making it liable for such claims to the petitioners.

Page 86

LABOR RELATIONS
Atty. Jefferson M. Marquez
Anent the deduction of SSS loan and the value of the shoes from petitioner Virgilio Agabons 13th month pay, we find the same to be unauthorized. The evident intention of Presidential Decree No. 851 is to grant an additional income in the form of the 13th month pay to employees not already receiving the same so as to further protect the level of real wages from the ravages of world-wide inflation. Clearly, as additional income, the 13th month pay is included in the definition of wage under Article 97(f) of the Labor Code, to wit: (f) Wage paid to any employee shall mean the remuneration or earnings, however designated, capable of being expressed in terms of money whether fixed or ascertained on a time, task, piece , or commission basis, or other method of calculating the same, which is payable by an employer to an employee under a written or unwritten contract of employment for work done or to be done, or for services rendered or to be rendered and includes the fair and reasonable value, as determined by the Secretary of Labor, of board, lodging, or other facilities customarily furnished by the employer to the employee from which an employer is prohibited under Article 113 of the same Code from making any deductions without the employees knowledge and consent. In the instant case, private respondent failed to show that the deduction of the SSS loan and the value of the shoes from petitioner Virgilio Agabons 13th month pay was authorized by the latter. The lack of authority to deduct is further bolstered by the fact that petitioner Virgilio Agabon included the same as one of his money claims against private respondent. The Court of Appeals properly reinstated the monetary claims awarded by the Labor Arbiter ordering the private respondent to pay each of the petitioners holiday pay for four regular holidays from 1996 to 1998, in the amount of P6,520.00, service incentive leave pay for the same period in the amount of P3,255.00 and the balance of Virgilio Agabons thirteenth month pay for 1998 in the amount of P2,150.00.

3. Jaka Food Processing vs. Pacot, G.R. No. 151378, March 28, 2005 Facts: Respondents Darwin Pacot, Robert Parohinog, David Bisnar, Marlon Domingo, Rhoel Lescano and Jonathan Cagabcab were earlier hired by petitioner JAKA Foods Processing Corporation (JAKA, for short) until the latter terminated their employment on August 29, 1997 because the corporation was in dire financial straits. It is not disputed, however, that the termination was effected without JAKA complying with the requirement under Article 283 of the Labor Code regarding the service of a written notice upon the employees and the Department of Labor and Employment at least one (1) month before the intended date of termination. In time, respondents separately filed with the regional Arbitration Branch of the National Labor Relations Commission (NLRC) complaints for illegal dismissal, underpayment of wages and nonpayment of service incentive leave and 13th month pay against JAKA and its HRD Manager, Rosana Castelo. After due proceedings, the Labor Arbiter rendered a decision declaring the termination illegal and ordering JAKA and its HRD Manager to reinstate respondents with full backwages, and separation pay if reinstatement is not possible. More specifically the decision dispositively reads: In time, respondents separately filed with the regional Arbitration Branch of the National Labor Relations Commission (NLRC) complaints for illegal dismissal, underpayment of wages and nonpayment of service incentive leave and 13th month pay against JAKA and its HRD Manager, Rosana Castelo. After due proceedings, the Labor Arbiter rendered a decision declaring the termination illegal and ordering JAKA and its HRD Manager to reinstate respondents with full backwages, and separation pay if reinstatement is not possible. Issues: Does the absence of the notice of hearing in dismissal due to authorize cause amounts to illegal dismissal? Are the dismissed employees, because of companys serious losses, entitled to separation pay? Ruling: A dismissal for just cause under Article 282 implies that the employee concerned has committed, or is guilty of, some violation against the employer, i.e. the employee has committed some serious misconduct, is guilty of some fraud against the employer, or, as in Agabon, he has neglected his duties. Thus, it can be said that the employee himself initiated the dismissal process. On another breath, a dismissal for an authorized cause under Article 283 does not necessarily imply delinquency or culpability on the part of the employee. Instead, the dismissal process is initiated by the employers exercise of his management prerogative, i.e. when the employer opts to install labor saving devices, when he decides to cease business operations or when, as in this case, he undertakes to implement a retrenchment program. The clear-cut distinction between a dismissal for just cause under Article 282 and a dismissal for authorized cause under Article 283 is further reinforced by the fact that in the first, payment of separation pay, as a rule, is not required, while in the second, the law requires payment of separation pay. For these reasons, there ought to be a difference in treatment when the ground for dismissal is one of the just causes under Article 282, and when based on one of the authorized causes under Article 283. Accordingly, it is wise to hold that: (1) if the dismissal is based on a just cause under Article 282 but the employer failed to comply with the notice requirement, the sanction to be imposed upon him should be tempered because the dismissal process was, in effect, initiated by an act imputable to the employee; and (2) if the dismissal is based on an authorized cause under

Page 87

LABOR RELATIONS
Atty. Jefferson M. Marquez
Article 283 but the employer failed to comply with the notice requirement, the sanction should be stiffer because the dismissal process was initiated by the employers exercise of his management prerogative. It is, therefore, established that there was ground for respondents dismissal, i.e., retrenchment, which is one of the authorized causes enumerated under Article 283 of the Labor Code. Likewise, it is established that JAKA failed to comply with the notice requirement under the same Article. Considering the factual circumstances in the instant case and the above ratiocination, we, therefore, deem it proper to fix the indemnity at P50,000.00. We likewise find the Court of Appeals to have been in error when it ordered JAKA to pay respondents separation pay equivalent to one (1) month salary for every year of service. This is because in Reahs Corporation vs. NLRC we made the following declaration: The rule, therefore, is that in all cases of business closure or cessation of operation or undertaking of the employer, the affected employee is entitled to separation pay. This is consistent with the state policy of treating labor as a primary social economic force, affording full protection to its rights as well as its welfare. The exception is when the closure of business or cessation of operations is due to serious business losses or financial reverses; duly proved, in which case, the right of affected employees to separation pay is lost for obvious reasons. xxx.

4. Mauricio v. NLRC, G.R. No. 164635, November 17, 2005 Facts: 1. Petitioner Mauricio, started working as an Administrative Assistant in the Legal Department of the Manila Banking Corporation on July 1, 1999 as a probationary employee. 2. As a pre-employment requirement, the bank directed the submission by petitioner of, among other things, a 1x1 ID picture, 2 x 2 ID picture, two reference letters, and clearance from the employee's previous employment. 3. Petitioner failed to submit the required documents. The bank gave her up to December 15, 1999 to comply, and advised her that the processing of her regularization as employee would be held in abeyance. 4. Despite the deadline given her, petitioner still failed to comply with the requirements, drawing the bank to send her a Memorandum giving her until December 29, 1999 to submit the requirements, and informing that her failure to do so would cause the termination of her employment effective December 29, 1999. 5. Petitioner, by letter of December 28, 1999, informed the bank that she could not secure a clearance from her previous employer, the Manila Bankers Life Insurance Corporation, a sister company of the bank, as she had a pending case with it. She thus requested that any action relative to her employment be held in abeyance as she was still following up the early resolution of the case. 6. By reply memorandum, the bank denied petitioner's request on the ground that the submission of the pre-employment requirements, one of which is a clearance from the previous employer if one is previously employed, is a standing policy of Manilabank applicable especially to bank officers. The bank further reasoned that, to allow an exemption to the rule that the same should be submitted prior to the expiration of the 6month probationary employment will create a precedent which will prejudice an established hiring policy, not to mention the legal implication of waiver on the part of the bank to further require submission of the clearance after the lapse of the probationary employment. However, the bank was still open in the event that petitioner will secure the required clearance form (sic) Manila Bankers Life and would consider her future employment with Manilabank. 7. Petitioner filed on January 21, 2000 a complaint for illegal dismissal, unpaid salary, and moral and exemplary damages against the bank. Issue: W/N Mauricios probationary employment was validly terminated by Manila Banking Corporation Ruling: Yes, Mauricios probationary employment was validly terminated by the bank. In denying Muricios petition, SC affirmed the Labor Arbiters decision. To quote: The submission of clearance from a previous employer is a reasonable requirement to qualify as a regular employee upon the expiration of the six months probationary employment. This reasonable regulation is mandatory in the sense that it speaks of the employee's character before he

Page 88

LABOR RELATIONS
Atty. Jefferson M. Marquez
or she becomes a regular employee. For sure, no employer in his right mind would engage the regular service of an employee unless he is certain of the moral character of a probationary employee applying as regular employee. To say that the requirement is a mere formality is an oversimplification of the long standing policy in the bank industry. In the case of San Miguel Brewery Sales Force Union vs. Ople (170 SCRA 25), the Honorable Supreme Court ruled that: Except as limited by special laws, an employer is free to regulate according to his own discretion and judgment, all aspects of employment, including hiring. The pre-employment requirements of respondent bank which was made known to complainant upon the inception of her employment were not shown to be unreasonable so as to render ineffectual complainant's denial as regular employee. In fact, complainant was given a six month period to produce the clearance requirement and as early as November 26, 1999, complainant was directed to submit the documents required for regularization but she dilly-dallied. It was only on December 28, 1999 or a day before the deadline that complainant wrote her previous employer about the issuance of the clearance requirement but considering that complainant has a pending questionably petty cash liquidation problem upon audit, the clearance cannot be issued before the expiration of the six months probationary period. The fact that complainant's previous employer cannot issue a clearance because of pending questionable transaction in which complainant is involved, it is within the province of respondents to deny the regularization of complainant.

5. Industrial Timber Corp. vs. Ababon, G.R. No. 164518, Janury 25, 2006 and March 28, 2007 Facts: Industrial Plywood Group Corporation (IPGC) is the owner of a plywood plant located at Agusan, Pequeo, Butuan City, leased to Industrial Timber Corporation (ITC) on August 30, 1985 for a period of five years. Thereafter, ITC commenced operation of the plywood plant and hired 387 workers. On March 16, 1990, ITC notified the Department of Labor and Employment (DOLE) and its workers that effective March 19, 1990 it will undergo a no plant operation due to lack of raw materials and will resume only after it can secure logs for milling. Meanwhile, IPGC notified ITC of the expiration of the lease contract in August 1990 and its intention not to renew the same. On June 26, 1990, ITC notified the DOLE and its workers of the plants shutdown due to the non-renewal of anti-pollution permit that expired in April 1990. This fact and the alleged lack of logs for milling constrained ITC to lay off all its workers until further notice. This was followed by a final notice of closure or cessation of business operations on August 17, 1990 with an advice for all the workers to collect the benefits due them under the law and CBA. On October 15, 1990, IPGC took over the plywood plant after it was issued a Wood Processing Plant Permit No. WPR-1004-081791-042, which included the antipollution permit, by the Department of Environment and Natural Resources (DENR) coincidentally on the same day the ITC ceased operation of the plant. This prompted Virgilio Ababon, et al. to file a complaint against ITC and IPGC for illegal dismissal, unfair labor practice and damages. They alleged, among others, that the cessation of ITCs operation was intended to bust the union and that both corporations are one and the same entity being controlled by one owner. Issue: Whether or not Ababon, et al. were illegally dismissed due to the closure of ITCs business; and whether they are entitled to separation pay, backwages, and other monetary awards. Ruling: Under Article 283 of the Labor Code, three requirements are necessary for a valid cessation of business operations: (a) service of a written notice to the employees and to the DOLE at least one month before the intended date thereof; (b) the cessation of business must be bona fide in character; and (c) payment to the employees of termination pay amounting to one month pay or at least one-half month pay for every year of service, whichever is higher. As borne out from the records, respondent ITC actually underwent no plant operation since 19 March 1990 due to lack of log supply. This fact is admitted by complainants (Minutes of hearing, 28 October 1991). Since then several subsequent incidents prevented respondent ITC to resume its business operations e.g. expiration and non-renewal of the wood processing plant permit, anti-pollution permit, and the lease contract on the plywood plant. Without the raw materials respondent ITC has nothing to produce. Without the permits it cannot lawfully operate the plant. And without the contract of lease respondent ITC has no option but to cease operation and turn over the plant to the lessor. Having established that ITCs closure of the plywood plant was done in good faith and that it was due to causes beyond its control, the conclusion is inevitable that said closure is

Page 89

LABOR RELATIONS
Atty. Jefferson M. Marquez
valid. Consequently, Ababon, et al. could not have been illegally dismissed to be entitled to full backwages. Thus, we find it no longer necessary to discuss the issue regarding the computation of their backwages. However, they are entitled to separation pay equivalent to one month pay or at least one-half month pay for every year of service, whichever is higher. Although the closure was done in good faith and for valid reasons, The Supreme Court find that ITC did not comply with the notice requirement. While an employer is under no obligation to conduct hearings before effecting termination of employment due to authorized cause, however, the law requires that it must notify the DOLE and its employees at least one month before the intended date of closure. In the case at bar, ITC notified its employees and the DOLE of the no plant operation on March 16, 1990 due to lack of raw materials. This was followed by a shut down notice dated June 26, 1990 due to the expiration of the anti-pollution permit. However, this shutdown was only temporary as ITC assured its employees that they could return to work once the renewal is acted upon by the DENR. On August 17, 1990, the ITC sent its employees a final notice of closure or cessation of business operations to take effect on the same day it was released. We find that this falls short of the notice requirement for termination of employment due to authorized cause considering that the DOLE was not furnished and the notice should have been furnished both the employees and the DOLE at least one month before the intended date of closure. In Agabon v. National Labor Relations Commission and Jaka Food Processing Corporation v. Pacot, the Court sustained the dismissals for just cause under Article 282 and for authorized cause under Article 283 of the Labor Code, respectively, despite non-compliance with the statutory requirement of notice and hearing. The grounds for the dismissals in those cases, namely, neglect of duty and retrenchment, remained valid because the non-compliance with the notice and hearing requirement in the Labor Code did not undermine the validity of the grounds for the dismissals. Indeed, to invalidate a dismissal merely because of a procedural defect creates absurdity and runs counter to public interest. Where the dismissal is based on an authorized cause under Article 283 of the Labor Code but the employer failed to comply with the notice requirement, the sanction should be stiff as the dismissal process was initiated by the employers exercise of his management prerogative, as opposed to a dismissal based on a just cause under Article 282 with the same procedural infirmity where the sanction to be imposed upon the employer should be tempered as the dismissal process was, in effect, initiated by an act imputable to the employee.

6. Equitable Bank vs Sadac, G.R. No. 164772, June 8, 2006 Facts: Respondent Sadac was appointed Vice President of the Legal Department of petitioner Bank effective 1 August 1981, and subsequently General Counsel thereof on 8 December 1981. On 26 June 1989, nine lawyers of petitioner Banks Legal Department, in a letter-petition to the Chairman of the Board of Directors, accused respondent Sadac of abusive conduct and petitioned for a change in leadership of the department. On the ground of lack of confidence in respondent Sadac, under the rules of client and lawyer relationship, petitioner Bank instructed respondent Sadac to deliver all materials in his custody in all cases in which the latter was appearing as its counsel of record. In reaction thereto, respondent Sadac requested for a full hearing and formal investigation but the same remained unheeded. On 9 November 1989, respondent Sadac filed a complaint for illegal dismissal with damages against petitioner Bank and individual members of the Board of Directors thereof. After learning of the filing of the complaint, petitioner Bank terminated the services of respondent Sadac. Finally, on 10 August 1989, respondent Sadac was removed from his office and ordered disentitled to any compensation and other benefits. The issues on the existence of employer-employee relationship as well as the legality of the dismissal were decided by the Court in the case of Equitable Banking Corporation vs. NLRC, G.R. No. 102467, June 13, 1997, wherein it was held that there was employer-employee relationship between petitioner Bank and respondent Sadac and that Sadac's dismissal was illegal, his dismissal not being grounded on any of the causes stated in Article 282 of the Labor Code and there being disregard of the procedural requirements in terminating Sadac's employment. In said case, respondent Sadac was held to be entitled to backwages from termination of employment until turning sixty (60) years of age (in 1995) and, thereupon, to retirement benefits in accordance with law. He shall also be paid an additional amount of P5,000.00. The controversy in the present case arises on the computation of backwages wherein respondent Sadac included the general salary increases in said computation while petitioner Bank insists that the general salary increases should not be included in the computation of backwages but rather the computation should be based on the wage rate at the time of dismissal. Issue: Whether or not general salary increases should be included in the base figure to be used in the computation of backwages. Ruling: General salary increases are not included in the base figure to be used in the computation of backwages. Article 279 mandates that an employees full backwages shall be inclusive of allowances and other benefits or their monetary equivalent. A salary increase cannot be interpreted as either an allowance or a benefit. Salary increases are not akin to allowances or benefits, and cannot be confused with either. The term "allowances" is sometimes used synonymously with "emoluments," as indirect or contingent remuneration, which may or may not be earned, but which is sometimes in the nature of compensation, and sometimes in the nature of reimbursement. Allowances and benefits are

Page 90

LABOR RELATIONS
Atty. Jefferson M. Marquez
granted to the employee apart or separate from, and in addition to the wage or salary. In contrast, salary increases are amounts which are added to the employees salary as an increment thereto for varied reasons deemed appropriate by the employer. Salary increases are not separate grants by themselves but once granted, they are deemed part of the employees salary. To extend the coverage of an allowance or a benefit to include salary increases would be to strain both the imagination of the Court and the language of law. As observed by the NLRC, "to otherwise give the meaning other than what the law speaks for by itself, will open the floodgates to various interpretations." Indeed, if the intent were to include salary increases as basis in the computation of backwages, the same should have been explicitly stated in the same manner that the law used clear and unambiguous terms in expressly providing for the inclusion of allowances and other benefits. Furthermore, salary increases are a mere expectancy. They are, by its nature volatile and are dependent on numerous variables, including the companys fiscal situation and even the employees future performance on the job, or the employees continued stay in a position subject to management prerogative to transfer him to another position where his services are needed. In short, there is no vested right to salary increases. That respondent Sadac may have received salary increases in the past only proves fact of receipt but does not establish a degree of assuredness that is inherent in backwages. From the foregoing, the plain conclusion is that respondent Sadacs computation of his full backwages which includes his prospective salary increases cannot be permitted. In addition, a cursory reading of the dispositive portion of the Courts Decision of 13 June 1997 in G.R. No. 102467, awarding backwages to respondent Sadac, shows that the award of backwages therein is unqualified. Judicial precedents tell us that an unqualified award of backwages means that the employee is paid at the wage rate at the time of his dismissal. Thus, the base figure to be used in the computation of backwages is pegged at the wage rate at the time of the employees dismissal, inclusive of regular allowances that the employee had been receiving such as the emergency living allowances and the 13th month pay mandated under the law.

7. Heirs of Sara Lee vs. Rey, G.R. No. 1499013, August 31, 2006 Facts: The House of Sara Lee (petitioner) is engaged in the direct selling of a variety of product lines for men and women, including cosmetics, intimate apparels, perfumes, ready to wear clothes and other novelty items, through its various outlets nationwide. In the pursuit of its business, the petitioner engages and contracts with dealers to sell the aforementioned merchandise. These dealers, known either as Independent Business Managers (IBMs) or Independent Group Supervisors (IGSs), depending on whether they sell individually or through their own group, would obtain at discounted rates the merchandise from the petitioner on credit and then sell the same products to their own customers at fixed prices also determined by the petitioner. Cynthia Rey (respondent), at the time of her dismissal from employment, or on June 25, 1996, held the position of Credit Administration Supervisor or CAS at the Cagayan de Oro City branch of the petitioner. Sometime in June 1995, while respondent was still working in Butuan City, she allegedly instructed the Accounts Receivable Clerk of the Cagayan de Oro outlet, a certain Ms. Magi Caroline Mendoza, to change the credit term of one of the IBMs of the petitioner, a certain Ms. Mariam Rey-Petilla, who happens to be respondents sister-in-law, from the 52-day limit to an unauthorized term of 60 days. . As a consequence of the discovery of the foregoing alleged anomalous practice of extending the credit terms of certain IBMs, management undertook an audit of the Cagayan de Oro City and Butuan City branches. On the basis of the hearing, the alleged voluntary admissions of respondent, and the findings of the auditors report, the petitioner, on June 25, 1996, formally dismissed the respondent for breach of trust and confidence. On September 24, 1996, as stated above, respondent filed her Complaint for illegal dismissal, backwages and damages, with the Labor Arbiter. On April 30, 1998, the Labor Arbiter rendered a decision in favor of the respondent. Aggrieved, the petitioner appealed to the NLRC. On October 29, 1998, the NLRC rendered its decision dismissing the appeal. The petitioner appealed to the CA under Rule 65. On August 25, 2000, the CA dismissed the Petition on the sole ground that factual issues are not proper subjects for a special civil action of certiorari. Issue: Is the respondent dismissed for a just cause? Ruling: Contrary to the findings of the NLRC and the CA, the Court holds that respondent was dismissed for a just cause. Law and jurisprudence have long recognized the right of employers to dismiss employees by reason of loss of trust and confidence. More so, in the case of supervisors or personnel occupying positions of responsibility, loss of trust justifies termination. Loss of confidence as a just cause for dismissal is premised on the fact that an employee concerned holds a position of trust and confidence. This situation applies where a person is entrusted with confidence on delicate matters, such as the custody, handling, or care and protection of the employers property. But, in order to constitute a just cause for dismissal, the act complained of must be work-related, such that the employee concerned is unfit to continue working for the employer. In the present case, the respondent is not an ordinary rank-and-file employee. The nature of her work requires a substantial amount of trust and confidence on the part of the employer. Being the Credit Administration Supervisor of the Cagayan de Oro and Butuan City branches of the petitioner, respondent occupied a highly sensitive and critical position and may thus be dismissed on the ground of loss of trust and confidence. The duties of the respondent included the strict monitoring of the 38- or 52-day rolling due date of each of its IBMs and IGSs, as well as the

Page 91

LABOR RELATIONS
Atty. Jefferson M. Marquez
supervision of the credit and collection of payments and outstanding accounts due to the petitioner from its dealers. More importantly, respondent has a direct hand in the preparation and computation of the Service Fees or sales commissions accruing to each dealer. The computation of these commissions depends on whether the dealer concerned was able to remit the sales proceeds within the 38-day or 52-day rolling deadline. Clearly, respondents position involves a high degree of responsibility requiring trust and confidence. The position carried with it the duty to observe proper company procedures in the fulfillment of her job, as it relates closely to the financial interests of the company. Respondents unauthorized extensions of the credit periods of the dealers are prejudicial to the interest of the petitioner and bear serious financial implications. Employers, generally, are allowed a wider latitude of discretion in terminating the employment of managerial personnel or those of similar rank performing functions which by their nature require the employers trust and confidence, than in the case of ordinary rankand-file employees. There can be no doubt continuance in the sensitive fiduciary position of Credit Administration Supervisor would be patently inimical to the interests of the petitioner. It would be oppressive and unjust to order the petitioner to take her back, for the law, in protecting the rights of the employee, authorizes neither oppression nor self-destruction of the employer.

8. Galaxi Steel Workers Union vs. NLRC, G.R. No. 165757, October 17, 2006, citing North Davao Mining Facts: Galaxie is a corporation engaged in the business of manufacturing and sale of re-bars and steel billets which are used primarily in the construction of high-rise buildings. On account of serious business losses which occurred in 1997 up to mid-1999 totaling around P127,000,000.00, Galaxie decided to close down its business operations. Galaxie thus filed on July 30, 1999 a written notice with the DOLE informing the latter of its intended closure and the consequent termination of its employees effective August 31, 1999. And it posted the notice of closure on the corporate bulletin board. Petitioners Galaxie Steel Workers Union and Galaxie employees filed a complaint for illegal dismissal, unfair labor practice, and money claims against Galaxie. The Labor Arbiter declared valid Galaxies closure of business but nevertheless ordered it to pay the employees separation pay, pro-rata 13th month pay, and vacation and sick leave credits. The NLRC upheld the Labor Arbiters decision but reversed the award of pro-rata 13th month pay and vacation and sick leave credits, the same not being among petitioners causes of action as in fact they were not even mentioned in their pleadings. And it reversed too the award for separation pay, the closure of Galaxies business being due to serious business losses. Nevertheless, the NLRC directed Galaxie to grant petitioners, by way of financial assistance, the same amount given to the employees who had executed quitclaims. Their motion for reconsideration having been denied, petitioners filed a petition for certiorari with the Court of Appeals, arguing that the NLRC acted with grave abuse of discretion in not finding Galaxie guilty of unfair labor practice and of violating petitioners right to notice of closure, and in deleting the award of separation pay. CA upheld the NLRC decision. Hence, the present petition for review. Issues: Whether or not Galaxie is guilty of unfair labor practice in closing its business operations shortly after petitioner union filed for certification election. Whether or not petitioners are entitled to separation pay. Whether or not the written notice posted by Galaxie on the company bulletin board sufficiently complies with the notice requirement under Article 283 of the Labor Code. Ruling: Petitioners contend that the Court of Appeals erred in not finding that Galaxies closure of business operations was motivated not by serious business losses but by their anti-union stance. It is settled that SC is not a trier of facts, a rule which applies with greater force in labor cases where the findings of fact of the NLRC are accorded respect and even finality, as long as they are supported by substantial evidence from which an independent evaluation of the facts may be made. In this case, the Labor Arbiter, the NLRC, and the CA were unanimous in ruling that Galaxies closure or cessation of business operations was due to serious business losses or financial reverses, and not because of any alleged anti-union position. The Court finds no reason to modify such finding. In any event, petitioners contend that Galaxie did not serve written notices of the closure of business operations upon its employees, it having merely posted a notice on the company bulletin board. Indeed, Galaxies documentary evidence shows that it had been experiencing serious financial losses at the time it closed business operations. As aptly found by the Court of Appeals: The NLRCs finding on the legality of the closure should be upheld for it is supported by substantial evidence consisting of the audited financial statements showing that Galaxie continuously incurred losses from 1997 up to mid-1999, to wit: P65,753,480.65 in 1997, P48,429,785.89 in 1998, and P13,204,389.97 in 1999; and of the various demand notices of payments from creditor banks. Besides, the petitioners had not presented evidence to the contrary; nor did they establish that the closure was motivated by Galaxies anti-union stance. True, the union was seeking the holding of a certification election at the time that Galaxie closed its business operation, but that, without more, was not sufficient to attribute anti-unionism against Galaxie. Upon the other hand, petitioners failed to present concrete evidence supporting their claim of unfair labor practice. Unfair labor practice refers to acts that violate the workers right to organize, and are defined in Articles 248 and

Page 92

LABOR RELATIONS
Atty. Jefferson M. Marquez
261 of the Labor Code. The prohibited acts relate to the workers right to self-organization and to the observance of Collective Bargaining Agreement without which relation the acts, no matter how unfair, are not deemed unfair labor practices. Respecting petitioners claim for separation pay, in North Davao Mining Corporation v. National Labor Relations Commission, this Court held that Article 283 governs the grant of separation benefits "in case of closures or cessation of operation" of business establishments "NOT due to serious business losses or financial reverses . . ." Where, the closure then is due to serious business losses, the Labor Code does not impose any obligation upon the employer to pay separation benefits. The denial of petitioners claim for separation pay was thus in order. With regard to the notice requirement, the Labor Arbiter found, and it was upheld by the NLRC and the Court of Appeals, that the written notice of closure or cessation of Galaxies business operations was posted on the company bulletin board one month prior to its effectivity. The mere posting on the company bulletin board does not, however, meet the requirement under Article 283 of "serving a written notice on the workers." The purpose of the written notice is to inform the employees of the specific date of termination or closure of business operations, and must be served upon them at least one month before the date of effectivity to give them sufficient time to make the necessary arrangements. In order to meet the foregoing purpose, service of the written notice must be made individually upon each and every employee of the company. Nevertheless, the validity of termination of services can exist independently of the procedural infirmity in the dismissal. After analyzing the consequences of the divergent doctrines on employment termination, the Court held that in cases involving dismissals for cause, but without observance of statutory due process, the better rule is to declare that the dismissal was for cause but imposing sanctions on the employer. By so doing, dispensing justice not just to employees but to employers as well is achieved. In Business Services of the Future Today, Inc. v. Court of Appeals, which reiterated the ruling in Agabon v. National Labor Relations Commission, this Court held that where the dismissal is for an authorized cause, the lack of statutory due process should not nullify the dismissal, or render it illegal, or ineffectual. However, the employer should indemnify the employee, in the form of nominal damages, for the violation of his right to statutory due process.

9. Sy vs. Metro Bank, G.R. No. 160618, November 2, 2006 Facts: Petitioner Dennis Sy was the branch manager in Bajada, Davao City of respondent Metropolitan Bank and Trust Company. Sy would have rendered 30 years of service by August 18, 1999 under the banks retirement plan, which provides that an employee must retire upon reaching 55 years of age or after rendering 30 years of service, whichever comes earlier. However, on February 5, 1999, a few months before he was supposed to retire, the bank reappointed him as branch manager for a term of one year from August 18, 1999 until August 18, 2000, with a corresponding salary increase effective August 16, 1999. In November 1999, the bank released the results of the audit conducted. Sy, on November 15, 1999, tendered an irrevocable letter of retirement, wherein he requested the timely release of his retirement pay and other benefits. His request was denied. The bank averred that Sy has allowed client-Spouses Ong to conduct kiting activities in their account. The bank placed Sy on preventive suspension. Sy responded that he only made a wrong credit judgment. The bank, not satisfied with his answer, notified Sy of other violations of company policies. Unconvinced of Sy explanation, the bank dismissed Sy. Sy then filed a case for illegal dismissal, which was dismissed by the labor arbiter for lack of merit. On appeal, the NLRC deemed Sy compulsorily retired. When brought up to the Court of Appeals, the labor arbiters decision was reinstated. Issues: Was Sy illegally terminated? If his dismissal was valid, would he still be entitled to retirement benefits? Ruling: No. Sy was validly dismissed on the ground of fraud and willful breach of trust under Article 282 of the Labor Code. Records show that as bank manager, he authorized kiting or drawing of checks against uncollected funds in wanton violation of the banks policies. It was sufficient basis for the bank to lose trust in him. Unlike a rank-and-file worker, where breach of trust as a ground for valid dismissal requires proof of involvement in the alleged anomaly and where mere uncorroborated accusation by the employer will not suffice, the sheer existence of a basis for believing that the employers trust has been breach is enough for the dismissal of a managerial employee. Petitioner, however, theorizes that having been compulsorily retired, he could no longer be dismissed by the bank. His premise is absurd. Indeed, he would have qualified for compulsory retirement under the banks Retirement Plan. However, he opted to accept the banks offer of extending his employment for another year with a corresponding salary increase. Thus, in effect, he had never retired. Unfortunately for him, while serving such extended term, the bank discovered his unauthorized grant of accommodation to accounts engaged in kiting activity. Such act is a clear breach of the trust in him by the

Page 93

LABOR RELATIONS
Atty. Jefferson M. Marquez
bank. He cannot now elude dismissal for a just cause by claiming he was already retired compulsorily. No. Under the Labor Code, only unjustly dismissed employees are entitled to retirement benefits and other privileges including reinstatement and backwages. Since petitioners dismissal was for a just cause, he is not entitled to any retirement benefit.

10. King of Kings Transport vs. NLRC, G.R. No. 166208, June 29, 2007 Facts: Respondent Mamac was hired as bus conductor of Don Mariano Transit Corporation (DMTC) on April 29, 1999. Respondent was required to accomplish a "Conductor's Trip Report" and submit it to the company after each trip. As a background, this report indicates the ticket opening and closing for the particular day of duty. After submission, the company audits the reports. Once an irregularity is discovered, the company issues an "Irregularity Report" against the employee, indicating the nature and details of the irregularity. Thereafter, the concerned employee is asked to explain the incident by making a written statement or counter-affidavit at the back of the same Irregularity Report. After considering the explanation of the employee, the company then makes a determination of whether to accept the explanation or impose upon the employee a penalty for committing an infraction. That decision shall be stated on said Irregularity Report and will be furnished to the employee. Upon audit of the October 28, 2001 Conductor's Report of respondent, KKTI noted an irregularity. It discovered that respondent declared several sold tickets as returned tickets causing KKTI to lose an income of eight hundred and ninety pesos. While no irregularity report was prepared on the October 28, 2001 incident, KKTI nevertheless asked respondent to explain the discrepancy. In his letter, respondent said that the erroneous declaration in his October 28, 2001 Trip Report was unintentional. He explained that during that day's trip, the windshield of the bus assigned to them was smashed; and they had to cut short the trip in order to immediately report the matter to the police. As a result of the incident, he got confused in making the trip report. On November 26, 2001, respondent received a letter terminating his employment effective November 29, 2001. The dismissal letter alleged that the October 28, 2001 irregularity was an act of fraud against the company. KKTI also cited as basis for respondent's dismissal the other offenses he allegedly committed since 1999. Respondent filed a Complaint for illegal dismissal, illegal deductions, nonpayment of 13th-month pay, service incentive leave, and separation pay. He denied committing any infraction and alleged that his dismissal was intended to bust union activities. Moreover, he claimed that his dismissal was effected without due process. KKTI contended that respondent was legally dismissed after his commission of a series of misconducts and misdeeds. It claimed that respondent had violated the trust and confidence reposed upon him by KKTI. Also, it averred that it had observed due process in dismissing respondent and maintained that respondent was not entitled to his money claims such as service incentive leave and 13th-month pay because he was paid on commission or percentage basis. Issue: Whether or not procedural requirements were complied with. Ruling: Due process under the Labor Code involves two aspects: first, substantive the valid and authorized causes of termination of employment under the Labor Code; and second, procedural the manner of dismissal. Non-compliance with the Due Process Requirements Art. 277 of the Labor Code provides the manner of termination of employment, thus: Art. 277. Miscellaneous Provisions. . . . (b)Subject to the constitutional right of workers to security of tenure and their right to be protected against dismissal except for a just and authorized cause without prejudice to the requirement of notice under Article 283 of this Code, the employer shall furnish the worker whose employment is sought to be terminated a written notice containing a statement of the causes for termination and shall afford the latter ample opportunity to be heard and to defend himself with the assistance of his representative if he so desires in accordance with company rules and regulations promulgated pursuant to guidelines set by the Department of Labor and Employment. Any decision taken by the employer shall be

Page 94

LABOR RELATIONS
Atty. Jefferson M. Marquez
without prejudice to the right of the worker to contest the validity or legality of his dismissal by filing a complaint with the regional branch of the National Labor Relations Commission. The burden of proving that the termination was for a valid or authorized cause shall rest on the employer. Accordingly, the implementing rule of the aforesaid provision states: SEC. 2.Standards of due process; requirements of notice. In all cases of termination of employment, the following standards of due process shall be substantially observed: I.For termination of employment based on just causes as defined in Article 282 of the Code: (a)A written notice served on the employee specifying the ground or grounds for termination, and giving said employee reasonable opportunity within which to explain his side. (b)A hearing or conference during which the employee concerned, with the assistance of counsel if he so desires is given opportunity to respond to the charge, present his evidence, or rebut the evidence presented against him. (c)A written notice of termination served on the employee, indicating that upon due consideration of all the circumstances, grounds have been established to justify his termination. In case of termination, the foregoing notices shall be served on the employee's last known address. To clarify, the following should be considered in terminating the services of employees: (1)The first written notice to be served on the employees should contain the specific causes or grounds for termination against them, and a directive that the employees are given the opportunity to submit their written explanation within a reasonable period. "Reasonable opportunity" under the Omnibus Rules means every kind of assistance that management must accord to the employees to enable them to prepare adequately for their defense. This should be construed as a period of at least five (5) calendar days from receipt of the notice to give the employees an opportunity to study the accusation against them, consult a union official or lawyer, gather data and evidence, and decide on the defenses they will raise against the complaint. Moreover, in order to enable the employees to intelligently prepare their explanation and defenses, the notice should contain a detailed narration of the facts and circumstances that will serve as basis for the charge against the employees. A general description of the charge will not suffice. Lastly, the notice should specifically mention which company rules, if any, are violated and/or which among the grounds under Art. 282 is being charged against the employees. (2)After serving the first notice, the employers should schedule and conduct a hearing or conference wherein the employees will be given the opportunity to: (1) explain and clarify their defenses to the charge against them; (2) present evidence in support of their defenses; and (3) rebut the evidence presented against them by the management. During the hearing or conference, the employees are given the chance to defend themselves personally, with the assistance of a representative or counsel of their choice. Moreover, this conference or hearing could be used by the parties as an opportunity to come to an amicable settlement. (3)After determining that termination of employment is justified, the employers shall serve the employees a written notice of termination indicating that: (1) all circumstances involving the charge against the employees have been considered; and (2) grounds have been established to justify the severance of their employment. In the instant case, KKTI admits that it had failed to provide respondent with a "charge sheet." However, it maintains that it had substantially complied with the rules, claiming that "respondent would not have issued a written explanation had he not been informed of the charges against him." We are not convinced. First, respondent was not issued a written notice charging him of committing an infraction. The law is clear on the matter. A verbal appraisal of the charges against an employee does not comply with the first notice requirement. In Pepsi Cola Bottling Co. v. NLRC, the Court held that consultations or conferences are not a substitute for the actual observance of notice and hearing. Second, the Court observed the irregularity reports against respondent for his other offenses that such contained merely a general description of the charges against him. The reports did not even state a company rule or policy that the employee had allegedly violated. Likewise, there is no mention of any of the grounds for termination of employment under Art. 282 of the Labor Code. Thus, KKTI's "standard" charge sheet is not sufficient notice to the employee. Third, no hearing was conducted. Regardless of respondent's written explanation, a hearing was still necessary in order for him to clarify and present evidence in support of his defense. Moreover, respondent made the letter merely to explain the circumstances relating to the irregularity. He was unaware that a dismissal proceeding was already being effected. Sanction for Non-compliance with Due Process Requirements

Page 95

LABOR RELATIONS
Atty. Jefferson M. Marquez
As stated earlier, after a finding that petitioners failed to comply with the due process requirements, the CA awarded full backwages in favor of respondent in accordance with the doctrine in Serrano v. NLRC. However, the doctrine in Serrano had already been abandoned in Agabon v. NLRC by ruling that if the dismissal is done without due process, the employer should indemnify the employee with nominal damages. Thus, for non-compliance with the due process requirements in the termination of respondent's employment, petitioner KKTI is sanctioned to pay respondent the amount of thirty thousand pesos (PhP30,000) as damages.

11. Johnson & Johnson v. Johnson Office & Sales Union, G.R. No. 172799, July 6, 2007

Facts: A complaint for illegal dismissal was filed by respondents Ma. Jesusa Bonsol and Rizalinda Hirondo against petitioners Johnson & Johnson (Phils.), Inc. and Janssen Pharmaceutica, one of the formers divisions. The Labor Arbiter dismissed the complaint, prompting respondents to elevate the matter to the NLRC. On 14 December 2001, the NLRC rendered a Resolution, modifying the decision of the Labor Arbiter. The NLRC ruled that the violations of company procedure committed by respondents did not constitute serious misconduct or willful disobedience warranting their dismissal; hence, respondents were entitled to reinstatement. Johnson and Johnson sought partial reconsideration but the NLRC denied the motion in a Resolution dated 11 February 2002. Neither party appealed from the resolution decision of the NLRC within the reglementary period. The Resolution dated 14 December 2001 became final and executory. 31 March 2004 At a conference held, petitioner Johnson and Johnson reiterated their intention to satisfy respondents monetary award but the latter refused and insisted on their reinstatement. Thereafter, petitioner filed a Manifestation and Motion, arguing that the 14 December 2001 Resolution granted them the right to choose between the payment of separation pay and the reinstatement of respondents based on the finding that while their termination was illegal, respondents were not entirely faultless as they did not follow the exact procedure in the performance of their duties. Petitioners also claimed that reinstatement was no longer feasible in view of the strained relations between the parties. 18 June 2004 the NLRC issued a Resolution, which directed the reinstatement of respondents pursuant to the 14 December 2001 Resolution. The NLRC recognized respondents right to choose between reinstatement and separation pay and disregarded petitioners claim of strained relations. Petitioners motion for reconsideration was denied in the Resolution dated 28 July 2004. The Court of Appeals affirmed the resolutions of the NLRC dated 18 June 2004 and 8 July 2004. Thus in a petitioner for certiorari before the SC, Johnson and Johnson contends that the intent of the 14 December 2001 Resolution was to grant them the option to reinstate respondents to their former positions without the payment of backwages, or in the alternative, to pay them separation pay, because the dispositive portion of the Resolution was directed toward or addressed to them (Johnson and Johnson), who are legally obliged to implement the ruling. According to petitioners, the NLRC erred and modified the Resolution dated 14 December 2001, which had become final and executory, when it stated in its 18 June 2004 Resolution that respondents have the right to choose between their reinstatement and getting paid the monetary award when no such categorical pronouncement can be gathered from the 14 December 2001 Resolution. Issue: Whether or not Johnson and Johnson, as the employer, has the option to reinstate respondents to their former positions without the payment of backwages, or in the alternative, to pay them separation pay Ruling: The petition has no merit. Well-entrenched is the rule that an illegally dismissed employee is entitled to reinstatement as a matter of right. Over the years, however, case law developed that where reinstatement is not feasible, expedient or practical, as where reinstatement would only exacerbate the tension and strained relations between the parties, or where the relationship between the employer and employee has been unduly strained by reason of

Page 96

LABOR RELATIONS
Atty. Jefferson M. Marquez
their irreconcilable differences, particularly where the illegally dismissed employee held a managerial or key position in the company, it would be more prudent to order payment of separation pay instead of reinstatement. In other words, the payment of separation compensation in lieu of the reinstatement of an employee who was illegally dismissed from work shall be allowed if and only if the employer can prove the existence of circumstances showing that reinstatement will no longer be for the mutual benefit of the employer and employee. The NLRC Resolution dated 14 December 2001 expressly recognized respondents right to reinstatement in view of the illegality of their termination. Thus, the dispositive portion of said resolution ordered respondents reinstatement without, however, the payment of backwages as a primary relief. Petitioners are mistaken in holding that they have the prerogative to choose whether to reinstate respondents to their former positions or to just pay their monetary award. Neither party can claim that it has the categorical right to choose between reinstatement and the payment of the monetary award. Ultimately, the NLRC has the authority to execute its judgment and to settle any issue that may arise pertaining to the manner or details of implementing its judgment. In the instant case, although the opposing parties yielded to the judgment of the NLRC and did not anymore elevate the labor dispute to the appellate court, they are now at odds as to how the 14 December 2001 Resolution should be implemented. Thus, the NLRC properly exercised its authority to resolve the controversy when it issued the Resolution dated 18 June 2004, where it categorically ordered the reinstatement of respondents to their former positions, in consonance with its earlier ruling. The NLRC upheld the continuing primacy of reinstatement as the available relief and made short shrift of petitioners avowal that separation pay should be awarded in lieu of reinstatement. Effectively, the NLRC and the Court of Appeals disregarded petitioners claim that the relation between the parties was so strained that only the payment of the monetary award was feasible under the circumstances. The Court defers, as it should, to the common finding of the NLRC and Court of Appeals since the issue of the existence of strained relations between the parties is factual in nature.

12. Asian Terminal vs. NLRC, G.R. No. 158458, December 19, 2007, citing Standard Electric Mfg. vs. Standard Electric Employees Union, G.R. No. 166111, August 25, 2005 Facts: Romeo Labrague (respondent) was a stevedore antigo employed with Asian Terminals, Inc. since the 1980's. Beginning September 9, 1993, respondent failed to report for work allegedly because he was arrested and placed in detention for reasons not related to his work. After respondent had been absent for more than one year, Asian Terminals, Inc., through Atty. Rodolfo G. Corvite, Jr., (petitioners) sent him (respondent) a letter, dated December 27, 1994, at his last known address at Area H, Parola, Tondo, Manila, requiring him to explain within 72 hours why he should not suffer disciplinary penalty for his prolonged absence. The following month, petitioner sent respondent another notice of similar tenor. Finally, on February 8, 1995, petitioner issued a memorandum stating: For having incurred absence without official leave (AWOL) from 03 September 1993 up to the present after you were put behind bars due to your involvement in a killing incident, your employment is hereby terminated for cause effective IMMEDIATELY. Following his acquittal and release from detention, respondent reported for work on July 3, 1996 but was advised by petitioners to file a new application so that he may be rehired.Thus, respondent filed with the NLRC a complaint for illegal dismissal, separation pay, non-payment of labor standard benefits, damages and attorney's fees. Ruling: In declaring the dismissal of respondent illegal, the concurrent view of the CA, NLRC and LA is that the latter's prolonged absence was excusable, for it was brought about by his detention for almost three years for a criminal charge that was later declared baseless. They held that his prolonged absence was not coupled with an intention to relinquish his employment, and therefore did not constitute abandonment. Petitioners argue that they were justified in dismissing respondent after the latter incurred a three-year absence without leave, and refused to report for work despite several notices. Petitioners argue that respondent's prolonged absence was not justified or excused by his so-called detention, which remained a mere allegation that was never quite substantiated by any form of official documentation. The foregoing arguments of petitioners are specious. It cannot be gainsaid that respondent was in detention during the entire period of his absence from work and, more importantly, that his situation was known to petitioners. It is of record that in the February 8, 1995 termination notice it issued, petitioners expressly acknowledged that respondent began incurring absences without leave after [he was] put behind bars due to [his] involvement in a killing incident. It clearly indicates that petitioners knew early on of the situation of respondent. It also explains why in its reply before the LA, appeal before the NLRC and petition for certiorari before CA, petitioners never questioned the truth about respondent's detention. Petitioners' skepticism about respondent's detention is a mere afterthought not proper for consideration in a petition for review under Rule 45, which bars reappraisal of facts not disputed before the lower courts or already settled in their proceedings, and unanimously at that. It is beyond dispute then that the underlying reason for respondent's absences was his detention. The question is whether the CA erred in holding that such absences did not amount to abandonment as to furnish petitioners cause to dismiss respondent. To justify the dismissal of respondent for abandonment, petitioners should

Page 97

LABOR RELATIONS
Atty. Jefferson M. Marquez
have established by concrete evidence the concurrence of two elements: first, that respondent had the intention to deliberately and without justification abandon his employment or refuse to resume his work; and second, that respondent performed overt acts from which it may be deduced that he no longer intended to work. Petitioners failed to discharge such burden of proof. Respondent's absences, even after notice to return to work, cannot be equated with abandonment, especially when we take into account that the latter incurred said absences unwillingly and without fault. Absences incurred by an employee who is prevented from reporting for work due to his detention to answer some criminal charge is excusable if his detention is baseless, in that the criminal charge against him is not at all supported by sufficient evidence. In Magtoto v. National Labor Relations Commission as well as Pedroso v. Castro, we declared such absences as not constitutive of abandonment, and held the dismissal of the employee-detainee invalid. We recently reiterated this ruling in Standard Electric Manufacturing Corporation v. Standard Electric Employees Union-NAFLU-KMU, viz.: The facts in Pedroso v. Castro are similar to the set of facts in the present case. The petitioners therein were arrested and detained by the military authorities by virtue of a Presidential Commitment Order allegedly for the commission of Conspiracy to Commit Rebellion under Article 136 of the RPC. As a result, their employer hired substitute workers to avoid disruption of work and business operations. They were released when the charges against them were not proven. After incarceration, they reported back to work, but were refused admission by their employer. The Labor Arbiter and the NLRC sustained the validity of their dismissal. Nevertheless, this Court again held that the dismissed employees should be reinstated to their former positions, since their separation from employment was founded on a false or non-existent cause; hence, illegal. Respondent Javiers absence from August 9, 1995 cannot be deemed as an abandonment of his work. Abandonment is a matter of intention and cannot lightly be inferred or legally presumed from certain equivocal acts. To constitute as such, two requisites must concur: first, the employee must have failed to report for work or must have been absent without valid or justifiable reason; and second, there must have been a clear intention on the part of the employee to sever the employer-employee relationship as manifested by some overt acts, with the second element being the more determinative factor. Abandonment as a just ground for dismissal requires clear, willful, deliberate, and unjustified refusal of the employee to resume his employment. Mere absence or failure to report for work, even after notice to return, is not tantamount to abandonment. Moreover, respondent Javiers acquittal for rape makes it more compelling to view the illegality of his dismissal. The trial court dismissed the case for insufficiency of evidence, and such ruling is tantamount to an acquittal of the crime charged, and proof that respondent Javiers arrest and detention were without factual and legal basis in the first place. Similarly, respondent herein was prevented from reporting for work by reason of his detention. That his detention turned out to be without basis, as the criminal charge upon which said detention was ordered was later dismissed for lack of evidence, made the absences he incurred as a consequence thereof not only involuntary but also excusable. It was certainly not the intention of respondent to absent himself, or his fault that he was detained on an erroneous charge. In no way may the absences he incurred under such circumstances be likened to abandonment. The CA, therefore, correctly held that the dismissal of respondent was illegal, for the absences he incurred by reason of his unwarranted detention did not amount to abandonment.

13. Smart Communications v. Astorga, G.R. No. 148142, January 28, 2008 Facts: Regina M. Astorga (Astorga) was employed by respondent SMART on May 8, 1997 as District Sales Manager of the Corporate Sales Marketing Group/ Fixed Services Division (CSMG/FSD). In February 1998, SMART launched an organizational realignment to achieve more efficient operations. This was made known to the employees. Part of the reorganization was the outsourcing of the marketing and sales force. Thus, SMART entered into a joint venture agreement with NTT of Japan, and formed SMART-NTT Multimedia, Incorporated (SNMI). Since SNMI was formed to do the sales and marketing work, SMART abolished the CSMG/FSD, Astorgas division. To soften the blow of the realignment, SNMI agreed to absorb the CSMG personnel who would be recommended by SMART. SMART then conducted a performance evaluation of CSMG personnel and those who garnered the highest ratings were favorably recommended to SNMI. Astorga landed last in the performance evaluation, thus, she was not recommended by SMART. SMART, nonetheless, offered her a supervisory position in the Customer Care Department, but she refused the offer because the position carried lower salary rank and rate. Despite the abolition of the CSMG/FSD, Astorga continued reporting for work. But on March 3, 1998, SMART issued a memorandum advising Astorga of the termination of her employment on ground of redundancy, effective April 3, 1998. Astorga received it on March 16, 1998. The termination of her employment prompted Astorga to file a Complaint for illegal dismissal, non-payment of salaries and other benefits with prayer for moral and exemplary damages against SMART and Ann Margaret V. Santiago (Santiago). She claimed that abolishing CSMG and, consequently, terminating her employment was illegal for it violated her right to security of tenure. She also posited that it was illegal for an

Page 98

LABOR RELATIONS
Atty. Jefferson M. Marquez
employer, like SMART, to contract out services which will displace the employees, especially if the contractor is an in-house agency. SMART responded that there was valid termination. It argued that Astorga was dismissed by reason of redundancy, which is an authorized cause for termination of employment, and the dismissal was effected in accordance with the requirements of the Labor Code. The redundancy of Astorgas position was the result of the abolition of CSMG and the creation of a specialized and more technically equipped SNMI, which is a valid and legitimate exercise of management prerogative. Issue: WON the dismissal of Astorga is valid. Ruling: YES. Astorgas dismissal is founded upon authorized cause. Astorga was terminated due to redundancy, which is one of the authorized causes for the dismissal of an employee. The nature of redundancy as an authorized cause for dismissal is explained in the leading case of Wiltshire File Co., Inc. v. National Labor Relations Commission, viz: x x x redundancy in an employers personnel force necessarily or even ordinarily refers to duplication of work. That no other person was holding the same position that private respondent held prior to termination of his services does not show that his position had not become redundant. Indeed, in any well organized business enterprise, it would be surprising to find duplication of work and two (2) or more people doing the work of one person. We believe that redundancy, for purposes of the Labor Code, exists where the services of an employee are in excess of what is reasonably demanded by the actual requirements of the enterprise. Succinctly put, a position is redundant where it is superfluous, and superfluity of a position or positions may be the outcome of a number of factors, such as overhiring of workers, decreased volume of business, or dropping of a particular product line or service activity previously manufactured or undertaken by the enterprise. The characterization of an employees services as superfluous or no longer necessary and, therefore, properly terminable, is an exercise of business judgment on the part of the employer. The wisdom and soundness of such characterization or decision is not subject to discretionary review provided, of course, that a violation of law or arbitrary or malicious action is not shown. Astorga claims that the termination of her employment was illegal and tainted with bad faith. She asserts that the reorganization was done in order to get rid of her. But except for her barefaced allegation, no convincing evidence was offered to prove it. This Court finds it extremely difficult to believe that SMART would enter into a joint venture agreement with NTT, form SNMI and abolish CSMG/FSD simply for the sole purpose of easing out a particular employee, such as Astorga. Moreover, Astorga never denied that SMART offered her a supervisory position in the Customer Care Department, but she refused the offer because the position carried a lower salary rank and rate. If indeed SMART simply wanted to get rid of her, it would not have offered her a position in any department in the enterprise. Astorga also states that the justification advanced by SMART is not true because there was no compelling economic reason for redundancy. But contrary to her claim, an employer is not precluded from adopting a new policy conducive to a more economical and effective management even if it is not experiencing economic reverses. Neither does the law require that the employer should suffer financial losses before he can terminate the services of the employee on the ground of redundancy. We agree with the CA that the organizational realignment introduced by SMART, which culminated in the abolition of CSMG/FSD and termination of Astorgas employment was an honest effort to make SMARTs sales and marketing departments more efficient and competitive. As the CA had taken pains to elucidate: x x x a careful and assiduous review of the records will yield no other conclusion than that the reorganization undertaken by SMART is for no purpose other than its declared objective as a labor and cost savings device. Indeed, this Court finds no fault in SMARTs decision to outsource the corporate sales market to SNMI in order to attain greater productivity. [Astorga] belonged to the Sales Marketing Group under the Fixed Services Division (CSMG/FSD), a distinct sales force of SMART in charge of selling SMARTs telecommunications services to the corporate market. SMART, to ensure it can respond quickly, efficiently and flexibly to its customers requirement, abolished CSMG/FSD and shortly thereafter assigned its functions to newly-created SNMI Multimedia Incorporated, a joint venture company of SMART and NTT of Japan, for the reason that CSMG/FSD does not have the necessary technical expertise required for the value added services. By transferring the duties of CSMG/FSD to SNMI, SMART has created a more competent and specialized organization to perform the work required for corporate accounts. It is also relieved SMART of all administrative costs management, time and money-needed in maintaining the CSMG/FSD. The determination to outsource the duties of the CSMG/FSD to SNMI was, to Our mind, a sound business judgment based on relevant criteria and is therefore a legitimate exercise of management prerogative. Indeed, out of our concern for those lesser circumstanced in life, this Court has inclined towards the worker and upheld his cause in most of his

Page 99

LABOR RELATIONS
Atty. Jefferson M. Marquez
conflicts with his employer. This favored treatment is consonant with the social justice policy of the Constitution. But while tilting the scales of justice in favor of workers, the fundamental law also guarantees the right of the employer to reasonable returns for his investment. In this light, we must acknowledge the prerogative of the employer to adopt such measures as will promote greater efficiency, reduce overhead costs and enhance prospects of economic gains, albeit always within the framework of existing laws. Accordingly, we sustain the reorganization and redundancy program undertaken by SMART. However, as aptly found by the CA, SMART failed to comply with the mandated one (1) month notice prior to termination. The record is clear that Astorga received the notice of termination only on March 16, 1998 or less than a month prior to its effectivity on April 3, 1998. Likewise, the Department of Labor and Employment was notified of the redundancy program only on March 6, 1998. Article 283 of the Labor Code clearly provides: Art. 283. Closure of establishment and reduction of personnel. The employer may also terminate the employment of any employee due to the installation of labor saving devices, redundancy, retrenchment to prevent losses or the closing or cessation of operation of the establishment or undertaking unless the closing is for the purpose of circumventing the provisions of this Title, by serving a written notice on the workers and the Ministry of Labor and Employment at least one (1) month before the intended date thereof x x x. SMARTs assertion that Astorga cannot complain of lack of notice because the organizational realignment was made known to all the employees as early as February 1998 fails to persuade. Astorgas actual knowledge of the reorganization cannot replace the formal and written notice required by the law. In the written notice, the employees are informed of the specific date of the termination, at least a month prior to the effectivity of such termination, to give them sufficient time to find other suitable employment or to make whatever arrangements are needed to cushion the impact of termination. In this case, notwithstanding Astorgas knowledge of the reorganization, she remained uncertain about the status of her employment until SMART gave her formal notice of termination. But such notice was received by Astorga barely two (2) weeks before the effective date of termination, a period very much shorter than that required by law. Be that as it may, this procedural infirmity would not render the termination of Astorgas employment illegal. The validity of termination can exist independently of the procedural infirmity of the dismissal. The CA, therefore, committed no reversible error in sustaining Astorgas dismissal and at the same time, awarding indemnity for violation of Astorga's statutory rights. However, we find the need to modify, by increasing, the indemnity awarded by the CA to Astorga, as a sanction on SMART for non-compliance with the one-month mandatory notice requirement, in light of our ruling in Jaka Food Processing Corporation v. Pacot, viz.: [I]f the dismissal is based on a just cause under Article 282 but the employer failed to comply with the notice requirement, the sanction to be imposed upon him should be tempered because the dismissal process was, in effect, initiated by an act imputable to the employee, and (2) if the dismissal is based on an authorized cause under Article 283 but the employer failed to comply with the notice requirement, the sanction should be stiffer because the dismissal process was initiated by the employers exercise of his management prerogative. We deem it proper to increase the amount of the penalty on SMART to P50,000.00. As provided in Article 283 of the Labor Code, Astorga is, likewise, entitled to separation pay equivalent to at least one (1) month salary or to at least one (1) months pay for every year of service, whichever is higher. The records show that Astorgas length of service is less than a year. She is, therefore, also entitled to separation pay equivalent to one (1) month pay. However, the award of backwages to Astorga by the CA should be deleted for lack of basis. Backwages is a relief given to an illegally dismissed employee. Thus, before backwages may be granted, there must be a finding of unjust or illegal dismissal from work. Since Astorgas dismissal is for an authorized cause, she is not entitled to backwages. The CAs award of backwages is totally inconsistent with its finding of valid dismissal.

14. Enriquez v. Bank of the Philippine Islands, G.R. No. 172812, February 12, 2008 Facts: Enriquez and Sia were the branch manager and assistant branch manager, respectively, of the BPI-Bacolod Singcang Branch. Enriquez was first employed by BPI in 1971 and had been an employee thereof for 32 years at the time of her termination, whereas Sia had been with since 1974, or for a total of 29 years at the time of his dismissal. Respondent Luis A. Puentevella (Puentevella) is one of respondents principal officers and

Page 100

LABOR RELATIONS
Atty. Jefferson M. Marquez
was impleaded in his personal capacity. Petitioners version of the facts: Petitioners maintain that on 27 December 2002, their branch experienced a heavy volume of transactions owing to the fact that it was the last banking day of the year. When banking hours came to a close, teller Geraldine Descartin (Descartin) purportedly discovered that she had a cash shortage of P36,000.00 and informed Sia about it. Sia, in turn, informed Enriquez of the problem and was directed to review the days transactions to trace its cause. Descartin claimed that the discrepancy was due to an innocent oversight and recalled that the unaccounted shortage was due to the failure of her mother-in-law, Remedios Descartin (Remedios), to sign the withdrawal slip when the latter withdrew P36,000.00 earlier that day. With that explanation, Enriquez directed Descartin and her co-teller Evelyn Fregil (Fregil) to submit their written memorandum of the incident. Descartin was permitted to leave the bank to look for Remedios so that the latter could sign the withdrawal slip. At around 7:00 p.m., she returned to the bank with the signed withdrawal slip and debited the amount from the clients account. Thus, petitioners aver, the transaction was regularized before the end of the day. It is the position of petitioners that as there was neither shortage nor loss to the bank because the initial discrepancy was accounted for and that it was due to a mere oversight, they put the matter to rest. In the meantime, Sia began to wind up his affairs as 27 December 2002 was his last working day with the bank before going on terminal leave prior to his optional retirement. BPIs version: BPI has a different version of what transpired on 27 December 2002. According to them, teller Descartins shortage of P36,000.00, which she confided to her co-teller Fregil, was incurred because she had temporarily borrowed the money that week to pay her financial obligations but intended to return the same on the first week of January. Teller Fregil reported the matter to Sia and Enriquez, both of whom suggested that teller Descartin fill the shortage with a loan from her family. Teller Descartin replied that her family did not have the money, she instead borrowed the amount from her in-laws. Thus, at 5:21 p.m., teller Descartin posted the unsigned withdrawal slip for the amount of P36,000.00 against the joint account of her parents-in-law. As the amount exceeded the floor limit for tellers which would require the approval of a superior officer, either Enriquez or Sia approved the transaction at 5:22 p.m. as reflected on the account records. Teller Descartin thereafter left the bank to secure the signature of her mother-in-law Remedios and returned at past 7:00 p.m. with the signed withdrawal slip. Thus, sometime in February 2003, respondent Puentevella initiated further investigation on the incident. Later, on 3 March 2003, teller Fregil retracted her original statement and instead executed another letter claiming that there was a cover-up of the shortage on the day in question. Respondents assert that the investigation conducted by the Auditing Division of BPI bolstered teller Fregils claims of irregularity as the audit report disclosed that petitioners failed to make the necessary report on the shortage and instead assisted in covering-up teller Descartins wrongdoing. On 25 April 2003, petitioners were instructed to report to the BPI head office for polygraph testing. While they expressed their willingness to be interviewed, petitioners objected to the polygraph test. On 27 June 2003, petitioners received show-cause memos directing them to explain in writing why they should not be sanctioned for conflict of interest and breach of trust. Petitioners submitted their respective replies in which they denied the charges against them. On14 July 2003, a committee of respondent bank conducted a hearing of the case and as part of the investigation, separately interviewed petitioners and tellers Descartin and Fregil. On 3 September 2003, petitioners were dismissed from employment on grounds of breach of trust and confidence and dishonesty. Issue: Whether or not there were valid grounds for termination of employment. Ruling: BPI had just cause to terminate their employment. There is no denying that loss of trust and confidence is a valid ground for termination of employment. Hence, the basic requisite for dismissal on the ground of loss of confidence is that the employee concerned holds a position of trust and confidence or is routinely charged with the care and custody of the employers money or property. Moreover, the breach must be related to the performance of the employees function. Also, it must be shown that the employee is a managerial employee, since the term trust and confidence is restricted to said class of employees. The SC found sufficient basis in evidence to accord full probative value to Teller Fregils retraction letter which she later affirmed through subsequent affidavits. The independent audit conducted by the auditing division of BPI notably supports her claim that the wrongdoing was concealed by petitioners from respondent bank. Moreover, a review of the tellers transaction summary of teller Descartin reinforces the conclusion that the shortage in her pico box was due to a temporary borrowing, the cover-up of which was sanctioned by petitioners.

Page 101

LABOR RELATIONS
Atty. Jefferson M. Marquez
It is likewise asserted by petitioners that under BPIs bank policy, failure to report a shortage is not a ground to terminate employment. The argument is short-sighted. BPIs policy on tellers shortages is unambiguous. It requires that all shortages be declared properly and booked accordingly on the same day they are incurred. Furthermore, the same must be reported by the branch head to the designated bank officers and departments not later than the second banking day from the date of booking. Taken together with the attending circumstances of the case, the failure of petitioners to report the cash shortage of teller Descartin, even if done in good faith, nonetheless resulted in their abetting the dishonesty committed by the latter. Under the personnel policies of BPI, this act of petitioners justifies their dismissal even on the first offense. Even assuming the version of petitioners as the truth, the fact remains that they willfully decided against reporting the shortage that occurred. As a result, in either situation, petitioners acts have caused respondents to have a legitimate reason to lose the trust reposed in them as senior managerial employees. Their participation in the cover-up of the misconduct of teller Descartin makes them unworthy of the trust and confidence demanded by their positions. It is well-settled that the power to dismiss an employee is a recognized prerogative that is inherent in the employers right to freely manage and regulate his business. An employer cannot be expected to retain an employee whose lack of morals, respect and loyalty to his employer or regard for his employers rules and appreciation of the dignity and responsibility of his office has so plainly and completely been bared. Thus, to compel respondent bank to keep petitioners in its employ after the latter have betrayed the confidence given to them would be unjust to respondent bank. The expectation of trust is more so magnified in the instant case in light of the nature of respondent banks business. The banking industry is imbued with public interest and is mandated by law to serve its clients with extraordinary care and diligence. To be able to fulfill this duty, it in turn must rely on the honesty and loyalty of its employees. Clearly, as a measure of self-preservation against acts patently inimical to its interests, respondent bank had every right to dismiss petitioners for breach of trust, loss of confidence and dishonesty. Indeed, in cases of this nature, the fact that petitioners had been employees of BPI for a long time, if it is to be considered at all, should be taken against them. Their manifest condonation and even concealment of an offense prejudicial to their employers interest committed by a subordinate under their supervision reflect a regrettable lack of loyalty which they should have reinforced, instead of betrayed. While the Constitution is committed to the policy of social justice and the protection of the working class, it should not be supposed that every labor dispute will be automatically decided in favor of labor. Management also has its own rights which, as such, are entitled to respect and enforcement in the interest of simple fair play.

15. RB Michael Press vs. Galit, G.R. No. 153510, February 13, 2008 Facts: Respondent was employed by petitioner R.B. Michael Press as an offset machine operator, whose work schedule was from 8:00 a.m. to 5:00 p.m., Mondays to Saturdays, and he was paid PhP230 a day. During his employment, Galit was tardy for a total of 190 times, totaling to 6,117 minutes, and was absent without leave for a total of nine and a half days. On February 22, 1999, respondent was ordered to render overtime service in order to comply with a job order deadline, but he refused to do so. The following day, respondent reported for work but petitioner Escobia told him not to work, and to return later in the afternoon for a hearing. When he returned, a copy of an Office Memorandum was served on him, as follows: To:Mr. Nicasio Galit From:ANNALENE REYES-ESCOBIA Re:WARNING FOR DISMISSAL; NOTICE OF HEARING This warning for dismissal is being issued for the following offenses: (1)habitual and excessive tardiness (2)committing acts of discourtesy, disrespect in addressing superiors (3)failure to work overtime after having been instructed to do so (4)Insubordination willfully disobeying, defying or disregarding company authority The offenses you've committed are just causes for termination of employment as provided by the Labor Code. You were given verbal warnings

Page 102

LABOR RELATIONS
Atty. Jefferson M. Marquez
before, but there had been no improvement on your conduct. Further investigation of this matter is required, therefore, you are summoned to a hearing at 4:00 p.m. today. The hearing will determine your employment status with this company. (SGD) ANNALENE REYES-ESCOBIA Manager On February 24, 1999, respondent was terminated from employment. The employer, through petitioner Escobia, gave him his two-day salary and a termination letter averring that Galit was dismissed due to the following offenses: (1) habitual and excessive tardiness; (2) commission of discourteous acts and disrespectful conduct when addressing superiors; (3) failure to render overtime work despite instruction to do so; and (4) insubordination, that is, willful disobedience of, defiance to, or disregard of company authority.. Respondent subsequently filed a complaint for illegal dismissal and money claims before the National Labor Relations Commission (NLRC). Issues: (1) WON there was just cause to terminate the employment of respondent, and whether due process was observed in the dismissal process; (2) WON respondent is entitled to backwages and other benefits despite his refusal to be reinstated. Ruling: Respondent's tardiness cannot be considered condoned by petitioners Habitual tardiness is a form of neglect of duty. Lack of initiative, diligence, and discipline to come to work on time everyday exhibit the employee's deportment towards work. Habitual and excessive tardiness is inimical to the general productivity and business of the employer. This is especially true when the tardiness and/or absenteeism occurred frequently and repeatedly within an extensive period of time. The mere fact that the numerous infractions of respondent have not been immediately subjected to sanctions cannot be interpreted as condonation of the offenses or waiver of the company to enforce company rules. A waiver is a voluntary and intentional relinquishment or abandonment of a known legal right or privilege. It has been ruled that "a waiver to be valid and effective must be couched in clear and unequivocal terms which leave no doubt as to the intention of a party to give up a right or benefit which legally pertains to him." Hence, the management prerogative to discipline employees and impose punishment is a legal right which cannot, as a general rule, be impliedly waived. Thus it is incumbent upon the employee to adduce substantial evidence to demonstrate condonation or waiver on the part of management to forego the exercise of its right to impose sanctions for breach of company rules. In the case at bar, respondent did not adduce any evidence to show waiver or condonation on the part of petitioners. Insubordination or willful disobedience For willful disobedience to be a valid cause for dismissal, these two elements must concur: (1) the employee's assailed conduct must have been willful, that is, characterized by a wrongful and perverse attitude; and (2) the order violated must have been reasonable, lawful, made known to the employee, and must pertain to the duties which he had been engaged to discharge. In the present case, there is no question that petitioners' order for respondent to render overtime service to meet a production deadline complies with the second requisite. Art. 89 of the Labor Code empowers the employer to legally compel his employees to perform overtime work against their will to prevent serious loss or damage: Art. 89.EMERGENCY OVERTIME WORK Any employee may be required by the employer to perform overtime work in any of the following cases: xxx xxx xxx (c)When there is urgent work to be performed on machines, installations, or equipment, in order to avoid serious loss or damage to the employer or some other cause of similar nature; xxx xxx xxx In the present case, petitioners' business is a printing press whose production schedule is sometimes flexible and varying. It is only reasonable that workers are sometimes asked to render overtime work in order to meet production deadlines. The issue now is, whether respondent's refusal or failure to render overtime work was willful; that is, whether such refusal or failure was characterized by a wrongful and perverse attitude. In Lakpue Drug Inc. v. Belga, willfulness was described as "characterized by a wrongful and perverse mental attitude rendering the employee's act inconsistent with proper subordination." The fact that respondent refused to provide overtime work despite his knowledge that there is a production deadline that needs to be met, and that without him, the offset machine operator, no further printing can be had, shows his wrongful and perverse mental attitude; thus, there is willfulness.

Page 103

LABOR RELATIONS
Atty. Jefferson M. Marquez
After a re-examination of the facts, we rule that respondent unjustifiably refused to render overtime work despite a valid order to do so. The totality of his offenses against petitioner R.B. Michael Press shows that he was a difficult employee. His refusal to render overtime work was the final straw that broke the camel's back, and, with his gross and habitual tardiness and absences, would merit dismissal from service. Due process: twin notice and hearing requirement On the issue of due process, petitioners claim that they had afforded respondent due process. Petitioners maintain that they had observed due process when they gave respondent two notices and that they had even scheduled a hearing where he could have had explained his side and defended himself. We are not persuaded. We held in Agabon v. NLRC: Procedurally, (1) if the dismissal is based on a just cause under Article 282, the employer must give the employee two written notices and a hearing or opportunity to be heard if requested by the employee before terminating the employment: a notice specifying the grounds for which dismissal is sought a hearing or an opportunity to be heard and after hearing or opportunity to be heard, a notice of the decision to dismiss; and (2) if the dismissal is based on authorized causes under Articles 283 and 284, the employer must give the employee and the Department of Labor and Employment written notices 30 days prior to the effectivity of his separation Under the twin notice requirement, the employees must be given two (2) notices before his employment could be terminated: (1) a first notice to apprise the employees of their fault, and (2) a second notice to communicate to the employees that their employment is being terminated. Not to be taken lightly of course is the hearing or opportunity for the employee to defend himself personally or by counsel of his choice. A scrutiny of the disciplinary process undertaken by petitioners leads us to conclude that they only paid lip service to the due process requirements. The undue haste in effecting respondent's termination shows that the termination process was a mere simulation the required notices were given, a hearing was even scheduled and held, but respondent was not really given a real opportunity to defend himself; and it seems that petitioners had already decided to dismiss respondent from service, even before the first notice had been given. Anent the written notice of charges and hearing, it is plain to see that there was merely a general description of the claimed offenses of respondent. The hearing was immediately set in the afternoon of February 23, 1999 the day respondent received the first notice. Therefore, he was not given any opportunity at all to consult a union official or lawyer, and, worse, to prepare for his defense.

16. Cosmos Bottling Corporation v. Nagrama, G.R. No 164403, March 4, 2008 Facts: Respondent Pablo Nagrama, Jr. was initially employed by petitioner as a maintenance mechanic on June 24, 1993 at the Cosmos Plant in Cauayan, Isabela. On September 17, 1996, he was elected by the local union as chief shop steward. Respondent was designated by petitioner as waste water treatment operator effective September 27, 1999. Petitioner hired Clean Flow Philippines, Inc. to conduct training seminars to acquaint petitioners personnel on the operations of the water treatment plant. Respondent was instructed to attend the seminar to be held on September 27-30, 1999. He failed to attend the first two (2) days of the seminar. In a letter by his immediate supervisor, Josephine D. Calacien, dated September 29, 1999, respondent was informed that charges of abandonment of duty and gross insubordination had been lodged against him. He was required to submit his written explanation. Respondent filed his explanation on September 30, 1999. He contended that he had to attend to an administrative hearing for fellow unionists which were held at Santiago, Isabela; that before he went, he first secured permission from the plant controller. He averred that as a union official, he is obligated to attend to the problems of his fellow union members. Issue: Is the dismissal based on the grounds of abandonment and gross insubordination valid? Ruling:

Page 104

LABOR RELATIONS
Atty. Jefferson M. Marquez
There is no abandonment and gross insubordination. Two (2) elements must be satisfied for an employee to be guilty of abandonment. The first is the failure to report for work or absence without valid or justifiable reason. The second is a clear intention to sever the employer-employee relationship. The second element is the more determinative factor and must be evinced by overt acts. Likewise, the burden of proof is on the employer to show the employees clear and deliberate intent to discontinue his employment without any intention of returning; mere absence is not sufficient. First, respondents absence was justified under the circumstances. He was a shop steward, which recent jurisprudence qualifies as a union officer. As an officer, he had a valid reason to attend the hearing of his union brothers. He also asked for and was given permission as can be seen from the minutes of his hearing. Petitioner does not contest this fact. Permission negates any possibility of respondent abandoning his job. As to the second requisite, We are not convinced that respondent intended to sever the employer-employee relationship with Cosmos. He immediately complied with the memo requiring him to explain his absence. His failure to report directly to his Quality Assurance Supervisor and Analyst can be dismissed as failure to properly understand the instructions he was given. Moreover, respondent filed a complaint for illegal dismissal.] A complaint for illegal dismissal shows a desire to continue work. Verily, a review of the evidence shows that both elements of abandonment are lacking. For gross insubordination, also called willful disobedience of a lawful order, to lie, two (2) requisites are also necessary. First, the assailed conduct must have been intentional and characterized by a wrongful and perverse attitude.] Second, the order violated must have been reasonable, lawful, and made known to the employee and should pertain to the duties which he has been engaged to discharge.http://sc.judiciary.gov.ph/jurisprudence/2008/march2008/164403.htm - _ftn83 There is no question that orders to attend the seminar are lawful instructions by petitioner. The first element of gross insubordination, however, is lacking. A review of the records shows that respondents failure to report to his quality assurance supervisor and failure to fully attend the seminar was in no way tainted by a wrongful or perverse attitude. His failure to secure a clearance from Clean Flow was due to his attendance to his union duties. Hence, there is no gross insubordination.

17. School of the Holy Spirit of Q.C. vs. Taguiam, G.R. No. 165565, July 14, 2008 Facts: Corazon P. Taguiam was the Class Adviser of Grade 5-Esmeralda of the petitioner, School of the Holy Spirit of Quezon City. On March 10, 2000, the class president, wrote a letter to the grade school principal requesting permission to hold a year-end celebration at the school grounds. The principal authorized the activity and allowed the pupils to use the swimming pool. In this connection, respondent distributed the parent's/guardian's permit forms to the pupils. Corazon P. Taguiam admitted that Chiara Mae Federico's permit form was unsigned. Nevertheless, she concluded that Chiara Mae was allowed by her mother to join the activity since her mother personally brought her to the school with her packed lunch and swimsuit. Before the activity started, she warned the pupils who did not know how to swim to avoid the deeper area. However, while the pupils were swimming, two of them sneaked out. Respondent went after them to verify where they were going. Unfortunately, while respondent was away, Chiara Mae drowned. When she returned, the maintenance man was already administering cardiopulmonary resuscitation on Chiara Mae. The child was still alive when respondent rushed her to the General Malvar Hospital where she was pronounced dead on arrival. Corazon P. Taguiam was dismissed for gross negligence resulting to loss of confidence. Issue: Was the dismissal based on the ground as stated valid? Ruling: Yes the dismissal was valid. Under Article 282 of the Labor Code, gross and habitual neglect of duties is a valid ground for an employer to terminate an employee. Gross negligence implies a want or absence of or a failure to exercise slight care or diligence, or the entire absence of care. It evinces a thoughtless disregard of consequences without exerting any effort to avoid them. Habitual neglect implies repeated failure to perform one's duties for a period of time, depending upon the circumstances. Respondent had been grossly negligent. First , it is undisputed that Chiara Mae's permit form was unsigned. Yet, respondent allowed her to join the activity because she assumed that Chiara Mae's mother has allowed her to join it by personally bringing her to the school with her packed lunch and swimsuit. The purpose of a permit form is precisely to ensure that the parents have allowed their child to join the school activity involved. Respondent cannot simply ignore this by resorting to

Page 105

LABOR RELATIONS
Atty. Jefferson M. Marquez
assumptions. Respondent admitted that she was around when Chiara Mae and her mother arrived. She could have requested the mother to sign the permit form before she left the school or at least called her up to obtain her conformity. Second, it was respondent's responsibility as Class Adviser to supervise her class in all activities sanctioned by the school. Thus, she should have coordinated with the school to ensure that proper safeguards, such as adequate first aid and sufficient adult personnel, were present during their activity. She should have been mindful of the fact that with the number of pupils involved, it would be impossible for her by herself alone to keep an eye on each one of them. Notably, respondent's negligence, although gross, was not habitual. In view of the considerable resultant damage, however, the cause is sufficient to dismiss respondent. This is not the first time that the SC have departed from the requirements laid down by the law that neglect of duties must be both gross and habitual. In Philippine Airlines, Inc. v. NLRC, we ruled that Philippine Airlines (PAL) cannot be legally compelled to continue with the employment of a person admittedly guilty of gross negligence in the performance of his duties although it was his first offense. In that case, we noted that a mere delay on PAL's flight schedule due to aircraft damage entails problems like hotel accommodations for its passengers, re-booking, the possibility of law suits, and payment of special landing fees not to mention the soaring costs of replacing aircraft parts. In another case, Fuentes v. National Labor Relations Commission, we held that it would be unfair to compel Philippine Banking Corporation to continue employing its bank teller. In that case, we observed that although the teller's infraction was not habitual, a substantial amount of money was lost. The deposit slip had already been validated prior to its loss and the amount reflected thereon is already considered as current liabilities in the bank's balance sheet. Indeed, the sufficiency of the evidence as well as the resultant damage to the employer should be considered in the dismissal of the employee. In this case, the damage went as far as claiming the life of a child. As a result of gross negligence in the present case, petitioners lost its trust and confidence in respondent. Loss of trust and confidence to be a valid ground for dismissal must be based on a willful breach of trust and founded on clearly established facts. A breach is willful if it is done intentionally, knowingly and purposely, without justifiable excuse, as distinguished from an act done carelessly, thoughtlessly, heedlessly or inadvertently. Otherwise stated, it must rest on substantial grounds and not on the employer's arbitrariness, whims, caprices or suspicion; otherwise, the employee would eternally remain at the mercy of the employer. It should be genuine and not simulated; nor should it appear as a mere afterthought to justify earlier action taken in bad faith or a subterfuge for causes which are improper, illegal or unjustified. It has never been intended to afford an occasion for abuse because of its subjective nature. There must, therefore, be an actual breach of duty committed by the employee which must be established by substantial evidence. All told, there being a clear showing that respondent was culpable for gross negligence resulting to loss of trust and confidence, her dismissal was valid and legal.

18. Universal Staffing Services Inc. v. NLRC, G.R. No. 177576, July 21, 2008 Facts: Respondent Grace M. Morales (Morales) applied for and was hired as receptionist by petitioner Universal Staffing Services, Inc. (USSI) in behalf of its principal Jin Xiang International Labour Supply of United Arab Emirates (U.A.E.). The contract duly approved by the Philippine Overseas Employment Administration (POEA), provided for an employment term of two (2) years with a monthly salary of Dhs1, 100.00. After Ten (10) months of work in Al Sandos Suites (Al Sandos) Abu Dhabi, U.A.E. Morales employment was terminated allegedly due to her poor work performance. Morales received Dhs1,300.00 as full and final settlement of all her claims on January 1, 2003, and was repatriated on January 7, 2003. Claiming that she was illegally terminated, Morales filed a complaint for illegal dismissal and non-payment of overtime and vacation pay against USSI and Al Sandos Hotel Management with the Labor Arbiter. Issue: Whether or not Morales is illegally dismissed Ruling: USSI insists that Morales dismissal was based on a valid and legal ground. The Labor Arbiter lent credence to USSIs posture and dismissed Morales complaint, but the NLRC and the CA reversed the Arbiters findings. Morales was dismissed for her alleged poor performance. As a general concept, "poor performance" is equivalent to inefficiency and incompetence in the performance of official duties. Under Article 282 of the Labor Code, an unsatisfactory rating can be a just cause for dismissal only if it amounts to gross and habitual neglect of duties. Thus, the fact that an employee's performance is found to be poor or unsatisfactory does not necessarily mean that the employee is grossly and habitually negligent of his duties. Gross negligence implies a want or absence of or failure to exercise slight care or diligence, or the entire absence of care. It evinces a thoughtless disregard of consequences

Page 106

LABOR RELATIONS
Atty. Jefferson M. Marquez
without exerting any effort to avoid them. No substantial evidence was presented to substantiate the cause of Morales dismissal. First, USSI failed to cite particular acts or instances that would validate its claim of Morales poor performance. Second, no convincing proof was offered to substantiate Morales alleged poor performance. The principle echoed and reechoed in jurisprudence is that the onus of proving that the employee was dismissed for a just cause rests on the employer, and the latters failure to discharge that burden would result in a finding that the dismissal is unjustified. Furthermore, Morales was not accorded due process. Under Article 277(b) of the Labor Code, the employer must send the employee who is about to be terminated, a written notice stating the cause/s for termination and must give the employee the opportunity to be heard and to defend himself. There was no showing that Al Sandos warned Morales of her alleged poor performance. Likewise, Morales was not served the first notice apprising her of the particular acts or omissions on which her dismissal was based together with the opportunity to explain her side. The only notice given to Morales was the letter dated December 14, 2002 informing her that she was already terminated. Certainly, there can be no other conclusion than that Morales was illegally dismissed and her employment contract was illegally terminated. With this finding, it is imperative that Morales be granted the monetary benefits due her. WHEREFORE, the petition is PARTIALLY GRANTED. Grace M. Morales is declared illegally dismissed. Petitioner Universal Staffing Services, Inc. is ordered to pay Morales three (3) months salary or Dhs3,300.00, or its peso equivalent. The awards of overtime and holiday pay, as well as attorneys fees, are DELETED.

19. Flight Attendants and Steward Association of the Philippines (FASAP) v. Philippine Airlines, G.R. No. 178083, G.R. No. 178083, July 22, 2008 Facts: Petitioner FASAP is the duly certified collective bargaining representative of PAL flight attendants and stewards, or collectively known as PAL cabin crew personnel. Respondent PAL is a domestic corporation organized and existing under the laws of the Republic of the Philippines, operating as a common carrier transporting passengers and cargo through aircraft. On June 15, 1998, PAL retrenched 5,000 of its employees, including more than 1,400 of its cabin crew personnel, to take effect on July 15, 1998. PAL adopted the retrenchment scheme allegedly to cut costs and mitigate huge financial losses as a result of a downturn in the airline industry brought about by the Asian financial crisis. During said period, PAL claims to have incurred P90 billion in liabilities, while its assets stood at P85 billion. In implementing the retrenchment scheme, PAL adopted its so-called Plan 14 whereby PALs fleet of aircraft would be reduced from 54 to 14, thus requiring the services of only 654 cabin crew personnel. PAL admits that the retrenchment is wholly premised upon such reduction in fleet, and to the strike staged by PAL pilots since this action also translated into a reduction of flights. PAL claims that the scheme resulted in savings x x x amounting to approximately P24 million per month savings that would greatly alleviate PALs financial crisis. On June 22, 1998, FASAP filed a Complaint against PAL and Patria T. Chiong (Chiong) for unfair labor practice, illegal retrenchment with claims for reinstatement and payment of salaries, allowances and backwages of affected FASAP members, actual, moral and exemplary damages with a prayer to enjoin the retrenchment program then being implemented. Meanwhile, months after the June 15, 1998 mass dismissal of its cabin crew personnel, PAL began recalling to service those it had previously retrenched. Thus, in November 1998 and up to March 1999 several of those retrenched were called back to service. To date, PAL claims to have recalled 820 of the retrenched cabin crew personnel. FASAP, however, claims that only 80 were recalled as of January 2001. Issue: Whether PALs retrenchment scheme was justified. Ruling:

Page 107

LABOR RELATIONS
Atty. Jefferson M. Marquez
No. Under the Labor Code, retrenchment or reduction of employees is authorized as follows: ART. 283. Closure of establishment and reduction of personnel. - The employer may also terminate the employment of any employee due to the installation of labor-saving devices, redundancy, retrenchment to prevent losses or the closing or cessation of operation of the establishment or undertaking unless the closing is for the purpose of circumventing the provisions of this Title, by serving a written notice on the workers and the Ministry of Labor and Employment at least one (1) month before the intended date thereof. In case of termination due to the installation of laborsaving devices or redundancy, the worker affected thereby shall be entitled to a separation pay equivalent to at least his one (1) month pay or to at least one (1) month pay for every year of service, whichever is higher. In case of retrenchment to prevent losses and in cases of closures or cessation of operations of establishment or undertaking not due to serious business losses or financial reverses, the separation pay shall be equivalent to one (1) month pay or at least one-half (1/2) month pay for every year of service, whichever is higher. A fraction of at least six (6) months shall be considered one (1) whole year. The law recognizes the right of every business entity to reduce its work force if the same is made necessary by compelling economic factors which would endanger its existence or stability. Where appropriate and where conditions are in accord with law and jurisprudence, the Court has authorized valid reductions in the work force to forestall business losses, the hemorrhaging of capital, or even to recognize an obvious reduction in the volume of business which has rendered certain employees redundant. Nevertheless, while it is true that the exercise of this right is a prerogative of management, there must be faithful compliance with substantive and procedural requirements of the law and jurisprudence, for retrenchment strikes at the very heart of the workers employment, the lifeblood upon which he and his family owe their survival. Retrenchment is only a measure of last resort, when other less drastic means have been tried and found to be inadequate. The burden clearly falls upon the employer to prove economic or business losses with sufficient supporting evidence. Its failure to prove these reverses or losses necessarily means that the employees dismissal was not justified. Any claim of actual or potential business losses must satisfy certain established standards, all of which must concur, before any reduction of personnel becomes legal. These are: (1) That retrenchment is reasonably necessary and likely to prevent business losses which, if already incurred, are not merely de minimis, but substantial, serious, actual and real, or if only expected, are reasonably imminent as perceived objectively and in good faith by the employer; (2) That the employer served written notice both to the employees and to the Department of Labor and Employment at least one month prior to the intended date of retrenchment; (3) That the employer pays the retrenched employees separation pay equivalent to one (1) month pay or at least one-half (1/2) month pay for every year of service, whichever is higher; (4) That the employer exercises its prerogative to retrench employees in good faith for the advancement of its interest and not to defeat or circumvent the employees right to security of tenure; and, (5) That the employer used fair and reasonable criteria in ascertaining who would be dismissed and who would be retained among the employees, such as status, efficiency, seniority, physical fitness, age, and financial hardship for certain workers. In view of the facts and the issues raised, the resolution of the instant petition hinges on a determination of the existence of the first, fourth and the fifth elements set forth above, as well as compliance therewith by PAL, taking to mind that the burden of proof in retrenchment cases lies with the employer in showing valid cause for dismissal: that legitimate business reasons exist to justify retrenchment. FIRST ELEMENT: The employers prerogative to layoff employees is subject to certain limitations. The law speaks of serious business losses or financial reverses. Sliding incomes or decreasing gross revenues are not necessarily losses, much less serious business losses within the meaning of the law. The fact that an employer may have sustained a net loss, such loss, per se, absent any other evidence on its impact on the business, nor on expected losses that would have been incurred had operations been continued, may not amount to serious business losses mentioned in the law. The employer must show that its losses increased through a period of time and that the condition of the company will not likely improve in the near future, or that it expected no abatement of its losses in the coming years. Put simply, not every loss incurred or expected to be incurred by a company will justify retrenchment.

Page 108

LABOR RELATIONS
Atty. Jefferson M. Marquez
The employer must also exhaust all other means to avoid further losses without retrenching its employees. Retrenchment is a means of last resort; it is justified only when all other less drastic means have been tried and found insufficient. Even assuming that the employer has actually incurred losses by reason of the Asian economic crisis, the retrenchment is not completely justified if there is no showing that the retrenchment was the last recourse resorted to. Where the only less drastic measure that the employer undertook was the rotation work scheme, or the threeday-work-per-employee-per-week schedule, and it did not endeavor at other measures, such as cost reduction, lesser investment on raw materials, adjustment of the work routine to avoid scheduled power failure, reduction of the bonuses and salaries of both management and rankand-file, improvement of manufacturing efficiency, and trimming of marketing and advertising costs, the claim that retrenchment was done in good faith to avoid losses is belied. Alleged losses if already realized, and the expected imminent losses sought to be forestalled, must be proved by sufficient and convincing evidence. The reason for requiring this is readily apparent: any less exacting standard of proof would render too easy the abuse of this ground for termination of services of employees; scheming employers might be merely feigning business losses or reverses in order to ease out employees. In establishing a unilateral claim of actual or potential losses, financial statements audited by independent external auditors constitute the normal method of proof of profit and loss performance of a company. A Statement of Profit and Loss submitted to prove alleged losses, without the accompanying signature of a certified public accountant or audited by an independent auditor, is nothing but a self-serving document which ought to be treated as a mere scrap of paper devoid of any probative value. In the instant case, PAL failed to substantiate its claim of actual and imminent substantial losses which would justify the retrenchment of more than 1,400 of its cabin crew personnel. Although the Philippine economy was gravely affected by the Asian financial crisis, however, it cannot be assumed that it has likewise brought PAL to the brink of bankruptcy. Likewise, the fact that PAL underwent corporate rehabilitation does not automatically justify the retrenchment of its cabin crew personnel. To prove that PAL was financially distressed, it could have submitted its audited financial statements but it failed to present the same with the Labor Arbiter. Instead, it narrated a litany of woes without offering any evidence to show that they translated into specific and substantial losses that would necessitate retrenchment. Interestingly, PAL submitted its audited financial statements only when the case was the subject of certiorari proceedings in the Court of Appeals by attaching in its Comment a copy of its consolidated audited financial statements for the years 2002, 2003 and 2004. However, these are not the financial statements that would have shown PALs alleged precarious position at the time it implemented the massive retrenchment scheme in 1998. PAL should have submitted its financial statements for the years 1997 up to 1999; and not for the years 2002 up to 2004 because these financial statements cover a period markedly distant to the years in question, which make them irrelevant and unacceptable. FOURTH ELEMENT: Concededly, retrenchment to prevent losses is an authorized cause for terminating employment and the decision whether to resort to such move or not is a management prerogative. However, the right of an employer to dismiss an employee differs from and should not be confused with the manner in which such right is exercised. It must not be oppressive and abusive since it affects one's person and property. On the requirement that the prerogative to retrench must be exercised in good faith, we have ruled that the hiring of new employees and subsequent rehiring of retrenched employees constitute bad faith; that the failure of the employer to resort to other less drastic measures than retrenchment seriously belies its claim that retrenchment was done in good faith to avoid losses; and that the demonstrated arbitrariness in the selection of which of its employees to retrench is further proof of the illegality of the employers retrenchment program, not to mention its bad faith. When PAL implemented Plan 22, instead of Plan 14, which was what it had originally made known to its employees, it could not be said that it acted in a manner compatible with good faith. It offered no satisfactory explanation why it abandoned Plan 14; instead, it justified its actions of subsequently recalling to duty retrenched employees by making it appear that it was a show of good faith; that it was due to its good corporate nature that the decision to consider recalling employees was made. The truth, however, is that it was unfair for PAL to have made such a move; it was capricious and arbitrary, considering that several thousand employees who had long been working for PAL had lost their jobs, only to be recalled but assigned to lower positions (i.e., demoted), and, worse, some as new hires, without due regard for their long years of service with the airline. The irregularity of PALs implementation of Plan 14 becomes more apparent when it rehired 140 probationary cabin attendants whose services it had previously terminated, and yet proceeded to terminate the services of its permanent cabin crew personnel. In sum, we find that PAL had implemented its retrenchment program in an arbitrary manner and with evident bad faith, which prejudiced the tenurial rights of the cabin crew personnel.

Page 109

LABOR RELATIONS
Atty. Jefferson M. Marquez
FIFTH ELEMENT: In selecting employees to be dismissed, fair and reasonable criteria must be used, such as but not limited to: (a) less preferred status (e.g., temporary employee), (b) efficiency and (c) seniority. In the implementation of its retrenchment scheme, PAL evaluated the cabin crew personnels performance during the year preceding the retrenchment (1997), based on the following set of criteria or rating variables found in the Performance Evaluation Form of the cabin crew personnels Grooming and Appearance Handbook: A. B. INFLIGHT PROFICIENCY EVALUATION 30% JOB PERFORMANCE 35% Special Award +5 Commendations +2 Appreciation +1 Disciplinary Actions Reminder (-3), Warning/Admonition & Reprimands (-5), Suspension (-20), Passenger Complaints (-30), Appearance (-10) ATTENDANCE 35% Perfect Attendance +2 Missed Assignment -30 Sick Leaves in excess of allotment and other leaves in excess of allotment -20 Tardiness -10 1[93]

C.

The appellate court held that there was no need for PAL to consult with FASAP regarding standards or criteria that the airline would utilize in the implementation of the retrenchment program; and that the criteria actually used which was unilaterally formulated by PAL using its Performance Evaluation Form in its Grooming and Appearance Handbook was reasonable and fair. Indeed, PAL was not obligated to consult FASAP regarding the standards it would use in evaluating the performance of the each cabin crew. However, the criteria utilized by PAL in the actual retrenchment were not reasonable and fair. Indeed, the NLRC made a detailed listing of the retrenchment scheme based on the ICCD Masterank and Seniority 1997 Ratings. It found the following: 1. 2. 3. 4. 5. Number of employees retrenched due to inverse seniority rule and other reasons -- 454 Number of employees retrenched due to excess sick leaves -- 299 Number of employees who were retrenched due to excess sick leave and other reasons -- 61 Number of employees who were retrenched due to other reasons -- 107 Number of employees who were demoted -- 552

Total -- 1,473.4 Prominent from the above data is the retrenchment of cabin crew personnel due to other reasons which, however, are not specifically stated and shown to be for a valid cause. This is not allowed because it has no basis in fact and in law. Moreover, in assessing the overall performance of each cabin crew personnel, PAL only considered the year 1997. This makes the evaluation of each cabin attendants efficiency rating capricious and prejudicial to PAL employees covered by it. By discarding the cabin crew personnels previous years of service and taking into consideration only one years worth of job performance for evaluation, PAL virtually did away with the concept of seniority, loyalty and past efficiency, and treated all cabin attendants as if they were on equal footing, with no one more senior than the other. In sum, PALs retrenchment program is illegal because it was based on wrongful premise (Plan 14, which in reality turned out to be Plan 22, resulting in retrenchment of more cabin attendants than was necessary) and in a set of criteria or rating variables that is unfair and unreasonable when implemented. It failed to take into account each cabin attendants respective service record, thereby disregarding seniority and loyalty in the evaluation of overall employee performance.

Page 110

LABOR RELATIONS
Atty. Jefferson M. Marquez
DISPOSITION: WHEREFORE, the instant petition is GRANTED. The assailed Decision of the Court of Appeals in CA-G.R. SP No. 87956 dated August 23, 2006, which affirmed the Decision of the NLRC setting aside the Labor Arbiters findings of illegal retrenchment and its Resolution of May 29, 2007 denying the motion for reconsideration, are REVERSED and SET ASIDE and a new one is rendered: 1. FINDING respondent Philippine Airlines, Inc. GUILTY of illegal dismissal;

2. ORDERING Philippine Air Lines, Inc. to reinstate the cabin crew personnel who were covered by the retrenchment and demotion scheme of June 15, 1998 made effective on July 15, 1998, without loss of seniority rights and other privileges, and to pay them full backwages, inclusive of allowances and other monetary benefits computed from the time of their separation up to the time of their actual reinstatement, provided that with respect to those who had received their respective separation pay, the amounts of payments shall be deducted from their backwages. Where reinstatement is no longer feasible because the positions previously held no longer exist, respondent Corporation shall pay backwages plus, in lieu of reinstatement, separation pay equal to one (1) month pay for every year of service; 3. ORDERING Philippine Airlines, Inc. to pay attorneys fees equivalent to ten percent (10%) of the total monetary award.

Costs against respondent PAL.

20. John Hancock Life Insurance Corp. vs. Davis, G.R. No. 169549, Sept. 3, 2008 Facts: Joanna Cantre Davis was agency administration officer of John Hancock Life Insurance Corporation. On October 18, 2000, Patricia Yuseco, JHLICs corporate affairs manager, discovered that her wallet was missing. She immediately reported the loss of her credit cards to AIG and BPI Express. To her surprise, she was informed that "Patricia Yuseco" had just made substantial purchases using her credit cards in various stores in the City of Manila. She was also told that a proposed transaction in Abenson's-Robinsons Place was disapproved because "she" gave the wrong information upon verification. Because loss of personal property among its employees had become rampant in its office, petitioner sought the assistance of NBI. The NBI, obtained a security video from Abenson's showing the person who used Yuseco's credit cards. Yuseco and other witnesses positively identified the person in the video as Davis NBI and Yuseco filed a complaint for qualified theft against Davis but because the affidavits presented by the NBI (identifying respondent as the culprit) were not properly verified, the city prosecutor dismissed the complaint due to insufficiency of evidence. Meanwhile, petitioner placed Davis under preventive suspension and instructed her to cooperate with its ongoing investigation. Davis filed a complaint for illegal dismissal alleging that petitioner terminated her employment without cause. The labor arbiter, in May 21, 2002, found that Davis committed serious misconduct (she was the principal suspect for qualified theft committed inside petitioner's office during work hours). There was a valid cause for her dismissal. Thus, the complaint was dismissed for lack of merit. Upon appeal, NLRC affirmed the labor arbiter in July 31, 2003 and denied her motion for reconsideration in October 30, 2003. Upon petition for certiorari filed with the CA, CA on July 4, 2005 granted the petition holding that the labor arbiter and NLRC merely adopted the findings of the NBI regarding respondent's culpability. Because the affidavits of the witnesses were not verified, they did not constitute substantial evidence. The labor arbiter and NLRC should have assessed evidence independently as "unsubstantiated suspicions, accusations and conclusions of employers (did) not provide legal justification for dismissing an employee". Petitioner moved for reconsideration but it was denied. Hence, this petition where petitioner argues that the ground for an employee's dismissal need only be proven by substantial evidence. Thus, the dropping of charges against an employee (especially on a technicality such as lack of proper verification) or his subsequent acquittal does not preclude an employer from dismissing him due to serious misconduct. Issue: Whether or not petitioner substantially proved the presence of valid cause for respondent's termination. Ruling: Supreme Court granted the petition and ruled that petitioner validly dismissed Davis for cause analogous to serious misconduct. Article 282 of the Labor Code provides: Termination by Employer. An employer may terminate an employment for any of the following causes: (a) Serious misconduct or willful disobedience by the employee of the lawful orders of his employer or his representatives in connection with his work; (e) Other causes analogous to the foregoing.

Page 111

LABOR RELATIONS
Atty. Jefferson M. Marquez
Misconduct involves "the transgression of some established and definite rule of action, forbidden act, a dereliction of duty, willful in character, and implies wrongful intent and not mere error in judgment". For misconduct to be serious and therefore a valid ground for dismissal, it must be: of grave and aggravated character and not merely trivial or unimportant and connected with the work of the employee. In this case, petitioner dismissed Davis based on the NBI's finding that the latter stole and used Yuseco's credit cards. But since the theft was not committed against petitioner itself but against one of its employees, respondent's misconduct was not work-related and therefore, she could not be dismissed for serious misconduct. Nonetheless, Article 282 (e) of the Labor Code talks of other analogous causes or those which are susceptible of comparison to another in general or in specific detail. For an employee to be validly dismissed for a cause analogous to those enumerated in Article 282, the cause must involve a voluntary and/or willful act or omission of the employee. A cause analogous to serious misconduct is a voluntary and/or willful act or omission attesting to an employee's moral depravity. Theft committed by an employee against a person other than his employer, if proven by substantial evidence, is a cause analogous to serious misconduct. Did petitioner substantially prove the existence of valid cause for respondent's separation? Yes. The labor arbiter and the NLRC relied not only on the affidavits of the NBI's witnesses but also on that of respondent. They likewise considered petitioner's own investigative findings. Clearly, they did not merely adopt the findings of the NBI but independently assessed evidence presented by the parties. Their conclusion (that there was valid cause for respondent's separation from employment) was therefore supported by substantial evidence.

21. Merin vs. NLRC, G.R. No. 171790, October 17, 2008 Facts: Respondent Mamac was hired as bus conductor of Don Mariano Transit Corporation (DMTC) on April 29, 1999. Respondent was required to accomplish a "Conductor's Trip Report" and submit it to the company after each trip. As a background, this report indicates the ticket opening and closing for the particular day of duty. After submission, the company audits the reports. Once an irregularity is discovered, the company issues an "Irregularity Report" against the employee, indicating the nature and details of the irregularity. Thereafter, the concerned employee is asked to explain the incident by making a written statement or counter-affidavit at the back of the same Irregularity Report. After considering the explanation of the employee, the company then makes a determination of whether to accept the explanation or impose upon the employee a penalty for committing an infraction. That decision shall be stated on said Irregularity Report and will be furnished to the employee. Upon audit of the October 28, 2001 Conductor's Report of respondent, KKTI noted an irregularity. It discovered that respondent declared several sold tickets as returned tickets causing KKTI to lose an income of eight hundred and ninety pesos. While no irregularity report was prepared on the October 28, 2001 incident, KKTI nevertheless asked respondent to explain the discrepancy. In his letter, respondent said that the erroneous declaration in his October 28, 2001 Trip Report was unintentional. He explained that during that day's trip, the windshield of the bus assigned to them was smashed; and they had to cut short the trip in order to immediately report the matter to the police. As a result of the incident, he got confused in making the trip report. On November 26, 2001, respondent received a letter terminating his employment effective November 29, 2001. The dismissal letter alleged that the October 28, 2001 irregularity was an act of fraud against the company. KKTI also cited as basis for respondent's dismissal the other offenses he allegedly committed since 1999. Respondent filed a Complaint for illegal dismissal, illegal deductions, nonpayment of 13th-month pay, service incentive leave, and separation pay. He denied committing any infraction and alleged that his dismissal was intended to bust union activities. Moreover, he claimed that his dismissal was effected without due process. KKTI contended that respondent was legally dismissed after his commission of a series of misconducts and misdeeds. It claimed that respondent had violated the trust and confidence reposed upon him by KKTI. Also, it averred that it had observed due process in dismissing respondent and maintained that respondent was not entitled to his money claims such as service incentive leave and 13th-month pay because he was paid on commission or percentage basis. Issue: Whether or not procedural requirements were complied with. Ruling: Due process under the Labor Code involves two aspects: first, substantive the valid and authorized causes of termination of employment under the Labor Code; and second, procedural the manner of dismissal.

Page 112

LABOR RELATIONS
Atty. Jefferson M. Marquez
Non-compliance with the Due Process Requirements Art. 277 of the Labor Code provides the manner of termination of employment, thus: Art. 277. Miscellaneous Provisions. . . . (b)Subject to the constitutional right of workers to security of tenure and their right to be protected against dismissal except for a just and authorized cause without prejudice to the requirement of notice under Article 283 of this Code, the employer shall furnish the worker whose employment is sought to be terminated a written notice containing a statement of the causes for termination and shall afford the latter ample opportunity to be heard and to defend himself with the assistance of his representative if he so desires in accordance with company rules and regulations promulgated pursuant to guidelines set by the Department of Labor and Employment. Any decision taken by the employer shall be without prejudice to the right of the worker to contest the validity or legality of his dismissal by filing a complaint with the regional branch of the National Labor Relations Commission. The burden of proving that the termination was for a valid or authorized cause shall rest on the employer. Accordingly, the implementing rule of the aforesaid provision states: SEC. 2.Standards of due process; requirements of notice. In all cases of termination of employment, the following standards of due process shall be substantially observed: I.For termination of employment based on just causes as defined in Article 282 of the Code: (a)A written notice served on the employee specifying the ground or grounds for termination, and giving said employee reasonable opportunity within which to explain his side. (b)A hearing or conference during which the employee concerned, with the assistance of counsel if he so desires is given opportunity to respond to the charge, present his evidence, or rebut the evidence presented against him. (c)A written notice of termination served on the employee, indicating that upon due consideration of all the circumstances, grounds have been established to justify his termination. In case of termination, the foregoing notices shall be served on the employee's last known address. To clarify, the following should be considered in terminating the services of employees: (1)The first written notice to be served on the employees should contain the specific causes or grounds for termination against them, and a directive that the employees are given the opportunity to submit their written explanation within a reasonable period. "Reasonable opportunity" under the Omnibus Rules means every kind of assistance that management must accord to the employees to enable them to prepare adequately for their defense. This should be construed as a period of at least five (5) calendar days from receipt of the notice to give the employees an opportunity to study the accusation against them, consult a union official or lawyer, gather data and evidence, and decide on the defenses they will raise against the complaint. Moreover, in order to enable the employees to intelligently prepare their explanation and defenses, the notice should contain a detailed narration of the facts and circumstances that will serve as basis for the charge against the employees. A general description of the charge will not suffice. Lastly, the notice should specifically mention which company rules, if any, are violated and/or which among the grounds under Art. 282 is being charged against the employees. (2)After serving the first notice, the employers should schedule and conduct a hearing or conference wherein the employees will be given the opportunity to: (1) explain and clarify their defenses to the charge against them; (2) present evidence in support of their defenses; and (3) rebut the evidence presented against them by the management. During the hearing or conference, the employees are given the chance to defend themselves personally, with the assistance of a representative or counsel of their choice. Moreover, this conference or hearing could be used by the parties as an opportunity to come to an amicable settlement. (3)After determining that termination of employment is justified, the employers shall serve the employees a written notice of termination indicating that: (1) all circumstances involving the charge against the employees have been considered; and (2) grounds have been established to justify the severance of their employment. In the instant case, KKTI admits that it had failed to provide respondent with a "charge sheet." However, it maintains that it had substantially complied with the rules, claiming that "respondent would not have issued a written explanation had he not been informed of the charges against him." We are not convinced. First, respondent was not issued a written notice charging him of committing an infraction. The law is clear on the matter. A verbal appraisal of the charges against an employee does not comply with the first notice requirement. In Pepsi Cola Bottling Co. v. NLRC, the Court held that consultations or conferences are not a substitute for the actual observance of notice and hearing.

Page 113

LABOR RELATIONS
Atty. Jefferson M. Marquez
Second, the Court observed the irregularity reports against respondent for his other offenses that such contained merely a general description of the charges against him. The reports did not even state a company rule or policy that the employee had allegedly violated. Likewise, there is no mention of any of the grounds for termination of employment under Art. 282 of the Labor Code. Thus, KKTI's "standard" charge sheet is not sufficient notice to the employee. Third, no hearing was conducted. Regardless of respondent's written explanation, a hearing was still necessary in order for him to clarify and present evidence in support of his defense. Moreover, respondent made the letter merely to explain the circumstances relating to the irregularity. He was unaware that a dismissal proceeding was already being effected. Sanction for Non-compliance with Due Process Requirements As stated earlier, after a finding that petitioners failed to comply with the due process requirements, the CA awarded full backwages in favor of respondent in accordance with the doctrine in Serrano v. NLRC. However, the doctrine in Serrano had already been abandoned in Agabon v. NLRC by ruling that if the dismissal is done without due process, the employer should indemnify the employee with nominal damages. Thus, for non-compliance with the due process requirements in the termination of respondent's employment, petitioner KKTI is sanctioned to pay respondent the amount of thirty thousand pesos (PhP30,000) as damages.

22. Yrasuegui vs. Phil Airlines, G.R. No. 168081, Oct. 17, 2008 Facts: Armando G. Yrasuegui was a former international flight steward of Philippine Airlines, Inc. (PAL). He stands five feet and eight inches (58) with a large body frame. The proper weight for a man of his height and body structure is from 147 to 166 pounds, the ideal weight being 166 pounds, as mandated by the Cabin and Crew Administration Manual of PAL. His weight problem dates back to 1984 when PAL advised him to go on an extended vacation leave from December 29, 1984 to March 4, 1985 to address his weight concerns. For failure to meet the weight standards another leave without pay from March 5, 1985 to November 1985 was imposed. He met the required weight and was allowed to work but his weight problem recurred, thus another leave without pay from October 17, 1988 to February 1989. From 1989 to 1992 his weight fluctuated from 209lb, 215lb, 217lb, 212lb, and 205. During that period he was requested to lose weight and to report for weight checks which he constantly failed to do. In the meantime his status was off-duty. Finally in 1993, petitioner was formally informed by PAL that due to his inability to attain his ideal weight, and considering the utmost leniency extended to him which spanned a period covering a total of almost five (5) years, his services were considered terminated effective immediately. He then filed a complaint for illegal dismissal against PAL. The Labor Arbiter ruled that he was illegally dismissed and entitles to reinstatement, backwages and attorneys fees. The NLRC affirmed the LA. The CA reversed the NLRC. Issue: Whether or not petitioner was illegally dismissed. Ruling: The obesity of petitioner is a ground for dismissal under Article 282(e) of the Labor Code. The weight standards of PAL constitute a continuing qualification of an employee in order to keep the job. Tersely put, an employee may be dismissed the moment he is unable to comply with his ideal weight as prescribed by the weight standards. The dismissal would fall under Article 282(e) of the Labor Code. As explained by the CA: x x x [T]he standards violated in this case were not mere orders of the employer; they were the prescribed weights that a cabin crew must maintain in order to qualify for and keep his or her position in the company. In other words, they were standards that establish continuing qualifications for an employees position. The failure to meet the employers qualifying standards is in fact a ground that does not squarely fall under grounds (a) to (d) and is therefore one that falls under Article 282(e) the other causes analogous to the foregoing. By its nature, these qualifying standards are norms that apply prior to and after an employee is hired. x x x We hold that the obesity of petitioner, when placed in the context of his work as flight attendant, becomes an analogous cause under Article 282(e) of the Labor Code that justifies his dismissal from the service. His obesity may not be unintended, but is nonetheless voluntary. II. The dismissal of petitioner can be predicated on the bona fide occupational qualification defense. Employment in particular jobs may not be

Page 114

LABOR RELATIONS
Atty. Jefferson M. Marquez
limited to persons of a particular sex, religion, or national origin unless the employer can show that sex, religion, or national origin is an actual qualification for performing the job. The qualification is called a bona fide occupational qualification (BFOQ). A common carrier, from the nature of its business and for reasons of public policy, is bound to observe extraordinary diligence for the safety of the passengers it transports. Thus, it is only logical to hold that the weight standards of PAL show its effort to comply with the exacting obligations imposed upon it by law by virtue of being a common carrier. The primary objective of PAL in the imposition of the weight standards for cabin crew is flight safety. The task of a cabin crew or flight attendant is not limited to serving meals or attending to the whims and caprices of the passengers. The most important activity of the cabin crew is to care for the safety of passengers and the evacuation of the aircraft when an emergency occurs. Passenger safety goes to the core of the job of a cabin attendant. Truly, airlines need cabin attendants who have the necessary strength to open emergency doors, the agility to attend to passengers in cramped working conditions, and the stamina to withstand grueling flight schedules. On board an aircraft, the body weight and size of a cabin attendant are important factors to consider in case of emergency. Aircrafts have constricted cabin space, and narrow aisles and exit doors. Thus, the arguments of respondent that [w]hether the airlines flight attendants are overweight or not has no direct relation to its mission of transporting passengers to their destination; and that the weight standards has nothing to do with airworthiness of respondents airlines, must fail. The job of a cabin attendant during emergencies is to speedily get the passengers out of the aircraft safely. Being overweight necessarily impedes mobility. Indeed, in an emergency situation, seconds are what cabin attendants are dealing with, not minutes. Three lost seconds can translate into three lost lives. Evacuation might slow down just because a wide-bodied cabin attendant is blocking the narrow aisles. Petitioner is entitled to separation pay. Normally, a legally dismissed employee is not entitled to separation pay. This may be deduced from the language of Article 279 of the Labor Code that [a]n employee who is unjustly dismissed from work shall be entitled to reinstatement without loss of seniority rights and other privileges and to his full backwages, inclusive of allowances, and to his other benefits or their monetary equivalent computed from the time his compensation was withheld from him up to the time of his actual reinstatement. Luckily for petitioner, this is not an ironclad rule. Exceptionally, separation pay is granted to a legally dismissed employee as an act social justice, or based on equity. In both instances, it is required that the dismissal (1) was not for serious misconduct; and (2) does not reflect on the moral character of the employee. Here, We grant petitioner separation pay equivalent to one-half (1/2) months pay for every year of service. It should include regular allowances which he might have been receiving.

23. Sagales v. Rustans Commercial Corporation, G.R. No. 166554, November 27, 2008 Facts: Petitioner Julito Sagales was employed by respondent Rustans Commercial Corporation occupying the position of Chief Cook at the Yum Yum Tree Coffee Shop from October 1970 until July 26, 2001, when he was terminated. He was paid a basic monthly salary of P9,880.00. He was also receiving service charge of not less than P3,000.00 a month and other benefits under the law. Petitioner was also a consistent recipient of numerous citations for his performance. After receiving his latest award, petitioner conveyed to respondent his intention of retiring, after reaching 31 years in service. Petitioner, however, was not allowed to retire with his honor intact. On June 18, 2001, Security Guard Magtangob apprehended petitioner in the act of taking out from Rustans Supermarket a plastic bag containing 1.335 kilos of squid heads worth P50.00. Petitioner was not able to show any receipt when confronted. Thus, petitioner was brought to the Makati Police Criminal Investigation Division where he was detained. Petitioner was later ordered released pending further investigation but respondent placed petitioner under preventive suspension. During the inquest proceedings for qualified theft before Assistant Prosecutor Pineda, petitioner contended that although he was in possession of the plastic bag containing the squid heads, he did not steal them because he actually paid for them. As proof, petitioner presented a receipt. The only fault he committed was his failure to immediately show the purchase receipt when he was accosted because he misplaced it when he changed his clothes. He also alleged that the squid heads were already scraps as these were not intended for cooking. Neither were the squid heads served to customers. He bought the squid heads so that they could be eaten instead of being thrown away. If he intended to steal from respondent, he could have stolen other valuable items instead of scrap. Pineda believed the version of petitioner and recommended the dismissal of the case for lack of evidence. A formal investigation was conducted by the Legal Department on July 6, 2001. Respondent did not find merit in the explanation of petitioner. Thus, petitioner was dismissed from service on July 26, 2001. At that time, petitioner had been under preventive suspension for 1 month. Aggrieved, petitioner filed a complaint for illegal dismissal against respondent. He also prayed for unpaid salaries/wages, overtime pay, as well as moral and exemplary damages, attorneys fees, and service charges.

Page 115

LABOR RELATIONS
Atty. Jefferson M. Marquez
Labor Arbiter Pati dismissed the complaint. According to the Labor Arbiter, the nature of the responsibility of petitioner was not that of an ordinary employee. It then went on to categorize petitioner as a supervisor in a position of responsibility where trust and confidence is inherently infused. As such, it behooved him to be more knowledgeable if not the most knowledgeable in company policies on employee purchases of food scrap items in the kitchen. Per the evidence presented by respondent, petitioner breached company policy which justified his dismissal. On appeal, NLRC reversed the decision of the Labor Arbiter. The NLRC held that the petitioner is a mere rank-and-file employee. The evidence is also wanting that petitioner committed the crime charged. The NLRC did not believe that petitioner would trade off almost 31 years of service for P50.00 worth of squid heads. Hence, petitioner was illegally dismissed as respondent failed to establish a just cause for dismissal. However, the claim for damages was denied for lack of evidence. Respondent brought the matter to the CA. In reversing the NLRC, the CA opined that the position of petitioner was supervisory in nature. The CA also held that the evidence presented by respondent clearly established loss of trust and confidence on petitioner. Lastly, the CA, although taking note of the long years of service of petitioner and his numerous awards, refused to award separation pay in his favor. According to the CA, the award of separation pay cannot be sustained under the social justice theory because the instant case involves theft of the employers property. Petitioner left with no other recourse, availed of the present remedy. Issue:S: (1) WON petitioners position is supervisory in nature which is covered by the trust and confidence rule. (2) WON the evidence on record sufficient to conclude that petitioner committed the crime charged. (3) WON the penalty of dismissal is proper. Ruling: I. The position of petitioner is supervisory in nature which is covered by the trust and confidence rule. The nature of the job of an employee becomes relevant in termination of employment by the employer because the rules on termination of managerial and supervisory employees are different from those on the rank-and-file. Managerial employees are tasked to perform key and sensitive functions, and thus are bound by more exacting work ethics. As a consequence, managerial employees are covered by the trust and confidence rule. The same holds true for supervisory employees occupying positions of responsibility. There is no doubt that the position of petitioner as chief cook is supervisory in nature. A chief cook directs and participates in the preparation and serving of meals; determines timing and sequence of operations required to meet serving times; and inspects galley and equipment for cleanliness and proper storage and preparation of food. Naturally, a chief cook falls under the definition of a supervisor, i.e., one who, in the interest of the employer, effectively recommends managerial actions which would require the use of independent judgment and is not merely routinary or clerical. It has not escaped Our attention that petitioner changed his stance as far as his actual position is concerned. In his position paper, he alleged that at the time of his dismissal, he was Chief Cook.[46] However, in his memorandum, he now claimed that he was an Asst. Cook. The ploy is clearly aimed at giving the impression that petitioner is merely a rank-and-file employee. The change in nomenclature does not, however, help petitioner, as he would still be covered by the trust and confidence rule. Of course, the ruling assumes greater significance if petitioner is the chief cook. A chief cook naturally performs greater functions and has more responsibilities than an assistant cook. In eo quod plus sit simper inest et minimus. The greater always includes the less. Ang malawak ay laging sumasakop sa maliit. II. The evidence on record is sufficient to conclude that petitioner committed the crime charged. Security of tenure is a paramount right of every employee that is held sacred by the Constitution. The reason for this is that labor is deemed to be property within the meaning of constitutional guarantees. Indeed, as it is the policy of the State to guarantee the right of every worker to security of tenure as an act of social justice, such right should not be denied on mere speculation of any similar or unclear nebulous basis. Indeed, the right of every employee to security of tenure is all the more secured by the Labor Code by providing that the employer shall not terminate the services of an employee except for a just cause or when authorized by law. Otherwise, an employee who is illegally dismissed shall be entitled to reinstatement without loss of seniority rights and other privileges and to his full backwages, inclusive of allowances, and to his other benefits or their monetary equivalent computed from the time his compensation was withheld from him up to the time of his actual reinstatement.

Page 116

LABOR RELATIONS
Atty. Jefferson M. Marquez
Necessarily then, the employer bears the burden of proof to show the basis of the termination of the employee. In the case at bar, respondent has discharged its onus of proving that petitioner committed the crime charged. We quote with approval the observation of the CA in this regard: On this matter, petitioner presents as evidence the verified statement of security guard Aranas. Aranas positively saw the private in the act of bringing out the purloined squid heads. Similarly, the statement of security guard Magtangob attested to the commission by private respondent of the offense charged. Further, the verified statement of Samson, store manager of petitioner corporation who is in charge of all personnel, including employees of the Yum Yum Tree Coffee Shop of which private respondent was a former assistant cook, attested to the fact of private respondent seeking apology for the commission of the act. Likewise, the statement of Zenaida Castro (Castro), cashier of petitioner corporations supermarket, Makati Branch, Ayala Center, Makati City, confirmed that indeed the 1.335 kilos of squid heads amounting to fifty pesos (P50.00)per kilo, had not been paid for. The contention of petitioner that respondent merely imputed the crime against him because he was set to retire is difficult, if not impossible, to believe. Worth noting is the fact that petitioner failed to impute any ill will or motive on the part of the witnesses against him. We stress that the quantum of proof required for the application of the loss of trust and confidence rule is not proof beyond reasonable doubt. It is sufficient that there must only be some basis for the loss of trust and confidence or that there is reasonable ground to believe, if not to entertain the moral conviction, that the employee concerned is responsible for the misconduct and that his participation in the misconduct rendered him absolutely unworthy of trust and confidence. It is also of no moment that the criminal complaint for qualified theft against petitioner was dismissed. It is well settled that the conviction of an employee in a criminal case is not indispensable to the exercise of the employers disciplinary authority. III. The penalty of dismissal is too harsh under the circumstances. The free will of management to conduct its own business affairs to achieve its purpose cannot be denied. The only condition is that the exercise of management prerogatives should not be done in bad faith or with abuse of discretion. Truly, while the employer has the inherent right to discipline, including that of dismissing its employees, this prerogative is subject to the regulation by the State in the exercise of its police power. In this regard, it is a hornbook doctrine that infractions committed by an employee should merit only the corresponding penalty demanded by the circumstance. The penalty must be commensurate with the act, conduct or omission imputed to the employee and must be imposed in connection with the disciplinary authority of the employer. In the case at bar, petitioner deserves compassion more than condemnation. At the end of the day, it is undisputed that: (1) petitioner has worked for respondent for almost thirty-one (31) years; (2) his tireless and faithful service is attested by the numerous awards he has received from respondent; (3) the incident on June 18, 2001 was his first offense in his long years of service; (4) the value of the squid heads worth P50.00 is negligible; (5) respondent practically did not lose anything as the squid heads were considered scrap goods and usually thrown away in the wastebasket; (6) the ignominy and shame undergone by petitioner in being imprisoned, however momentary, is punishment in itself; and (7) petitioner was preventively suspended for one month, which is already a commensurate punishment for the infraction committed. Truly, petitioner has more than paid his due. In any case, it would be useless to order the reinstatement of petitioner, considering that he would have been retired by now. Thus, in lieu of reinstatement, it is but proper to award petitioner separation pay computed at one-month salary for every year of service, a fraction of at least six (6) months considered as one whole year. In the computation of separation pay, the period where backwages are awarded must be included.

24. Garcia vs. PAL, G.R. No. 164856, Jan. 20, 2009, En Banc, citing Genuino vs. NLRC, G.R. No. 142732-33, December 4, 2007 Facts: This case stemmed from an administrative charge filed by PAL against employees, herein petitioners after allegedly being caught in the act of sniffing shabu in the workplace. After due notice, PAL dismissed petitioners prompting the latter to file a complaint for illegal dismissal which was resolved by the Labor Arbiter in their favor ordering inter alia their reinstatement. Subsequently, respondent company was placed under corporate rehabilitation. From the Labor Arbiter, respondent appealed to NLRC which reversed said decision. Later, a writ of execution as regards the reinstatement was issued by the Labor Arbiter. Respondent then filed an urgent petition for injunction on the ground that it cannot

Page 117

LABOR RELATIONS
Atty. Jefferson M. Marquez
comply with the reinstatement order due to its corporate rehabilitation. Issues: 1. Whether a subsequent finding of a valid dismissal by NLRC removes the basis for implementing the reinstatement aspect of the Labor Arbiters decision? 2. Whether respondent company is justified in refusing to comply with such reinstatement order in view of its corporate rehabilitation? Ruling: On the first issue, jurisprudential trend has maintained that even if the order of reinstatement of the Labor Arbiter is reversed on appeal, it is obligatory on the part of the employer to reinstate and pay the wages of the dismissed employee during the period of appeal until reversal by the higher court. The employee is not required to reimburse whatever salary he may have received for he is entitled to such. The opposite view is articulated in Genuino vs NLRC which states: If the decision of the Labor Arbiter is later reversed on appeal upon the finding that the ground for dismissal is valid, then the employer has the right to require the dismissed employee on payroll reinstatement to refund the salaries he or she received while the case was pending appeal, xxx. Considering that Genuino was not reinstated to work or placed on payroll reinstatement, and her dismissal is based on a just cause, then she is not entitled to be paid the salaries xxx. However, the dearth of authority supporting Genuino renders inutile the rationale of reinstatement pending appeal. Pursuant to police power, the State may authorize an immediate implementation, pending appeal of a decision reinstating a dismissed or separated employee since that saving act is designed to stop, although temporarily since the appeal may be decided in favor of the appellant, a continuing threat or danger to the survival or even the life of the dismissed or separated employee and his family. Thus, the Refund Doctrine easily demonstrates how a favorable decision by the Labor Arbiter could harm more than help a dismissed employee. The employee, to make both ends meet, would necessarily have to use up the salaries received during the pendency of the appeal, only to end up having to refund the sum in case of a final unfavorable decision. The provision of Art. 223 is clear that an award by the Labor Arbiter for reinstatement shall be immediately executory even pending appeal and the posting of a bond by the employer shall not stay the execution for reinstatement. The legislative intent is quite obvious i.e. to make an award of reinstatement immediately enforceable, even pending appeal. The Court reaffirms such prevailing principle that even if the order of reinstatement is reversed on appeal, it is obligatory on the part of the employer to reinstate and pay the wages of the dismissed employee during the period of appeal until reversal by the higher court. After the labor arbiters decision is reversed by a higher court, the employee may be barred from collecting the accrued wages if it is shown that the delay in enforcing the reinstatement was without fault of the employer. The test is two-fold: a) there must be actual delay or the fact that the order of reinstatement pending appeal was not executed prior to its reversal and b) the delay must not be due to the employers unjustified act or omission. If the delay is due to employers unjustified refusal, the employer may still be required to pay the salaries notwithstanding the reversal of the labor arbiters decision. In Genuino, the former NLRC Rules of Procedure was still applied in which it did not lay down a mechanism to promptly effectuate the self-executory order of reinstatement, making it difficult to establish that the employer actually refused to comply. The new NLRC Rules of Procedure which took effect on Jan. 7, 2006 now require the employer to submit a report of compliance within 10 calendar days from receipt of the labor arbiters decision. The employee need not file a motion for the issuance of a writ of execution since the labor arbiter shall thereafter motu proprio issues the writ. It is settled that upon appointment by SEC of a rehabilitation receiver, all actions for claims before any tribunal against the corporation shall ipso jure be suspended. Case law recognizes that unless there is a restraining order, the implementation of the order of reinstatement is ministerial and mandatory. The suspension of claims partakes of the nature of a restraining order that constitutes legal justification for respondents non-compliance with the reinstatement order. Respondents failure to exercise the alternative options of actual reinstatement and payroll reinstatement was thus justified. The petition is denied. The CA decision annulling the NLRC resolutions affirming the validity of the Writ of Execution and Notice of Garnishment is affirmed.

25. La Union Cement Workers Union et al., vs NLRC et al., G.R. No. 174621, January 30, 2009 Facts: Private respondent Bacnotan Cement Corporation (respondent company), now known as Holcim Philippines, Inc., is engaged in the manufacture of cement. Prior to 1994, respondent company had been utilizing the "wet process technology" in its operations. Sometime in 1992, respondent company introduced the "dry process technology" as part of its modernization program. In 1995, the new "dry process technology" became fully

Page 118

LABOR RELATIONS
Atty. Jefferson M. Marquez
operational. After a comparative study of the two production lines, respondent company discovered that the "dry process technology" or the dry line proved to be more efficient as the cost was minimized by P15.00 per cement bag while the "wet process technology" or the wet line consumed more fuel and had to undergo frequent repairs and shutdowns due to its obsolescence. To implement the closure of the wet line, respondent company and petitioner La Union Cement Workers Union (petitioner Union) entered into a Memorandum of Agreement on 19 July 1997, whereby respondent company committed to grant separation pay equivalent to 150% of the monthly basic pay for every year of service plus the additional fixed amount of P27,000.00 to employees who would be terminated as a result of the closure of the wet line. In an open letter dated 11 August 1997, Magdaleno B. Albarracin, Jr., the respondent companys Senior Executive Vice President, notified the employees of the its decision to mothball the wet line and the termination of those whose employment would become unnecessary as a result of the closure. On 15 August 1997, respondent company sent a letter to the office of Ricardo S. Martinez, Regional Director of the Department of Labor and Employment (DOLE), informing him about respondent companys decision to shut down the wet line and furnishing him the list of affected employees. On 16 August 1997, respondent company sent notices of termination to more or less 200 employees including petitioner Almoite. Upon the receipt of the separation pay, a number of the affected employees signed individual Release Waiver and Quitclaim. Sometime in November 1997, petitioner Union and some 80 of its members including petitioner Almoite filed complaints for unfair labor practice, illegal lay-off and illegal dismissal against respondent company before the NLRC Regional Arbitration Branch 1 in San Fernando, La Union. The petitioners alleged that while the closure affected only the wet line, among the employees terminated were operating the dry line or performing support services for both wet and dry lines. They further alleged that after the closure of the wet line, respondent company contracted out the services performed by the employees who were terminated. Only 31 of the 80 employees pursued the complaints before the Labor Arbiter. After submission of the parties position papers and pleadings, Labor Arbiter Irenarco R. Rimando rendered a Decision on 19 March 1999 dismissing the complaints. Labor Arbiter Rimando found that respondent company complied with the requisite notice and severance pay mandated under Article 283 of the Labor Code. As regards the claim that the services performed by the complainants were eventually assumed by employees who were retained or were contracted out, Labor Arbiter Rimando ruled that the employer had the prerogative to utilize its remaining workforce to the maximum. Petitioners appealed to the NLRC, arguing that respondent company failed to prove with substantial evidence that the retrenchment was absolutely necessary and unavoidable mainly because the affected employees were also performing support services in the wet line. Public respondent NLRC affirmed in toto the decision of Labor Arbiter Rimando. It held that the appeal was brought by petitioner Union and not by its members who were the real parties-in-interest and, thus, must be dismissed outright. The NLRC held that the retrenchment on the ground of redundancy was valid in any case. It concluded that the scaling down of activities requiring support services was a consequence of the closure of the wet line; hence, the termination of the excess employees performing such support services followed as a matter of course. Ruling: In a Resolution dated 13 December 2006, the Court dismissed the petition with respect to petitioner Union for insufficiency or defective verification and certification of non-forum shopping, as only the president of petitioner Union signed the same in violation of Sections 4 and 5, Rule 7 of the Rules of Court. The judgment of dismissal has become final and executory with respect to petitioner Union on 08 February 2007. The instant petition raises two issues: namely, whether petitioner Union is the real party-in-interest in this case and whether petitioner Almoites termination was valid. The question of petitioner Unions capacity to sue on behalf of its members has become moot and academic in view of the judgment of dismissal of the instant petition which has already become final and executory with respect to petitioner Union. Thus, the remaining issue to be resolved in this petition pertains to petitioner Almoites claim that petitioner Union has failed to prove that his work as an oiler for both the wet and dry lines has become redundant with the closure of the wet line. Petitioner Almoites claim is clearly a factual question which is beyond the province of a Rule 45 petition. In any event, the Court finds no cogent reason to disturb the judgment of the Court of Appeals affirming the Labor Arbiter and NLRC rulings that the termination of petitioner Almoite and the other employees of respondent company. As explained by the NLRC, the termination of petitioner Almoite was a necessary consequence of the partial closure of operations of respondent company. Petitioner Almoites work as an oiler for both the wet line and dry line has become redundant or superfluous following the closure of the wet line. By and large, the determination of whether to maintain or phase out an entire department or section or to reduce personnel lies with the management. Thus, his termination on the ground of redundancy is an authorized cause for termination under Article 283 of the Labor Code. The Court quotes with approval the following conclusions of the NLRC: x x x There is no dispute as to the fact that there was a partial closure or cessation of operations with the mothballing of the old wet-process production line of the company a situation which falls among the authorized causes for termination allowed under Article 283 of the Labor Code. x x x Neither is there any dispute that the logical and consequence [sic]of such partial cessation of operations was to render certain employees redundant. Obviously enough, since there was a curtailment in operations, certain activities were rendered either excess or no longer necessary, hence, redundant. The only ostensible argument presented by appellant is the bare allegation that most of them were not exclusively assigned to the wet process line but were performing support services for both the wet line and the dry line. Therefore, they argue that they could not be declared redundant by virtue of the closure of the wet line alone. This line of argument is non sequitur, fallacious and totally untenable. It proceeds from the erroneous premise that only those exclusively assigned to the wet line can be declared redundant. The mere fact that an employee was performing support services for both the wet and the dry line does not in any way exclude him from being declared as redundant. On the contrary, with the closure of the wet line and the consequent scaling down of activities requiring support services, it stands to reason that there was already an excess of employees performing support services. Respondent had therefore all the reason to include such employees among those whom it considered redundant. WHEREFORE, the instant petition for review on certiorari is DENIED and the decision and resolution of the Court of Appeals in CA-G.R. SP No. 90597 are AFFIRMED.

Page 119

LABOR RELATIONS
Atty. Jefferson M. Marquez

26. Mendros, Jr. vs. Mitsubishi Motors Phils Corp., G.R. No. 169780, Feb. 16, 2009 Facts: MMPC hired Mendros in 1994 as regular body repman, later promoted to assembler major in the companys manufacturing division. Due to severe drastic slump of its vehicle sales brought about by the financial crisis in 1997, MMPC as per audited financial statements, sustained a loss of PhP 470 million in 1997, and PhP 771 million in 1998. MMPC implemented various cost-cutting measures, some of which are employment-hiring freezing, separation of casuals and trainees, manpower services reduction, plant shutdowns, and reduced work week for managerial and other monthly salaried personnel. In Feb 1998, it instituted the first stage of its retrenchment program affecting 531 hourly manufacturing employees, which proved in adequate. It then launched a temporary lay-off program of 170 hourly employees, including Mendros. The latter received a letter informing him of the temporary suspension of his employment for six months from Jan 4 to July 2, 1999. In the interim, he was updated of the business conditions by MMPC. In June 1, 1999, MMPC notified DOLE that the temporary lay-off is being made permanent effective July 2, 1999 due to continuing adverse market conditions. Mendros filed for a case of illegal dismissal. In its position papers, MMPC defined the criteria used in considering employees for retrenchment, and attached its financial statements for 1997-1996 and 1998-1997 prepared by SGV & Co. The Labor Arbiter decided in favor of MMPC. Upon appeal, the NLRC reversed said decision and declared that the dismissal was illegal stating that the merit rating system adopted by MMPC as additional criterion for retrenchment was erroneous and arbitrary, being against the CBA. According to the NLRC, the CBA listed only seniority and needs of the company as determinative factors. The NLRC said that MMPC had not notified Mendros of the additional criterion and of the findings of the merit evaluation, thus nullifying the retrenchment. The CA decided in favor of MMPC, and reinstated the LAs ruling. Issue: Whether or not the temporary lay-off and eventual retrenchment was valid. Ruling: The retrenchment was valid. The right or management to retrench workers to meet clear and continuing economic threats or during periods of economic recession to prevent losses is recognized by Art. 283 of the LC. The requirements for a valid retrenchment are: that the retrenchment is reasonably necessary and likely to prevent business losses which, if already incurred, are not merely de minimis, but substantial, serious, and real, or only if expected, are reasonably imminent as perceived objectively and in good faith by the employer; that the employer serves written notice both to the employees concerned and the DOLE at least a month before the intended date of retrenchment; that the employer pays the retrenched employee separation pay in an amount prescribed by the Code; that the employer exercises its prerogative to retrench in good faith; and that it uses fair and reasonable criteria in ascertaining who would be retrenched or retained. The losses expected should be substantial and not merely de minimis in extent. If the loss purportedly sought to be forestalled by retrenchment is clearly shown to be insubstantial and inconsequential in character, the bonafide nature of the retrenchment would appear to be seriously in question. Secondly, the substantial loss apprehended must be reasonably imminent, as such imminence can be perceived objectively and in good faith by the employer. There should be a certain degree of urgency for the retrenchment, which is after all a drastic recourse with serious consequences. Because of the consequential nature of retrenchment, it must, thirdly, be reasonably necessary and likely to effectively prevent the expected losses. The employer should have taken other measures prior or parallel to retrenchment to forestall losses, i.e., cut other costs than labor costs. An employer who lays off substantial numbers of workers while continuing to dispense fat executive bonuses and perquisites or so-called golden parachutes, can scarcely claim to be retrenching in good faith to avoid losses. To impart operational meaning to the constitutional policy of providing full protection to labor, the employers prerogative to bring down labor costs by retrenching must be exercised essentially as a measure of last resort, after less drastic meanse.g., reduction of both management and rank-and-file bonuses and salaries, going on reduced time, costs, etc.have been tried and found wanting. Lastly, alleged losses if already realized, and the expected imminent losses sought to be forestalled, must be proved by sufficient and convincing evidence. The reason for requiring this quantum of proof is readily apparent: any less exacting standard of proof would render too easy the abuse of this ground for termination of services of employees. There can hardly be any dispute that MMPC suffered substantial and heavy losses in FY 1997 and continued to bleed in 1998. As shown in its AFS for those fiscal years, it incurred an aggregate loss of PhP 1.242 billion for its two-year operation. Any suggestion that a billion peso plus loss is de minimis in extent has to be dismissed for sheer absurdity. It bears to state that Art. 283 of the Code uses the phrase retrenchment to prevent losses. The phrase necessarily implies that retrenchment may be effected even in the event only of imminent, impending, or expected losses. The employer

Page 120

LABOR RELATIONS
Atty. Jefferson M. Marquez
need not wait for substantial losses to materialize before exercising ultimate and drastic option to prevent such losses. In the case at bench, MMPC was already financially hemorrhaging before finally resorting to retrenchment. Mendros argues that since Art. V, Sec 5(c) of the CBA provides for only two factors, (1) seniority (last-in, first-out) and (2) the needs of the company, to be considered in retrenching MMPC employees, the company is bereft of authority to arbitrarily impose other factors or criteria in effecting his retrenchment. The Court is not persuaded. Sec. 1, Art. V of the CBA, allows MMPC, in the exercise of its customary management functions and prerogatives on matters of promotions, transfer, layoff, and recall, to consider as guiding norms the following factors or criteria: Seniority, Efficiency and Attitude, Job Knowledge and Potential, and Attendance. And to complement this prerogative, the company, in the same section, is given the discretion to exercise just and fair evaluation of such factors, meaning that the company is accorded a reasonable latitude to assign a corresponding weight to each factor. The Sec. 5(c) needs of the company factor, if viewed by its self without linking it to the Sec. 1 criteria, would be a meaningless, if not unreasonable, standard. The proper view, therefore, is that the Sec. 1 criteria qualify the factors of seniority and needs of the company in Sec. 5(c). Sec. 5(c) should be understood in the light of Sec. 1 which, to stress, provides seniority, efficiency and attitude, job knowledge and potential, and attendance as among the factors that should guide the company in choosing the employees to be laid-off or kept. All other things being equal, a company would necessarily need to retain those who had rendered dedicated and highly efficient service and whose knowledge, attendance, and potential hew with company standards. Any other measure would be senseless in the business viewpoint. Accordingly, the merit rating used by MMPC based on Sec. 5 in conjunction with and as qualified by the factors provided under Sec. 1 is fair and reasonable, and, to be sure, well within the contemplation of the parties CBA. Mendros lament about not being furnished a copy of the 1997-1996 and 1998-1997 AFS and other financial documents, as well as the finding of the merit evaluation rating, at the time he was notified of his lay-off cannot be accorded tenability. There is no law or rule that requires an employer to furnish an employee to be retrenched copies of its AFS and other documents. The appropriate forum for an employee to contest the reality or good faith character of the retrenchment asserted as ground for dismissal from employment is before the DOLE.

27. Rosa vs. Ambassador Hotel, G.R. No. 177059, March 13, 2009 Facts: Petitioners file a complaint with the Department of Labor and Employment-NCR which prompted an inspection of the hotels premises by a labor inspector, respondent was found to have been violating labor standards laws and was thus ordered to pay them some money claims. This purportedly angered respondents management which retaliated by suspending and/or constructively dismissing them by drastically reducing their work days through the adoption of a work reduction/rotation scheme. Criminal cases for estafa were likewise allegedly filed against several of the employees involved, some of which cases were eventually dismissed by the prosecutors office for lack of merit. The labor arbiter found respondent and its manager Yolanda L. Chan guilty of illegal dismissal and ordered them to pay petitioners separation pay at month for every year of service with full backwages, and 10% of the monetary award as attorneys fees. Respondent appealed to the NLRC, which affirmed the labor arbiters ruling with the modification that five of the complainants, namely Diana P. Castillo, Lorena L. Hildao, Gilbert Ongjoco, Salvador So and Ma. Pilar A. Barcenilla, were directed to report back to work, and respondent was directed to accept them without having to pay them backwages. With respect to petitioners, the NLRC held that Edgar de Leon was actually dismissed but illegally on November 7, 2001 and that with respect to the four other petitioners, they were constructively dismissed on April 15, 2002 by virtue of respondents memorandum of even date. On respondents motion for reconsideration, the NLRC modified its decision by, among other things, absolving respondents manager Yolanda L. Chan of any personal liability. Respondent appealed and maintained that its act of reducing the number of work days per week was valid, as it was done to save its business from bankruptcy due to economic reverses. The appellate court reversed the NLRC decision and dismissed petitioners complaints holding that there was no constructive dismissal because petitioners simply disappeared from work upon learning of the work reduction/rotation scheme; and that in their position paper submitted before the NLRC, petitioners only prayed for separation pay and not for reinstatement, hence, following settled jurisprudence, the latter relief has been foreclosed. Issue: Whether or not petitioners simply disappeared from their work constitute abandonment? Ruling: Case law holds that constructive dismissal occurs when there is cessation of work because continued employment is rendered impossible, unreasonable or unlikely; when there is a demotion in rank or diminution in pay or both; or when a clear discrimination, insensibility, or disdain by an employer becomes unbearable to the employee. Respondents sudden, arbitrary and unfounded adoption of the two-day work scheme which greatly reduced petitioners salaries renders it liable for constructive dismissal. Respecting the appellate courts ruling that petitioners simply disappeared from their work, hence, they are guilty of abandonment, the same does not lie. Absence must be accompanied by overt acts unerringly pointing to the fact that the employee simply does not want to work anymore. And the burden of proof to show that there was unjustified refusal to go back to work rests on the employer. Abandonment is a matter of intention and cannot lightly be inferred or legally

Page 121

LABOR RELATIONS
Atty. Jefferson M. Marquez
presumed from certain equivocal acts. For abandonment to exist, two requisites must concur: first, the employee must have failed to report for work or must have been absent without valid or justifiable reason; and second, there must have been a clear intention on the part of the employee to sever the employer-employee relationship as manifested by some overt acts. The second element is the more determinative factor. Abandonment as a just ground for dismissal thus requires clear, willful, deliberate, and unjustified refusal of the employee to resume employment. Mere absence or failure to report for work, even after notice to return, is not tantamount to abandonment. Respondent, which has the onus of proving that petitioners abandoned their work, failed to discharge the same, however. Upon the other hand, petitioners immediate filing of complaints for illegal suspension and illegal dismissal after the implementation of the questioned work scheme, which scheme was adopted soon after petitioners complaints against respondent for violation of labor standards laws were found meritorious, negates respondents claim of abandonment. An employee who takes steps to protest his dismissal cannot by logic be said to have abandoned his work.

28. Motorola Phils. v. Ambrocio, G.R. No. 173279, March 30, 2009 Facts: Sometime in 1997, Motorola Philippines, Inc. (MPI), decided to close its Paraaque plant in order to consolidate its operations at its Carmona, Cavite plant, offering to its affected employees a redundancy/separation package consisting benefits and emoluments. Out of about 900 employees who availed of the package and were consequently separated from employment, 236 employees including respondents herein, filed two separate complaints against MPI, for payment of retirement pay equivalent to one month salary per year of service, alleging that they were entitled thereto under Sec. III-B of MPIs Retirement Plan. MPI alleged that the applicable retirement plan was not Sec. III-B, but Policy 1215, which provides that In case of voluntary separation from the company due to Labor Saving devices or redundancy, retrenchment program initiated by the Company as a result of a merger or to prevent losses or other similar causes, the company shall provide a separation pay equivalent to one (1) months pay per year of service, inclusive of any service benefit eligibility under the Retirement Plan. MPI thus insisted that respondents had already received such one-month pay, the same having been included in the cash component of the separation/redundancy package, which consisted of two-months pay per year of service, paid to them. LA ruled in favor of the respondents. NLRC however has a reversed ruling holding that the benefits received by respondents for involuntary separation under MPIs retirement plan included the service pay benefits under either Sec. III-B of the Retirement Plan or Policy 1215 which both grant exactly the same benefit in case of involuntary separation one months pay for every year of service, and since none of respondents retired but were actually involuntarily separated due to redundancy, then they cannot avail of such pay. CA reversed NLRC's decision. Issue: WON respondent-employees are entitled to their claim of another separate one-month pay per year of service in pursuant to Sec. III-B of MPIs Retirement Plan. Ruling: Petition granted. Respondents have no cause of action as against petitioners with respect to their claim for additional retirement benefits. Article 283 of the Labor Code, as amended, provides: ART. 283. Closure of establishment and reduction of personnel. The employer may also terminate the employment of any employee due to the installation of labor saving devices, redundancy, retrenchment to prevent losses or the closing or cessation of operation of the establishment or undertaking unless the closing is for the purpose of circumventing the provisions of this Title, by serving a written notice on the workers and the [Department] of Labor and Employment at least one (1) month before the intended date thereof. In case of termination due to the installation of labor-saving devices or redundancy, the worker affected thereby shall be entitled to a separation pay equivalent to at least his one (1) month pay or to at least one (1) month pay for every year of service, whichever is higher. In case of retrenchment to prevent losses and in cases of closures or cessation of operations of establishment or undertaking not due to serious business losses or financial reverses, the separation pay shall be equivalent to one (1) month pay or at least one-half (1/2) month pay for every year of service, whichever is higher. A fraction of at least six (6) months shall be considered one (1) whole year." (Emphasis supplied) Separation pay has been defined as the amount that an employee receives at the time of his severance and is designed to provide the employee with the wherewithal during the period he is looking for another employment,16and is recoverable only in the instances enumerated under Articles 283 and 284 of the Labor Code, as amended, or in illegal dismissal cases when reinstatement is no longer possible.

Page 122

LABOR RELATIONS
Atty. Jefferson M. Marquez
Retirement pay, on the other hand, presupposes that the employee entitled to it has reached the compulsory retirement age or has rendered the required number of years as provided for in the collective bargaining agreement (CBA), the employment contract or company policy, or in the absence thereof, in Republic Act No. 7641 or the Retirement Law. It is admitted that respondents were terminated pursuant to a redundancy, and not due to retirement program, hence, they were entitled to a separation pay of one month salary per year of service. As correctly ruled by the NLRC, by whatever version of MPIs Retirement Plan would be made applicable, respondents are entitled to a separation pay of one month salary per year of service. Thus, when respondents were paid a separation pay of two months salary for every year of service under the Redundancy Package, they already received what was due them under the law and in accordance with MPIs plan.

29. Perez et al., vs. Phil Telegraph & Telephone Company et al., G.R. No. 152048, April 7, 2009 Facts: Felix B. Perez and Amante G. Doria were employed by Philippine Telegraph and Telephone Company (PT&T) as shipping clerk and supervisor, respectively, in PT&Ts Shipping Section, Materials Management Group. Acting on an alleged unsigned letter regarding anomalous transactions at the Shipping Section, PT&T formed a special audit team to investigate the matter. It was discovered that the Shipping Section jacked up the value of the freight costs for goods shipped and that the duplicates of the shipping documents allegedly showed traces of tampering, alteration and superimposition. Perez and Doria were placed on preventive suspension for 30 days for their alleged involvement in the anomaly. Their suspension was extended for 15 days twice. A memorandum was issued by PT&T dismissing them from service for having falsified company documents. Perez and Doria filed a complaint for illegal suspension and illegal dismissal, alleging that they were dismissed on the date they received the memorandum. They likewise contended that due process was not observed in the absence of a hearing in which they could have explained their side and refuted the evidence against them. Issue: Whether or not the dismissal of Perez and Doria was legal. Ruling: The dismissal is illegal. PT&T did not observe due process when it failed to comply with the two-notice requirement for terminating employees. ART. 277. Miscellaneous provisions. x x x (b) Subject to the constitutional right of workers to security of tenure and their right to be protected against dismissal except for a just and authorized cause and without prejudice to the requirement of notice under Article 283 of this Code, the employer shall furnish the worker whose employment is sought to be terminated a written notice containing a statement of the causes for termination and shall afford the latter ample opportunity to be heard and to defend himself with the assistance of his representative if he so desires in accordance with company rules and regulations promulgated pursuant to guidelines set by the Department of Labor and Employment. Any decision taken by the employer shall be without prejudice to the right of the worker to contest the validity or legality of his dismissal by filing a complaint with the regional branch of the National Labor Relations Commission. The burden of proving that the termination was for a valid or authorized cause shall rest on the employer. Perez and Doria were neither apprised of the charges against them nor given a chance to defend themselves. They were simply and arbitrarily separated from work and served notices of termination in total disregard of their rights to due process and security of tenure. Where the dismissal was without just or authorized cause and there was no due process, Article 279 of the Labor Code, mandates that the employee is entitled to reinstatement without loss of seniority rights and other privileges and full backwages, inclusive of allowances, and other benefits or their monetary equivalent computed from the time the compensation was not paid up to the time of actual reinstatement. However, reinstatement is no longer possible because of the length of time that has passed from the date of the incident to final resolution. *** In this case, due process was not observed because of the violation of the twin notice requirement. However, the absence of a hearing does not always result to a denial of due process. The court had the following to say: While a formal hearing or conference is ideal, it is not an absolute, mandatory or exclusive avenue of due process. An employees right to be heard in termination cases is satisfied not only by a formal face to face confrontation but by any meaningful opportunity to controvert the charges against him and to submit evidence in support thereof. "To be heard" does not mean verbal argumentation alone inasmuch as one may be heard

Page 123

LABOR RELATIONS
Atty. Jefferson M. Marquez
just as effectively through written explanations, submissions or pleadings. Therefore, while the phrase "ample opportunity to be heard" may in fact include an actual hearing, it is not limited to a formal hearing only. In other words, the existence of an actual, formal "trial-type" hearing, although preferred, is not absolutely necessary to satisfy the employees right to be heard. The employer may provide an employee with ample opportunity to be heard and defend himself with the assistance of a representative or counsel in ways other than a formal hearing. The employee can be fully afforded a chance to respond to the charges against him, adduce his evidence or rebut the evidence against him through a wide array of methods, verbal or written. The following are the guiding principles in connection with the hearing requirement in dismissal cases: (a) "ample opportunity to be heard" means any meaningful opportunity (verbal or written) given to the employee to answer the charges against him and submit evidence in support of his defense, whether in a hearing, conference or some other fair, just and reasonable way. (b) a formal hearing or conference becomes mandatory only when requested by the employee in writing or substantial evidentiary disputes exist or a company rule or practice requires it, or when similar circumstances justify it. (c) the "ample opportunity to be heard" standard in the Labor Code prevails over the "hearing or conference" requirement in the implementing rules and regulations. Petitioners likewise contended that due process was not observed in the absence of a hearing in which they could have explained their side and refuted the evidence against them. There is no need for a hearing or conference. We note a marked difference in the standards of due process to be followed as prescribed in the Labor Code and its implementing rules. The Labor Code, on one hand, provides that an employer must provide the employee ample opportunity to be heard and to defend himself with the assistance of his representative if he so desires: Nonetheless, Section 2(d), Rule I of the Implementing Rules of Book VI of the Labor Code should not be taken to mean that holding an actual hearing or conference is a condition sine qua non for compliance with the due process requirement in termination of employment. The test for the fair procedure guaranteed under Article 277(b) cannot be whether there has been a formal pretermination confrontation between the employer and the employee. The "ample opportunity to be heard" standard is neither synonymous nor similar to a formal hearing. To confine the employees right to be heard to a solitary form narrows down that right. It deprives him of other equally effective forms of adducing evidence in his defense. Certainly, such an exclusivist and absolutist interpretation is overly restrictive. The "very nature of due process negates any concept of inflexible procedures universally applicable to every imaginable situation." Significantly, Section 2(d), Rule I of the Implementing Rules of Book VI of the Labor Code itself provides that the so-called standards of due process outlined therein shall be observed "substantially," not strictly. This is a recognition that while a formal hearing or conference is ideal, it is not an absolute, mandatory or exclusive avenue of due process. An employees right to be heard in termination cases under Article 277(b) as implemented by Section 2(d), Rule I of the Implementing Rules of Book VI of the Labor Code should be interpreted in broad strokes. It is satisfied not only by a formal face to face confrontation but by any meaningful opportunity to controvert the charges against him and to submit evidence in support thereof. A hearing means that a party should be given a chance to adduce his evidence to support his side of the case and that the evidence should be taken into account in the adjudication of the controversy. "To be heard" does not mean verbal argumentation alone inasmuch as one may be heard just as effectively through written explanations, submissions or pleadings. Therefore, while the phrase "ample opportunity to be heard" may in fact include an actual hearing, it is not limited to a formal hearing only. In other words, the existence of an actual, formal "trial-type" hearing, although preferred, is not absolutely necessary to satisfy the employees right to be heard.

30. Telecommunications Distributors Specialists Inc. et al., vs. Garriel, G.R. No. 174981, May 25, 2009, citing 2009 Perez Facts: Raymund Garriel was a Customer Service Assistant (CSA) working at Telecommunications Distributors Specialists Inc.(TDSI). He was in charge of selling cellphones to customers as well as cellphone line accounts to subscribers from which he collected cash payments for. During these various transactions, three notable incidents occurred. The first was when he forged the signature of a subscriber, Ms. Ratcliffe, when she forgot to sign the coverage waiver in her application. The second incident was similar to the first, involving one Mila Huilar, where again, Garriel forged her signature on her coverage waiver. The third incident involved Garriel in selling a defective cellphone personally owned by him to a subscriber named Helcon Mabesa. He did not issue any official receipt for the transaction. These incidents were brought to the attention of the human resource department by these three customers. Garriel was then given a notice to explain why he violated company rules and procedures. Garriel denies all the accusations before him. An investigation ensued. Thereafter, Garriel was dismissed on grounds of serious misconduct and loss of trust and confidence. Garriel files an illegal dismissal case against the company. Issue: 1. Was Garriel illegally dismissed? 2. Was due process observed in dismissing Garriel? Ruling:

Page 124

LABOR RELATIONS
Atty. Jefferson M. Marquez
There was a valid dismissal. An employee entrusted with the companys goods and properties, who thereafter violates company policies with regard to the making fictitious documentation, willfully and knowingly selling defective products, and conniving with co-employees in creating a cash shortage on the report to pay for his own cellphone constitute serious misconduct as well as loss of trust and confidence as grounds for a valid dismissal. The company had complied with the twin due process requirement in terminating an employee. He was given a notice to explain coupled with the list of violations of its company policy. The essence of due process is simply the opportunity to be heard. After being heard, an investigation was conducted to determine the veracity of the facts alleged on both sides. The notice of dismissal was effected thereafter. Principles: 1. Grounds for Loss of Trust and Confidence apply only to employees occupying positions of trust and confidence or those who are routinely charged with the care and custody of the employers money or property. Requisites: a. the loss of confidence must not be simulated; b. it should not be used as a subterfuge for causes which are illegal, improper or unjustified; c. it may not be arbitrarily asserted in the face of overwhelming evidence to the contrary; d. it must be genuine, not a mere afterthought, to justify earlier action taken in bad faith, and, e. the employee involved holds a position of trust and confidence. 2. Guiding principles in connection with the hearing requirement in dismissal cases: A. ample opportunity to be heard means any meaningful opportunity, verbal or written, given to the employee to answer the charges against him and submit evidence in support of his defense, whether in a hearing, conference or some other fair, just and reasonable way. B. a formal hearing or conference becomes MANDATORY only when: 1. requested by the employee in writing or 2. substantial evidentiary disputes exist, or 3. a company rule or practice requires it, or 4. similar circumstances justify it. C. the amply opportunity to be heard standards in the Labor Code PREVAILS over the hearing or conference requirement in the IRR. 3. Comment on Social Justice: The policy of social justice is not intended to countenance wrongdoing simply because it is committed by the underprivileged. At best it may mitigate the penalty but it certainly will not condone the offense. xxx Those who invoke social justice may do so only if their hands are clean and their motives blameless and not simply because they happen to be poor. This great policy of our Constitution is not meant for the protection of those who have proved they are not worthy of it, like the workers who have tainted the cause of labor with the blemishes of their own character. (Ha Yuan Restaurant v. NLRC)

31. Triumph International Philippines v. Apostol, G.R. No. 164423, June 16, 2009 Facts: Respondent Apostol was hired as assistant manager by petitioner Triumph International (Phils.), Inc. (TIPI) in March 1991, and was terminated by TIPI on 21 January 2000. On the other hand, respondent Opulencia was hired as a warehouse helper by TIPI sometime in 1990, and was the companys warehouse supervisor at the time of the termination of his employment on 21 January 2000. Apostol was the immediate superior of Opulencia. On 14 and 15 August 1999, TIPI conducted an inventory cycle count of its direct and retail sales in its Muoz warehouse. The inventory cycle count yielded discrepancies between its result and the stock list balance Sugue (TIPIs Marketing Services Manager) sent a show-cause letter to Apostol, TIPIs Assistant Manager-Warehouse and Distribution, requiring him to explain in writing the negative variance based on the inventory cycle count. On 21 January 2000, TIPI, through Sugue, served notices to Apostol and Opulencia, stating that their employment had been terminated for committing infractions of the companys rules and regulations. Specifically, Apostol was found to have committed Offense No. 3 (Fraud or willful breach by an employee of the trust reposed in him by the Company) and Offense No. 25 (Using, uttering or saying profane, indecent, abusive, derogatory and/or indecorous words or language against the employer or supervisor), while Opulencia was found to have committed Offense No. 3 only. On 28 January 2000, Apostol and Opulencia filed with the Labor Arbiter a complaint for illegal dismissal and non-payment of salaries and other benefits against TIPI. Labor Arbiter rendered a Decision dismissing the Complaint for lack of merit. On appeal, the NLRC affirmed the Decision of the Labor Arbiter. On 20 February 2004, the Court of Appeals rendered judgment, reversing and setting aside the NLRC Decision. Issues:

Page 125

LABOR RELATIONS
Atty. Jefferson M. Marquez
WON respondents were illegally dismissed. Ruling: NO. We find the appeal meritorious. In cases of termination of employees, the well-entrenched policy is that no worker shall be dismissed except for just or authorized cause provided by law and after due process. Dismissals of employees have two facets: first, the legality of the act of dismissal, which constitutes substantive due process; and second, the legality in the manner of dismissal, which constitutes procedural due process. TIPI complied with the Procedural Due Process The grounds to which respondents were held liable are among the just causes for termination of employment under Article 282 of the Labor Code. Termination of employment based on Article 282 mandates that the employer substantially comply with the requirements of due process under the rules implementing the Labor Code (Sec. 2(d), Rule 1, Book VI of the Omnibus Rules), to wit: x x x (i) A written notice served on the employee specifying the ground or grounds for termination, and giving said employee reasonable opportunity within which to explain his side; (ii) A hearing or conference during which the employee concerned, with the assistance of counsel if he so desires is given opportunity to respond to the charge, present his evidence or rebut the evidence presented against him; and (iii) a written notice of termination served on the employee, indicating that upon, due consideration of all the circumstances, grounds have been established to justify his termination. x x x There is no question that TIPI, in dismissing Apostol and Opulencia, complied with the above requirements of procedural due process. The Court of Appeals even pointed out in its decision some of the documentary proofs of such compliance. We quote the pertinent portion of the Court of Appeals decision, viz: x x x In the present case, the evidence shows that the private respondent [TIPI] had substantially complied with the requirements of procedural due process. The private respondent sent the following to the petitioners: (a) show cause letters addressed to the petitioners [Apostol and Opulencia] requiring them to explain in writing within 48 hours upon receipt, the discrepancy on the cycle count conducted on the Muoz warehouse on August 14-15, 1999 and placing both of them on leave with pay until further notice pending investigation on the matter; (b) memorandum dated October 22, 1999 addressed to petitioner Apostol showing the findings after the investigation was conducted by the private respondent, requiring him to explain within 24 hours from receipt why he should not be terminated from his employment for loss of confidence; and (c) the notices of termination dated January 21, 2000. Substantive Due Process- R were dismissed for valid and just cause Thus, we are left with the question on whether the alleged causes for dismissal of respondents Apostol and Opulencia are supported by substantial evidence. Apostol and Opulencia were dismissed mainly on ground of fraud or willful breach of trust. As previously mentioned, fraud or willful breach of the employers trust is a just cause for termination of employment under Article 282(c) of the Labor Code. This provision is premised on the fact that the employee concerned holds a position of trust and confidence, a situation which exists where such employee is entrusted by the employer with confidence on delicate matters, such as care and protection, handling or custody of the employers property. But, in order to constitute a just cause for dismissal, the act complained of must be work-related such as would show the employee concerned to be unfit to continue working for the employer. Recent decisions of this Court have distinguished the treatment of managerial employees from that of the rank-and-file personnel, insofar as the application of the doctrine of loss of trust and confidence is concerned. Thus, with respect to rank-and-file personnel, loss of trust and confidence, as ground for valid dismissal, requires proof of involvement in the alleged events in question, and that mere uncorroborated assertions and accusations by the employer will not be sufficient. But as regards a managerial employee, the mere existence of a basis for believing that such employee has breached the trust of his employer would suffice for his dismissal. Hence, in the case of managerial employees, proof beyond reasonable doubt is not required. It is sufficient that there is some basis for the employers loss of trust and confidence, such as when the employer has reasonable ground to believe that the employee concerned is responsible for the purported misconduct, and the nature of his participation therein renders him unworthy of the trust and confidence demanded of his position. Nonetheless,

Page 126

LABOR RELATIONS
Atty. Jefferson M. Marquez
the evidence must be substantial and must establish clearly and convincingly the facts on which the loss of confidence rests and not on the employers arbitrariness, whims, and caprices or suspicion. In this case, Apostol and Opulencia were not ordinary rank and file employees; they were managerial and supervisory employees. Apostol was TIPIs assistant manager for warehouse and distribution, while Opulencia was a warehouse supervisor. They were entrusted with the management and handling of the companys warehouse goods. Thus, respondents were found by TIPI to have made unauthorized and unreported adjusting entries to the stocklist without proper investigation and reconciliation with the Accounting Department, without prior authorization by management, and without preparation of formal reports indicating the parties responsible for the adjustments and those who approved the same. This, according to TIPI, is a clear violation of the companys internal control procedures, which resulted to the loss of the companys trust and confidence in the respondents. Respondents do not deny making adjustment entries to the stocklist. In fact, both admitted making such adjustments in the office memoranda and affidavits submitted as evidence in this case. The question, therefore, is whether respondents Apostol and Opulencia, in making such adjustments, violated TIPI internal control procedures. After a careful evaluation of the evidence on record, we are convinced that the respondents made unauthorized adjustments in TIPIs stocklist, in violation of the companys internal control procedures. This act warrants respondents dismissal for willful breach of employers trust. Considering the foregoing, we find that respondents Apostol and Opulencia were dismissed by TIPI for a valid and just cause. The relationship of employer and employee, especially where the employee has access to the employers property, necessarily involves trust and confidence. Where the rules laid down by the employer to protect its property are violated by the very employee who is entrusted and expected to follow and implement the rules, the employee may be validly dismissed from service. Finding the dismissal of respondents Apostol and Opulencia, based on willful breach of employers trust, valid, we deem it unnecessary to further rule on TIPIs other ground for Apostols dismissal, i.e., uttering indecent, abusive and derogatory words against his supervisor. Note, however, that such act of an employee, if substantially proven, may be considered as serious misconduct which would warrant the termination of his employment.

32. Technological Institute of the Phils Teachers and Employees Organization vs. Court of Appeals, et al., G.R. No. 158703, June 26, 2009 Facts: Petitioner Magdalena T. Salon (Salon) was a College Instructor 3 of the Humanities and Social Science Department (HSSD) of respondent Technological Institute of the Philippines (TIP) and a member of the Technological Institute of the Philippines Teachers and Employees Organization (TIPTEO). Sometime in year 2000, the TIP received complaints from students claiming that Salon was collecting P1.50 per page for the test paper used in the subject she was teaching at the time. She reportedly asked her students not to write on the test papers; these test papers were not returned to the students after the test. In addition, a complaint was filed against her for anomalously changing the grade of a particular student who was at the same time a son of a co-faculty member from 5.0 (failed) to 6.0 (dropped). Salon answered that she only collected P0.50 per page to reimburse herself from personal expenses. Further, she admitted of having changed the grade in order to vindicate the student from the wrath of his father. Salon believed that a dropped grade is better than a failed mark. There were enumerated TIP Memoranda in the case, geared towards avoidance of unduly burdening the students from unreasonable finances. But what is really determinative is TIP Memorandum No. P-66, which requires a prior permission from the proper school authority should the teacher directly sell his/her examination papers to students, usually for reimbursement, provided, the cost shall be within the rate prescribed by the school. The Voluntary Arbitrator ruled in favor of Salon. On appeal under Rule 43, said decision was affirmed by CA. However, on motion for reconsideration, CA reversed itself, ruling in favor of TIP and against Salon, but granted separation pay. Issue: 1. Whether or not Salon is guilty of violating TIPs Memorandum No. P-66, for unauthorized selling of examination papers 2. Whether or not Salon is guilty of serious misconduct for falsifying Manalos (the son a co-faculty member) grade and violating the grading rules under the Manual of Regulations for Private Schools Ruling:

Page 127

LABOR RELATIONS
Atty. Jefferson M. Marquez
Salon is guilty of the two valid causes. On the sale of Papers: The cost of the sale of test papers by Salon to her students was within the prescribed parameters of the school. However, it cannot be denied that Salon did not first obtain the prior permission of the proper school authority, a condition precedent required by TIP Memorandum no. P-66. Clearly, she transgressed the school regulation. On grade tampering: It is a violation against the Manual of Regulation for Private Schools whose Section 79 provides: Sec. 79. Basis for Grading. The final grade or rating given to a pupil or student in a subject should be based on his scholastic record. Any addition or diminution to the grade x x x shall not be allowed. The present violation involves elements of falsification and dishonesty. Knowing fully what Manalo deserved, Salon gave him a grade of 6.0 instead of a failing grade. In the process, she changed in short, falsified her own records by changing the submitted record and the supporting documents. Viewed in any light, this is Serious Misconduct under Article 282(a) of the Labor Code, and a just cause for termination of employment. On separation pay: CA erred on this matter. The violation constituted serious misconduct or a cause relating to the employees moral character. The fact that Salon committed an offense for the first time, or she has served for 10 years in TIP, and the allegation that there was no malice or bad intention on her part are misplaced arguments. Thus, separation pay is not proper. WHEREFORE, premises considered, we hereby DENY the petition for lack of merit. We hereby AFFIRM the amended decision of the Court of Appeals promulgated on May 22, 2003, but DELETE the award of separation pay. Costs against the petitioners.

33. Llamas v. Ocean Gateway Maritime and Management Services Inc., G.R. No. 179293, August 14, 2009 Facts: Ocean Gateway Maritime and Management, Inc (OGMM). hired Eden Llamas as an accounting manager. OGMM's Chief Executive Officer Macaraig called petitioners attention because of her failure, despite repeated demands, to accomplish the long overdue monthly and annual company financial reports and to remit the companys contributions to the SSS and PhilHealth. But Llamas failed to comply with the instruction as money for the purpose was not credited to the companys account at the bank. Macaraig sent a memorandum to petitioner charging her with gross and habitual neglect of duty and/or misconduct or willful disobedience and insubordination, detailing therein the bases of the charges, and requiring her to submit a written explanation why she should not be penalized or dismissed from employment but OMGG found the explanation unsatisfactory Issues: WON the dismissal was for a just cause WON due process was observed Ruling: THE DISMISSAL WAS FOR A JUST CAUSE. Under Article 282 (b) of the Labor Code, negligence must be both gross and habitual to justify the dismissal of an employee. Gross negligence is characterized by want of even slight care, acting or omitting to act in a situation where there is a duty to act, not inadvertently but willfully and intentionally with a conscious indifference to consequences insofar as other persons may be affected. In the present case, Llamas, as respondents Accounting Manager, failed to discharge her important duty of remitting SSS/PhilHealth contributions not once but quadruple times, resulting in respondents incurring of penalties totaling P18,580.41, not to mention the employees/members contributions being unupdated. Her claim of being overworked and undermanned does not persuade. As noted by respondent, the company had been in operation for less than three (3) months at the time the negligence and delays were committed, with only a few transactions and only with one principal, Malaysian Merchant Marine Bhd., hence, its financial and accounting books should not have been difficult to prepare. Moreover, as claimed by respondent

Page 128

LABOR RELATIONS
Atty. Jefferson M. Marquez
which was not refuted by petitioner, she failed to remit the contributions as early as November 2001 during which time, however, on-the-job trainees were still with the company, hence, her claim of being undermanned behind such failure does not lie. As to the delay in the remittance of SSS/PhilHealth contributions for January 2002, which petitioner claims to be due to the fact that the money intended for payment was not yet credited as of February 20, 2002 to respondents bank account, as well as to her absence the following day or on February 21, 2002 due to hypertension, the Court is not persuaded, given that at that time, she had already been in delay in the performance of her duties. On petitioners declaration that I believe that I did something good for our office when our declaration of gross income submitted to City Hall for the renewal of our municipal license was lower than our actual gross income for which the office had paid a lower amount, the Court finds the same as betraying a streak of dishonesty in her. It partakes of serious misconduct. x x x Misconduct has been defined as improper or wrong conduct. It is the transgression of some established and definite rule of action, a forbidden act, a dereliction of duty, willful in character, and implies wrongful intent and not mere error of judgment. The misconduct to be serious must be of such grave and aggravated character and not merely trivial and unimportant. Such misconduct, however serious, must nevertheless be in connection with the employees work to constitute just cause for his separation. Thus, for misconduct or improper behavior to be a just cause for dismissal, (a) it must be serious; (b) must relate to the performance of the employees duties; and (c) must show that the employee has become unfit to continue working for the employer. Indeed, an employer may not be compelled to continue to employ such person whose continuance in the service would be patently inimical to his employers interest.[12] (Emphasis supplied) For her act of understating the companys profits or financial position was willful and not a mere error of judgment, committed as it was in order to save costs, which to her warped mind, was supposed to benefit respondent. It was not merely a violation of company policy, but of the law itself, and put respondent at risk of being made legally liable. Verily, it warrants her dismissal from employment as respondents Accounting Manager, for as correctly ruled by the appellate court, an employer cannot be compelled to retain in its employ someone whose services is inimical to its interests. 2. As to the second issue of whether due process was accorded petitioner, the Court rules in the affirmative. Far from being arbitrary, the termination of her services was effected after she was afforded the opportunity to, as she did, submit her explanation on why she should not be disciplined or dismissed, which explanation, it bears reiteration, was, however, found unsatisfactory.

34. Lowe Inc., v. CA, G.R. 164813 & 174590, August 14, 2009 Facts: Lowe is an advertising agency, and Maria Elizabeth "Mariles" L. Gustilo (Gustilo) 9 is the Chief Executive Officer and President of Lowe, while Raul M. Castro (Castro) is the Executive Creative Director . Both were included in the complaint for illegal dismissal in their capacity as officers of Lowe. At the height of the influx of advertising projects, Lowe hired Mutuc as a Creative Director to help out the four other Creative Directors of Lowe. Mutuc was given a salary of P100,000 a month and Mutuc became a regular employee. A year after, most of Lowes clients reduced their advertising budget. In response to the situation, Lowe implemented cost-cutting measures including a redundancy program and Lowe terminated Mutucs services because her position was declared redundant. Mutuc filed a complaint for illegal dismissal, nonpayment of 13th month pay with prayer for the award of moral and exemplary damages plus attorneys fees against Lowe. Labor Arbiter dismissed Mutucs complaint NLRC reversed the decision. Both Lowe and Mutuc filed petitions for certiorari before the Court of Appeals. The Court of Appeals modified the NLRCs decision rendered in favor of Mutuc, hence this petition. Issues: 1. WON Mutuc was validly dismissed by reason of redundancy 2. WON Mutuc is entitled only to separation pay and proportionate 13th month pay Ruling:

Page 129

LABOR RELATIONS
Atty. Jefferson M. Marquez
Yes. Redundancy exists when the service of an employee is in excess of what is reasonably demanded by the actual requirements of the business. For a valid implementation of a redundancy program, the employer must comply with the following requisites: (1) written notice served on both the employee and the DOLE at least one month prior to the intended date of termination; (2) payment of separation pay equivalent to at least one month pay or at least one month pay for every year of service, whichever is higher; (3) good faith in abolishing the redundant position; and (4) fair and reasonable criteria in ascertaining what positions are to be declared redundant. (See Art. 283) The controversy lies on whether Lowe used any fair and reasonable criteria in declaring Mutucs position redundant and whether there was bad faith in the abolition of her position. The determination of the continuing necessity of a particular officer or position in a business corporation is a management prerogative, and the courts will not interfere unless arbitrary or malicious action on the part of management is shown. Indeed, an employer has no legal obligation to keep more employees than are necessary for the operation of its business. The Court recognizes that a host of relevant factors comes into play in determining who among the employees should be retained or separated. Among the accepted criteria in implementing a redundancy program are: (1) preferred status; (2) efficiency; and (3) seniority. Lowe employed fair and reasonable criteria in declaring Mutucs position redundant. Mutuc, who was hired only on 23 June 2000, did not deny that she was the most junior of all the executices of Lowe. Mutuc also did not present contrary evidence to disprove that she was the least efficient and least competent among all the Creative Directors. Yes

35. Estacio v. Pampanga I Electric Cooperative, G.R. No. 183196, August 19, 2009 Facts: This is a Petition for Review on Certiorari. Respondent PELCO I is an electric cooperative duly organized, incorporated, and registered pursuant to Presidential Decree No. 269. Respondent Engr. Allas is the General Manager Petitioner Estacio had been employed at respondent PELCO I as a bill custodian since 1977, while petitioner Manliclic had been working for respondent PELCO I as a bill collector since June 1992. On 22 August 2002, Nelia D. Lorenzo (Lorenzo), the Internal Auditor of respondent PELCO I, submitted her Audit Findings: Evaluation of the results of physical inventory of bills through reconciliation of records such as aging schedule of consumer accounts receivable balance, collection reports and other related documents revealed 87 bills amounting to One Hundred Twenty Six Thousand Seven Hundred Fifty and 93/100 (P126,750.93) remained unremitted as of August 20, 2002. Accounting of which includes the accountability of Ms. Estacio amounting to One Hundred Twenty Three Thousand Eight Hundred Seven and 14/100 (P123,807.14) representing 86 bills.2[5] Respondent Engr. Allas issued a Memorandum to petitioner Estacio informing her of the audit findings, and directing her to explain in writing, within 72 hours upon receipt thereof, why no disciplinary action should be imposed upon her for Gross Negligence of Duty. In her written explanation, petitioner Estacio averred that she had no control over and should not be held answerable for the failure of the bill collectors at the San Luis Area Office to remit their daily collections. Unsatisfied with petitioner Estacios explanation, respondent Engr. Allas issued a Memorandum charging Estacio with gross negligence of duty. A formal investigation/hearing then ensued, during which petitioner Estacio was duly represented by counsel.

Page 130

LABOR RELATIONS
Atty. Jefferson M. Marquez
Respondent Engr. Allas rendered a Decision which adopted the recommendation of the investigation committee dismissing petitioner Estacio from service, with forfeiture of her benefits, with the modification deleting the charge of dishonesty. Petitioner Estacio sought a reconsideration of the said decision but it was denied by respondent Engr. Allas. Labor Arbiter ruled in favor of respondents. NLRC disagreed with the Labor Arbiter. Issue: WON Estacio and Manliclic were legally dismissed? Ruling: Yes. There was a legal dismissal of petitioner Estacio. Court rules that there is valid cause for petitioner Manliclic dismissal from service. It is undisputed that petitioners were accorded due process. Through the Memoranda issued by respondent Engr. Allas, petitioners were duly informed of the results of the audit conducted by Internal Auditor Lazaro, which were unfavorable to petitioners. The Court also finds that there was valid cause for petitioner Estacios dismissal. Gross negligence connotes want or absence of or failure to exercise even slight care or diligence, or the total absence of care. It evinces a thoughtless disregard of consequences without exerting any effort to avoid them. To warrant removal from service, the negligence should not merely be gross, but also habitual. Petitioner Estacio, despite the opportunities given to her, did not offer any satisfactory explanation or evidence in her defense. Her only reason for failing to comply with the requisite daily accounting and reporting of the bill collections was the terrible weather condition. Petitioner Estacios failure to make a complete accounting and reporting of the bill collections plainly demonstrated her disregard for one of her fundamental duties as a bill custodian. Petitioner Manliclics honesty and integrity are the primary considerations for his position as a bill collector because, as such, he has in his absolute control and possession -- prior to remittance -- a highly essential property of the cooperative. The amount misappropriated by petitioner Manliclic is irrelevant. More than the resulting material damage or prejudice, it is petitioner Manliclics very act of misappropriation that is offensive to respondent PELCO petitioner Manliclic committed a breach of the trust reposed in him by his employer, respondent PELCO I. This constitutes valid cause for his dismissal from service Court of Appeals is AFFIRMED

36. Maralit v. PNB, G.R. No. 163788, August 24, 2009 Facts: Ester B. Maralit (Maralit) worked for respondent Philippine National Bank (PNB) from 27 August 1968 to 31 December 1998(30 years). She began as a casual clerk and climbed her way to become branch manager. PNB, offered its personnel an early retirement plan known as , Special Separation Incentive Plan (SSIP), the guidelines for the availment of the SSIP, that personnel with pending administrative cases or who are under preliminary investigation may avail of the SSIP. However, payment of their benefits shall be made only after the resolution of their cases and only if they are not disqualified from receiving such benefits. PNB charged Maralit with serious misconduct, gross violation of bank rules and regulations, and conduct prejudicial to the best interest of the bank. PNB stated: By giving undue and unwarranted preference, advantage or benefit to a private party through manifest partiality and evident bad faith committed by you while performing the duties as Manager as follows: (1) in a memorandum dated 8 July 1998, a certain Gay Ophelia T. Alano reported Maralits irregular transactions; (2) in a memorandum dated 9 July 1998, Vice President Florencio C. Lat of Branch Operations and Consumer Banking Division for Southern Luzon and Bicol referred Maralits irregular transactions to the IAG for immediate investigation; (3) Maralit submitted a memorandum dated 9 July 1998 admitting the irregular transactions; (4) on 9 July 1998, Vice President Milagros Pastrana of Branch Administrative Office for Southern Luzon and Bicol recommended that Maralit be temporarily assigned to the Naga Branch; (5) on 15 July 1998, Maralit assumed her new assignment at the Naga Branch; (6) on 28 July 1998, PNB reported to the BSP the P54,950,000 drawings against uncollected deposits; (7) in a memorandum dated 8 September 1998, the IAG found that Maralit violated bank policies which resulted in the return of unfunded checks amounting to P54,950,000 and recommended that Maralit be required to submit her written answer under oath; (8) on 15 September 1998, Maralit filed her application for early retirement; (9) in its 29 September 1998 memorandum, PNB stated, Attached is

Page 131

LABOR RELATIONS
Atty. Jefferson M. Marquez
the Internal Audit report dated September 8, 1998; and (10) in its 16 October 1998 memorandum, PNB stated, In connection with the Special Audit report of Internal Audit Group dated September 8, 1998. Internal Audit Group (IAG) asks Maralit for a written answer under oath to the above charges together with whatever affidavits and other documentary evidence you may wish to submit within five (5) days from receipt of this memorandum why she should not be penalized for Serious Misconduct, Willful Breach of Trust and Gross Violation of Bank Rules and Regulations under Article 282 of the Labor Code. In its 16 October 1998 memorandum, PNB placed Maralit under preventive suspension. PNB stated: Maralit placed under preventive suspension for thirty (30) days effective upon receipt hereof pursuant to Section 3, Rule XIV of the Omnibus Rules Implementing the Labor Code. On 20 November 1998, PNB conditionally approved Maralits application for early retirement effective at the close of business hours on 31 December 1998. PNB stated that, Payment of Special Separation Incentive and other Benefits shall be made only upon final resolution of the administrative case against you, provided that the decision in said investigation does not disqualify you from the enjoyment of said benefits. Under the SSIP, Maralit was entitled to P1,359,086.02 retirement benefits. Maralit submitted her answer dated 11 January 1999. She stated that The favorable accommodations granted, claiming that were made in good faith and intended for the higher interests of the Bank. She admitted that the accommodations were deviations from Banks policies, uncollected deposits were a wanton violation of the policy of the Bangko Sentral ng Pilipinas (BSP) and PNB. She had no discretion to do prohibited acts, and PNBs interest was unreasonably put at risk. In 2000 PNB found Maralit, finding her guilty of serious misconduct, gross violation of bank rules and regulations, and conduct prejudicial to the best interest of the bank. PNB dismissed Maralit from the service with forfeiture of her retirement benefits effective at the close of business hours on 31 December 1998. Maralit filed with the arbitration branch of the NLRC a complaint for non-payment of retirement benefits and separation pay, and for damages against PNB. The Labor Arbiter held that Maralit was entitled to P1,359,086.02 retirement benefits,P200,000 exemplary damages, and P155,908.60 attorneys fees. The Labor Arbiter found that (1) Maralit was not under preliminary investigation when she filed her application for early retirement; (2) had Maralit known that she would be administratively charged, she would not have availed of the SSIP so that she could continue receiving her monthly salary; (3) when PNB approved Maralits application for early retirement, the Administrative Adjudication Panel had not decided the administrative case against her; (4) there was no hearing or conference held where Maralit could respond to the charge, present her evidence, or rebut the evidence presented against her; and (5) PNB illegally dismissed Maralit and committed an act oppressive to labor. PNB appealed to the NLRC, claiming that the Labor Arbiter gravely abused his discretion and erred in his factual findings. NLRC affirmed with modification the Labor Arbiters 22 January 2001 Decision. The NLRC deleted the award of P200,000 exemplary damages. The NLRC held that (1) there was no grave abuse of discretion on the part of the Labor Arbiter, (2) the material facts as found by the Labor Arbiter were consistent with the evidence, and (3) the award of exemplary damages lacked basis. PNB claimed that the NLRC committed grave abuse of discretion when it affirmed the Labor Arbiters 22 January 2001 Decision because (1) Maralit was not entitled to retirement benefits, (2) Maralit was afforded due process, and (3) Maralit was not entitled to attorneys fees. The Court of Appeals held that the NLRC committed grave abuse of discretion when it affirmed the Labor Arbiters 22 January 2001 Decision. The Court of Appeals found that Maralit was under preliminary investigation when she filed her application for early retirement and that she was afforded due process. Issues: Whether or not Ester Maralit has retired or was dismissed from the service effective December 31, 1998? Whether or not she has committed Serious Misconduct, Willful Breach of Trust and Gross Violation of Bank Rules and Regulations under Article 282 of the Labor Code that will forfeit her retirement benefits? Ruling: The evidence shows that Maralit was afforded due process. The essence of due process is an opportunity to be heard or, as applied to administrative proceedings, an opportunity to explain ones side. A formal or trial-type hearing is not essential. PNB gave Maralit ample opportunity to explain her side. In its 29 September 1998 memorandum, PNB directed Maralit to submit her written answer under oath together

Page 132

LABOR RELATIONS
Atty. Jefferson M. Marquez
with affidavits and other documentary evidence: The administrative investigation in PNB undergoes a three-tiered process which commences with an audit report made by the Internal Audit Division (IAD). Three investigative bodies are separate and independent from each other, and they proceed without influence from the other bodies having their own respective mandates and processes. Maralit invoking the principle on estoppel, that an employer who accepts or approves the retirement of an employee loses the right to dismiss such employee in a subsequent action. Retirement and Dismissal are entirely different and incompatible from each other. Each is a distinct and separate mode of extinguishing an employer-employee relationship, and has its own legal effects in our jurisdiction. Consequently, they cannot be taken together for the purpose of terminating employment relation. The Court is unimpressed. The evidence shows that Maralit was under preliminary investigation when she filed her application for early retirement. PNB consistently stated that payment of Maralits retirement benefits shall be paid only after final resolution of the administrative case against her, provided that she is not disqualified to receive such benefits. PNB may rightfully terminate Maralits services for a just cause, including serious misconduct. Serious misconduct is improper conduct, a transgression of some established and definite rule of action, a forbidden act, or a dereliction of duty. Having been dismissed for a just cause, Maralit is not entitled to her retirement benefits. There is substantial evidence showing that there was valid cause for the bank to dismiss petitioners employment for loss of trust and confidence. Petitioner was a bank accountant, which is a position of trust and confidence. The amount involved is significant, almost P4.5 million.

37. Quevedo v. Benguet Electric Cooperative, G.R. No. 168927, September 11, 2009 Facts: Petitioners are former employees of respondent Benguet Electric Cooperative, Incorporated (BENECO). BENECO started automating its operations, rendering redundant the functions performed by some employees, including petitioners. Instead however of terminating outright petitioners and paying them statutory benefits under the Labor Code, BENECO offered them an option to retire under a newly created optional retirement program which would provide them with more benefits than what is statutorily required. Petitioners accepted respondents offer and thereby received benefits. Nearly four months after their severance from their employment however, petitioners filed a complaint for illegal termination of employment against BENECO. They claimed that notwithstanding the fact that they had no intention of retiring, they were forced to do so because BENECO would have them terminated had they insisted otherwise. They also questioned the validity of BENECOs downsizing measure. The Labor Abiter dismissed their claims for lack of merit. However, upon reaching the NLRC, said court found for them and ordered BENECO to reinstate them. The Court of Appeals, in a petition for certiorari made by herein respondent, reversed NLRC decision which prompted petitioner to file the instant case. Issue: Whether or not petitioners were illegally dismissed from their employment by the respondent Ruling: There was no illegal dismissal. It is inappropriate to discuss whether the requirements for terminating an employee had been complied with since the petition is focused on whether the retirement was made voluntarily. While termination of employment and retirement from service are common modes of ending employment, they are mutually exclusive, with varying juridical bases and resulting benefits. Retirement from service is contractual (i.e. based on the bilateral agreement of the employer and employee), while termination of employment is statutory (i.e. governed by the Labor Code and other related laws as to its grounds, benefits and procedure). The benefits resulting from termination vary, depending on the cause. For retirement, Article 287 of the Labor Code gives leeway to the parties to stipulate above a floor of benefits. As to whether the retirement in this case was done voluntarily, the line between voluntary and involuntary retirement is thin but it is one which this Court has drawn. Voluntary retirement cuts employment ties leaving no residual employer liability; involuntary retirement amounts to a discharge,

Page 133

LABOR RELATIONS
Atty. Jefferson M. Marquez
rendering the employer liable for termination without cause. The employees intent is the focal point of analysis. In determining such intent, the fairness of the process governing the retirement decision, the payment of stipulated benefits, and the absence of badges of intimidation or coercion are relevant parameters. The records presented show sufficient compliance with the criteria. Furthermore, the manner by which BENECO arrived at its decision to downsize and at the same time spare petitioners the lesser benefits under Article 283 of the Labor Code by creating a more generous retirement package was regular, transparent and fully documented. Petitioners were afforded opportunity to seek reconsideration of BENECOs decision to downsize, albeit without success as BENECO stood pat on its management decision. Nor were petitioners here denied the stipulated benefits. The records show that on average, the benefits each of the petitioners received were more than twice their statutory counterpart under Article 183. The marked difference between these two bundles of benefits not only factored in petitioners decision to retire under the EVR program but also explained the lapse of nearly four months before petitioners sued BENECO. Finally, petitioners accepted BENECOs offer without reservation and received payments without protest. True, petitioners requested BENECO to reconsider its decision to abolish their positions but this is a natural inclination to keep ones livelihood. It does not rise to that level of intimidation or coercion sufficient to vitiate consent. Petitioners nevertheless argue that their inevitable termination forced their hands, leaving them no choice but to retire from service. Although superficially appealing, this argument rests on an unfair, but predictably biased, assumption. petitioners were not compelled to retire, they simply chose, between two equally valid options, the exit route offering bigger benefits. As already decided by this Court in Benguet Electric Cooperative v. Fianza : [T]he abolition of a position deemed no longer necessary is a management prerogative, and this Court, absent any findings of malice and arbitrariness on the part of management, will not efface such privilege if only to protect the person holding that office.

38. Placido et al. v. NLRC, G.R. No. 180888, September 18, 2009 Facts: Petitioners Placido and Caragay had been employed as cable splicers by PLDT. PLDT had been receiving reports of theft and destruction of its cables. PLDT inspector and security guard, responding to a report that cables were being stripped and burned in one of the residences, proceeded to the said area where they saw petitioners service vehicle parked infront of the house. Petitioners were seen stripping and burning cables inside the compound of the house which turned out to belong to Caragays mother. With the assistance of police and barangay officials, PLDT recovered the cables bearing the "PLDT" marking. PLDT filed an Information for Qualified Theft against petitioners. PLDT also required petitioners to explain within 72 hours why no severe disciplinary action should be taken against them for Serious Misconduct and Dishonesty. Petitioners submitted a joint explanation denying the charges against them. By their claim, they were on their way back from the house of one Quezada from whom they were inquiring about a vehicle when they were detained by the PLDT investigator. On petitioners request, a formal hearing was scheduled. Their request for a copy of the Security Investigation was denied, however, on the ground that they are only entitled to "be informed of the charges, and they cannot demand for the report as it is still on the confidential stage. Petitioners counsel later reiterated the request for a setting of a hearing and an audiotape of the June 25, 2001 hearing, but the same was denied. A third time request for another hearing was likewise denied. PLDT sent notices of termination to petitioners, prompting them to file a complaint for illegal dismissal before the Labor Arbiter. Labor Arbiter held that petitioners were illegally dismissed which was reversed by the NLRC. Pettitioners appealed to CA which affirmed the NLRC Decision holding that since the cables bore the "PLDT" marking, they were presumed to be owned by PLDT, hence, the burden of evidence shifted on petitioners to prove that they were no longer owned by PLDT, but they failed. Issue: WON petitioners were denied due process and were illegally dismissed Ruling: No, petitioners were not denied due process. They were legally dismissed. Article 277 of the Labor Code provides: (b) Subject to the constitutional right of workers to security of tenure and their right to be protected against dismissal except for a just or authorized cause and without prejudice to the requirement of notice under Article 283 of this Code, the employer shall furnish the workers whose employment is sought to be terminated a written notice containing a statement of the causes for termination and shall afford the latter ample

Page 134

LABOR RELATIONS
Atty. Jefferson M. Marquez
opportunity to be heard and defend himself with the assistance of his representative if he so desires in accordance with company rules and regulations promulgated pursuant to the guidelines set by the Department of Labor and Employment. Any decision taken by the employer shall be without prejudice to the right of the worker to contest the validity or legality of his dismissal by filing a complaint with the regional branch of the National Labor Relations Commission. The burden of proving that the termination was for a valid or authorized cause shall rest on the employer. And the Omnibus Rules Implementing the Labor Code require a hearing and conference during which the employee concerned is given the opportunity to respond to the charge, and present his evidence or rebut the evidence presented against him. Thus Rule I, Section 2(d), provides: Section 2. Security of Tenure. (d) In all cases of termination of employment, the following standards of due process shall be substantially observed: For termination of employment based on just causes as defined in Article 282 of the Labor Code: (i) A written notice served on the employee specifying the ground or grounds for termination, and giving said employee reasonable opportunity within which to explain his side. (ii) A hearing or conference during which the employee concerned, with the assistance of counsel if he so desires, is given opportunity to respond to the charge, present his evidence or rebut the evidence presented against him.1avvphi1 (iii) A written notice of termination served on the employee, indicating that upon due consideration of all the circumstances, grounds have been established to justify his termination. The abovequoted provision of Section 2(d) should not be taken to mean, however, that holding an actual hearing or conference is a condition sine qua non for compliance with the due process requirement in case of termination of employment. For the test for the fair procedure guaranteed under the above-quoted Article 277(b) of the Labor Code is not whether there has been a formal pretermination confrontation between the employer and the employee. The "ample opportunity to be heard" standard is neither synonymous nor similar to a formal hearing. To confine the employees right to be heard to a solitary form narrows down that right. The essence of due process is simply an opportunity to be heard or, as applied to administrative proceedings, an opportunity to explain one's side or an opportunity to seek a reconsideration of the action or ruling complained of. What the law prohibits is absolute absence of the opportunity to be heard, hence, a party cannot feign denial of due process where he had been afforded the opportunity to present his side. A formal or trial type hearing is not at all times and in all instances essential to due process, the requirements of which are satisfied where the parties are afforded fair and reasonable opportunity to explain their side of the controversy. In the present case, petitioners were, among other things, given several written invitations to submit themselves to PLDTs Investigation Unit to explain their side, but they failed to heed them. A hearing was conducted where petitioners attended along with their union MKP representatives during which the principal witnesses to the incident were presented. Petitioners were thus afforded the opportunity to confront those witnesses and present evidence in their behalf, but they failed to do so. SC found that as the cables bore the "PLDT" marking, the presumption is that PLDT owned them. The burden of evidence thus lay on petitioners to prove that they acquired the cables lawfully but this they failed to discharge.

39. Martinez v. B&B Fish Broker, G.R. No. 179985, September 18, 2009 Facts: Odilon L. Martinez (petitioner) was employed as a cashier by B&B Fish Broker, a partnership owned and managed by respondent Norberto M. Lucinario (Lucinario) and Jose Suico. Lucinario called petitioners attention to his alleged shortages in his cash collections and ordered him to, as he did, take a leave the following day. When petitioner reported back for work, he was relieved of his position and reassigned as company custodian. As cashier, petitioners duties consisted of issuing receipts on items taken and bought and balancing of the cash on hand and receipts issued at the close of the business day. After a few days, petitioner filed an application for a four-day leave effective on even date due to an inflamed jaw. His application, addressed to Lucinario, was received by a co-employee, Arielle Penaranda. Subsequently, petitioner discovered that his name had been removed from the company logbook and was prevented from logging in. And he was informed that his application for a four-day leave of absence had been denied. The following day petitioner, having understood that the removal of his name from the logbook amounted to the termination of his employment, tried to confer with Lucinario but to no avail, hence, filed a complaint against B&B Fish Broker and/or Lucinario, for illegal dismissal, underpayment and non-payment of wages with prayer for

Page 135

LABOR RELATIONS
Atty. Jefferson M. Marquez
reinstatement, before the Arbitration Branch of the National Labor Relations Commission. Denying petitioners charge that his services were illegally terminated, Lucinario claimed, in effect, that petitioner abandoned his job. Issue: Whether or not Petitioner was illegally dismissed? Ruling: The petition is impressed with merit. While Lucinario contends that petitioner abandoned his job, the bulk of his (Lucinarios) evidence relates to petitioners incurring of shortages in his collections to justify the transfer of petitioners assignment from cashier to company custodian and his alleged previous suspension. Parenthetically, documentary evidence relating thereto, which could lend light on petitioners performance, was not presented. On to Lucinarios claim that petitioner abandoned his employment: It is axiomatic that in a petition for review on certiorari, only questions of law may be raised. The rule admits of certain exceptions, however, one of which is when there is variance on the appreciation of facts of the case. In the present case, the Labor Arbiter ruled that there is no illegal dismissal, yet she ordered petitioners reinstatement. The NLRC found otherwise that petitioner was illegally dismissed. On appeal, the appellate court reversed the findings of the NLRC. This constrains the Court to reassess the evidence of the parties. Abandonment is a form of neglect of duty, one of the just causes for an employer to terminate an employee. It is a hornbook precept that in illegal dismissal cases, the employer bears the burden of proof. For a valid termination of employment on the ground of abandonment, Lucinario must prove, by substantial evidence, the concurrence of petitioners failure to report for work for no valid reason and his categorical intention to discontinue employment. Lucinario, however, failed to establish any overt act on the part of petitioner to show his intention to abandon employment. As reflected above, petitioner, after being informed of his alleged shortages in collections and despite his relegation to that of company custodian, still reported for work. He later applied for a 4-day leave of absence. On his return, he discovered that his name was erased from the logbook, was refused entry into the company premises, and learned that his application for a 4-day leave was not approved. He thereupon exerted efforts to communicate with Lucinario on the status of his employment, but to no avail. To the Court, these circumstances do not indicate abandonment. Finally, that petitioner immediately filed the illegal dismissal complaint with prayer for reinstatement should dissipate any doubts that he wanted to return to work. What thus surfaces is that petitioner was constructively dismissed. No actual dismissal might have occurred in the sense that petitioner was not served with a notice of termination, but there was constructive dismissal, petitioner having been placed in a position where continued employment was rendered impossible and unreasonable by the circumstances indicated above.

40. Flight Attendants and Steward Association of the Phils vs. Phil Airlines, G.R. No. 178083, October 2, 2009, see July 22, 2008, main decision

Facts: Petitioner FASAP is the duly certified collective bargaining representative of PAL flight attendants and stewards, or collectively known as PAL cabin crew personnel. Respondent PAL is a domestic corporation organized and existing under the laws of the Republic of the Philippines, operating as a common carrier transporting passengers and cargo through aircraft. On June 1998, PAL retrenched 5,000 of its employees, including more than 1,400 of its cabin crew personnel. PAL adopted the retrenchment scheme allegedly to cut costs and mitigate huge financial losses as a result of a downturn in the airline industry brought about by the Asian financial crisis. Prior to the full implementation of the assailed retrenchment program, FASAP and PAL conducted a series of consultations and meetings and explored all possibilities of cushioning the impact of the impending reduction in cabin crew personnel. However, the parties failed to agree on how the scheme would be implemented. Thus PAL unilaterally resolved to utilize the criteria set forth in the Collective Bargaining Agreement in retrenching cabin crew personnel: that is, that retrenchment shall be based on the individual employees efficiency rating and seniority. On June 1998, FASAP filed a Complaint against PAL for unfair labor practice, illegal retrenchment with claims for reinstatement and payment of salaries, allowances and backwages of affected FASAP

Page 136

LABOR RELATIONS
Atty. Jefferson M. Marquez
members, actual, moral and exemplary damages with a prayer to enjoin the retrenchment program then being implemented. Issue: Whether or not PALs retrenchment scheme was justified Ruling: While it is true that the exercise of this right is a prerogative of management, there must be faithful compliance with substantive and procedural requirements of the law and jurisprudence. The burden falls upon the employer to prove economic or business losses with sufficient supporting evidence. Any claim of actual or potential business losses must satisfy certain established standards, all of which must concur, before any reduction of personnel becomes legal. These are: (1) That retrenchment is reasonably necessary and likely to prevent business losses which, if already incurred, are not merely de minimis, but substantial, serious, actual and real, or if only expected, are reasonably imminent as perceived objectively and in good faith by the employer; (2) That the employer served written notice both to the employees and to the Department of Labor and Employment at least one month prior to the intended date of retrenchment; (3) That the employer pays the retrenched employees separation pay equivalent to one (1) month pay or at least one-half () month pay for every year of service, whichever is higher; (4) That the employer exercises its prerogative to retrench employees in good faith for the advancement of its interest and not to defeat or circumvent the employees right to security of tenure; and, (5) That the employer used fair and reasonable criteria in ascertaining who would be dismissed and who would be retained among the employees, such as status, efficiency, seniority, physical fitness, age, and financial hardship for certain workers. PAL failed to comply with the first requirement as in the instant case, PAL failed to substantiate its claim of actual and imminent substantial losses which would justify the retrenchment of more than 1,400 of its cabin crew personnel. As to the fourth requirement, PAL had implemented its retrenchment program in an arbitrary manner and with evident bad faith, which prejudiced the tenurial rights of the cabin crew personnel. As to the fifth requirement, in assessing the overall performance of each cabin crew personnel, PAL only considered the year 1997. This makes the evaluation of each cabin attendants efficiency rating capricious and prejudicial to PAL employees covered by it. WHEREFORE, the instant petition is GRANTED.

41. Eats-Cetera Food Services Outlet v. Letran, G.R. No. 179507, October 2, 2009 Facts: Espadero had been employed by Eats-cetera Food Services Outlet since June 30, 2001 as cashier. When she reported for duty, Espadero discovered that her time card was already punched in. After asking around, she found out that a certain Joselito Cahayagan was the one who punched in her time card. Espadero, however, failed to report the incident to her supervisor, Clarissa Reduca (Reduca), who reported the incident to the personnel manager, Greta dela Hostria. Espadero contended that she was dismissed outright without being given ample opportunity to explain her side. She claimed that on November 21, 2002, petitioners called her and asked her to make a letter of admission as a condition for her reemployment. After writing a letter [of apology about not being able to report the incident immediately], Espadero was told to wait for an assignment. The following day, on November 22, 2002, the company issued a Memorandum terminating her for violation of Rule 24 of the company rules and regulations. Because of this, Espadero decided to file a complaint for illegal dismissal before the NLRC. Petitioners maintained that the company rules and regulations, as well as the corresponding penalties in case of violation thereof, were made known to Espadero before and upon her actual employment as cashier; that contrary to her claim, petitioners gave Espadero ample opportunity to explain her side; and presented the affidavit of supervisor Reduca [containing the incident report]; they conducted an impartial investigation of the incident and found substantial evidence that Espadero was in cahoots with a co-worker in punching in her time card. For this reason, petitioners decided to terminate her. Labor Arbiter declared petitioners liable for illegally terminating Espadero. Upon appeal, the NLRC reversed the Labor Arbiters findings. Aggrieved, respondents filed a petition for certiorari before the CA, who rendered a ruling affirming the Labor Arbiters pronouncement that Espadero was not afforded due process. The appellate court also observed that the punishment of dismissal was too harsh and unjustified. Issues: whether Espadero was afforded her right to due process prior to being dismissed from her job;

Page 137

LABOR RELATIONS
Atty. Jefferson M. Marquez
whether Espaderos infraction was serious enough to warrant the penalty of dismissal. Ruling: Article 282 of the Labor Code includes serious misconduct, fraud and willful breach of trust among the just causes for termination. But prior to termination on such grounds, the employer must satisfy both substantive and procedural due process. Not only must the employee be afforded a reasonable opportunity to be heard and to submit any evidence he may have in support of his defense, but the dismissal must be for a just or authorized cause as provided by law. THE PROCEDURAL REQUIREMENTS are set forth in Section 2(d), Rule I of the Implementing Rules of Book VI of the Labor Code, to wit: SEC. 2. Security of Tenure. x x x. x x x x (d) In all cases of termination of employment, the following standards of due process shall be substantially observed: For termination of employment based on just causes as defined in Article 282 of the Labor Code: A written notice served on the employee specifying the ground or grounds for termination, and giving said employee reasonable opportunity within which to explain his side. A hearing or conference during which the employee concerned, with the assistance of counsel if he so desires is given opportunity to respond to the charge, present his evidence, or rebut the evidence presented against him. A written notice of termination served on the employee, indicating that upon due consideration of all the circumstances, grounds have been established to justify his termination. Petitioners complied with the second notice requirement. Greta dela Hostria, as personnel manager, issued a Memorandum stating with clarity the reason for Espaderos dismissal. SUBSTANTIVELY, we also sustain petitioners reasoning that Espaderos position as a cashier - is one that requires a high degree of trust and confidence, and that her infraction reasonably taints such trust and confidence reposed upon her by her employer. A POSITION OF TRUST AND CONFIDENCE - has been defined as one where a person is entrusted with confidence on delicate matters, or with the custody, handling, or care and protection of the employers property and/or funds. One such position is that of a cashier. A cashier is a highly sensitive position which requires absolute trust and honesty on the part of the employee. It is for this reason that the Court has sustained the dismissal of cashiers who have been found to have breached the trust and confidence of their employers. In one case, the Court upheld the validity of the dismissal of a school cashier despite her 19 years of service after evidence showed that there was a discrepancy in the amount she was entrusted to deposit with a bank. In Metro Drug Corporation v. National Labor Relations Commission, we explained: LOSS OF CONFIDENCE - as a ground for dismissal does not entail proof beyond reasonable doubt of the employees misconduct. It is enough that there be some basis for such loss of confidence or that the employer has reasonable grounds to believe, if not to entertain the moral conviction[,] that the employee concerned is responsible for the misconduct and that the nature of his participation therein rendered him absolutely unworthy of the trust and confidence demanded by his position. The rule, therefore, is that - if there is sufficient evidence to show that the employee occupying - a position of trust and confidence - is guilty of a breach of trust, or that his employer - has ample reason to distrust him, the labor tribunal cannot justly deny the employer - the authority to dismiss such employee. In the instant case, petitioners cannot be faulted for losing their trust in Espadero. As an employee occupying a job - which requires utmost fidelity to her employers, she failed to report to her immediate supervisor - the tampering of her time card. Whether her failure was deliberate - or due to sheer negligence, and whether Espadero was or was not - in cahoots with a co-worker, the fact remains that - the tampering was not promptly reported and could, very likely, not have been known by petitioners, or, at least, could have been discovered at a much later period, if it had not been reported by Espaderos supervisor - to the personnel manager. Petitioners, therefore, cannot be blamed for losing their trust in Espadero.

Page 138

LABOR RELATIONS
Atty. Jefferson M. Marquez
Moreover, the peculiar nature of Espaderos position aggravates her misconduct. MISCONDUCT has been defined as improper or wrong conduct; the transgression of some established or definite rule of action, a forbidden act, a dereliction of duty, willful in character, and implies wrongful intent and not mere error in judgment. The misconduct, to be serious, must be of such a grave character and not merely trivial or unimportant. To constitute just cause for termination, it must be in connection with the employees work. With the degree of trust expected of Espadero, such infraction can hardly be classified as one that is trivial or unimportant. Her failure to promptly report the incident reflects a cavalier regard for the responsibility required of her in the discharge of the duties of her position.

42. Plantation Bay Resort and Spa, et al. vs. Dubrico, G.R. No. 182216, December 4, 2009 Facts: In compliance with RA 9165 (Comprehensive Dangerous Drugs Act), Plantation Bay conducted surprise random drug tests on 122 unsuspecting victims. .errr. . .employees. The tests were done with the assistance of PNP SOCO (scene of the crime operations) with 2 labs conducting the tests: (1) MARTELL drug lab administered the initial tests and (2) PHIL. DRUG SCREENING LAB conducted the confirmatory tests. Respondents Romel Dubrico, Godfrey Ngujo and Julius Villaflor were among 21 employees found positive for use of methamphetamine hydrochloride (shabu). In compliance with several memoranda, they submitted their explanations on the results of the tests, which Plantation Bay found unsatisfactory hence, they were dismissed. Labor Arbiter found them guilty of serious misconduct and ruled that there was no illegal dismissal. NLRC reversed saying there was illegal dismissal and that respondents were not really using drugs! CA affirmed NLRC decision based on evidence which showed a discrepancy between the tests conducted by Phil. Drug and Martell. Plantation Bay objected to the employees questioning the veracity of the tests only in the NLRC Motion for Recon, an issue not raised during the proceedings. Additionally, they maintain that in terminating the services of respondents, they relied on the results of the random drug tests undertaken by an accredited and licensed drug testing facility, and if the results turned out to be questionable or erroneous, they should not be made liable therefor. Issue: 1. Whether or not the NLRC erred in considering the new issue on the veracity of the tests conducted. 2. Whether or not there was illegal dismissal on the part of Plantation Bay. Ruling: Petition is bereft of merit. (a) NLRC did not err. Technical rules of procedure are not strictly adhered to in labor cases consistent with the Constitutional mandate to afford protection to labor. The NLRC did not err in considering the issue of the veracity of the confirmatory tests even if the same was raised only in respondents Motion for Reconsideration of its Decision, it being crucial in determining the validity of respondents dismissal from their employment. Technical rules of procedure are not strictly adhered to in labor cases. In the interest of substantial justice, new or additional evidence may be introduced on appeal before the NLRC. Such move is proper, provided due process is observed, as was the case here, by giving the opposing party sufficient opportunity to meet and rebut the new or additional evidence introduced. (b)Yes. Illegal dismissal. Petition likewise fails on the merits. Plantation Bay failed to prove that employees used drugs based on the doubtful test results. Exhibit A (Note that the confirmatory test showed earlier results that the initial test LOL) Name Romel Dubrico Godfrey Ngujo Julius Villaflor Drug Test Urine sample received on 09/29/04 at 5:14 p.m. Urine sample received on 09/29/04 at 5:24 p.m. Urine sample received on 09/29/04 at 5:32 p.m. Confirmatory Test Issued on 09/29/04 at 3:57 p.m. Issued on 09/29/04 at 3:57 p.m. Issued on 09/29/04 at 4:15 p.m.

Where there is no showing of a clear, valid and legal cause for termination, the law considers the case a matter of illegal dismissal. The burden is on the employer to prove that the termination was for a valid and legal cause. WHEREFORE, the Petition is DENIED.

Page 139

LABOR RELATIONS
Atty. Jefferson M. Marquez

43. Fulache v. ABS-CBN Broadcasting Corporation, G.R. No. 183810, January 21, 2010 Facts: Involved in this case are two separate cases. The first one is with regards to the regularization of the petitioners while the second one is with regards to the illegal dismissal of such petitioners REGULARIZATION CASE ILLEGAL DISMISSAL CASE

Petitioners filed two separate complaints against ABSCBN for While the appeal of ABSCBN to the NLRC was PENDING, petitioners regularization, unfair labor practice, and several money claims. were dismissed by ABSCBN. Petitioners were drivers, cameraman, teleprompters, editors, personal In defense, ABS-CBN alleged that even before the labor arbiter assistants, and VTR editors. rendered his decision in the regularization case, it had already The petitioners alleged that ABS-CBN and the ABS-CBN Rank-and- undertaken a comprehensive review of its existing organizational File Employees Union executed a CBA where they had been excluded structure to address its operational requirements. It then decided to from its coverage as ABS-CBN considered them temporary and not course through legitimate service contractors all driving, messengerial, regular employees, in violation of the Labor Code. They claimed they janitorial, utility, make-up, wardrobe and security services for both the had already rendered more than a year of service in the company and, Metro Manila and provincial stations, to improve its operations and to therefore, should have been recognized as regular employees entitled make them more economically viable. Petitioners were not singled out to security of tenure and to the privileges and benefits enjoyed by for dismissal; as drivers, they were dismissed because they belonged regular employees. They asked that they be paid overtime, night shift to a job category that had already been contracted out. differential, holiday, and rest day and service incentive leave pay. Labor Arbiter upheld the validity of ABS-CBN's contracting out of They also prayed for an award of moral damages and attorneys fees. certain work or services in its operations. The labor arbiter found that ABS-CBN alleged that the petitioners services were contracted on petitioners had been dismissed due to redundancy, an authorized various dates by its Cebu station as independent contractors/off cause under the law. Labor Arbiter awarded them separation pay of camera talents, and they were not entitled to regularization in these one (1) months salary for every year of service. capacities. ABSCBN appealed the above decision to the NLRC. Labor Arbiter rendered a decision holding that the petitioners were regular employees of ABS-CBN, not independent contractors, and are entitled to the benefits and privileges of regular employees. ABSCBN appealed such decision to the NLRC. NLRC made a JOINT DECISION regarding the regularization and the illegal dismissal case. NLRC ruled that there was an employer-employee relationship NLRC reversed the labor arbiters ruling in the illegal dismissal case; it between the petitioners and ABS-CBN as the company exercised found that petitioners had been illegally dismissed and awarded them control over the petitioners in the performance of their work; the backwages and separation pay in lieu of reinstatement. petitioners were regular employees because they were engaged to perform activities usually necessary or desirable in ABS-CBN's trade or business; they cannot be considered contractual employees since they were not paid for the result of their work, but on a monthly basis and were required to do their work in accordance with the companys schedule. Petitioners and ABSCBN both filed for a MOTION FOR RECONSIDERATION of the above decision before the NLRC. NLRC stood by the ruling that the petitioners were regular employees entitled to the benefits and privileges of regular employees. Issue:/S: Whether or not petitioners were regular employees of ABSCBN? Whether or not petitioners were illegally dismissed by ABSCBN? Ruling: On the illegal dismissal case, the petitioners, while recognized as regular employees, were declared legally dismissed due to redundancy.

Page 140

LABOR RELATIONS
Atty. Jefferson M. Marquez
REGULARIZATION CASE Supreme Court agreed with the earlier decisions that petitioners were indeed regular employees of ABSCBN. As regular employees, the petitioners fall within the coverage of the bargaining unit and are therefore entitled to CBA benefits as a matter of law and contract. ILLEGAL DISMISSAL CASE The termination of employment of the four drivers occurred under highly questionable circumstances and with plain and unadulterated bad faith. The records show that the regularization case was in fact the root of the resulting bad faith as this case gave rise and led to the dismissal case. First, the regularization case was filed leading to the labor arbiters decision declaring the petitioners to be regular employees. ABS-CBN appealed the decision and maintained its position that the petitioners were independent contractors. In the course of this appeal, ABS-CBN took matters into its own hands and terminated the petitioners services, clearly disregarding its own appeal then pending with the NLRC. To justify the termination of service, the company cited redundancy as its authorized cause but offered no justificatory supporting evidence. It merely claimed that it was contracting out the petitioners activities in the exercise of its management prerogative. ABS-CBN fell into a downward spiral of irreconcilable legal positions. ABS-CBN forgot labor law and its realities. It forgot that by claiming redundancy as authorized cause for dismissal, it impliedly admitted that the petitioners were regular employees whose services, by law, can only be terminated for the just and authorized causes defined under the Labor Code. It similarly forgot that an exercise of management prerogative can be valid only if it is undertaken in good faith and with no intent to defeat or circumvent the rights of its employees under the laws or under valid agreements. The dismissal was clearly attended by bad faith.

PETITION GRANTED. Petitioners are regular employees of ABSCBN and they are thus entitled to the benefits under the CBA. Petitioners were likewise illegally dismissed and are thus entitled to reinstatement, backwages, attorneys fees, as well as moral damages for the attendance of bad faith in such dismissal.

44. Ancheta vs. Destiny Financial Plans Inc. et al., G.R. No. 179702, Feb. 16, 2010 Facts: On December 1, 2002, respondent Destiny Financial Plans, Inc., a pre-need insurance company, hired petitioner as Head of its Marketing Group, with a compensation package of Ninety Thousand Pesos (P90,000.00) a month. On February 2, 2004, a Marketing Committee meeting was called by respondent Arsenio Bartolome (Bartolome) at the conference room of respondent company. Present at the meeting were petitioner, respondent Bartolome, various leaders of the marketing team, and the operations director of the company. During the meeting, respondent Bartolome made several announcements. However, to the surprise of petitioner, respondent Bartolome announced that petitioner was to resign from the respondent company. On February 11, 2004, petitioner received a letter from respondent company, asking him to explain within forty-eight (48) hours why his services should not be terminated for loss of confidence in his ability to perform the functions of Marketing Director of the company. On February 13, 2004, petitioner submitted his letter of explanation to respondent company. On February 17, 2004, the board of directors of respondent company terminated petitioners services on the ground of loss of confidence. Thus, on March 16, 2004, petitioner filed before the Labor Arbiter a complaint for illegal dismissal, with prayer for reinstatement, payment of full backwages, payment of 13th month pay, moral and exemplary damages, and attorneys fees, against respondent. On April 28, 2005, the Labor Arbiter rendered a Decision in favor of petitioner Ancheta. National Labor Relations Commission (NLRC) reversed the decision of the Labor Arbiter. Aggrieved, petitioner filed a petition for certiorari under Rule 65 of the Rules of Court before the CA. On April 19, 2007, the CA rendered a Decision, affirming with modification the decision of the NLRC. Issue: Whether petitioners employment was validly terminated because of loss of confidence. Ruling:

Page 141

LABOR RELATIONS
Atty. Jefferson M. Marquez
Two requisites must concur in order that there be a valid dismissal from employment, namely: (1) the dismissal must be for any of the causes expressed in Article 282 of the Labor Code; and (2) the employee must be given an opportunity to be heard and to defend himself. Under Article 282(c) of the Labor Code, an employer can terminate the employment of the employee concerned for "fraud or willful breach by an employee of the trust reposed in him by his employer or duly authorized representative." The doctrine of loss of confidence requires the concurrence of the following: (1) loss of confidence should not be simulated; (2) it should not be used as a subterfuge for causes which are improper, illegal, or unjustified; (3) it may not be arbitrarily asserted in the face of overwhelming evidence to the contrary; (4) it must be genuine, not a mere afterthought to justify an earlier action taken in bad faith; and (5) the employee involved holds a position of trust and confidence. Loss of confidence, as a just cause for termination of employment, is premised on the fact that the employee concerned holds a position of responsibility, trust and confidence. He must be invested with confidence on delicate matters, such as the custody, handling, care, and protection of the employer's property and/or funds. In order to constitute a just cause for dismissal, the act complained of must be "work-related" such as would show the employee concerned to be unfit to continue working for the employer. Petitioner was a managerial employee of respondent company, holding a highly sensitive position. Being the Head of the Marketing Group of respondent company, he was in charge, among others, of the over-all production and sales performance of the company. Thus, as aptly pointed out by the CA, his performance was practically the lifeblood of the corporation, because its earnings depended on the sales of the marketing group, which he used to head. The position held by petitioner required the highest degree of trust and confidence of his employer in the formers exercise of managerial discretion insofar as the conduct of the latters business was concerned. Petitioners inability to perform the functions of his office to the satisfaction of his employer and the formers poor judgment as marketing head caused the company huge financial losses. If these were not timely addressed and corrected, the company could have collapsed, to the detriment of its policy holders, stockholders, employees, and the public in general. The power to dismiss an employee is a recognized prerogative inherent in the employers right to freely manage and regulate his business. The law, in protecting the rights of the laborers, authorizes neither oppression nor self-destruction of the employer. The worker's right to security of tenure is not an absolute right, for the law provides that he may be dismissed for cause. In this case, as admitted by petitioner, he was hired because of his expertise in the pre-need industry. His competence and satisfactory performance as head of the marketing group assumed primordial importance for his continued employment in the company. His dismal performance was causing the company financial losses; thus, it was not wise for the company to continue his services. To be sure, an employer cannot be compelled to continue with the employment of workers when continued employment will prove inimical to the employers interest. The SC agrees to CA that private respondents did not strictly comply with the two notice requirement in dismissing petitioner Ancheta. While private respondents sent a show cause letter to petitioner Ancheta, the same letter precipitately implemented termination procedures, i.e., demanded the return of the Executive elevator key which allows petitioner Ancheta access to the office premises and the surrender of the company car assigned to him, even as petitioner Ancheta had yet to answer and air his side. Such betrays the fact that the said show cause letter was but a formality and petitioner Anchetas dismissal is a foregone conclusion. It is thus apparent that private respondents did not comply with the procedural requirements of due process in dismissing petitioner Ancheta. Respondents failure to observe due process in the termination of employment of petitioner for a just cause does not invalidate the dismissal but makes respondent company liable for non-compliance with the procedural requirements of due process. The violation of petitioners right to statutory due process warrants the payment of nominal damages, the amount of which is addressed to the sound discretion of the court, taking into account the relevant circumstances. Petition Denied.

45. Javellana, Jr. vs. Belen, G.R. Nos. 181913 & 182158, March 5, 2010 Facts: Belen was hired by Javellana as company driver and assigned him the tasks of picking up and delivering live hogs, feeds, and lime stones used for cleaning the pigpens. On August 19, 1999 Javellana gave him instructions to (a) pick up lime stones in Tayabas, Quezon; (b) deliver live hogs at Barrio Quiling, Talisay, Batangas; (c) have the delivery truck repaired; and (d) pick up a boar at Joliza Farms in Norzagaray, Bulacan. Petitioner Belen further alleged that his long and arduous day finally ended at 4:30 a.m. of the following day, August 20, 1999. But after just three hours of sleep, respondent Javellana summoned him to the office. When he arrived at 8:20 a.m., Javellana had left. After being told that the latter would not be back until 4:00 p.m., Belen decided to go home and get some more sleep. Petitioner Belen was promptly at the office at 4:00 p.m. but respondent Javellana suddenly blurted out that he was firing Belen from work. Deeply worried that he might not soon get another job, Belen asked for a separation pay. When Javellana offered him only P5,000.00, he did not accept it. Javellana claimed, on the other hand, that he hired petitioner Belen in 1995, not as a company driver, but as family driver. Belen did not do work for his farm on a regular basis, but picked up feeds or delivered livestock only on rare occasions when the farm driver and vehicle were unavailable. Regarding petitioner Belen's dismissal from work, respondent Javellana insisted that he did it for a reason. Belen intentionally failed to report for work on August 20, 1999 and this warranted his dismissal. Issue:

Page 142

LABOR RELATIONS
Atty. Jefferson M. Marquez
Does the amount that the Labor Arbiter awarded petitioner Belen represent all that he will get when the decision in his case becomes final or does it represent only the amount that he was entitled to at the time the Labor Arbiter rendered his decision, leaving room for increase up to the date the decision in the case becomes final? Ruling: Article 279 of the Labor Code, as amended by Section 34 of Republic Act 6715 instructs: Art. 279. Security of Tenure. - In cases of regular employment, the employer shall not terminate the services of an employee except for a just cause or when authorized by this Title. An employee who is unjustly dismissed from work shall be entitled to reinstatement without loss of seniority rights and other privileges and to his full backwages, inclusive of allowances, and to his other benefits or their monetary equivalent computed from the time his compensation was withheld from him up to the time of his actual reinstatement. Clearly, the law intends the award of backwages and similar benefits to accumulate past the date of the Labor Arbiter's decision until the dismissed employee is actually reinstated. But if, as in this case, reinstatement is no longer possible, this Court has consistently ruled that backwages shall be computed from the time of illegal dismissal until the date the decision becomes final. As it happens, the parties filed separate petitions before this Court. The petition in G.R. 181913, filed by respondent Javellana, questioned the CA's finding of illegality of dismissal while the petition in G.R. 182158, filed by petitioner Belen, challenged the amounts of money claims awarded to him. The Court denied the first with finality in its resolution of September 22, 2008; the second is the subject of the present case. Consequently, Belen should be entitled to backwages from August 20, 1999, when he was dismissed, to September 22, 2008, when the judgment for unjust dismissal in G.R. 181913 became final. Separation pay, on the other hand, is equivalent to one month pay for every year of service, a fraction of six months to be considered as one whole year. Here that would begin from January 31, 1994 when petitioner Belen began his service. Technically the computation of his separation pay would end on the day he was dismissed on August 20, 1999 when he supposedly ceased to render service and his wages ended. But, since Belen was entitled to collect backwages until the judgment for illegal dismissal in his favor became final, here on September 22, 2008, the computation of his separation pay should also end on that date. Further, since the monetary awards remained unpaid even after it became final on September 22, 2008 because of issues raised respecting the correct computation of such awards, it is but fair that respondent Javellana be required to pay 12% interest per annum on those awards from September 22, 2008 until they are paid. The 12% interest is proper because the Court treats monetary claims in labor cases the equivalent of a forbearance of credit. It matters not that the amounts of the claims were still in question on September 22, 2008. What is decisive is that the issue of illegal dismissal from which the order to pay monetary awards to petitioner Belen stemmed had been long terminated.

46. WPP Marketing Communications Inc., et al., vs. Galera, G.R. No. 169207, March 25, 2010 Facts: Petitioner is Jocelyn Galera, an American citizen who was recruited from the US by private respondent John Steedman, Chairman-WPP Worldwide and Chief Executive Officer of Mindshare, Co., a corporation based in Hong Kong, China, to work in the Philippines for private respondent WPP Marketing Communications, Inc. (WPP). On December 14, 2000, GALERA alleged she was verbally notified by private STEEDMAN that her services had been terminated from private respondent WPP. A termination letter followed the next day. On 3 January 2001, Galera filed a complaint for illegal dismissal, holiday pay, service incentive leave pay, 13th month pay, incentive plan, actual and moral damages, and attorney's fees against WPP and/or John Steedman (Steedman), Mark Webster (Webster) and Nominada Lansang (Lansang). The Labor Arbiter's Ruling for illegal dismissal and damages in favor of GALERA. The First Division of the NLRC reversed the ruling of Arbiter Madriaga. Yet it was reversed again by CA. Issue: Whether Galera is an Employee or a Corporate Officer. Whether WPP illegally dismissed Galera. Ruling: Employee. Galera, on the belief that she is an employee, filed her complaint before the Labor Arbiter. On the other hand, WPP, Steedman, Webster and Lansang contend that Galera is a corporate officer; hence, any controversy regarding her dismissal is under the jurisdiction of the Regional Trial Court. We agree with Galera. Corporate officers are given such character either by the Corporation Code or by the corporation's by-laws. Galera's appointment as a corporate officer (Vice-President with the operational title of Managing Director of Mindshare) during a special meeting of WPP's Board of Directors is an appointment to a non-existent corporate office. At the time of Galera's appointment, WPP

Page 143

LABOR RELATIONS
Atty. Jefferson M. Marquez
already had one Vice-President in the person of Webster and all five directorship positions provided in the by-laws are already occupied. Another indicator that she was a regular employee and not a corporate officer is Section 14 of the contract, which clearly states that she is a permanent employee not a Vice-President or a member of the Board of Directors. Another convincing indication that she was only a regular employee and not a corporate officer is the disciplinary procedure, which states that her right of redress is through Mindshare's Chief Executive Officer for the Asia-Pacific. This implies that she was not under the disciplinary control of private respondent WPP's Board of Directors (BOD), which should have been the case if in fact she was a corporate officer because only the Board of Directors could appoint and terminate such a corporate officer. WPP's dismissal of Galera lacked both substantive and procedural due process. Apart from Steedman's letter dated 15 December 2000 to Galera, WPP failed to prove any just or authorized cause for Galera's dismissal. Steedman's letter to Galera reads: The operations are currently in a shamble. There is lack of leadership and confidence in your abilities from within, our agency partners and some clients. Most of the staff I spoke with felt they got more guidance and direction from Minda than yourself. In your role as Managing Director, that is just not acceptable. I believe your priorities are mismanaged. The recent situation where you felt an internal strategy meeting was more important than a new business pitch is a good example. You failed to lead and advise on the two new business pitches. In both cases, those involved sort (sic) Minda's input. As I discussed with you back in July, my directive was for you to lead and review all business pitches. It is obvious [that] confusion existed internally right up until the day of the pitch. The quality output is still not to an acceptable standard, which was also part of my directive that you needed to focus on back in July. I do not believe you understand the basic skills and industry knowledge required to run a media special operation. WPP, Steedman, Webster, and Lansang, however, failed to substantiate the allegations in Steedman's letter. Galera, on the other hand, presented documentary evidence 22 in the form of congratulatory letters, including one from Steedman, which contents are diametrically opposed to the 15 December 2000 letter. The law further requires that the employer must furnish the worker sought to be dismissed with two written notices before termination of employment can be legally effected: (1) notice which apprises the employee of the particular acts or omissions for which his dismissal is sought; and (2) the subsequent notice which informs the employee of the employer's decision to dismiss him. Failure to comply with the requirements taints the dismissal with illegality. 23 WPP's acts clearly show that Galera's dismissal did not comply with the two-notice rule.

47. Mercado v. AMA Computer College, G.R. No. 183572, April 13, 2010 Facts: Five former faculty members of AMA Computer College in Paraaque City executed individual Teachers Contracts for each of the trimesters they were engaged to teach. For the school year 2000-2001, when AMACC implemented new faculty screening guidelines, the petitioners failed to obtain a passing rating based on the performance standards. Thus, AMACC did not give them any salary increase. Due to this, they filed a complaint for underpayment of wages. Consequently, they were dismissed as their contracts have expired & were not renewed. The Labor Arbiter ruled that they had been illegally dismissed. The NLRC affirmed the LAs decision. In addition, however, it observed that the applicable law is Section 92 of the Manual of Regulations for Private Schools (which mandates a probationary period of nine consecutive trimesters of satisfactory service for academic personnel in the tertiary level where collegiate courses are offered on a trimester basis), not Article 281 of the Labor Code (which prescribes a probationary period of six months) as the LA ruled. Despite this, the NLRC still affirmed the LAs finding of illegal dismissal on the basis of standards that were only introduced near the end of their probationary period and not at the time of engagement. The CA dismissed the action for illegal dismissal ruling that under the Manual for Regulations for Private Schools, a teaching personnel in a private educational institution (1) must be a full time teacher; (2) must have rendered three consecutive years of service; and (3) such service must be satisfactory before he or she can acquire permanent status. Since they had not completed three (3) consecutive years of service (i.e. six regular semesters or nine consecutive trimesters of satisfactory service) and were still within their probationary period, then they cannot acquire permanent status. The non-renewal of contract is a valid management prerogative. Issue: Whether or not the dismissal is valid. Ruling: AMACC failed to prove by substantial evidence that there was just cause for the non-renewal of the petitioners contracts.

Page 144

LABOR RELATIONS
Atty. Jefferson M. Marquez
The Labor Code is supplemented with respect to the period of probation by special rules found in the Manual of Regulations for Private Schools. On the matter of probationary period, Section 92 of these regulations provides: Section 92. Probationary Period. Subject in all instances to compliance with the Department and school requirements, the probationary period for academic personnel shall not be more than three (3) consecutive years of satisfactory service for those in the elementary and secondary levels, six (6) consecutive regular semesters of satisfactory service for those in the tertiary level, and nine (9) consecutive trimesters of satisfactory service for those in the tertiary level where collegiate courses are offered on a trimester basis. Other than on the period, the following quoted portion of Article 281 of the Labor Code still fully applies: The services of an employee who has been engaged on a probationary basis may be terminated for a just cause when he fails to qualify as a regular employee in accordance with reasonable standards made known by the employer to the employee at the time of his engagement. An employee who is allowed to work after a probationary period shall be considered a regular employee. The common practice is for the employer and the teacher to enter into a contract, effective for one school year. At the end of the school year, the employer has the option not to renew the contract, particularly considering the teachers performance. If the contract is not renewed, the employment relationship terminates. If the contract is renewed, usually for another school year, the probationary employment continues. Again, at the end of that period, the parties may opt to renew or not to renew the contract. If renewed, this second renewal of the contract for another school year would then be the last year since it would be the third school year of probationary employment. At the end of this third year, the employer may now decide whether to extend a permanent appointment to the employee, primarily on the basis of the employee having met the reasonable standards of competence and efficiency set by the employer. For the entire duration of this three-year period, the teacher remains under probation. Upon the expiration of his contract of employment, being simply on probation, he cannot automatically claim security of tenure and compel the employer to renew his employment contract. The school, however, cannot forget that its system of fixed-term contract is a system that operates during the probationary period and for this reason is subject to the terms of Article 281 of the Labor Code. Given the clear constitutional and statutory intents, we cannot but conclude that in a situation where the probationary status overlaps with a fixed-term contract not specifically used for the fixed term it offers, Article 281 should assume primacy and the fixed-period character of the contract must give way. While we can grant that the standards were duly communicated to the petitioners and could be applied beginning the 1st trimester of the school year 2000-2001, glaring and very basic gaps in the schools evidence still exist. The exact terms of the standards were never introduced as evidence; neither does the evidence show how these standards were applied to the petitioners. Hence, the dismissal was illegal.

48. Pantoja vs. SCA Hygiene Products Corp., G.R. No. 163554, April 23, 2010 Facts: SCA Hygiene Products Corp. ( SCA Hygiene for short), a corporation engaged in sale, making and distribution of tissue products and industrial paper, hired Pantoja on March 1987 as back tender taking charge of operations in one of SCA Hygienes mill ( Paper Mill No.4).On March 1999, Pantoja received a Notice of Transfer offering him a position at Paper Mill No. 5 under the same terms and conditions of employment for an anticipated shutdown of Paper Mill No.4 to streamline and phase out the companys industrial paper manufacturing operations in Paper Mill No.4 due to financial difficulties brought about by the low volume of sales and orders for industrial paper products. But Pantoja (and some others offered with transfers ) refused to be transferred of which his services were terminated by reason of redundancy of position. Pantoja then received separation pay (which was handsomely over and above what was provided by law) and executed a release and quitclaim in the corps favor. However, on June 2000, Pantoja filed a complaint for illegal dismissal against SCA Hygiene for lack of valid cause. Pantoja interposed that no permanent shutdown of Paper Mill No. 4 due to its continuous operation since his termination, presenting in evidence Paper Mill Personnel Schedule for Mill No.4 for June, July and August 2000; thus, corp. was in bad faith trying to circumvent his tenurial security when no substantial reason exist. Labor Arbiter dismissed Pantojas complaint stating his rejection of transfer and receipt of the separation pay belie Pantojas illegal dismissal. On appeal by Pantoja, NLRC reversed the Arbiters decision stating the redundancy program is legally infirm on feigned shutdown of operations. On reconsideration by SCA Hygiene asseverating that on 1999 when Mill No.4 was shut down due to low production output, there was a necessity to occasionally run from time to time the machines only for the purpose of maintaining and preserving the same and does not mean that Paper Mill No. 4 continued to be operational. Yet, NLRC remain unpersuaded. On appeal by SCA Hygiene, CA reinstated Labor Arbiters decision. Issue:

Page 145

LABOR RELATIONS
Atty. Jefferson M. Marquez
Whether or not Pantoja was illegally dismissed Ruling: Pantoja is not illegally dismissed. SCA Hygienes right of management prerogative was exercised in good faith. In International Harvester Macleod, Inc. v. Intermediate Appellate Court, the determination of the need to phase out a particular department and consequent reduction of personnel and reorganization as a labor and cost saving device is a recognized management prerogative which the courts will not generally interfere with. Circumstances pointing good faith on SCA Hygienes part - the abolishment of Paper Mill No. 4 was a business judgment arrived at due to low demand for the production of industrial paper at the time. Despite an apparent reason to implement a retrenchment program as a cost-cutting measure, SCA Hygiene, did not outrightly dismiss the workers affected by the closure of Paper Mill No. 4 but gave them an option to be transferred to posts of equal rank and pay. As can be seen, retrenchment was utilized by respondent only as an available option in case the affected employee would not want to be transferred. SCA Hygiene did not proceed directly to retrench. This, to our mind, is an indication of good faith on respondents part as it exhausted other possible measures other than retrenchment. Besides, the employers prerogative to bring down labor costs by retrenching must be exercised essentially as a measure of last resort, after less drastic means have been tried and found wanting. Giving the workers an option to be transferred without any diminution in rank and pay specifically belie petitioners allegation that the alleged streamlining scheme was implemented as a ploy to ease out employees, thus, the absence of bad faith. No evidence, however, was presented to prove that there was continuous operation after the shutdown in the year 1999. On record, Paper Mill No. 4 resumed its operation in 2000 due to a more favorable business climate. The resumption of its industrial paper manufacturing operations does not, however, make respondents streamlining/reorganization plan illegal because, again, the abolishment of Paper Mill No. 4 in 1999 was a business judgment arrived at to prevent a possible financial drain at that time. As long as no arbitrary or malicious action on the part of an employer is shown, the wisdom of a business judgment to implement a cost saving device is beyond this courts determination. Work reassignment of an employee as a genuine business necessity is a valid management prerogative. Even though the transfer would not involve any diminution of rank and pay, still Pantoja refused the transfer and instead, accepted the separation pay voluntarily. The consideration for the quitclaim is credible and reasonable, the waiver represents a valid and binding undertaking. No force and duress attended in its execution. The corp. even gave Pantoja a separation pay more than what the law requires from respondent.

49. BPI v. NLRC, G.R. No. 179801, June 18, 2010 Facts: Records show that respondent, Arambulo, was initially employed as Clerk in 1972 at Citytrust Banking Corporation, which eventually merged with the Bank of Philippine Islands (BPI). She later became Lead Teller, then as Sales Manager, and subsequently, as Bank Manager in BPI-San Pablo, Laguna Branch in 1996. On 4 October 2001, she was reprimanded for the improper handling and retention of a clients account. She was transferred to BPI Family Bank in Los Baos, Laguna on 21 November 2001. On 26 April 2002, a client of BPI-San Pablo, Laguna Branch requested for a certification of her savings account. Her balance reflected an amount less than the actual amount deposited. Hence, BPI conducted an investigation and discovered that its bank teller, Teotima Helen Azucena (Azucena) was making unauthorized withdrawals. A show cause memorandum was served to Azucena asking her to explain the unauthorized withdrawals. In her written response, Azucena implicated respondent, in that the latter, on many occasions, would make temporary cash borrowings and would return the money at the end of the day through withdrawals from her own or other clients accounts. There were times when respondent would fail to return the money withdrawn resulting in shortages on the part of Azucena. When respondent was transferred to Los Baos, Laguna, Azucena added that the same practice was continued by her son, Artie Arambulo. BPI conducted a thorough investigation and discovered that respondent had approved several withdrawals from various accounts of clients whose signatures were forged. The assistant branch manager of the said branch also imputed fault to Arambulo. Arambulo admitted that she prepared the unsigned withdrawal slips on the account of Mr. Vicente Amante (Mr. Amante) totalingP700,000.00 upon request of the latter. She also explained that she processed the withdrawal slips of Mr. Emeterio Dikitan, with the latter signing later on, to expedite his transaction with the bank. She denied any knowledge with regard to the unfunded checks of Mr. Amante that were supposedly deposited to other depositors account. She argued that the posting is done by the teller and only amounts over P150,000.00 pass through her. BPI conducted a hearing and on January 13, 2003, Arambulo was served with notice of termination on the ground of LOSS OF TRUST AND

Page 146

LABOR RELATIONS
Atty. Jefferson M. Marquez
CONFIDENCE for gross violation of policies and procedures. Arambulo filed a complaint for illegal dismissal. The NLRC and CA ruled that respondents dismissal for cause in accordance with the law. It was established that respondent had approved withdrawals which were later proven to be forged but ordered the payment of separation pay. Issue: WON ARAMBULO IS ENTITLED TO SEPARATION PAY. Ruling: NO. (Court applied its ruling in Toyota vs NLRC) While as a general rule, an employee who has been dismissed for any of the just causes enumerated under Article 282 of the Labor Code is not entitled to separation pay, the Court has allowed in numerous cases the grant of separation pay or some other financial assistance to an employee dismissed for just causes on the basis of equity. In the leading case of Philippine Long Distance Telephone Co. v. NLRC, the Court stated that separation pay shall be allowed as a measure of social justice only in those instances where the employee is validly dismissed for causes other than serious misconduct or those reflecting on his moral character. In granting separation pay to respondent, the NLRC and Court of Appeals both adhered to this jurisprudential precept and cleared respondent of bad faith. However, the succeeding case of Toyota Motor Phils. Corp. Workers Association v. NLRC reaffirmed the general rule that separation pay shall be allowed as a measure of social justice only in those instances where the employee is validly dismissed for causes other than serious misconduct, willful disobedience, gross and habitual neglect of duty, fraud or willful breach of trust, commission of a crime against the employer or his family, or those reflecting on his moral character. These five grounds are just causes for dismissal as provided in Article 282 of the Labor Code. Verily, it may not be amiss to emphasize that if an employee has been dismissed for a just cause under Article 282 of the Labor Code, he is not entitled to separation pay. In the instant case, respondent was dismissed on the ground of loss of trust and confidence. It is significant to stress that for there to be a valid dismissal based on loss of trust and confidence: the breach of trust must be willful, meaning it must be done intentionally, knowingly, and purposely, without justifiable excuse. The basic premise for dismissal on the ground of loss of confidence is that the employees concerned hold a position of trust and confidence. It is the breach of this trust that results in the employers loss of confidence in the employee. The case of Aromin v. NLRC is in all fours. In said case, Aromin was the assistant vice-president of BPI when he was validly dismissed for loss of trust and confidence. Invoking the pronouncement in Toyota, the Court disallowed the payment of separation pay on the ground that Aromin was found guilty of willful betrayal of trust, a serious offense akin to dishonesty.

50. Phil. Rural Reconstruction Movement vs. Pulgar, G.R. No. 169227, July 5, 2010 Facts: Pulgar was the manager of PRRM's branch office the Tayabas Bay Field Office (TBFO) in Quezon Province. When Pulgar was reassigned to PRRM's central office, PRRM, through Goyena Solis (Solis), conducted an investigation into alleged financial anomalies committed at the TBFO. In her investigation report, Solis stated that part of the funds allotted to the TBFO was missing or not properly accounted for. The report also stated that some of the receipts that the TBFO submitted to liquidate the organization's financial transactions were fictitious and manufactured. The PRRM management sent Pulgar a copy of the report, together with a memorandum, asking him to explain these findings.

Page 147

LABOR RELATIONS
Atty. Jefferson M. Marquez
In a letter dated February 24, 1997, Pulgar admitted that TBFO's reported expenses did not reflect its actual expenses. He explained that as field manager, he presumed he had the discretion to determine when and how the funds would be used, as long as the use was devoted to the implementation of TBFO projects. Thus, there were instances when he used the funds intended for one project to sustain the activities of other projects. Pulgar further admitted that some of the receipts he submitted to liquidate TBFO's expenses were not genuine; he claimed that he had to produce fake receipts to comply with the central office's requirements and deadlines, otherwise the release of TBFO's subsequent funds would be delayed. Pulgar also disclosed that he had, on his own initiative, opened a separate bank account at the Capitol Bank 6 for TBFO's savings; the account had a remaining balance of P206,958.50. Lastly, Pulgar manifested his willingness to attend a meeting with the senior officers, scheduled on February 28, 1997, to further explain his side. PRRM maintains that while the investigation was ongoing, Pulgar went on leave on March 3-10, March 20-25, and April 1-15, 1997. After the lapse of his last leave on April 15, 1997, Pulgar no longer reported to work, leading PRRM to believe that Pulgar had abandoned his work to evade any liability arising from the investigation. PRRM was therefore surprised to learn that Pulgar had filed an illegal dismissal case on April 3, 1997. Pulgar tells another tale. According to him, on March 17, 1997, he submitted a letter to PRRM to complain that he was not given the right to confront and question Solis, but his letter went unanswered. Thereafter, on March 31, 1997, he was not allowed to enter the premises of the organization. Pulgar also alleges that PRRM's representatives removed his personal properties and records from his office, placed them in boxes and kept them in storage. Believing he was constructively dismissed by PRRM's actions, Pulgar filed a complaint against PRRM on April 3, 1997 for illegal dismissal, illegal suspension, and nonpayment of service incentive leave pay and 13th month pay. Pulgar also asked for actual damages, moral damages, and attorney's fees. The CA observed that PRRM presented no evidence to prove that Pulgar abandoned his job. Reasoning that filing an illegal dismissal complaint is inconsistent with the charge of abandonment, the appellate court concluded that Pulgar had been illegally dismissed. Issue: Whether or not Pulgar was illegally dismissed from employment. Ruling: No. In concluding that Pulgar was constructively dismissed from employment, the CA relied on two main factors: (a) Pulgar's claim that he was barred from entering the premises on March 31, 1997; and (b) the fact that Pulgar immediately filed a complaint for illegal dismissal against PRRM. At first glance, the CA's decision appears correct. But the facts are not as simple as they appear to be. Primarily, we underscore the fact that when Pulgar filed an illegal dismissal complaint on April 3, 1997, he was still on leave from the organization. In other words, from PRRM's standpoint, Pulgar was still its employee when he filed the illegal dismissal case against the organization. Pulgar claims that he was forced to file an illegal dismissal complaint against PRRM while he was on leave because he was not allowed to enter the office premises on March 31, 1997. But aside from making this allegation, Pulgar failed to provide any other details on how he was prevented from entering the premises. Was he physically prevented from entering the premises by a security guard? Did the senior officers of PRRM refuse to let him into the office when he reported to work? We are left to guess the particulars of how PRRM prevented Pulgar from entering the premises, leaving us to doubt the veracity of this allegation. To bolster his contention that he was constructively dismissed, Pulgar asserts that his personal things were taken from his office, placed in boxes and put in storage. To support this allegation, he attached three photographs. But the only thing seen in these photographs is a storage room with sealed boxes on the floor. Taken at face value, there is nothing in the photographs that proves that the boxes in the storage room even contain Pulgar's personal things. Absent such proof, we cannot use these pictures to prove that Pulgar was constructively dismissed from employment. We further note that at the time PRRM was conducting an investigation into the alleged anomalies committed in the liquidation and use of PRRM funds at the TBFO during Pulgar's management, Pulgar went on a number of leaves, specifically on March 3-10, 1997, then on March 20-25, 1997, and finally on April 1-15, 1997. The timing and frequency of these leaves, while not indicative of Pulgar's intention to sever his employment, at the very least, imply Pulgar's active efforts to evade the organization's ongoing investigation. Significantly, while Pulgar claims he was constructively dismissed when he was barred from the premises on March 31, 1997, he still filed his

Page 148

LABOR RELATIONS
Atty. Jefferson M. Marquez
application for leave for April 1-15, 1997. The fact alone that Pulgar was able to return to the office to file his application for leave for April 1-15, 1997 raises doubt as to his purported ban from the premises. Also worth mentioning is the fact that Pulgar continued to receive his salary from PRRM even after March 31, 1997, or the date of his alleged constructive dismissal. In fact, Pulgar received his salary up until April 15, 1997, when his vacation and sick leaves had been consumed. These circumstances, taken together, lead us to conclude that PRRM did not terminate Pulgar's employment. On the contrary, what appears from the evidence is that it was Pulgar himself who terminated his employment with PRRM when he filed an illegal dismissal complaint against the organization while he was on leave. While we recognize the rule that in illegal dismissal cases, the employer bears the burden of proving that the termination was for a valid or authorized cause, in the present case, however, the facts and the evidence do not establish a prima facie case that the employee was dismissed from employment. Before the employer must bear the burden of proving that the dismissal was legal, the employee must first establish by substantial evidence the fact of his dismissal from service. Logically, if there is no dismissal, then there can be no question as to its legality or illegality. Bare allegations of constructive dismissal, when uncorroborated by the evidence on record, cannot be given credence. Although under normal circumstances, an employee's act of filing an illegal dismissal complaint against his employer is inconsistent with abandonment; in the present case, we simply cannot use that one act to conclude that Pulgar did not terminate his employment with PRRM, and in the process ignore the clear, substantial evidence presented by PRRM that proves otherwise. While the Constitution is committed to the policy of social justice and the protection of the working class, it should not be supposed that every labor dispute will be automatically decided in favor of labor. Management also has its rights which are entitled to respect and enforcement in the interest of simple fair play. We have previously ruled on the Labor Arbiter's jurisdiction to rule on all money claims, including those of the employer, arising out of the employer-employee relationship.

51. Maribago Bluewater Beach Resort v. Dual, G.R. No. 180660, July 20, 2010 Facts: Petitioner Maribago is a corporation operating a resort hotel and restaurant in Barangay Maribago, Lapu-Lapu City. In 19953, it hired respondent Dual as waiter and promoted him later as outlet cashier of its Poolbar/Allegro Restaurant. Sometime in 2005, a group of Japanese guests and their companions dined at Allegro. Captain waiter Hiyas took their dinner orders comprising of six (6) sets of lamb and six (6) sets of fish. As per company procedure, Hiyas forwarded one copy of the order slip to the kitchen and another copy to respondent. Pursuant to the order slip, fourteen (14) sets of dinner were prepared by the chef. Hiyas and waiter Genaro Mission, Jr. served twelve (12) set dinners to the guests, and another two (2) sets to their guides free of charge (total of 14 sets of dinner). After dinner, the guests asked for their bill. Since Hiyas was attending to other guests, he gave a signal to Mission to give the bill. Mission asked respondent Dual for the sales transaction receipt and presented this to the guests. The guests paid the amount indicated on the receipt and thereafter left in a hurry. The receipt printed at 10:40 p.m. shows that only P3,036.00 was remitted by cashier Dual corresponding to six (6) sets of dinner. In view of the discrepancy between the order slip and the receipt issued, petitioner Maribago, through its Human Resource Development (HRD) manager, issued memoranda, requiring respondent Dual, Alvin Hiyas, Ernesto Avenido and Basilio Alcoseba to explain why they should not be penalized for violating House Rule 4.1 (dishonesty in any nature). During the clarificatory hearing, butcher Alegrado testified that waiter Alcoseba went to the butchery looking for the order slip for table no. 113. At around 9:45 p.m., waiter Alcoseba caused the alteration of the order slip to reflect that six (6) orders were cancelled. Alegrado allegedly asked Alcoseba if the cook was already aware of the cancellation, to which the latter answered "oo, kahibaw na" (yes, he is already aware). Alcoseba stated that he was not privy to the cancellation of orders since he was busy attending to his room service duty. He claims that - he saw the cancelled food orders at the waiter's station but insists that - he did not have any part in the alteration of the order slip. During the clarificatory hearing, however, he admitted that he altered the order slip by cancelling six (6) set dinners. After the investigation, respondent Dual was found guilty of dishonesty for his fabricated statements and for asking one of the waiters (Mission) to corroborate his allegations. He was terminated per memorandum dated 22 January 2005. Alcoseba was also terminated for dishonesty based on his admission that he altered the order slip.

Page 149

LABOR RELATIONS
Atty. Jefferson M. Marquez
Dual filed a complaint for unfair labor practice, illegal dismissal, non-payment of 13th month and separation pay, and damages. The Labor Arbiter found that respondent's termination was without valid cause and ruled that respondent is entitled to separation pay; The NLRC set aside the Labor Arbiter's decision and dismissed the complaint for lack of merit, saying that complainant's act of depriving respondent of its lawful revenue is tantamount to fraud against the company which warrants dismissal from the service. Falsification of commercial documents as a means to malverse company funds constitutes fraud against the company; The Court of Appeals reversed the decision and resolution of the NLRC. Finding no sufficient valid cause to justify respondent's dismissal, the Court of Appeals ordered petitioner to pay respondent full backwages and separation pay. Petitioner places - the crux of the controversy - on the proven tampering of the transaction receipt which happened in respondent's workstation. Respondent, on the other hand, reiterates his story that the order slip was already altered when Mission gave it to him; that he was able to confirm the cancellation of some orders from Alcoseba and Hiyas; that the receipt he printed was based on the order slip for six (6) sets of dinner; that Mission gave him P3,100.00 as payment and he returned P64.00 as change. Issue: Whether or not the court of appeals committed a grave and reversible error in reversing the national labor relations commission and directing petitioner to pay respondent full backwages from the time he was illegally dismissed, up to the finality of [its] decision and separation pay of one month salary for every year of service. In essence, the issue is whether the Court of Appeals erred in ruling that respondent was illegally dismissed. Ruling: The law requires that an employer shall not terminate the services of an employee except for a just or authorized cause. Otherwise, an employee unjustly dismissed from work is entitled to reinstatement and full backwages. The law also requires the employer - to observe due process in termination cases. w In Agabon v. National Labor Relations Commission,34cralaw we ruled that- violation of the employee's statutory right to due process makes the employer liable to pay indemnity in the form of nominal damages. The law further requires that the burden of proving the cause for termination rests with the employer.cra35 In this case, we are in agreement that petitioner's evidence proved that respondent is guilty of DISHONESTY and of stealing money entrusted to him as cashier. Instead of reporting P10,100.00 as payment by the guests for their dinner, respondent cashier only reported P3,036.00 as shown by the receipt which he admitted to have issued. The receipt which bears his name "NITO" was printed at "22:40" (10:40 p.m.) or 1 hour and 40 minutes after the guests had left at 9:00 p.m. Two other receipts were issued for the same amount at "22:39:55" and "22:40:01". Moreover, respondent's claim that - he received P3,100.00 only and gave Mission P64.00 as change is not shown by the receipt that he issued. The issued receipt does not show that change was given. In addition, the amount indicated in the receipt does not coincide with Dual's contention that only four (4) dishes were cancelled and two (2) dishes were given free of charge. If such were the case, then the amount charged to the guests should have been for eight (8) sets of dinner and not six (6) sets. As established during the clarificatory hearing, twelve (12) sets of dinner were served to guests and two (2) dinner sets were given to the tour guides free of charge. It is clearly indicated in the altered order slip that six (6) out of the twelve (12) sets of dinner were cancelled. The allegation of Dual that - six (6) dinner sets were indeed cancelled as evidenced by the dishes he allegedly saw in the utensil station is negated by the testimonies of the kitchen staff (Chef Armand Galica, Butcher Alegrado and Dessert-in-charge John Marollano) that twelve (12) set meals were served and consumed. These testimonies coincide with the claim of waiters Hiyas and Mission that fourteen (14) sets of dinner were served. The serving of food eliminates the argument of cancellation.

Page 150

LABOR RELATIONS
Atty. Jefferson M. Marquez
The alibi of cancellation has no leg to stand on. The standard operating procedure of Maribago dictates that in cases of cancellation, the order slip - has to be countersigned by the attending waiter (which in this case should have been Chief Waiter Hiyas) but such was not so in this case. The foregoing facts - explain why Dual and Alcoseba tried twice to convince Mission to cover up their crime. They even asked Mission to take the fall by asking him to admit that he altered the order slip from twelve (12) sets of dinner to six (6) sets. In fine, what is damning to the cause of Dual - is the receipt which he admittedly issued. The receipt was issued long after the guests had left (9:00 p.m.) and after the alteration of the order slip (9:45 p.m.) was done. Such fact led us to the conclusion that - he consented to and participated in the anomaly. Respondent's acts constitute SERIOUS MISCONDUCT which is a just cause for termination under the law. THEFT committed by an employee is a valid reason for his dismissal by the employer. Although as a rule this Court leans over backwards to help workers and employees continue with their employment or to mitigate the penalties imposed on them, ACTS OF DISHONESTY - in the handling of company property, petitioner's income in this case, are a different matter. Withal, the law, in protecting the rights of the laborers, authorizes neither oppression nor self-destruction of the employer. While the Constitution is committed to the policy of social justice and the protection of the working class, it should not be supposed that every labor dispute will be automatically decided in favor of labor. The management also has its own rights, as such, are entitled to respect and enforcement in the interest of simple fair play. Out of its concern for those with less privileges in life, the Supreme Court has inclined more often than not - toward the worker and upheld his cause - in his conflicts with the employer. Such favoritism, however, has not blinded the Court to the rule that justice is in every case for the deserving, to be dispensed in the light of the established facts and applicable law and doctrine.

52. New Puerto Commercial vs. Lopez, G.R. No. 169999, July 26, 2010 In order to validly dismiss an employee, he must be accorded both substantive and procedural due process by the employer. Procedural due process requires that the employee be given a notice of the charge against him, an ample opportunity to be heard, and a notice of termination. Even if the aforesaid procedure is conducted after the filing of the illegal dismissal case, the legality of the dismissal, as to its procedural aspect, will be upheld provided that the employer is able to show that compliance with these requirements was not a mere afterthought. Facts: In 1999, petitioner New Puerto Commercial hired respondents Felix Gavan and Rodel Lopez as a delivery panel driver and as a roving salesman, respectively. Under a rolling store scheme, petitioners assigned respondents to sell goods stocked in a van on cash or credit to the sari-sari stores of far-flung barangays and municipalities outside Puerto Princesa City, Palawan. Respondents were duty-bound to collect the accounts receivables and remit the same upon their return to petitioners' store on a weekly basis. In 2000, respondents filed a Complaint for illegal dismissal and non-payment of monetary benefits against petitioners with the Regional Office of the DOLE in Puerto Princesa City. On November 20, 2000, a conciliation conference was held but the parties failed to reach an amicable settlement. As a result, the complaint was endorsed for compulsory arbitration at the RAB of the NLRC. Previously or on November 28, 2000, petitioners sent respondents notices to explain why they should not be dismissed for gross misconduct based on (1) the alleged misappropriation of their sales collections, and (2) their absence without leave for more than a month. The notice also required respondents to appear before petitioners' lawyer on December 2, 2000 to give their side with regard to the foregoing charges. Respondents refused to attend said hearing. On December 18, 2000, petitioners served notices of termination on respondents on the grounds of gross misconduct and absence without leave for more than one month. The Labor Arbiter dismissed the complaint for illegal dismissal. The NLRC affirmed the ruling of the Labor Arbiter. It ruled that damages cannot

Page 151

LABOR RELATIONS
Atty. Jefferson M. Marquez
be awarded in favor of respondents because their dismissal was for just causes. The CA affirmed with modification the ruling of the NLRC, to wit, it awarded nominal damages of P30,000.00 each to petitioners because they were denied due process. Issue: Whether the respondents were denied procedural due process justifying the award of nominal damages in accordance with the ruling in Agabon v. National Labor Relations Commission Ruling: The petition is meritorious. When the requirements of procedural due process are satisfied, the award of nominal damages is improper. In termination proceedings of employees, procedural due process consists of the twin requirements of notice and hearing. The employer must furnish the employee with two written notices before the termination of employment can be effected: (1) the first apprises the employee of the particular acts or omissions for which his dismissal is sought; and (2) the second informs the employee of the employer's decision to dismiss him. The requirement of a hearing is complied with as long as there was an opportunity to be heard, and not necessarily that an actual hearing was conducted. As we explained in Perez v. Philippine Telegraph and Telephone Company: An employee's right to be heard in termination cases under Article 277 (b) as implemented by Section 2 (d), Rule I of the Implementing Rules of Book VI of the Labor Code should be interpreted in broad strokes. It is satisfied not only by a formal face to face confrontation but by any meaningful opportunity to controvert the charges against him and to submit evidence in support thereof. A hearing means that a party should be given a chance to adduce his evidence to support his side of the case and that the evidence should be taken into account in the adjudication of the controversy. "To be heard" does not mean verbal argumentation alone inasmuch as one may be heard just as effectively through written explanations, submissions or pleadings. Therefore, while the phrase "ample opportunity to be heard" [in Article 277 of the Labor Code] may in fact include an actual hearing, it is not limited to a formal hearing only. In other words, the existence of an actual, formal "trial-type" hearing, although preferred, is not absolutely necessary to satisfy the employee's right to be heard. It was duly established, as affirmed by the appellate court itself, that respondents failed to report for work starting from October 22, 2000 for respondent Lopez and October 28, 2000 for respondent Gavan, then at the time of the filing of the complaint with the labor office on November 3, 2000, respondents were not yet dismissed from employment. Prior to this point in time, there was, thus, no necessity to comply with the twin requirements of notice and hearing. The mere fact that the notices were sent to respondents after the filing of the labor complaint does not, by itself, establish that the same was a mere afterthought. The surrounding circumstances of this case adequately explain why the requirements of procedural due process were satisfied only after the filing of the labor complaint. Sometime in the third week of October 2000, petitioners received information that respondents were not remitting their sales collections to the company. Thereafter, petitioners initiated an investigation by sending one of their trusted salesmen, Bagasala, in the route being serviced by respondents. To prevent a possible cover up, respondents were temporarily reassigned to a new route to service. Subsequently, respondents stopped reporting for work (i.e., starting from October 22, 2000 for respondent Lopez and October 28, 2000 for respondent Gavan) after they got wind of the fact that they were being investigated for misappropriation of their sales collection, and, on November 3, 2000, respondents filed the subject illegal dismissal case to pre-empt the outcome of the ongoing investigation. On November 18, 2000, Bagasala returned from his month-long investigation in the far-flung areas previously serviced by respondents and reported that respondents indeed failed to remit P2,257.03 in sales collections. As a result, on November 28, 2000, termination proceedings were commenced against respondents by sending notices to explain with a notice of hearing scheduled on December 2, 2000. As narrated earlier, respondents failed to give their side despite receipt of said notices. Petitioners sent another set of notices to respondents on December 7, 2000 to attend a hearing on December 15, 2000 but respondents again refused to attend. Thus, on December 18, 2000, petitioners served notices of termination on respondents for gross misconduct in misappropriating their sales collections and absence without leave for more than a month. As can be seen, under the peculiar circumstances of this case, it cannot be concluded that the sending of the notices and setting of hearings were a mere afterthought because petitioners were still awaiting the report from Bagasala when respondents pre-empted the results of the ongoing investigation by filing the subject labor complaint. For this reason, there was sufficient compliance with the twin requirements of notice and hearing even if the notices were sent and the hearing conducted after the filing of the labor complaint. Thus, the award of nominal damages by the appellate court is improper.

53. Artificio vs. NLRC, G.R. No. 172988, July 26, 2010

Page 152

LABOR RELATIONS
Atty. Jefferson M. Marquez
Facts: Petitioner Jose P. Artificio was employed as security guard by respondent RP Guardians Security Agency, Inc., a corporation duly organized and existing under Philippine Laws and likewise duly licensed to engage in the security agency business. Sometime in June 2002, Artificio had a heated argument with a fellow security guard, Merlino B. Edu (Edu). On 25 July 2002, Edu submitted a confidential report 5 to Antonio A. Andres (Andres), Administration & Operations Manager, requesting that Artificio be investigated for maliciously machinating Edu's hasty relief from his post and for leaving his post during night shift duty to see his girlfriend at a nearby beerhouse. On 29 July 2002, another security guard, Gutierrez Err (Err), sent a report 6 to Andres stating that Artificio arrived at the office of RP Guardians Security Agency, Inc. on 25 June 2002, under the influence of liquor. When Artificio learned that no salaries would be given that day, he badmouthed the employees of RP Guardians Security Agency, Inc. and threatened to "arson" their office. On even date, Andres issued a Memorandum temporarily relieving Artificio from his post and placing him under preventive suspension pending investigation for conduct unbecoming a security guard, such as, abandonment of post during night shift duty, light threats and irregularities in the observance of proper relieving time. He also directed Artificio to report to the office of RP Guardians Security Agency, Inc. and submit his written answer immediately upon receipt of the memorandum. In another memorandum, Andres informed Artificio that a hearing will be held on 12 August 2002. Without waiting for the hearing to be held, Artificio filed on 5 August 2002, a complaint for illegal dismissal, illegal suspension, non-payment of overtime pay, holiday pay, premium pay for holiday and rest days, 13th month pay, and damages. He also prayed for payment of separation pay in lieu of reinstatement. 10 Labor Arbiter rendered a decision dated 6 October 2003, finding respondents guilty of illegal suspension and dismissal. It was also held that Artificio should have been allowed to confront Edu and Err before he was preventively suspended. Since the complainant does not seek reinstatement, he is entitled to limited backwages and separation pay. On appeal, the NLRC, set aside the decision of the Labor Arbiter ruling that the Labor Arbiter erred in considering preventive suspension as a penalty. The motion for reconsideration filed by Artificio was denied for lack of merit Artificio next filed a petition for certiorari before the Court of Appeals which rendered a decision affirming the NLRC decision. Artificio filed a motion for reconsideration which the Court of Appeals again denied. Issues: 1. Wether or not Petitioner Artificio's preventive suspension was justified 2. Whether or not, he is entitled to backwages and separation pay Ruling: 1. Yes. Sections 8 and 9 of Rule XXIII, Implementing Book V of the Omnibus Rules Implementing the Labor Code provides that preventive suspension is justified where the employee's continued employment poses a serious and imminent threat to the life or property of the employer or of the employee's co-workers. Without this kind of threat, preventive suspension is not proper. In this case, Artificio's preventive suspension was justified since he was employed as a security guard tasked precisely to safeguard respondents' client. His continued presence in respondents' or its client's premises poses a serious threat to respondents, its employees and client in light of the serious allegation of conduct unbecoming a security guard such as abandonment of post during night shift duty, light threats and irregularities in the observance of proper relieving time. Besides, Management has the prerogative to discipline its employees and to impose appropriate penalties on erring workers pursuant to company rules and regulations. This Court has upheld a company's management prerogatives so long as they are exercised in good faith for the advancement of the employer's interest and not for the purpose of defeating or circumventing the rights of the employees under special laws or under valid agreements. 20 Significantly, Artificio regrettably chose not to present his side at the administrative hearing scheduled to look into the factual issues that accompanied the accusation against him. In fact, he avoided the investigation into the charges by filing his illegal dismissal complaint ahead of the scheduled investigation. He, on his own decided that his preventive suspension was in fact illegal dismissal and that he is entitled to backwages and separation pay. Indeed, Artificio would even reject reinstatement revealing his bent to have his own way through his own means. As aptly noted by the NLRC, Artificio preempted the investigation that could have afforded him the due process of which he would then say he was denied. 2. Yes for Backwages. No for separation pay.

Page 153

LABOR RELATIONS
Atty. Jefferson M. Marquez
That resolved, we next proceed to the benefits due Artificio. Having determined that the imposition on Artificio of preventive suspension was proper and that such suspension did not amount to illegal dismissal, we see no basis for the grant of backwages. Nonetheless, given the attendant circumstances in this case, namely, that Artificio had been working with the company for a period of sixteen (16) years and without any previous derogatory record, the ends of social and compassionate justice would be served if Artificio be given some equitable relief in the form of separation pay. 22 Artificio is entitled to separation pay considering that while reinstatement is an option, Artificio himself has never, at anytime after the notice of preventive suspension intended to remain in the employ of private respondents.

54. Calipay vs. NLRC, G.R. No. 166411, August 3, 2010 Facts: On July 16, 1999, a Complaint 3 for illegal dismissal, unfair labor practice, underpayment of wages and 13th month pay, non-payment of service incentive leave pay, overtime pay, premium pay for holiday, rest day, night shift allowances and separation pay was filed by herein petitioner Elpidio Calipay, together with Alfredo Mission and Ernesto Dimalanta against herein private respondents Triangle Ace Corporation (Triangle) and Jose Lee. Calipay and the other complainants alleged in their Position Paper that in the course of their employment, they were not given any specific work assignment; they performed various kinds of work imposed upon them by Lee; in discharging their functions, they were required by Lee to work for nine (9) hours a day, beginning from 7:00 a.m. and ending at 6:00 p.m. with a break of one hour at 12:00 noon; they were also required to report from Monday to Sunday; for work rendered from Mondays to Saturdays beyond the normal eight (8) working hours in a day, they were paid a uniform daily wage in the amount of P140.00 even during holidays; for work performed on Sundays, they were not paid any wage due to the policy of Lee that his workers must provide work without pay at least a day in the week under his so-called "bayanihan system"; in receiving their wages, they were not given any duly accomplished payslips; instead, they were forced to sign a blank form of their daily time records and salary vouchers. It was further alleged that in May 1998, Lee confronted Calipay and Mission regarding their alleged participation and assistance in Dimalanta's claim for disability benefits with the Social Security System; despite their denials, Lee scolded Calipay and Mission; this incident later led to their dismissal in the same month. Labor Arbiter handling the case rendered a Decision dismissing the Complaint for lack of merit. Calipay and the other complainants filed an appeal with the National Labor Relations Commission (NLRC) modifying the Labor Arbiter's decision and ordering respondents Triangle Ace Corporation Inc./Jose Lee to reinstatement. Aggrieved, private respondents filed a Motion for Reconsideration was Given due course and the decision of the Labor Arbiter was reinstated and affirmed. As a consequence, Calipay and the other complainants moved for the reconsideration, but the same was denied by the NLRC. Appealed to CA which rendered its Decision dismissing the petition. Calipay filed a Motion for Reconsideration, but the CA denied Hence, the instant petition of Calipay raising the following issues: Issue: Whether or not there was abandonment of work a just ground for dismissal Held: Calipay and the other complainants failed to sufficiently refute these findings of the Labor Arbiter in their appeal filed with the NLRC. They simply insisted that they did not report for work, because they were already terminated. However, they did not present any evidence to prove their allegation. On the other hand, as held by the Labor Arbiter, private respondents were able to present the DTRs and Salary Vouchers of Calipay and the other complainants showing that they indeed reported for work even after their alleged termination from employment. 26 Calipay and the other complainants also failed to present evidence to prove their allegation that they were forced to sign blank forms of their DTRs and Salary

Page 154

LABOR RELATIONS
Atty. Jefferson M. Marquez
Vouchers. On the basis of the foregoing, the Court arrives at the conclusion that the filing of the complaint for illegal dismissal appears only as a convenient afterthought on the part of petitioner and the other complainants after they were dismissed in accordance with law. Jurisprudence has held time and again that abandonment is totally inconsistent with the immediate filing of a complaint for illegal dismissal, more so if the same is accompanied by a prayer for reinstatement. 27 In the present case, however, petitioner filed his complaint more than one year after his alleged termination from employment. Moreover, petitioner and the other complainants' inconsistency in their stand is also shown by the fact that in the complaint form which they personally filled up and filed with the NLRC, they only asked for payment of separation pay and other monetary claims. They did not ask for reinstatement. It is only in their Position Paper later prepared by their counsel that they asked for reinstatement. This is an indication that petitioner and the other complainants never had the intention or desire to return to their jobs. In fact, there is no evidence to prove that petitioner and his former co-employees ever attempted to return to work after they were dismissed from employment. On the other hand, private respondents were able to present memoranda or show-cause letters served on petitioner and the other complainants at their last known address requiring them to explain their absence, with a warning that their failure would be construed as abandonment of work. Also, private respondents served on petitioner and the other complainants a notice of termination as required by law. Private respondents' compliance with said requirements, taken together with the other circumstances above-discussed, only proves petitioner and the other complainants' abandonment of their work.

55. Nacague v. Sulpicio Lines, G.R. No. 172589, August 8, 2010 Facts: On 15 June 1995, Sulpicio Lines, Inc. (Sulpicio Lines) hired Nacague as hepe de viaje or the representative of Sulpicio Lines on board its vessel M/V Princess of the World (the ship). On 14 February 2003, Ceasar T. Chico, a housekeeper on the ship, submitted a report regarding the drug paraphernalia found inside the Mopalla Suite Room and the threat on his life made by Nacague and Chief Mate Reynaldo Doroon after he found the drug paraphernalia. On 15 February 2003, Sulpicio Lines sent a notice of investigation to Nacague informing him of the charges against him for use of illegal drugs and threatening a co-employee. When the ship docked in the port of Manila on 18 February 2003, some crew members of the ship, together with Nacague, were subjected to a random drug test. They were taken to S.M. Lazo Medical Clinic (S.M. Lazo Clinic) and were required to submit urine samples. The result of the random drug test revealed that Nacague was positive for methamphetamine hydrochloride or shabu. On 20 February 2003, Sulpicio Lines subjected Nacague to a formal investigation. Nacague denied using illegal drugs. On 23 February 2003, Nacague went to Chong Hua Hospital in Cebu City to undergo a voluntary drug test. The drug test with Chong Hua Hospital yielded a negative result. Nacague submitted this test result to Sulpicio Lines. However, on 7 March 2003, Sulpicio Lines sent a memorandum to Nacague terminating him from the service. The memorandum reads: After a careful consideration of your case with the evidence available, including your explanation, and with the positive drug test result, management finds you culpable of grave misconduct and loss of trust and confidence. In view thereof, the company is constrained to terminate your employment effective today, March 7, 2003. Feeling aggrieved, Nacague filed a complaint for illegal suspension, illegal dismissal and for reinstatement with backwages. LAIssue: WON the termination is valid. Ruling: Nacague maintains that the S.M. Lazo Clinic drug test was not credible because Sulpicio Lines failed to show that S.M. Lazo Clinic is an authorized drug testing center. Nacague also alleges that the urine samples were gathered carelessly without proper labels to identify their owners and that S.M. Lazo Clinic did not ask Nacague if he was taking any medication that might alter the results of the drug test. Nacague adds that Republic Act No. 9165 (R.A. No. 9165) and the Department of Labor and Employment Order No. 53-03 (Department Order No. 53-03) require two drug tests a screening test and a confirmatory test. Nacague maintains that, since only a screening test was conducted, he was illegally dismissed based on an incomplete drug test. Nacague argues that Sulpicio Lines failed to discharge its burden of proving that the termination of his employment was legal. On the other hand, Sulpicio Lines questions the belated attempt of Nacague to question the credibility of S.M. Lazo Clinic. Sulpicio Lines also argues that since Nacague knew that the residue of the drug would no longer be detectable in his body

Page 155

LABOR RELATIONS
Atty. Jefferson M. Marquez
after five days, Nacague underwent another drug test with the Chong Hua Hospital. Sulpicio Lines insists that the most accurate drug test is the random drug test conducted by S.M. Lazo Clinic and that the test with Chong Hua Hospital was a planned test. Under Article 279 of the Labor Code, an employer may terminate the services of an employee for just causes or for authorized causes. Furthermore, under Article 277(b) of the Labor Code, the employer must send the employee who is about to be terminated, a written notice stating the causes for termination and must give the employee the opportunity to be heard and to defend himself. Thus, to constitute valid dismissal from employment, two requisites must concur: (1) the dismissal must be for a just or authorized cause; and (2) the employee must be afforded an opportunity to be heard and to defend himself. Section 36 of R.A. No. 9165 provides that drug tests shall be performed only by authorized drug testing centers. Moreover, Section 36 also prescribes that drug testing shall consist of both the screening test and the confirmatory test. Section 36 of R.A. No. 9165 reads: SEC. 36. Authorized Drug Testing. Authorized drug testing shall be done by any government forensic laboratories or by any of the drug testing laboratories accredited and monitored by the DOH to safeguard the quality of test results. The DOH shall take steps in setting the price of the drug test with DOH accredited drug testing centers to further reduce the cost of such drug test. The drug testing shall employ, among others, two (2) testing methods, the screening test which will determine the positive result as well as the type of drug used and the confirmatory test which will confirm a positive screening test. x x x (Emphasis supplied) Department Order No. 53-03 further provides: Drug Testing Program for Officers and Employees Drug testing shall conform with the procedures as prescribed by the Department of Health (DOH) (www.doh.gov.ph). Only drug testing centers accredited by the DOH shall be utilized. A list of accredited centers may be accessed through the OSHC website (www.oshc.dole.gov.ph). Drug testing shall consist of both the screening test and the confirmatory test; the latter to be carried out should the screening test turn positive. The employee concerned must be informed of the test results whether positive or negative. (Emphasis supplied) The law is clear that drug tests shall be performed only by authorized drug testing centers. In this case, Sulpicio Lines failed to prove that S.M. Lazo Clinic is an accredited drug testing center. Sulpicio Lines did not even deny Nacagues allegation that S.M. Lazo Clinic was not accredited. Also, only a screening test was conducted to determine if Nacague was guilty of using illegal drugs. Sulpicio Lines did not confirm the positive result of the screening test with a confirmatory test. Sulpicio Lines failed to indubitably prove that Nacague was guilty of using illegal drugs amounting to serious misconduct and loss of trust and confidence. Sulpicio Lines failed to clearly show that it had a valid and legal cause for terminating Nacagues employment. When the alleged valid cause for the termination of employment is not clearly proven, as in this case, the law considers the matter a case of illegal dismissal. However, reinstatement is no longer feasible due to strained relations between Nacague and Sulpicio Lines and that Nacague should instead be granted separation pay.

56. Century Canning Corp. vs. Ramil, G.R. No. 171630, August 8, 2010 Facts: On March 3, 1999, respondent prepared a CAPEX form for external fax modems and terminal server, per order of Technical Operations Manager Jaime Garcia, Jr. and endorsed it to Marivic Villanueva, Secretary of Executive Vice-President Ricardo T. Po, for the latter's signature. The CAPEX form, however, did not have the complete details 3 and some required signatures. The following day, March 4, 1999, with the form apparently signed by Po, respondent transmitted it to Purchasing Officer Lorena Paz in Taguig Main Office. Paz processed the paper and found that some details in the CAPEX form were left blank. She also doubted the genuineness of the signature of Po, as appearing in the form. Paz then transmitted the CAPEX form to Purchasing Manager Virgie Garcia and informed her of the questionable signature of Po. Consequently, the request for the equipment was put on hold due to Po's forged signature. However, due to the urgency of purchasing badly needed equipment, respondent was ordered to make another CAPEX form, which was immediately transmitted to the Purchasing Department. ACcHIa Suspecting him to have committed forgery, respondent was asked to explain in writing the events surrounding the incident. He vehemently denied any participation in the alleged forgery. Respondent was, thereafter, suspended on April 21, 1999. Subsequently, he received a Notice of Termination from Armando C. Ronquillo, on May 20, 1999, for loss of trust and confidence. Respondent filed a case for illegal dismissal. LA dismissed the complaint for lack of merit. NLRC found in favor of respondent. NLRC reversed itself and rendered a new Decision upholding

Page 156

LABOR RELATIONS
Atty. Jefferson M. Marquez
LA decision. The CA, rendered judgment in favor of respondent and reinstated the earlier decision of the NLRC. Issue: Whether or not the dismissal was valid. Ruling: Respondent alleged in his position paper that after preparing the CAPEX form on March 3, 1999, he endorsed it to Marivic Villanueva for the signature of the Executive Vice-President Ricardo T. Po. The next day, March 4, 1999, respondent received the CAPEX form containing the signature of Po. Petitioner never controverted these allegations in the proceedings before the NLRC and the CA despite its opportunity to do so. Petitioner's belated allegations in its reply filed before this Court that Marivic Villanueva denied having seen the CAPEX form cannot be given credit. Points of law, theories, issues and arguments not brought to the attention of the lower court, administrative agency or quasi-judicial body need not be considered by a reviewing court, as they cannot be raised for the first time at that late stage. When a party deliberately adopts a certain theory and the case is decided upon that theory in the court below, he will not be permitted to change the same on appeal, because to permit him to do so would be unfair to the adverse party. The law mandates that the burden of proving the validity of the termination of employment rests with the employer. Failure to discharge this evidentiary burden would necessarily mean that the dismissal was not justified and, therefore, illegal. Unsubstantiated suspicions, accusations, and conclusions of employers do not provide for legal justification for dismissing employees. In case of doubt, such cases should be resolved in favor of labor, pursuant to the social justice policy of labor laws and the Constitution. Petitioner based respondent's dismissal on its unsubstantiated suspicions and conclusion that since respondent was the custodian and the one who prepared the CAPEX forms, he had the motive to commit the forgery. We have previously held that employers are allowed a wider latitude of discretion in terminating the services of employees who perform functions which by their nature require the employers' full trust and confidence and the mere existence of basis for believing that the employee has breached the trust of the employer is sufficient, this does not mean that the said basis may be arbitrary and unfounded. The right of an employer to dismiss an employee on the ground that it has lost its trust and confidence in him must not be exercised arbitrarily and without just cause. Loss of trust and confidence, to be a valid cause for dismissal, must be based on a willful breach of trust and founded on clearly established facts. The basis for the dismissal must be clearly and convincingly established, but proof beyond reasonable doubt is not necessary. It must rest on substantial grounds and not on the employer's arbitrariness, whim, caprice or suspicion; otherwise, the employee would eternally remain at the mercy of the employer. Petitioner's reliance on respondent's previous tardiness in reporting for work as a ground for his dismissal is likewise not meritorious. The correct rule has always been that such previous offense may be used as valid justification for dismissal from work only if the infractions are related to the subsequent offense upon which the basis of termination is decreed. His previous offenses were entirely separate and distinct from his latest alleged infraction of forgery. Hence, the same could no longer be utilized as an added justification for his dismissal. Besides, respondent had already been sanctioned for his prior infractions. To consider these offenses as justification for his dismissal would be penalizing respondent twice for the same offense. Respondent's illegal dismissal carries the legal consequences defined under Article 279 of the Labor Code, that is, an employee who is unjustly dismissed from work shall be entitled to reinstatement without loss of seniority rights and other privileges, and to the payment of his full backwages, inclusive of allowances, and to his other benefits or their monetary equivalent, computed from the time his compensation was withheld from him up to the time of his actual reinstatement. However, the Court finds that it would be best to award separation pay instead of reinstatement, in view of the strained relations between petitioner and respondent. Respondent was dismissed due to loss of trust and confidence and it would be impractical to reinstate an employee whom the employer does not trust, and whose task is to handle and prepare delicate documents. Under the doctrine of strained relations, the payment of separation pay has been considered an acceptable alternative to reinstatement when the latter option is no longer desirable or viable. On the one hand, such payment liberates the employee from what could be a highly oppressive work environment. On the other hand, the payment releases the employer from the grossly unpalatable obligation of maintaining in its employ a worker it could no longer trust. In view of the foregoing, respondent is entitled to the payment of full backwages, inclusive of allowances, and other benefits or their monetary equivalent, computed from the date of his dismissal on May 20, 1999 up to the finality of this decision, and separation pay in lieu of reinstatement

Page 157

LABOR RELATIONS
Atty. Jefferson M. Marquez
equivalent to one month salary for every year of service, computed from the time of his engagement by petitioner on August 1993 up to the finality of the decision.

57. D.M Consunji vs. Gobres, G.R. No. 169170, August 8, 2010 Facts: Respondents Antonio Gobres, Magellan Dalisay, Godofredo Paragsa, Emilio Aleta and Generoso Melo worked as carpenters in the construction projects of petitioner D.M. Consunji, Inc., a construction company, on several occasions and/or at various times. Their termination from employment for each project was reported to the Department of Labor and Employment (DOLE), in accordance with Policy Instruction No. 20, which was later superseded by Department Order No. 19, series of 1993. On October 14, 1998, respondents saw their names included in the Notice of Termination posted on the bulletin board at the project premises. Respondents filed a Complaint with the Arbitration Branch of the National Labor Relations Commission (NLRC) against petitioner D.M. Consunji, Inc. and David M. Consunji for illegal dismissal, and nonpayment of 13th month pay, five (5) days service incentive leave pay, damages and attorney's fees. Petitioner contended that since respondents were terminated by reason of the completion of their respective phases of work in the construction project, their termination was warranted and legal. Moreover, petitioner claimed that respondents have been duly paid their service incentive leave pay and 13th month pay through their respective bank accounts, as evidenced by bank remittances. Labor Arbiter ruled that the employees were project employees. NLRC affirmed the decision of the Labor Arbiter, and Court of Appeals affirmed the decision of NLRC but awarded damages for the failure of the company to observe procedural due process. Issue: Whether or not notice (procedural due process) is required in termination of project employees Ruling: No. A project employee is defined under Article 280 of the Labor Code as one whose "employment has been fixed for a specific project or undertaking the completion or termination of which has been determined at the time of the engagement of the employee or where the work or services to be performed is seasonal in nature and the employment is for the duration of the season." As project employees, respondents' termination is governed by Section 1 (c) and Section 2 (III), Rule XXIII (Termination of Employment), Book V of the Omnibus Rules Implementing the Labor Code. Section 1 (c), Rule XXIII, Book V of the Omnibus Rules Implementing the Labor Code states: Section 1.Security of tenure. (a) In cases of regular employment, the employer shall not terminate the services of an employee except for just or authorized causes as provided by law, and subject to the requirements of due process. xxx xxx xxx (c)In cases of project employment or employment covered by legitimate contracting or sub-contracting arrangements, no employee shall be dismissedprior to the completion of the project or phase thereof for which the employee was engaged, or prior to the expiration of the contract between the principal and contractor, unless the dismissal is for just or authorized cause subject to the requirements of due process or prior notice, or is brought about by the completion of the phase of the project or contract for which the employee was engaged. Records show that respondents were dismissed after the expiration of their respective project employment contracts, and due to the completion of the phases of work respondents were engaged for. Hence, the cited provision's requirements of due process or prior notice when an employee is dismissed for just or authorized cause (under Articles 282 and 283 of the Labor Code) prior to the completion of the project or phase thereof for which the employee was engaged do not apply to this case. AECDHS Further, Section 2 (III), Rule XXIII, Book V of the Omnibus Rules Implementing the Labor Code provides: Section 2.Standard of due process: requirements of notice. In all cases of termination of employment, the following standards of due process shall be substantially observed. 1 .For termination of employment based on just causes as defined in Article 282 of the Code: (a)A written notice served on the employee specifying the ground or grounds for termination, and giving to said employee reasonable opportunity within which to explain his side;

Page 158

LABOR RELATIONS
Atty. Jefferson M. Marquez
(b)A hearing or conference during which the employee concerned, with the assistance of counsel if the employee so desires, is given opportunity to respond to the charge, present his evidence or rebut the evidence presented against him; and (c)A written notice [of] termination served on the employee indicating that upon due consideration of all the circumstance, grounds have been established to justify his termination. In case of termination, the foregoing notices shall be served on the employee's last known address. II.For termination of employment as based on authorized causes defined in Article 283 of the Code, the requirements of due process shall be deemed complied with upon service of a written notice to the employee and the appropriate Regional Office of the Department at least thirty (30) days before the effectivity of the termination, specifying the ground or grounds for termination. III.If the termination is brought about by the completion of the contract or phase thereof, no prior notice is required. If the termination is brought about by the failure of an employee to meet the standards of the employer in the case of probationary employment, it shall be sufficient that a written notice is served the employee within a reasonable time from the effective date of termination. In this case, the Labor Arbiter, the NLRC and the Court of Appeals all found that respondents were validly terminated due to the completion of the phases of work for which respondents' services were engaged. The above rule clearly states, "If the termination is brought about by the completion of the contract or phase thereof, no prior notice is required." Cioco, Jr. v. C.E. Construction Corporation explained that this is because completion of the work or project automatically terminates the employment, in which case, the employer is, under the law, only obliged to render a report to the DOLE on the termination of the employment. Hence, prior or advance notice of termination is not part of procedural due process if the termination is brought about by the completion of the contract or phase thereof for which the employee was engaged. Petitioner, therefore, did not violate any requirement of procedural due process by failing to give respondents advance notice of their termination; thus, there is no basis for the payment of nominal damages.

58. Nagkaka-sang Lakas ng Manggagawa sa Keihin vs. Keihin Phils. Corp., G.R. No. 171115, August 9, 2010 Facts: Petitioner Helen Valenzuela (Helen) was a production associate in respondent Keihin Philippines Corporation (Keihin), a company engaged in the production of intake manifold and throttle body used in motor vehicles manufactured by Honda. On September 5, 2003, while Helen was about to leave the company premises, she saw a packing tape near her work area and placed it inside her bag because it would be useful in her transfer of residence. When the lady guard on duty inspected Helens bag, she found the packing tape inside her bag. The following day,respondent company issued a show cause notice7 to Helen accusing her of violating F.2 of the companys Code of Conduct, which says, "Any act constituting theft or robbery, or any attempt to commit theft or robbery, of any company property or other associates property. Penalty: D (dismissal)." Paul Cupon, Helens supervisor, called her to his office and directed her to explain in writing why no disciplinary action should be taken against her. Helen admitted the offense and even manifested that she would accept whatever penalty would be imposed upon her. She, however, did not reckon that respondent company would terminate her services for her admitted offense. On September 26, 2003, Helen received a notice of disciplinary action informing her that Keihin has decided to terminate her services on the ground of serious misconduct. On October 15, 2003, petitioners filed a complaintagainst respondent for illegal dismissal, non-payment of 13th month pay, with a prayer for reinstatement and payment of full back wages, as well as moral and exemplary damages. Issues: Whether or not the dismissal was illegal Ruling: I. SHE WAS GUILTY OF SERIOUS MISCONDUCT The petitioners argue that serious misconduct under existing law and jurisprudence could not be attributed to Helen because she was not motivated by malicious intent. it was Helens honest belief that the tape she took was of no use or value and that she did not hide the same. Misconduct is defined as "the transgression of some established and definite rule of action, a forbidden act, a dereliction of duty, willful in

Page 159

LABOR RELATIONS
Atty. Jefferson M. Marquez
character, and implies wrongful intent and not mere error in judgment." For serious misconduct to justify dismissal under the law, "(a) it must be serious, (b) must relate to the performance of the employees duties; and (c) must show that the employee has become unfit to continue working for the employer. It is noteworthy that prior to this incident, respondent issued two memoranda implementing an intensive inspection procedure and reminding all employees that those who will be caught stealing and performing acts of vandalism will be dealt with in accordance with the companys Code of Conduct. This was due to some problems of vandalism and loss in the company. The petitioners also argue that the penalty of dismissal is too harsh and disproportionate to the offense committed since the value of the thing taken is very minimal. Petitioners cite the case of Caltex Refinery Employees Association v. National Labor Relations Commission which involved the theft a bottle of lighter fluid. The supreme court then refrained from imposing the supreme penalty of dismissal since the employee had no violations "in his eight years of service and the value of the lighter fluid is very minimal compared to his salary. However, supreme court says that the case at bar is different from the Caltex case for two reasons: (1) In the Caltex case, the employee had been employed for eight years without incident and (2) is that respondent company was dealing with several cases of theft, vandalism, and loss of company and employees property when the incident involving Helen transpired. Petition is Denied FINAL NOTE: Supreme Court very briefly dealt with the defense of absence of due process which the court brushed aside by saying: With regard to the requirement of a hearing, the essence of due process lies in an opportunity to be heard. Such opportunity was afforded the petitioner when she was asked to explain her side of the story. In Metropolitan Bank and Trust Company v. Barrientos, we held that, "the essence of due process lies simply in an opportunity to be heard, and not that an actual hearing should always and indispensably be held." Similarly in Philippine Pasay Chung Hua Academy v. Edpan, we held that, "[e]ven if no hearing or conference was conducted, the requirement of due process had been met since he was accorded a chance to explain his side of the controversy."

59. Garcia v. Molina, G.R. No. 157383, August 18, 2010 Facts: Respondents Molina and Velasco, both Attorney V of the GSIS, received two separate Memoranda from petitioner charging them with grave misconduct. Specifically, Molina was charged for allegedly committing the following acts: 1) directly and continuously helping some alleged disgruntled employees to conduct concerted protest actions and/or illegal assemblies against the management and the GSIS President and General Manager; 2) leading the concerted protest activities held in the morning of May 22, 2002 during office hours within the GSIS compound; and 3) continuously performing said activities despite warning from his immediate superiors. In addition to the charge for grave misconduct for performing the same acts as Molina, Velasco was accused of performing acts in violation of the Rules on Office Decorum for leaving his office without informing his supervisor of his whereabouts; and gross insubordination for persistently disregarding petitioners instructions that Velasco should report to the petitioners office. These acts, according to petitioner, were committed in open betrayal of the confidential nature of their positions and in outright defiance of the Rules and Regulations on Public Sector Unionism. Considering the gravity of the charges against them, petitioner ordered the preventive suspension of respondents for ninety (90) days without pay, effective immediately. The following day, a committee was constituted to investigate the charges against respondents. Respondents filed with the Civil Service Commission (CSC) an Urgent Petition to Lift Preventive Suspension Order. They contended that the acts they allegedly committed were arbitrarily characterized as grave misconduct. Consistent with their stand that petitioner could not act as the complainant, prosecutor and judge at the same time, respondents filed with the CSC a Petition to Transfer Investigation to This Commission. Despite their urgent motions, the CSC failed to resolve respondents motions to lift preventive suspension order and to transfer the case from the GSIS to the CSC. Respondents filed with the CA a special civil action for certiotari and prohibition with prayer for Temporary Restraining Order (TRO). Respondents sought the annulment and setting aside of petitioners order directing the former to submit to the jurisdiction of the committee created to hear and investigate the administrative case filed against them. The CA rendered a decision in favor of respondents: Public respondents are hereby PERPETUALLY RESTRAINED from hearing and investigating the administrative case against petitioners, without prejudice to pursuing the same with the Civil Service Commission or any other agency of government as may be allowed for (sic) by law. Aggrieved, petitioner comes before the Court in this petition for review on certiorari under Rule 45 of the Rules of Court. In the meantime, the CSC resolved respondents Petition to Lift Order of Preventive Suspension and Petition to Transfer Investigation to the Commission. As to the lifting of the order of preventive suspension, the CSC considered the issue moot and academic considering that the period had lapsed and respondents had been allowed to resume their specific functions. This notwithstanding, the CSC opted to discuss the matter by way of obiter dictum. Without making a definitive conclusion as to the effect thereof in the case against respondents, the CSC declared that a preliminary investigation is a pre-requisite condition to the issuance of a formal charge. Aggrieved, respondents appealed to the CA

Page 160

LABOR RELATIONS
Atty. Jefferson M. Marquez
through a Petition for Review under Rule 43 of the Rules of Court. The CA rendered a Decision in favor of respondents. The CA declared null and void respondents formal charges for lack of the requisite preliminary investigation. In view thereof, the CA disagreed with the CSC that the question on the propriety of the preventive suspension order had become moot and academic. Rather, it concluded that the same is likewise void having emanated from the void formal charges. Consequently, the CA found that respondents were entitled to back salaries during the time of their illegal preventive suspension. Issue: Whether the preventive suspension orders issued against respondents Molina and Velasco are valid, well-founded and duly recognized by law. Ruling: The petitions are without merit. DENIED (G.R. No. 157383)/DISMISSED (G.R. No. 174137) It is likewise undisputed that the formal charges were issued without preliminary or fact-finding investigation. The filing by petitioner of formal charges against the respondents without complying with the mandated preliminary investigation or at least give the respondents the opportunity to comment violated the latter's right to due process. Hence, the formal charges are void ab initio and may be assailed directly or indirectly at anytime. Although administrative procedural rules are less stringent and often applied more liberally, administrative proceedings are not exempt from basic and fundamental procedural principles, such as the right to due process in investigations and hearings. 37 In particular, due process in administrative proceedings has been recognized to include the following: (1) the right to actual or constructive notice to the institution of proceedings which may affect a respondent's legal rights; (2) a real opportunity to be heard personally or with the assistance of counsel, to present witnesses and evidence in one's favor, and to defend one's rights; (3) a tribunal vested with competent jurisdiction and so constituted as to afford a person charged administratively a reasonable guarantee of honesty as well as impartiality; and (4) a finding by said tribunal which is supported by substantial evidence submitted for consideration during the hearing or contained in the records or made known to the parties affected.38 In the procedure adopted by petitioner, respondents were preventively suspended in the same formal charges issued by the former without the latter knowing that there were pending administrative cases against them. It is true that prior notice and hearing are not required in the issuance of a preventive suspension order.41 However, considering that respondents were preventively suspended in the same formal charges that we now declare null and void, then their preventive suspension is likewise null and void. Lastly, the CA committed no reversible error in ordering the payment of back salaries during the period of respondents preventive suspension. As the administrative proceedings involved in this case are void, no delinquency or misconduct may be imputed to respondents and the preventive suspension meted them is baseless. Consequently, respondents should be awarded their salaries during the period of their unjustified suspension.42 In granting their back salaries, we are simply repairing the damage that was unduly caused respondents, and unless we can turn back the hands of time, we can do so only by restoring to them that which is physically feasible to do under the circumstances.43 The principle of "no work, no pay" does not apply where the employee himself was unlawfully forced out of job.

60. Escario v. NLRC, G.R. No. 160302, September 27, 2010 Facts: The petitioners were among the regular employees of respondent Pinakamasarap Corporation (PINA), a corporation engaged in manufacturing and selling food seasoning. They were members of petitioner Malayang Samahan ng mga Manggagawa sa Balanced Foods (Union). At 8:30 in the morning of March 13, 1993, all the officers and some 200 members of the Union walked out of PINAs premises and proceeded to the barangay office to show support for Juanito Caete, an officer of the Union charged with oral defamation by Aurora Manor, PINAs personnel manager, and Yolanda Fabella, Manors secretary. It appears that the proceedings in the barangay resulted in a settlement, and the officers and members of the Union all returned to work thereafter. As a result of the walkout, PINA preventively suspended all officers of the Union because of the March 13, 1993 incident. PINA terminated the officers of the Union after a month. On April 14, 1993, PINA filed a complaint for unfair labor practice (ULP) and damages. The complaint was assigned to then Labor Arbiter Raul Aquino, who ruled in his decision dated July 13, 1994 that the March 13, 1993 incident was an illegal walkout constituting ULP; and that all the Unions officers, except Caete, had thereby lost their employment.

Page 161

LABOR RELATIONS
Atty. Jefferson M. Marquez
On April 28, 1993, the Union filed a notice of strike, claiming that PINA was guilty of union busting through the constructive dismissal of its officers. On May 9, 1993, the Union held a strike vote, at which a majority of 190 members of the Union voted to strike. The strike was held in the afternoon of June 15, 1993. PINA retaliated by charging the petitioners with ULP and abandonment of work, stating that they had violated provisions on strike of the collective bargaining agreement (CBA), such as: (a) sabotage by the insertion of foreign matter in the bottling of company products; (b) decreased production output by slowdown; (c) serious misconduct, and willful disobedience and insubordination to the orders of the Management and its representatives; (d) disruption of the work place by invading the premises and perpetrating commotion and disorder, and by causing fear and apprehension; (e) abandonment of work since June 28, 1993 despite notices to return to work individually sent to them; and (f) picketing within the company premises on June 15, 1993 that effectively barred with the use of threat and intimidation the ingress and egress of PINAs officials, employees, suppliers, and customers. On August 18, 1998, Labor Arbiter Jose G. de Vera (LA) rendered a decision, to wit: WHEREFORE, all the foregoing premises being considered, judgment is hereby rendered declaring the subject strike to be illegal. On appeal, the NLRC sustained the finding that the strike was illegal, but reversed the LAs ruling that there was abandonment, viz: However, we disagree with the conclusion that respondents union members should be considered to have abandoned their employment. On August 18, 2003, the CA affirmed the NLRC. In denying the petitioners claim for full backwages, the CA applied the third paragraph of Article 264(a) instead of Article 279 of the Labor Code, explaining that the only instance under Article 264 when a dismissed employee would be reinstated with full backwages was when he was dismissed by reason of an illegal lockout; that Article 264 was silent on the award of backwages to employees participating in a lawful strike; and that a reinstatement with full backwages would be granted only when the dismissal of the petitioners was not done in accordance with Article 282 (dismissals with just causes) and Article 283 (dismissals with authorized causes) of the Labor Code. Issue: The petitioners posit that they are entitled to full backwages from the date of dismissal until the date of actual reinstatement due to their not being found to have abandoned their jobs. They insist that the CA decided the question in a manner contrary to law and jurisprudence. The petitioners argue that the finding of no abandonment equated to a finding of illegal dismissal in their favor. Hence, they were entitled to full backwages. Ruling: The petitioners argument cannot be sustained. The petitioners participation in the illegal strike was precisely what prompted PINA to file a complaint to declare them, as striking employees, to have lost their employment status. However, the NLRC ultimately ordered their reinstatement after finding that they had not abandoned their work by joining the illegal strike. They were thus entitled only to reinstatement, regardless of whether or not the strike was the consequence of the employers ULP, considering that a strike was not a renunciation of the employment relation. As a general rule, backwages are granted to indemnify a dismissed employee for his loss of earnings during the whole period that he is out of his job. Considering that an illegally dismissed employee is not deemed to have left his employment, he is entitled to all the rights and privileges that accrue to him from the employment. The grant of backwages to him is in furtherance and effectuation of the public objectives of the Labor Code, and is in the nature of a command to the employer to make a public reparation for his illegal dismissal of the employee in violation of the Labor Code. That backwages are not granted to employees participating in an illegal strike simply accords with the reality that they do not render work for the employer during the period of the illegal strike. According to G&S Transport Corporation v. Infante: With respect to backwages, the principle of a "fair days wage for a fair days labor" remains as the basic factor in determining the award thereof. If there is no work performed by the employee there can be no wage or pay unless, of course, the laborer was able, willing and ready to work but was illegally locked out, suspended or dismissed or otherwise illegally prevented from working. xxx In Philippine Marine Officers Guild v. Compaia Maritima, as affirmed in Philippine Diamond Hotel and Resort v. Manila Diamond Hotel Employees Union, the Court stressed that for this exception to apply, it is required that the strike be legal, a situation that does not obtain in the case at bar. (emphasis supplied) The petitioners herein do not deny their participation in the June 15, 1993 strike. As such, they did not suffer any loss of earnings during their absence from work. Their reinstatement sans backwages is in order, to conform to the policy of a fair days wage for a fair days labor.

Page 162

LABOR RELATIONS
Atty. Jefferson M. Marquez
Under the principle of a fair days wage for a fair days labor, the petitioners were not entitled to the wages during the period of the strike (even if the strike might be legal), because they performed no work during the strike. Verily, it was neither fair nor just that the dismissed employees should litigate against their employer on the latters time. Thus, the Court deleted the award of backwages and held that the striking workers were entitled only to reinstatement in Philippine Diamond Hotel and Resort, Inc. (Manila Diamond Hotel) v. Manila Diamond Hotel Employees Union, considering that the striking employees did not render work for the employer during the strike.

61. Simizu Phils Contractors v. Callanta, G.R. No. 165923, September 29, 2010 Facts: Shimizu Phils. a corporation engaged in the construction business, employed Virgilio Callanta on August 23, 1994 as Safety Officer assigned at Yutaka-Giken Project and eventually as Project Administrator of petitioners Structural Steel Division (SSD) in 1995. Virgilio Callanta was informed that his services will be terminated effective July 9, 1997 due to the lack of any vacancy in other projects and the need to re-align the companys personnel requirements brought about by the imperatives of maximum financial commitments. He then filed an illegal dismissal complaint against petitioner assailing his dismissal as without any valid cause. Shimizu advanced that respondents services was terminated in accordance with a valid retrenchment program being implemented by the company since 1996 due to financial crisis that plague the construction industry. To prove its financial deficit, petitioner presented financial statements for the years 1995 to 1997 as well as the Securities and Exchange Commissions approval of petitioners application for a new paidin capital amounting to P330,000,000. Shimizu alleged that in order not to jeopardize the completion of its projects, the abolition of several departments and the concomitant termination of some employees were implemented as each project is completed. When respondents Honda Project was completed, petitioner offered respondent his separation pay which the latter refused to accept and instead filed an illegal dismissal complaint. Mr. Callanta claimed that Shimizu failed to comply with the requirements called for by law before implementing a retrenchment program thereby rendering it legally infirmed. First, it did not comply with the provision of the Labor Code mandating the service of notice of retrenchment. He pointed out that the notice sent to him never mentioned retrenchment but only project completion as the cause of termination. Also, the notice sent to the Department of Labor and Employment (DOLE) did not conform to the 30-day prior notice requirement. Second, petitioner failed to use fair and reasonable criteria in determining which employees shall be retrenched or retained. In the termination report submitted to DOLE, he was the only one dismissed out of 333 employees. Worse, junior and inexperienced employees were appointed/assigned in his stead to new projects thus also ignoring seniority in hiring and firing employees. Shimizu argued that when it submitted the retrenchment notice/termination report to DOLE, there was already substantial compliance with the requirement. Labor Arbiter rendered a decision holding that Mr. Callanta was validly retrenched. He found that sufficient evidence was presented to establish company losses; that petitioner offered respondent his separation pay; and that petitioner duly notified DOLE about the retrenchment. The Labor Arbiter further relied on petitioners factual version relating to respondents employment background with regard to his position and behavioral conduct. NLRC upheld the ruling that there was valid ground for respondents termination but modified the Labor Arbiters Decision by holding that petitioner violated respondents right to procedural due process. The NLRC found that petitioner failed to comply with the 30-day prior notice to the DOLE and that there is no proof that petitioner used fair and reasonable criteria in the selection of employees to be retrenched. Shimizu Philippine Contractor, Inc., is ordered to pay complainant-appellant Virgilio P. Callanta his separation pay equivalent to one (1) month pay for every year of service. For want of due notice, respondent is further directed to pay complainant an indemnity equivalent to one (1) month salary. CA reversed and set aside the NLRCs ruling. The CA opined that Shimizu failed to prove that there were employees other than respondent who were similarly dismissed due to retrenchment and that respondents alleged replacements held much higher ranks and were more deserving employees. Moreover, there were no proofs to sustain that petitioner used fair and reasonable criteria in determining which employees to retrench. According to the CA, petitioners failure to produce evidence raises the presumption that such evidence will be adverse to it. Consequently, the CA invalidated the retrenchment, held respondent to have been illegally dismissed, and ordered respondents reinstatement and payment of backwages. Issue:

Page 163

LABOR RELATIONS
Atty. Jefferson M. Marquez
Whether or not Shimizu has failed to obeserve fair and reasonable standards or criteria in effecting the dismissal or Mr. Callanta? Ruling: There was substantial compliance for a valid retrenchment; Shimizu used fair and reasonable criteria in effecting retrenchment. As an authorized cause for separation from service under Article 283 of the Labor Code, retrenchment is a valid exercise of management prerogative subject to the strict requirements set by jurisprudence: (1) That the retrenchment is reasonably necessary and likely to prevent business losses which, if already incurred, are not merely de minimis, but substantial, serious, actual and real, or if only expected, are reasonably imminent as perceived objectively and in good faith by the employer; (2) That the employer served written notice both to the employees and to the Department of Labor and Employment at least one month prior to the intended date of retrenchment; (3) That the employer pays the retrenched employees separation pay equivalent to one month pay or at least month pay for every year of service, whichever is higher; (4) That the employer exercises its prerogative to retrench employees in good faith for the advancement of its interest and not to defeat or circumvent the employees right to security of tenure; and (5) That the employer used fair and reasonable criteria in ascertaining who would be dismissed and who would be retained among the employees, such as status, efficiency, seniority, physical fitness, age, and financial hardship for certain workers. Both the Labor Arbiter and the NLRC found sufficient compliance with these substantive requirements, there being enough evidence to prove that petitioner was sustaining business losses, that separation pay was offered to respondent, and that notices of termination of service were furnished respondent and DOLE. However, the NLRC modified the Decision of the Labor Arbiter by granting respondent indemnity since the notice to DOLE was served short of the 30-day notice requirement and that there is no proof of the use of fair and reasonable criteria in the selection of employees to be retrenched or retained. The CA, then, reversed the Decision of the NLRC by ruling that the absence of fair and reasonable criteria in implementing the retrenchment invalidates altogether the retrenchment. In implementing its retrenchment scheme, petitioner was constrained to streamline its operations and to downsize its complements in a progressive manner in order not to jeopardize the completion of its projects. Thus, several departments like the Civil Works Division, Electromechanical Works Division and the Territorial Project Management Offices, among others, were abolished in the early part of 1996 and thereafter the Structural Steel Division, of which respondent was an Administrator. Respondent was among the last batch of employees who were retrenched and by the end of year 1997, all of the employees of the Structural Steel Division were severed from employment. Mr. Callanta argued that that he was singled out for termination as allegedly shown in petitioners monthly termination report for the month of July 1997 filed with the DOLE does not persuade this Court. Standing alone, this document is not proof of the total number of retrenched employees or that respondent was the only one retrenched. It merely serves as notice to DOLE of the names of employees terminated/ retrenched only for the month of July. In other words, it cannot be deemed as an evidence of the number of employees affected by the retrenchment program. Thus we cannot conclude that no other employees were previously retrenched. Shimizu implemented its retrenchment program in good faith because it undertook several measures in cutting down its costs, to wit, withdrawing certain privileges of petitioners executives and expatriates; limiting the grant of additional monetary benefits to managerial employees and cutting down expenses; selling of company vehicles; and infusing fresh capital into the company. Respondent did not attempt to refute that petitioner adopted these measures before implementing its retrenchment program. Shimizu was able to prove that it incurred substantial business losses, that it offered to pay respondent his separation pay, that the retrenchment scheme was arrived at in good faith, and lastly, that the criteria or standard used in selecting the employees to be retrenched was work efficiency which passed the test of fairness and reasonableness. The termination notice sent to DOLE did not comply with the 30-day notice requirement, thus, respondent is entitled to indemnity for violation of due process. However, petitioner admitted that the reports were submitted 21 days, in the case of the first notice, and 16 days, in the case of the second notice, before the intended date of respondents dismissal. The purpose of the one month prior notice rule is to give DOLE an opportunity to ascertain the veracity of the cause of termination. Noncompliance with this rule clearly violates the employees right to statutory due process. Consequently, we affirm the NLRCs award of indemnity to respondent for want of sufficient due notice. But to be consistent with our ruling in Jaka Food Processing Corporation v. Pacot, the indemnity in the form of nominal damages should be fixed in the amount of P50,000.00. WHERFORE, the petition is GRANTED., respectively, upholding the legality of respondents dismissal and awarding him separation pay equivalent to one (1) month pay or one-half (1/2) month pay for every year of service, whichever is higher,

Page 164

LABOR RELATIONS
Atty. Jefferson M. Marquez
are REINSTATED and AFFIRMED with MODIFICATION that the indemnity to be awarded to respondent is fixed in the amount of P50,000.00 as nominal damages. SO ORDERED.

62. Solidbank Corporation v. Gamier, G.R. No. 159461, November 15, 2010 Facts: Sometime in October 1999, petitioner Solidbank and respondent Solidbank Employees Union (Union) were set to renegotiate the economic provisions of their 1997-2001 Collective Bargaining Agreement (CBA) to cover the remaining two years thereof. Negotiations commenced on November 17, 1999 but seeing that an agreement was unlikely, the Union declared a deadlock on December 22, 1999 and filed a Notice of Strike on December 29, 1999. During the collective bargaining negotiations, some Union members staged a series of mass actions. In view of the impending actual strike, then Secretary of Labor and Employment Bienvenido E. Laguesma assumed jurisdiction over the labor dispute, pursuant to Article 263 (g) of the Labor Code, as amended. The assumption order dated January 18, 2000 directed the parties "to cease and desist from committing any and all acts that might exacerbate the situation. Dissatisfied with the Secretarys ruling, the Union officers and members decided to protest the same by holding a rally infront of the Office of the Secretary of Labor and Employment in Intramuros, Manila, simultaneous with the filing of their motion for reconsideration of the March 24, 2000 Order. Thus, on April 3, 2000, an overwhelming majority of employees, including the individual respondents, joined the "mass leave" and "protest action" at the Department of Labor and Employment (DOLE) office while the banks provincial branches in Cebu, Iloilo, Bacolod and Naga followed suit and "boycotted regular work. The union members also picketed the banks Head Office in Binondo on April 6, 2000, and Paseo de Roxas branch on April 7, 2000. As a result of the employees concerted actions, Solidbanks business operations were paralyzed. On the same day, then President of Solidbank, Deogracias N. Vistan, issued a memorandum addressed to all employees calling their absence from work and demonstration infront of the DOLE office as an illegal act, and reminding them that they have put their jobs at risk as they will be asked to show cause why they should not be terminated for participating in the union-instigated concerted action. The employees work abandonment/boycott lasted for three days, from April 3 to 5, 2000. On the third day of the concerted work boycott (April 5, 2000), Vistan issued another memorandum, this time declaring that the bank is prepared to take back employees who will report for work starting April 6, 2000 "provided these employees were/are not part of those who led or instigated or coerced their co-employees into participating in this illegal act." Out of the 712 employees who took part in the three-day work boycott, a total of 513 returned to work and were accepted by the bank. The remaining 199 employees insisted on defying Vistans directive, which included herein respondents Ernesto U. Gamier, Elena R. Condevillamar, Janice L. Arriola and Ophelia C. De Guzman. For their failure to return to work, the said 199 employees were each issued a show-cause memo directing them to submit a written explanation within twenty-four (24) hours why they should not be dismissed for the "illegal strike in defiance of the Assumption Order of the Secretary of Labor resulting to grave and irreparable damage to the Bank", and placing them under preventive suspension. Gamier, Condevillamar, Arriola and De Guzman filed complaints for illegal dismissal, moral and exemplary damages and attorneys fees. Issue: Whether the respondents were validly terminated? Ruling: The Court has consistently ruled that once the Secretary of Labor assumes jurisdiction over a labor dispute, such jurisdiction should not be interfered with by the application of the coercive processes of a strike or lockout. A strike that is undertaken despite the issuance by the Secretary of Labor of an assumption order and/or certification is a prohibited activity and thus illegal. Article 264 (a) of the Labor Code, as amended, also considers it a prohibited activity to declare a strike "during the pendency of cases involving the same grounds for the same strike." There is no dispute that when respondents conducted their mass actions on April 3 to 6, 2000, the proceedings before the Secretary of Labor were still pending as both parties filed motions for reconsideration of the March 24, 2000 Order. Clearly, respondents knowingly violated the aforesaid provision by holding a strike in the guise of mass demonstration simultaneous with concerted work abandonment/boycott. However, a worker merely participating in an illegal strike may not be terminated from employment. It is only when he commits illegal acts during a strike that he may be declared to have lost employment status. The Court have held that the responsibility of union officers, as main players in an illegal strike, is greater than that of the members and, therefore, limiting the penalty of dismissal only for the former for participation in an

Page 165

LABOR RELATIONS
Atty. Jefferson M. Marquez
illegal strike is in order. For the respondents who are union members, the rule is that an ordinary striking worker cannot be terminated for mere participation in an illegal strike. There must be proof that he or she committed illegal acts during a strike. In all cases, the striker must be identified. But proof beyond reasonable doubt is not required. Substantial evidence available under the attendant circumstances, which may justify the imposition of the penalty of dismissal, may suffice. Liability for prohibited acts is to be determined on an individual basis. Petitioners have not adduced evidence on such illegal acts committed by each of the individual respondents who are union members. Instead, petitioners simply point to their admitted participation in the mass actions which they knew to be illegal, being in violation of the Secretarys assumption order. The dismissal of herein respondent-union members are therefore unjustified in the absence of a clear showing that they committed specific illegal acts during the mass actions and concerted work boycott. The award of backwages is a legal consequence of a finding of illegal dismissal. Assuming that respondent-union members have indeed reported back to work at the end of the concerted mass actions, but were soon terminated by petitioners who found their explanation unsatisfactory, they are not entitled to backwages in view of the illegality of the said strike. Under the circumstances, respondents reinstatement without backwages suffices for the appropriate relief. But since reinstatement is no longer possible, given the lapse of considerable time from the occurrence of the strike, not to mention the fact that Solidbank had long ceased its banking operations, the award of separation pay of one (1) month salary for each year of service, in lieu of reinstatement, is in order.

63. Coca-Cola Export Corp. v. Gacayan, G.R. No. 149433, December 15, 2010 Facts: 1. Respondent Gacayan worked with petitioner for 9 years. She held the position of Senior Financial Accountant. Her employment was terminated for alleged loss of trust and confidence on the ground of fraudulently altering three receipts (1 Mdonalds receipt and 3 Shakeys Pizza receipts) which she submitted to support her claim for reimbursement of meal expenses. 2. The date of issuance of the Mdonalds receipt was altered. In her orders in Shakeys Pizza it was discovered that the receipt was actually for three orders of Bunch of Lunch, and not for Buddy Pack. 3. Respondent allegedly violated Section II, No. 15, paragraph (d) of the companys rules and regulations which punishes with dismissal the submission of any fraudulent item of expense. 4. She was dismissed for fraudulently submitting tampered and/or altered receipts in support of her petty cash reimbursements in gross violation of the companys rules and regulations. 5. She filed a complaint for illegal dismissal, non-payment of service incentive leave, sick leave and vacation leave with prayer for reinstatement, payment of backwages as well as for damages and attorneys fees, against petitioner. 6. The Labor Arbiter and NLRC ruled that the termination was valid but CA ruled that the penalty of dismissal imposed on respondent was too harsh and directed petitioner to immediately reinstate her. Issue: WON termination of Gacayan was valid Ruling: No, her termination was invalid. Respondents dismissal from employment was not grounded on any of the just causes enumerated under Article 282 of the Labor Code. Also, SC did not agree with petitioners contention that respondents repeated submission of altered or tampered receipts to support her claim for reimbursement constitutes a betrayal of the employers trust and confidence and a serious misconduct, thus, giving cause for the termination of her employment. It is important to note that the term trust and confidence is restricted to managerial employees. In Samson v. National Labor Relations Commission, the Court, citing Section 2(b), Rule I, Book III of the Omnibus Rules Implementing the Labor

Page 166

LABOR RELATIONS
Atty. Jefferson M. Marquez
Code, enumerated the conditions for one to be properly considered a managerial employee: (1) Their primary duty consists of the management of the establishment in which they are employed or of a department or sub-division thereof; (2) They customarily and regularly direct the work of two or more employees therein; [and] (3) They have the authority to hire or fire other employees of lower rank; or their suggestions and recommendations as to the hiring and firing and as to the promotion or any other change of status of other employees are given particular weight. In the instant case, respondent was the Senior Financial Accountant with the Job Description of a Financial Project Analyst. Respondent, among others, provides support in the form of financial analyses and evaluation of alternative strategies or action plans to assist management in strategic and operational decision-making, liaises with the Bottler to comply with Corporate Bottler financial reporting requirements and to ensure Bottlers plans are aligned with TCCECs, and assists management on various initiatives on ad hoc basis. In Nokom v. National Labor Relations Commission, this Court set the guidelines for the application of the doctrine of loss of confidence (a) Loss of confidence should not be simulated; (b) It should not be used as a subterfuge for causes which are improper, illegal or unjustified; (c) It may not be arbitrarily asserted in the face of overwhelming evidence to the contrary; and (d) It must be genuine, not a mere afterthought to justify earlier action taken in bad faith. In the instant case, the basis for terminating the employment of respondent was for gross violation of the companys rules and regulations, as specified in the termination letter. No mention was made regarding petitioners alleged loss of trust and confidence in respondent. Neither was there any explanation nor discussion of the alleged sensitive and delicate position of respondent requiring the utmost trust of petitioner. For loss of trust and confidence to be a valid ground for dismissal, it must be substantial and founded on clearly established facts. Loss of confidence must not be used as a subterfuge for causes which are improper, illegal or unjustified; it must be genuine, not a mere afterthought, to justify earlier action taken in bad faith. Because of its subjective nature, this Court has been very scrutinizing in cases of dismissal based on loss of trust and confidence because the same can easily be concocted by an abusive employer. Thus, when the breach of trust or loss of confidence theorized upon is not borne by clearly established facts, as in the instant case, such dismissal on the ground of loss and confidence cannot be countenanced. In the instant case, it was only in the Reply to Respondents Comment, that petitioner made mention of another ground for the dismissal of respondent, that of serious misconduct, when she submitted altered or tampered receipts to support her claim for reimbursement. Such allegation appears to be a mere afterthought, being tardily raised only in the Reply. The alleged infractions of respondent could hardly be considered serious misconduct. It is well to stress that in order to constitute serious misconduct which will warrant the dismissal of an employee, it is not sufficient that the act or conduct complained of has violated some established rules or policies. It is equally important and required that the act or conduct must have been done with wrongful intent. Such is, however, lacking in the instant case. In the instant case, petitioner alleged that under its rules and regulations, respondents submission of fraudulent items of expense is punishable by dismissal. However, petitioners rules cannot preclude the State from inquiring whether the strict and rigid application or interpretation thereof would be harsh to the employee. Even when an employee is found to have transgressed the employers rules, in the actual imposition of penalties upon the erring employee, due consideration must still be given to his length of service and the number of violations committed during his employ. Respondent had no previous record in her 9 years of service; this would have been her first offense. Respondent had also been a recipient of various commendations attesting to her competence and diligence in the performance of her duties, not only from petitioner, but also from petitioners counterparts in Poland and Thailand. Respondent also countered that she acted in good faith and with no wrongful intent when she submitted the receipts in support of her claim for reimbursement of meal allowance. According to respondent, only the dates or items were altered on the receipts. She did not claim more than what was allowed as meal expense for the days that she rendered overtime work. She believed that the submission of receipts was simply for records-keeping, since she actually rendered overtime work on the dates that she claimed for meal allowance. All told, this Court holds that the penalty of dismissal imposed on respondent is unduly oppressive and disproportionate to the infraction which she committed. A lighter penalty would have been more just.

Page 167

LABOR RELATIONS
Atty. Jefferson M. Marquez
Under Article 279 of the Labor Code, an employee who is unjustly dismissed from work shall be entitled to reinstatement without loss of seniority rights and other privileges and to his full backwages, inclusive of allowances, and to his other benefits or their monetary equivalent computed from the time his compensation was withheld from him up to the time of his actual reinstatement and not from the date of her dismissal from the service up to the date of finality of this decision as the CA ruled. SC ruled in favor of Gacayan. Petitioner was ordered to reinstate her.

64. Robinsons Galleria/Robinsons Supermarket v. Ranchez, G.R. No. 177937, January 19, 2011 Facts: Respondent was a probationary employee of petitioner Robinsons Galleria/Robinsons Supermarket Corporation (petitioner Supermarket) for a period of five (5) months. She underwent six (6) weeks of training as a cashier before she was hired as such on October 15, 1997. Two weeks after she was hired, respondent reported to her supervisor the loss of cash amounting to Twenty Thousand Two Hundred NinetyNine Pesos (P20,299.00) which she had placed inside the company locker. Petitioner Jess Manuel (petitioner Manuel), the Operations Manager of petitioner Supermarket, ordered that respondent be strip-searched by the company guards. However, the search on her and her personal belongings yielded nothing. Respondent acknowledged her responsibility and requested that she be allowed to settle and pay the lost amount. However, petitioner Manuel did not heed her request and instead reported the matter to the police. Petitioner Manuel likewise requested the Quezon City Prosecutors Office for an inquest. Subsequently, an information for Qualified Theft was filed. Thus, respondent filed a complaint for illegal dismissal and damages. Issue: Whether respondent was illegally terminated from employment by petitioners? Ruling: There is probationary employment when the employee upon his engagement is made to undergo a trial period during which the employer determines his fitness to qualify for regular employment based on reasonable standards made known to him at the time of engagement. A probationary employee, like a regular employee, enjoys security of tenure. However, in cases of probationary employment, aside from just or authorized causes of termination, an additional ground is provided under Article 281 of the Labor Code, i.e., the probationary employee may also be terminated for failure to qualify as a regular employee in accordance with reasonable standards made known by the employer to the employee at the time of the engagement. Thus, the services of an employee who has been engaged on probationary basis may be terminated for any of the following: (1) a just or (2) an authorized cause; and (3) when he fails to qualify as a regular employee in accordance with reasonable standards prescribed by the employer. Article 277(b) of the Labor Code mandates that subject to the constitutional right of workers to security of tenure and their right to be protected against dismissal, except for just and authorized cause and without prejudice to the requirement of notice under Article 283 of the same Code, the employer shall furnish the worker, whose employment is sought to be terminated, a written notice containing a statement of the causes of termination, and shall afford the latter ample opportunity to be heard and to defend himself with the assistance of a representative if he so desires, in accordance with company rules and regulations pursuant to the guidelines set by the Department of Labor and Employment. In the instant case, based on the facts on record, petitioners failed to accord respondent substantive and procedural due process. The haphazard manner in the investigation of the missing cash, which was left to the determination of the police authorities and the Prosecutors Office, left respondent with no choice but to cry foul. Administrative investigation was not conducted by petitioner Supermarket. On the same day that the missing money was reported by respondent to her immediate superior, the company already pre-judged her guilt without proper investigation, and instantly reported her to the police as the suspected thief, which resulted in her languishing in jail for two weeks. As correctly pointed out by the NLRC, the due process requirements under the Labor Code are mandatory and may not be supplanted by police investigation or court proceedings. The criminal aspect of the case is considered independent of the administrative aspect. Thus, employers

Page 168

LABOR RELATIONS
Atty. Jefferson M. Marquez
should not rely solely on the findings of the Prosecutors Office. They are mandated to conduct their own separate investigation, and to accord the employee every opportunity to defend himself. Furthermore, respondent was not represented by counsel when she was strip-searched inside the company premises or during the police investigation, and in the preliminary investigation before the Prosecutors Office. As to respondents monetary claims, Article 279 of the Labor Code provides that an employee who is unjustly dismissed from work shall be entitled to reinstatement without loss of seniority rights and other privileges, to full backwages, inclusive of allowances, and to other benefits or their monetary equivalent computed from the time his compensation was withheld from him up to the time of his actual reinstatement. However, due to the strained relations of the parties, the payment of separation pay has been considered an acceptable alternative to reinstatement, when the latter option is no longer desirable or viable. Thus, as an illegally or constructively dismissed employee, respondent is entitled to: (1) either reinstatement, if viable, or separation pay, if reinstatement is no longer viable; and (2) backwages. These two reliefs are separate and distinct from each other and are awarded conjunctively. In this case, since respondent was a probationary employee at the time she was constructively dismissed by petitioners, she is entitled to separation pay and backwages. Reinstatement of respondent is no longer viable considering the circumstances. However, the backwages that should be awarded to respondent shall be reckoned from the time of her constructive dismissal until the date of the termination of her employment. In all cases involving employees engaged on probationary basis, the employer shall make known to its employees the standards under which they will qualify as regular employees at the time of their engagement. Where no standards are made known to an employee at the time, he shall be deemed a regular employee, unless the job is self-descriptive, like maid, cook, driver, or messenger.

65. Hospital Management Services v. HMSI-Medical Center Manila Employees Asso., G.R. No. 176287, January 31, 2011 Facts: Respondent De Castro started working as a staff nurse at petitioner hospital since September 28, 1990, until she was dismissed on July 20, 1999. Between 2:00 a.m. to 3:00 a.m. of March 24, 1999, while respondent De Castro and ward-clerk orientee Gina Guillergan were at the nurse station on night duty (from 10:00 p.m. of March 23, 1999 to 6:00 a.m. of March 24, 1999), one Rufina Causaren, an 81-year-old patient confined at Room 724-1 of petitioner hospital for gangrenous wound on her right anterior leg and right forefoot and scheduled for operation on March 26, 1999, fell from the right side of the bed as she was trying to reach for the bedpan. Because of what happened, the niece of patient Causaren staying in the room was awakened and she sought assistance from the nurse station. Instead of personally seeing the patient, respondent De Castro directed ward-clerk orientee Guillergan to check the patient. The vital signs of the patient were normal. Later, the physician on duty and the nursing staff on duty for the next shift again attended to patient Causaren. Chief Nurse Josefina M. Villanueva informed Dr. Asuncion Abaya-Morido, president and hospital director, about the incident and requested for a formal investigation. On May 11, 1999, the legal counsel of petitioner hospital directed respondent De Castro and three other nurses on duty, Staff Nurse Janith V. Paderes and Nursing Assistants Marilou Respicio and Bertilla T. Tatad, to appear before the Investigation Committee. De Castro explained that at around 2:30 a.m. to 3:00 a.m., she was attending to a newly-admitted patient at Room 710 and, because of this, she instructed Nursing Assistant Tatad to check the vital signs of patient Causaren, with ward-clerk orientee Guillergan accompanying the latter. When the two arrived at the room, the patient was in a squatting position, with the right arm on the bed and the left hand holding on to a chair. In the Investigation Report, the Investigation Committee found that the subject incident happened between 11:00 a.m. to 11:30 a.m. of March 23, 1999. The three other nurses for the shift were not at the nurse station. Staff Nurse Paderes was then in another nurse station encoding the medicines for the current admissions of patients, while Nursing Assistant Respicio was making the door name tags of admitted patients and Nursing Assistant Tatad delivered some specimens to the laboratory. The committee recommended that despite her more than seven years of service, respondent De Castro should be terminated from employment for her lapse in responding to the incident and for trying to manipulate and influence her staff to cover-up the incident. As for Staff Nurse Paderes and Nursing Assistants Respicio and Tatad, the committee recommended that they be issued warning notices for failure to note the incident and endorse it to the next duty shift and, although they did not have any knowledge of the incident, they should be reminded not to succumb to pressure from their superiors in distorting the facts. On July 5, 1999, Janette A. Calixijan, HRD Officer of petitioner hospital, issued a notice of termination, duly noted by Dr. Abaya-Morido, upon respondent De Castro, effective at the close of office hours of July 20, 1999, for alleged violation of company rules and regulations, particularly

Page 169

LABOR RELATIONS
Atty. Jefferson M. Marquez
paragraph 16 (a), Item 3, Chapter XI of the Employee's Handbook and Policy Manual of 1996 (Employee's Handbook) (1) negligence to follow company policy on what to do with patient Rufina Causaren who fell from a hospital bed; (2) failure to record and refer the incident to the physician-[on- duty and] allow[ing] a significant lapse of time before reporting the incident; (3) deliberately instructing the staff to follow her version of the incident in order to cover up the lapse; and (4) negligence and carelessness in carrying out her duty as staff nurse-on-duty when the incident happened. On July 21, 1999, respondent De Castro, with the assistance of respondent Hospital Management Services Inc.-Medical Center Manila Employees Association-AFW, filed a Complaint[7] for illegal dismissal against petitioners with prayer for reinstatement and payment of full backwages without loss of seniority rights, P20,000.00 moral damages, P10,000.00 exemplary damages, and 10% of the total monetary award as attorney's fees. The Labor Arbiter rendered a Decision,[8] ordering the hospital to reinstate respondent De Castro to her former position or by payroll reinstatement, at the option of the former, without loss of seniority rights, but without backwages and, also, directing petitioners to notify her to report to work. The Labor Arbiter concluded that although respondent De Castro committed the act complained of, being her first offense, the penalty to be meted should not be dismissal from the service, but merely 7 to 14 days suspension as the same was classified as a less serious offense under the Employees Handbook. On appeal by respondent De Castro, the NLRC rendered a Decision reversing the findings of the Labor Arbiter and dismissing the complaint against the petitioners. It observed that respondent De Castro lacked diligence and prudence in carrying out her duty when, instead of personally checking on the condition of patient Causaren after she fell from the bed, she merely sent ward-clerk orientee Guillergan to do the same in her behalf and for influencing her staff to conceal the incident. The NLRC denied respondent De Castro's Motion for Reconsideration. The CA reversed and set aside the Decision of the NLRC and reinstated the Decision of the Labor Arbiter, with modification that respondent De Castro should be entitled to payment of full backwages and other benefits, or their monetary equivalent, computed from the expiration of the 14day-suspension period up to actual reinstatement. The CA ruled that while respondent De Castro's failure to personally attend to patient Causeran amounted to misconduct, however, being her first offense, such misconduct could not be categorized as serious or grave that would warrant the extreme penalty of termination from the service after having been employed for almost 9 years. It added that the subject infraction was a less serious offense classified under commission of negligent or careless acts during working time or on company property that resulted in the personal injury or property damage causing expenses to be incurred by the company stated in subparagraph 11, paragraph 3 (B), Chapter XI [on the Rules on Discipline] of the Employee's Handbook[9] of petitioner hospital. The CA denied the hospitals MR. Issue: Was De Castro guilty of serious misconduct? Was the penalty of termination reasonable? Ruling: The hospital is guilty of illegal dismissal. Article 282 (b) of the Labor Code provides that an employer may terminate an employment for gross and habitual neglect by the employee of his duties. The CA ruled that per the Employees Handbook of petitioner hospital, respondent De Castros infraction is classified as a less serious offense for commission of negligent acts during working time as set forth in subparagraph 11, paragraph 3 (B) of Chapter XI[10] thereof. Petitioners anchor respondent De Castros termination of employment on the ground of serious misconduct for failure to personally attend to patient Causaren who fell from the bed as she was trying to reach for the bedpan. Based on her evaluation of the situation, respondent De Castro saw no necessity to record in the chart of patient Causaren the fact that she fell from the bed as the patient did not suffer any injury and her vital signs were normal. She surmised that the incident was not of a magnitude that would require medical intervention as even the patient and her niece did not press charges against her by reason of the subject incident. It is incumbent upon respondent De Castro to ensure that patients, covered by the nurse station to which she was assigned, be accorded utmost health care at all times without any qualification or distinction. Respondent De Castros failure to personally assist patient Causaren, check her vital signs and examine if she sustained any injury, refer the matter to the patient's attending physician or any physician-on-duty, and note the incident in the report sheet for endorsement to the next shift for proper monitoring constitute serious misconduct that warrants her termination of employment. After attending to the toxic patients under her area of responsibility, respondent De Castro should have immediately proceeded to check the health condition of patient Causaren and, if necessary, request the physician-on-duty to diagnose her further. More importantly, respondent De Castro should make everything of record in the patients chart as there might be a possibility that while the patient may appear to

Page 170

LABOR RELATIONS
Atty. Jefferson M. Marquez
be normal at the time she was initially examined, an injury as a consequence of her fall may become manifest only in the succeeding days of her confinement. The patients chart is a repository of ones medical history and, in this regard, respondent De Castro should have recorded the subject incident in the chart of patient Causaren so that any subsequent discomfort or injury of the patient arising from the incident may be accorded proper medical treatment. Neglect of duty, to be a ground for dismissal, must be both gross and habitual. Gross negligence connotes want of care in the performance of one's duties. Habitual neglect implies repeated failure to perform one's duties for a period of time, depending upon the circumstances. A single or isolated act of negligence does not constitute a just cause for the dismissal of the employee. Despite our finding of culpability against respondent De Castro; however, we do not see any wrongful intent, deliberate refusal, or bad faith on her part when, instead of personally attending to patient Causaren, she requested Nursing Assistant Tatad and ward-clerk orientee Guillergan to see the patient, as she was then attending to a newly-admitted patient at Room 710. It was her judgment call, albeit an error of judgment, being the staff nurse with presumably more work experience and better learning curve, to send Nursing Assistant Tatad and ward-clerk orientee Guillergan to check on the health condition of the patient, as she deemed it best, under the given situation, to attend to a newly-admitted patient who had more concerns that needed to be addressed accordingly. Being her first offense, respondent De Castro cannot be said to be grossly negligent so as to justify her termination of employment. Moreover, petitioners allegation, that respondent De Castro exerted undue pressure upon her co-nurses to alter the actual time of the incident so as to exculpate her from any liability, was not clearly substantiated. Negligence is defined as the failure to exercise the standard of care that a reasonably prudent person would have exercised in a similar situation. The Court emphasizes that the nature of the business of a hospital requires a higher degree of caution and exacting standard of diligence in patient management and health care as what is involved are lives of patients who seek urgent medical assistance. An act or omission that falls short of the required degree of care and diligence amounts to serious misconduct which constitutes a sufficient ground for dismissal. However, in some cases, the Court had ruled that sanctioning an erring employee with suspension would suffice as the extreme penalty of dismissal would be too harsh. Considering that this was the first offense of respondent De Castro in her nine (9) years of employment with petitioner hospital as a staff nurse without any previous derogatory record and, further, as her lapse was not characterized by any wrongful motive or deceitful conduct, the Court deems it appropriate that, instead of the harsh penalty of dismissal, she would be suspended for a period of six (6) months without pay, inclusive of the suspension for a period of 14 days which she had earlier served. Thereafter, petitioner hospital should reinstate respondent Edna R. De Castro to her former position without loss of seniority rights, full backwages, inclusive of allowances and other benefits, or their monetary equivalent, computed from the expiration of her suspension of six (6) months up to the time of actual reinstatement.

66. Culili v. Eastern Telecommunications Phils., G.R. No. 165381, February 9, 2011 Facts: Nelson Culili was employed by Eastern Telecommunications a Senior Technician. In 1998, due to business losses, ETPI was compelled to implement a Right-Sizing Program which consisted of two phases: the first phase involved the reduction of ETPIs workforce to only those employees that were necessary and which ETPI could sustain; the second phase entailed a company-wide reorganization which would result in the transfer, merger, absorption or abolition of certain departments of ETPI. Among the departments abolished was the Service Quality Department. As a result, Culilis position was abolished due to redundancy. Upon filing a complaint, the Labor Arbiter rendered a decision finding ETPI guilty of illegal dismissal and unfair labor practice, which was affirmed by the NLRC. However, the Court of Appeals found that Culilis position was validly abolished due to redundancy. It was highly unlikely that ETPI would effect a company-wide reorganization simply for the purpose of getting rid of Culili. Also, ETPI cannot be held guilty of unfair labor practice as mere contracting out of services being performed by union members does not per se amount to unfair labor practice unless it interferes with the employees right to self-organization. Issue: Whether or not there was an illegal dismissal. Ruling: There was a valid dismissal on the ground of redundancy. There is redundancy when the service capability of the workforce is greater than what is reasonably required to meet the demands of the

Page 171

LABOR RELATIONS
Atty. Jefferson M. Marquez
business enterprise. A position becomes redundant when it is rendered superfluous by any number of factors such as over-hiring of workers, decrease in volume of business, or dropping a particular product line. Among the requisites of a valid redundancy program are: (1) the good faith of the employer in abolishing the redundant position; and (2) fair and reasonable criteria in ascertaining what positions are to be declared redundant such as but not limited to: preferred status, efficiency, and seniority. The records show that ETPI had sufficiently established not only its need to reduce its workforce and streamline its organization, but also the existence of redundancy in the position of a Senior Technician. It was decided that, in the judgment of ETPI management, the specialized functions of a Senior Technician whose sole function was essentially the repair and servicing of ETPIs telecommunications equipment was no longer needed since the Business and Consumer [Accounts] Department had to remain economical and focused yet versatile enough to meet all the multifarious needs of its small and medium sized clients. It is inconceivable that ETPI would effect a company-wide reorganization of this scale for the mere purpose of singling out Culili and terminating him. What ETPI did was to abolish the position itself for being too specialized and limited. SC finds Culilis dismissal was for a lawful cause and not an act of unfair labor practice, ETPI, however, was remiss in its duty to observe procedural due process in effecting the termination of Culili. In Mayon Hotel & Restaurant v. Adana, SC observed that the requirement of law mandating the giving of notices was intended: not only to enable the employees to look for another employment and therefore ease the impact of the loss of their jobs and the corresponding income, but more importantly, to give the Department of Labor and Employment (DOLE) the opportunity to ascertain the verity of the alleged authorized cause of termination. With regard to the impleaded corporate officers, they cannot be held liable for acts done in his official capacity because a corporation, by legal fiction, has a personality separate and distinct from its officers, stockholders, and members. To pierce this fictional veil, it must be shown that the corporate personality was used to perpetuate fraud or an illegal act, or to evade an existing obligation, or to confuse a legitimate issue. In illegal dismissal cases, corporate officers may be held solidarily liable with the corporation if the termination was done with malice or bad faith. Culili has failed to prove that his dismissal was orchestrated by the individual respondents herein for the mere purpose of getting rid of him. Hence, the dismissal is declared valid but Eastern Telecommunications Philippines, Inc. is ordered to pay petitioner Nelson A. Culili the amount of P50,000.00 as nominal damages for non-compliance with statutory due process, in addition to the mandatory separation pay required under Article 283 of the Labor Code.

67. Plastimer Industrial Corp. v. Gopo, G.R. No. 183390, February 16, 2011 Facts: The Personnel and Administration Manager of Plastimer issued a Memorandum informing all its employees of the decision of the Board of Directors to downsize and reorganize its business operations due to withdrawal of investments and shares of stocks which resulted in the change of its corporate structure. On 14 May 2004, the employees of Plastimer, including respondent Gopo and other employees were served written notices of their termination effective 13 June 2004. Plastimer and Plastimer Industrial Corporation Christian Brotherhood (PICCB), the incumbent sole and exclusive collective bargaining representative of all rank and file employees, entered into a Memorandum of Agreement (MOA) relative to the terms and conditions that would govern the retrenchment of the affected employees. On 26 May 2004, Plastimer submitted to the DOLE an Establishment Termination Report containing the list of the employees affected by the reorganization and downsizing. The affected employees, including respondents, signed individual Release Waiver and Quitclaim. Thereafter, respondents filed a complaint against Plastimer and its President Teo Kee Bin (petitioners) before the Labor Arbiter for illegal dismissal with prayer for reinstatement and full backwages, underpayment of separation pay, moral and exemplary damages and attorneys fees. Respondents alleged that they did not voluntarily relinquish their jobs and that they were required to sign the waivers and quitclaims without giving them an opportunity to read them and without explaining their contents; and that Plastimer failed to establish the causes/valid reasons for the retrenchment and to comply with the one-month notice to the DOLE as well as the standard prescribed under the Collective Bargaining Agreement between Plastimer and the employees. Petitioners countered that the retrenchment was a management prerogative and that respondents got their retrenchment or separation pay even before the effective date of their separation from service. The Labor Arbiter ruled in favor of petitioners. It held that petitioners were able to prove that there was a substantial withdrawal of stocks that led to the downsizing of the workforce; that notice to the affected employees were given on 14 May 2004, 30 days before its effective date on 14 June 2004, and it was only the notice to the DOLE that was filed short of the 30-day period; that respondents claimed their separation pay in

Page 172

LABOR RELATIONS
Atty. Jefferson M. Marquez
accordance with the MOA; and that respondents could not claim ignorance of the contents of the waivers and quitclaims because they were assisted by the union President and their counsel in signing them. On appeal, the NLRC affirmed the Labor Arbiters decision. The Court of Appeals reversed the NLRC decision and found that petitioners have been illegally dismissed. The Court of Appeals ruled that there was no valid cause for retrenchment; that while Plastimer claimed financial losses from 2001 to 2004, records showed an improvement of its finances in 2003; that Plastimer failed to use a reasonable and fair standard or criteria in ascertaining who would be dismissed and who would be retained among its employees; that the MOA between Plastimer and PICCB only recognized the need for partial retrenchment and the computation of retrenchment pay without disclosing the criteria in the selection of the employees to be retrenched; and that the union President and the PICCBs counsel were not present when the retrenched employees were made to sign the waivers and quitclaims. Hence, the petition before this Court. Issue: WON respondents were illegally retrenched by petitioners. Ruling: The petition has merit. This Court is not precluded from reviewing the factual issues when there are conflicting findings by the Labor Arbiter, the NLRC and the Court of Appeals. In this case, we find that the findings of the Labor Arbiter and the NLRC are more in accord with the evidence on record. One-Month Notice of Termination of Employment Article 283 of the Labor Code provides: ART. 283. Closure of establishment and reduction of personnel. - The employer may also terminate the employment of any employee due to the installation of labor-saving devices, redundancy, retrenchment to prevent losses or the closing or cessation of operation of the establishment or undertaking unless the closing is for the purpose of circumventing the provisions of this Title, by serving a written notice on the workers and the Department of Labor and Employment at least one (1) month before the intended date thereof. xxx In this case, Plastimer submitted the notice of termination of employment to the DOLE on 26 May 2004. However, notice to the affected employees were given to them on 14 May 2004 or 30 days before the effectivity of their termination from employment on 13 June 2004. While notice to the DOLE was short of the one-month notice requirement, the affected employees were sufficiently informed of their retrenchment 30 days before its effectivity. Petitioners failure to comply with the one-month notice to the DOLE is only a procedural infirmity and does not render the retrenchment illegal. In Agabon v. NLRC, we ruled that when the dismissal is for a just cause, the absence of proper notice should not nullify the dismissal or render it illegal or ineffectual. Instead, the employer should indemnify the employee for the violation of his statutory rights. Here, the failure to fully comply with the one-month notice of termination of employment did not render the retrenchment illegal but it entitles respondents to nominal damages. Validity of Retrenchment The fact that there was a net income in 2003 does not justify the Court of Appeals ruling that there was no valid reason for the retrenchment. Records showed that the net income of P6,185,707.05 for 2003 was not even enough for petitioners to recover from the P52,904,297.88 loss in 2002. Article 283 of the Labor Code recognizes retrenchment to prevent losses as a right of the management to meet clear and continuing economic threats or during periods of economic recession to prevent losses. There is no need for the employer to wait for substantial losses to materialize before exercising ultimate and drastic option to prevent such losses. Validity of Waivers and Quitclaims The Court has ruled that a waiver or quitclaim is a valid and binding agreement between the parties, provided that it constitutes a credible and reasonable settlement, and that the one accomplishing it has done so voluntarily and with a full understanding of its import. We agree with the Labor Arbiter and the NLRC that respondents were sufficiently apprised of their rights under the waivers and quitclaims that they signed. Each document contained the signatures of Marcaida, PICCB President, and Atty. Diwa, the counsel for the union, which proved

Page 173

LABOR RELATIONS
Atty. Jefferson M. Marquez
that respondents were duly assisted when they signed the waivers and quitclaims. Further, Marcaidas letter to Teo Kee Bin, dated 28 May 2004, proved that proper assistance was extended upon respondents. Hence, we rule that the waivers and quitclaims that respondents signed were valid. WHEREFORE, we SET ASIDE the Decision and Resolution of the Court of Appeals, and hereby REINSTATE the Decision of the Labor Arbiter and the Resolution of the NLRC upholding the validity of respondents retrenchment with MODIFICATION that petitioners pay each of the respondents the amount of P30,000 as nominal damages for non-compliance with statutory due process.

68. St. Marys Academy of Dipolog City vs. Palacio, G.R. No. 164913, September 8, 2010 Facts: On different dates in the late 1990s, petitioner hired respondents Calibod, Laquio, Santander, Saile and Montederamos, as classroom teachers, and respondent Palacio, as guidance counselor. In separate letters dated March 31, 2000, however, petitioner informed them that their reapplication for school year 2000-2001 could not be accepted because they failed to pass the Licensure Examination for Teachers (LET). According to petitioner, as non-board passers, respondents could not continue practicing their teaching profession pursuant to the Department of Education, Culture and Sports (DECS) Memorandum No. 10, S. 1998 7 which requires incumbent teachers to register as professional teachers pursuant to Section 27of Republic Act (RA) No. 7836, otherwise known as the Philippine Teachers Professionalization Act of 1994. The DECS Memorandum, pursuant to PRC Resolution No. 600, S. 1997 fixed the deadline for teachers to register on September 19, 2000. Further, as the aforesaid law provides for exceptions to the taking of examination, some of them possessed civil service eligibilities and special permits to teach. Also, it was established the petitioner retained other teachers who did not also possess the required eligibility. Together with four other classroom teachers namely Gail Josephine Padilla (Padilla), Virgilio Andalahao (Andalahao), Alma Decipulo (Decipulo), and Marlynn Palacio, who were similarly dismissed by petitioner on the same ground, respondents filed a complaint contesting their termination as highly irregular and premature. Petitioner claimed that it decided to terminate their services as early as March 31, 2000 because it would be prejudicial to the school if their services will be terminated in the middle of the school year. Issue: Whether or not there was illegal dismissal Ruling: The dismissal of Teresita Palacio, Calibod, Laquio, Santander, and Montederamos was premature and defeated their right to security of tenure. Sailes dismissal has legal basis for lack of the required qualification needed for continued practice of teaching Pursuant to RA7836, its resolution and subsequent memorandum, effective September 20, 2000, only holders of valid certificates of registration, valid professional licenses and valid special/temporary permits can engage in teaching in both public and private schools.24 Clearly, respondents, in the case at bar, had until September 19, 2000 to comply with the mandatory requirement to register as professional teachers. As respondents are categorized as those not qualified to register without examination, the law requires them to register by taking and passing the licensure examination. Petitioner claims that it terminated respondents employment as early as March 2000 because it would be highly difficult to hire professional teachers in the middle of the school year as replacements for respondents without compromising the operation of the school and education of the students. Petitioners intention and desire not to put the students education and school operation in jeopardy is neither a decisive consideration for respondents termination prior to the deadline set by law. Again, by setting a deadline for registration as professional teachers, the law has allowed incumbent teachers to practice their teaching profession until September 19, 2000, despite being unregistered and unlicensed. The prejudice that respondents retention would cause to the schools operation is only trivial if not speculative as compared to the consequences of respondents unemployment. Incidentally, petitioner did not dispute that it hired and retained other teachers who do not likewise possess the qualification and eligibility and even allowed them to teach during the school year 2000-2001. This indicates petitioners ulterior motive in hastily dismissing respondents. It is incumbent upon this Court to afford full protection to labor. Thus, while we take cognizance of the employers right to protect its interest, the same should be exercised in a manner which does not infringe on the workers right to security of tenure. "Under the policy of social justice, the

Page 174

LABOR RELATIONS
Atty. Jefferson M. Marquez
law bends over backward to accommodate the interests of the working class on the humane justification that those with less privilege in life should have more in law. Petition DENIED.

69. PLDT vs. Teves, G.R. No. 143511, November 15, 2010 Facts: Respondent was employed by petitioner Philippine Long Distance Telephone Company in 1981 as Clerk II until his termination from service on June 1, 1992. Petitioner terminated respondent through an Inter-Office Memorandum dated May 29, 1992 on account of his three (3) unauthorized leaves of absence committed within three (3) years in violation of petitioners rules and regulations. Respondent was absent from August 23 to September 3, 1990 as his wife gave birth and suffered complications. Respondent called up through a third party to inform petitioner that he would go on an extended leave. Upon his reporting for work, he wrote petitioner a letter confirming his leave of absence without pay for that period and stating the reasons thereof, with his wife's medical certificate attached. Dissatisfied, petitioner required respondent to submit further explanation which the latter did reiterating his previous explanation. However, it found respondents explanation to be unacceptable and unmeritorious for the latter's failure to call, notify or request petitioner for such leave and suspended respondent from work without pay for 20 days. Respondent was absent from May 29 to June 12, 1991 his eldest and youngest daughters getting sick and had to be confined in the hospital. Further, respondent alleged that he had relayed said message to an officemate who unfortunately did not also report for work. Petitioner found respondents explanation insufficient, respondent was suspended without pay for 45 days. Eight months thereafter, respondent availed of a seven-day leave of absence and extended such leave to complete his annual vacation leave. However, respondent failed to report for work from February 11 to February 19, 1992 which was made to prolong payment of his demandable financial obligations in the office. He further stated that he realized that what he did was wrong and only worsened his situation and asked for another chance. Petitioner found such explanation totally unacceptable. Thus, in an Inter-Office Memorandum dated May 29, 1992 addressed to respondent, the latter was terminated from service effective June 1, 1992 due to his third unauthorized absence within a three-year period. The LA found that respondents dismissal was legal. However, the NLRC found that the two previous incidents of respondents alleged unauthorized absences were justified, and that while his absence from February 11 to 19, 1992 was unacceptable and unreasonable, he should have been penalized therefor accordingly, but not with dismissal from service. The CA affirmed the NLRCs findings and concluded that respondents absences from February 11 to 19, 1992 was his first and only unauthorized absences during his 11 years of stay, and it did not merit the harsh penalty of dismissal. Issue: Whether or not there is sufficient ground for the termination of respondent. Held: We find that respondent's termination for committing three unauthorized absences within a three-year period had no basis; thus, there was no valid cause for respondent's dismissal. Even assuming that respondent's absenteeism constitutes willful disobedience, such offense does not warrant respondent's dismissal. Not every case of insubordination or willful disobedience by an employee reasonably deserves the penalty of dismissal. There must be a reasonable proportionality between the offense and the penalty. Petitioner's claim that the alleged previous infractions may be used as supporting justification to a subsequent similar offense, which would merit dismissal, finds no application in this case. Respondent's absence from August 23 to September 3, 1990 was justified and not unauthorized as there was prior notice. His absence from May 29 to June 12, 1991, although found to be unauthorized, was not at all unjustified. Thus, his absence during the period from February 11 to 19, 1991, being the only unauthorized and unjustified absence and his second unauthorized absence, should not merit the penalty of dismissal. While management has the prerogative to discipline its employees and to impose appropriate penalties on erring workers, pursuant to company rules and regulations, however, such management prerogatives must be exercised in good faith for the advancement of the employers interest

Page 175

LABOR RELATIONS
Atty. Jefferson M. Marquez
and not for the purpose of defeating or circumventing the rights of the employees under special laws and valid agreements. The Court is wont to reiterate that while an employer has its own interest to protect, and pursuant thereto, it may terminate an employee for a just cause, such prerogative to dismiss or lay off an employee must be exercised without abuse of discretion. Its implementation should be tempered with compassion and understanding. The employer should bear in mind that, in the execution of said prerogative, what is at stake is not only the employees position, but his very livelihood, his very breadbasket. Considering that respondent was illegally dismissed from service, he is entitled to be reinstated, without loss of seniority rights and the payment of backwages from the time respondents compensation was withheld from him until his reinstatement on November 12, 1997. However, since we find that respondent's absence from February 11 to 19, 1992 was unjustified and unauthorized, thus, his suspension for thirty days would be in order. Hence, the amount equivalent to the thirty-day suspension, which respondent should have served for his absence on February 11 to 19, 1992, should be deducted from the backwages to be awarded to him.

70. University of the Immaculate Concepcion vs. NLRC, G.R. No. 181146, January 26, 2011 Facts: Teodora C. Axalan is a regular faculty member holding the position of Associate Professor II in the University of the Immaculate Conception in Davao. She was dismissed due to 2 instances wherein she was allegedly absent without leave, attending seminars in Quezon City and Baguio City, respectively. On the first instance, Axalan claimed that she held online classes. She was convinced that she cannot be considered absent and opted not to write the letter of apology requested of her by the University President to avoid any administrative charge. On the second instance, Axalan claimed that she asked permission from the VP for Academics who denied giving the same. After conducting hearings and receiving evidence, the ad hoc grievance committee found Axalan to have incurred AWOL on both instances and recommended that Axalan be suspended without pay for six months on each AWOL charge. The university president approved the committees recommendation and wrote Axalan a letter informing her of her absences and of her total penalty of one-year suspension without pay for both AWOL charges effective immediately. Issue: Whether or not there was constructive dismissal. Held: NO, there was no constructive dismissal, Axalan having been validly validly suspended for cause and in accord with procedural due process. Constructive dismissal occurs when there is cessation of work because continued employment is rendered impossible, unreasonable, or unlikely as when there is a demotion in rank or diminution in pay or when a clear discrimination, insensibility, or disdain by an employer becomes unbearable to the employee leaving the latter with no other option but to quit. In this case however, there was no cessation of employment relations between the parties. It is unrefuted that Axalan promptly resumed teaching at the university right after the expiration of the suspension period. In other words, Axalan never quit. Hence, Axalan cannot claim that she was left with no choice but to quit, a crucial element in a finding of constructive dismissal. Thus, Axalan cannot be deemed to have been constructively dismissed. Significantly, at the time the Labor Arbiter rendered his Decision on 11 October 2004, Axalan had already returned to her teaching job at the university on 1 October 2004. The Labor Arbiters Decision ordering the reinstatement of Axalan, who at the time had already returned to work, is thus absurd. There being no constructive dismissal, there is no legal basis for the Labor Arbiters order of reinstatement as well as payment of backwages, salary differentials, damages, and attorneys fees. Thus, the third issue raised in the petition is now moot. The Court recognizes the right of employers to discipline its employees for serious violations of company rules after affording the latter due process and if the evidence warrants. The university, after affording Axalan due process and finding her guilty of incurring AWOL on two separate occasions, acted well within the bounds of labor laws in imposing the penalty of six-month suspension without pay for each incidence of AWOL.

Page 176

LABOR RELATIONS
Atty. Jefferson M. Marquez
As a learning institution, the university cannot be expected to take lightly absences without official leave among its employees, more so among its faculty members even if they happen to be union officers. To do so would send the wrong signal to the studentry and the rest of its teaching staff that irresponsibility is widely tolerated in the academe. The law protects both the welfare of employees and the prerogatives of management. Courts will not interfere with prerogatives of management on the discipline of employees, as long as they do not violate labor laws, collective bargaining agreements if any, and general principles of fairness and justice.

71. Simizu Phils Contractors v. Callanta, G.R. No. 165923, September 29, 2010 Facts: Shimizu Phils. a corporation engaged in the construction business, employed Virgilio Callanta on August 23, 1994 as Safety Officer assigned at Yutaka-Giken Project and eventually as Project Administrator of petitioners Structural Steel Division (SSD) in 1995. Virgilio Callanta was informed that his services will be terminated effective July 9, 1997 due to the lack of any vacancy in other projects and the need to re-align the companys personnel requirements brought about by the imperatives of maximum financial commitments. He then filed an illegal dismissal complaint against petitioner assailing his dismissal as without any valid cause. Shimizu advanced that respondents services was terminated in accordance with a valid retrenchment program being implemented by the company since 1996 due to financial crisis that plague the construction industry. To prove its financial deficit, petitioner presented financial statements for the years 1995 to 1997 as well as the Securities and Exchange Commissions approval of petitioners application for a new paidin capital amounting to P330,000,000. Shimizu alleged that in order not to jeopardize the completion of its projects, the abolition of several departments and the concomitant termination of some employees were implemented as each project is completed. When respondents Honda Project was completed, petitioner offered respondent his separation pay which the latter refused to accept and instead filed an illegal dismissal complaint. Mr. Callanta claimed that Shimizu failed to comply with the requirements called for by law before implementing a retrenchment program thereby rendering it legally infirmed. First, it did not comply with the provision of the Labor Code mandating the service of notice of retrenchment. He pointed out that the notice sent to him never mentioned retrenchment but only project completion as the cause of termination. Also, the notice sent to the Department of Labor and Employment (DOLE) did not conform to the 30-day prior notice requirement. Second, petitioner failed to use fair and reasonable criteria in determining which employees shall be retrenched or retained. In the termination report submitted to DOLE, he was the only one dismissed out of 333 employees. Worse, junior and inexperienced employees were appointed/assigned in his stead to new projects thus also ignoring seniority in hiring and firing employees. Shimizu argued that when it submitted the retrenchment notice/termination report to DOLE, there was already substantial compliance with the requirement. Labor Arbiter rendered a decision holding that Mr. Callanta was validly retrenched. He found that sufficient evidence was presented to establish company losses; that petitioner offered respondent his separation pay; and that petitioner duly notified DOLE about the retrenchment. The Labor Arbiter further relied on petitioners factual version relating to respondents employment background with regard to his position and behavioral conduct. NLRC upheld the ruling that there was valid ground for respondents termination but modified the Labor Arbiters Decision by holding that petitioner violated respondents right to procedural due process. The NLRC found that petitioner failed to comply with the 30-day prior notice to the DOLE and that there is no proof that petitioner used fair and reasonable criteria in the selection of employees to be retrenched. Shimizu Philippine Contractor, Inc., is ordered to pay complainant-appellant Virgilio P. Callanta his separation pay equivalent to one (1) month pay for every year of service. For want of due notice, respondent is further directed to pay complainant an indemnity equivalent to one (1) month salary. CA reversed and set aside the NLRCs ruling. The CA opined that Shimizu failed to prove that there were employees other than respondent who were similarly dismissed due to retrenchment and that respondents alleged replacements held much higher ranks and were more deserving employees. Moreover, there were no proofs to sustain that petitioner used fair and reasonable criteria in determining which employees to retrench. According to the CA, petitioners failure to produce evidence raises the presumption that such evidence will be adverse to it. Consequently, the CA invalidated the retrenchment, held respondent to have been illegally dismissed, and ordered respondents reinstatement and payment of backwages.

Page 177

LABOR RELATIONS
Atty. Jefferson M. Marquez
Issue: Whether or not Shimizu has failed to obeserve fair and reasonable standards or criteria in effecting the dismissal or Mr. Callanta? Ruling: There was substantial compliance for a valid retrenchment; Shimizu used fair and reasonable criteria in effecting retrenchment. As an authorized cause for separation from service under Article 283 of the Labor Code, retrenchment is a valid exercise of management prerogative subject to the strict requirements set by jurisprudence: (1) That the retrenchment is reasonably necessary and likely to prevent business losses which, if already incurred, are not merely de minimis, but substantial, serious, actual and real, or if only expected, are reasonably imminent as perceived objectively and in good faith by the employer; (2) That the employer served written notice both to the employees and to the Department of Labor and Employment at least one month prior to the intended date of retrenchment; (3) That the employer pays the retrenched employees separation pay equivalent to one month pay or at least month pay for every year of service, whichever is higher; (4) That the employer exercises its prerogative to retrench employees in good faith for the advancement of its interest and not to defeat or circumvent the employees right to security of tenure; and (5) That the employer used fair and reasonable criteria in ascertaining who would be dismissed and who would be retained among the employees, such as status, efficiency, seniority, physical fitness, age, and financial hardship for certain workers. Both the Labor Arbiter and the NLRC found sufficient compliance with these substantive requirements, there being enough evidence to prove that petitioner was sustaining business losses, that separation pay was offered to respondent, and that notices of termination of service were furnished respondent and DOLE. However, the NLRC modified the Decision of the Labor Arbiter by granting respondent indemnity since the notice to DOLE was served short of the 30-day notice requirement and that there is no proof of the use of fair and reasonable criteria in the selection of employees to be retrenched or retained. The CA, then, reversed the Decision of the NLRC by ruling that the absence of fair and reasonable criteria in implementing the retrenchment invalidates altogether the retrenchment. In implementing its retrenchment scheme, petitioner was constrained to streamline its operations and to downsize its complements in a progressive manner in order not to jeopardize the completion of its projects. Thus, several departments like the Civil Works Division, Electromechanical Works Division and the Territorial Project Management Offices, among others, were abolished in the early part of 1996 and thereafter the Structural Steel Division, of which respondent was an Administrator. Respondent was among the last batch of employees who were retrenched and by the end of year 1997, all of the employees of the Structural Steel Division were severed from employment. Mr. Callanta argued that that he was singled out for termination as allegedly shown in petitioners monthly termination report for the month of July 1997 filed with the DOLE does not persuade this Court. Standing alone, this document is not proof of the total number of retrenched employees or that respondent was the only one retrenched. It merely serves as notice to DOLE of the names of employees terminated/ retrenched only for the month of July. In other words, it cannot be deemed as an evidence of the number of employees affected by the retrenchment program. Thus we cannot conclude that no other employees were previously retrenched. Shimizu implemented its retrenchment program in good faith because it undertook several measures in cutting down its costs, to wit, withdrawing certain privileges of petitioners executives and expatriates; limiting the grant of additional monetary benefits to managerial employees and cutting down expenses; selling of company vehicles; and infusing fresh capital into the company. Respondent did not attempt to refute that petitioner adopted these measures before implementing its retrenchment program. Shimizu was able to prove that it incurred substantial business losses, that it offered to pay respondent his separation pay, that the retrenchment scheme was arrived at in good faith, and lastly, that the criteria or standard used in selecting the employees to be retrenched was work efficiency which passed the test of fairness and reasonableness. The termination notice sent to DOLE did not comply with the 30-day notice requirement, thus, respondent is entitled to indemnity for violation of due process. However, petitioner admitted that the reports were submitted 21 days, in the case of the first notice, and 16 days, in the case of the second notice, before the intended date of respondents dismissal. The purpose of the one month prior notice rule is to give DOLE an opportunity to ascertain the veracity of the cause of termination. Noncompliance with this rule clearly violates the employees right to statutory due process. Consequently, we affirm the NLRCs award of indemnity to respondent for want of sufficient due notice. But to be consistent with our ruling in Jaka Food Processing Corporation v. Pacot, the indemnity in the form of nominal damages should be fixed in the amount of P50,000.00.

Page 178

LABOR RELATIONS
Atty. Jefferson M. Marquez

72. Manila Mining Corp. Employees Association-FFW vs. Manila Mining Corp., G.R. No. 178222-23, September 29, 2010 Facts: Respondent Manila Mining Corporation (MMC) is a publicly-listed corporation engaged in large-scale mining for gold and copper ore. MMC is required by law to maintain a tailings containment facility to store the waste material generated by its mining operations. Consequently, MMC constructed several tailings dams to treat and store its waste materials. One of these dams was Tailings Pond No. 7 (TP No. 7), which was constructed in 1993 and was operated under a permit issued by the Department of Environment and Natural Resources (DENR), through its Environmental Management Bureau (EMB) in Butuan City, Agusan del Norte. Upon expiration of the tailings permit on 25 July 2001, DENR-EMB did not issue a permanent permit due to the inability of MMC to secure an Environmental Compliance Certificate (ECC). An essential component of an ECC is social acceptability or the consent of the residents in the community to allow TP No. 7 to operate, which MMC failed to obtain. Hence, it was compelled to temporarily shut down its mining operations, resulting in the temporary lay-off of more than 400 employees in the mine site. On 30 July 2001, MMC called for the suspension of negotiations on the CBA with the Union until resumption of mining operations. Among the employees laid-off, complainants Samuel Zuiga, Myrna Maquio, Doroteo Torre, Arsenio Mark Perez, Edmundo Galvez, Diana Ruth Rellores, Jonathan Araneta, Teresita Lagman, Reynaldo Anzures, Gerardo Opena, and Edwin Tuazon, together with the Union filed a complaint before the labor arbiter on even date praying for reinstatement, recognition of the Union as the sole and exclusive representative of its rank-andfile employees, and payment of moral and exemplary damages and attorneys fees. In their Position Paper, complainants challenged the validity of their lay-off on the averment that MMC was not suffering from business losses. They alleged that MMC did not want to bargain collectively with the Union, so that instead of submitting their counterproposal to the CBA, MMC decided to terminate all union officers and active members. Petitioners questioned the timing of their lay-off, and alleged that first, there was no showing that cost-cutting measures were taken by MMC; second, no criteria were employed in choosing which employees to lay-off; and third, the individuals laid-off were those who signed the attendance sheet of the union organizational meeting. Petitioners likewise claimed that they were denied due process because they were not given a 30-day notice informing them of the lay-off. Neither was the DOLE informed of this layoff, as mandated by law. Respondents justified the temporary lay-off as bona fide in character and a valid management prerogative pending the issuance of the permit to continuously operate TP No. 7. The labor arbiter ruled in favor of MMC and held that the temporary shutdown of the mining operation, as well as the temporary lay-off of the employees, is valid. On appeal, the National Labor Relations Commission (NLRC) modified the judgment of the labor arbiter and ordered the payment of separation pay equivalent to one month pay for every year of service. It ratiocinated that the temporary lay-off, which exceeded more than six (6) months, had the effect of severance of the employer-employee relationship. Issue: WON there was bad faith on the part of MMC in implementing the temporary lay-off resulting in the complainants constructive dismissal Held: The lay-off is neither illegal nor can it be considered as unfair labor practice. Despite all efforts exerted by MMC, it did not succeed in obtaining the consent of the residents of the community where the tailings pond would operate, one of the conditions imposed by DENR-EMB in granting its application for a permanent permit. It is precisely MMCs faultless failure to secure a permit which caused the temporary shutdown of its mining operations. For a charge of unfair labor practice to prosper, it must be shown that the employer was motivated by ill-will, bad faith or fraud, or was oppressive to labor. The employer must have acted in a manner contrary to morals, good customs, or public policy causing social humiliation, wounded feelings or grave anxiety. While the law makes it an obligation for the employer and the employees to bargain collectively with each other, such compulsion does not include the commitment to precipitately accept or agree to the proposals of the other. All it contemplates is that both parties

Page 179

LABOR RELATIONS
Atty. Jefferson M. Marquez
should approach the negotiation with an open mind and make reasonable effort to reach a common ground of agreement. Even as we declare the validity of the lay-off, we cannot say that MMC has no obligation at all to the laid-off employees. The validity of its act of suspending its operations does not excuse it from paying separation pay. Article 286 of the Labor Code allows the bona fide suspension of operations for a period not exceeding six (6) months. During the suspension, an employee is not deemed terminated. As a matter of fact, the employee is entitled to be reinstated once the employer resumes operations within the 6-month period. However, Article 286 is silent with respect to the rights of the employee if the suspension of operations lasts for more than 6 months. Thus is bred the issue regarding the responsibility of MMC toward its employees. The Court is not impressed with the claim that actual severe financial losses exempt MMC from paying separation benefits to complainants. In the first place, MMC did not appeal the decision of the Court of Appeals which affirmed the NLRCs award of separation pay to complainants. MMCs failure had the effect of making the awards final so that MMC could no longer seek any other affirmative relief. In the second place, the non-issuance of a permit forced MMC to permanently cease its business operations, as confirmed by the Court of Appeals. Under Article 283, the employer can lawfully close shop anytime as long as cessation of or withdrawal from business operations is bona fide in character and not impelled by a motive to defeat or circumvent the tenurial rights of employees, and as long as he pays his employees their termination pay in the amount corresponding to their length of service. The cessation of operations, in the case at bar is of such nature. It was proven that MMC stopped its operations precisely due to failure to secure permit to operate a tailings pond. Separation pay must nonetheless be given to the separated employees.

73. Lopez vs. Alturas Group of Companies, G.R. No. 191008, April 11, 2011 Facts: Petitioner Lopez is the truck driver of the respondent. He was given a notice to the effect that he would be terminated by the respondent on the ground that he was caught by the security guard stealing scrap irons by smuggling them out from the company premises. Later on, the respondent filed a criminal case of qualified theft against the respondent. Petitioner then filed an illegal dismissal case against the respondent on the ground that he was dismissed by the respondent without just cause and that he was not afforded due process as he was not given a counsel and an opportunity to confront witnesses. As the case was appealed, the NLRC ruled that there was a violation of due process in dismissing the petitioner as he was not afforded ample opportunity to refute the allegations lodged against him. Furthermore, petitioner advanced that there was no sufficient cause to terminate him as such ground was only a mere subterfuge of the respondent so as for him not to lodge a complaint of underpayment of wages. Issue: Whether or not the dismissal is illegal Ruling: No, the dismissal is not illegal. Dismissals have two facets: the legality of the act of dismissal, which constitutes substantive due process, and the legality of the manner of dismissal which constitutes procedural due process. As to substantive due process, the Court finds that respondent companys loss of trust and confidence arising from petitioners smuggling out of the scrap iron, conpounded by his past acts of unauthorized selling cartons belonging to respondent company, constituted just cause for terminating his services. Loss of trust and confidence as a ground for dismissal of employees covers employees occupying a position of trust who are proven to have breached the trust and confidence reposed on them The language of Article 282(c) of the Labor Code states that the loss of trust and confidence must be based on willful breach of the trust reposed in the employee by his employer. Such breach is willful if it is done intentionally, knowingly, and purposely, without justifiable excuse, as distinguished from an act done carelessly, thoughtlessly, heedlessly. And, in order to constitute a just cause for dismissal, the act complained of must be work-related and shows that the employee concerned is unfit to continue working for the employer. In addition, loss of confidence as a just cause for termination of employment is premised on the fact that the employee concerned holds a position of responsibility, trust and confidence or that the employee concerned is entrusted with confidence with respect to delicate matters, such as the handling or care and protection of the property and assets of the employer. The betrayal of this trust is the essence of the offense for which an employee is penalized. It is, however, with respect to the appellate courts finding that petitioner was not afforded procedural due process that the Court deviates from. Procedural due process has been defined as giving an opportunity to be heard before judgment is rendered. The Court finds that it was error for the NLRC to opine that petitioner should have been afforded counsel or advised of the right to counsel. The

Page 180

LABOR RELATIONS
Atty. Jefferson M. Marquez
right to counsel and the assistance of one in investigations involving termination cases is personally or with the assistance of a representative or counsel. He may also ask the employer to provide him copy of records material to his defense. His written explanation may also include a request that a formal hearing or conference be held. In such a case, the conduct of a formal hearing or conference becomes mandatory, just as it is where there exist substantial evidentiary disputes or where company rules or practice requires an actual hearing as part of employment pretermination procedure.

74. Apacible vs. Multimed Industries Inc., G.R. No. 178903, May 30, 2011 Facts: Petitioner Juliet Apacible was hired sometime in 1994 by respondent. She rose from the ranks to become Assistant Area Sales Manager for Cebu Operations, the position she held at the time she was separated from the service in 2003. On August 4, 2003, petitioner was informed by respondent Marlene Orozco (Marlene), her immediate superior, that she would be transferred to the company's main office in Pasig City on account of the ongoing reorganization. Petitioner requested that her transfer be made effective in October or November 2003 and that she be given time to discuss it with her husband and daughter. A week later, however, or on August 11, 2003, petitioner was informed that her transfer would be effective August 18, 2003. On even date, she was placed under investigation for the delayed released of BCRs (cash budget for customer representation in sealed envelopes which are given to loyal clients) which she received for distribution earlier in July 2003. Finding that the delay in releasing the BCRs amounted to loss of trust and confidence, petitioner claims that in a meeting with the respondents, she was given four options: resignation, termination, availment of an early retirement package worth P40,000, or transfer to Pasig City. Without availing of any option, petitioner took a leave of absence on August 28, 29 and September 1, 2003. On September 3, 2003, respondent company sent petitioner a memorandum-directive for her to immediately report to the head office in Pasig City and to return the company vehicle assigned to her to the Cebu Office within 24 hours. Petitioner did not heed the directive, however. She instead filed an application for sick leave until September 11, 2003, and another until September 27, 2003. On October 6, 2003, petitioner requested that she be given her daily work assignment in Cebu, which request was later to be denied by Olga by letter dated October 8, 2003. On October 7, 2003, petitioner was given a show cause notice for her to explain in writing why she should not be sanctioned for insubordination for failure to comply with the transfer order. On November 4, 2002, respondent company sent petitioner a notice of termination effective November 7, 2003 for insubordination, prompting petitioner to file a complaint for illegal dismissal, non-payment of overtime pay, 13th month pay, service incentive leave pay, separation pay, damages and attorney's fees before the Labor Arbiter. The Court of Appeals ruled that petitioner was not entitled to separation pay because, contrary to the NLRC's finding, she "lacked good faith." It noted that petitioner, from the start, knew and accepted the company policy on transfers whenever so required, and could not thus refuse "another valid reassignment by treating it as an imposition and burden." ISSUE: Whether petitioner is entitled separation pay by way of financial assistance. RULING: NO. Reno Foods, Inc. v. Nagkakaisang Lakas ng Manggagawa (NLM)-Katipunan 16 explains the propriety of granting separation pay in termination cases in this wise: The law is clear. Separation pay is only warranted when the cause for termination is not attributable to the employee's fault, such as those provided in Articles 283 and 284 of the Labor Code, as well as in cases of illegal dismissal in which reinstatement is no longer feasible. It is not allowed when an employee is dismissed for just cause, such as serious misconduct. xxx xxx xxx

Page 181

LABOR RELATIONS
Atty. Jefferson M. Marquez
It is true that there have been instances when the Court awarded financial assistance to employees who were terminated for just causes, on grounds of equity and social justice. The same, however, has been curbed and rationalized in Philippine Long Distance Telephone Company v. National Labor Relations Commission. In that case, we recognized the harsh realities faced by employees that forced them, despite their good intentions, to violate company policies, for which the employer can rightly terminate their employment. For these instances, the award of financial assistance was allowed. But, in clear and unmistakable language, we also held that the award of financial assistance shall not be given to validly terminated employees, whose offenses are iniquitous or reflective of some depravity in their moral character. When the employee commits an act of dishonesty, depravity, or iniquity, the grant of financial assistance is misplaced compassion. It is tantamount not only to condoning a patently illegal or dishonest act, but an endorsement thereof. It will be an insult to all the laborers who despite their economic difficulties, strive to maintain good values and moral conduct. In fact, in the recent case of Toyota Motors Philippines, Corp. Workers Association (TMPCWA) v. National Labor Relations Commission, we ruled that separation pay shall not be granted to all employees who are dismissed on any of the four grounds provided in Article 282 of the Labor Code. Such ruling was reiterated and further explained in Central Philippines Bandag Retreaders, Inc. v. Diasnes: To reiterate our ruling in Toyota, labor adjudicatory officials and the CA must demur the award of separation pay based on social justice when an employee's dismissal is based on serious misconduct or wilful disobedience; gross and habitual neglect of duty; fraud or wilful breach of trust; or commission of a crime against the person of the employer or his immediate family grounds under Art. 282 of the Labor Code that sanction dismissals of employees. They must be most judicious and circumspect in awarding separation pay or financial assistance as the constitutional policy to provide full protection to labor is not meant to be an instrument to oppress the employers. The commitment of the Court to the cause of labor should not embarrass us from sustaining the employers when they are right, as assistance to the undeserving and those who are unworthy of the liberality of the law. (italics in the original, emphasis and underscoring supplied) ASTIED Petitioner was, it bears reiteration, dismissed for wilfully disobeying the lawful order of her employer to transfer from Cebu to Pasig City. As correctly noted by the appellate court, petitioner knew and accepted respondent company's policy on transfers when she was hired and was in fact even transferred many times from one area of operations to another Bacolod City, Iloilo City and Cebu. Clearly, petitioner's adamant refusal to transfer, coupled with her failure to heed the order for her return the company vehicle assigned to her and, more importantly, allowing her counsel to write letters couched in harsh language to her superiors unquestionably show that she was guilty of insubordination, hence, not entitled to the award of separation pay.

75. Barroga vs. Data Center College, G.R. No. 174158, June 27, 2011 Facts: On November 11, 1991, petitioner was employed as an Instructor in Data Center College Laoag City branch in Ilocos Norte. In a Memorandum dated June 6, 1992, respondents transferred him to University of Northern Philippines (UNP) in Vigan, Ilocos Sur where the school had a tie-up program. Petitioner was informed through a letter dated June 6, 1992 that he would be receiving, in addition to his monthly salary, a P1,200.00 allowance for board and lodging during his stint as instructor in UNP-Vigan. In 1994, he was recalled to Laoag campus. On October 3, 2003, petitioner received a Memorandum transferring him to Data Center College Bangued, Abra branch as Head for Education/Instructor due to an urgent need for an experienced officer and computer instructor thereat. However, petitioner declined to accept his transfer to Abra citing the deteriorating health condition of his father and the absence of additional remuneration to defray expenses for board and lodging which constitutes implicit diminution of his salary. On November 10, 2003, petitioner filed a Complaint for constructive dismissal against respondents. Petitioner alleged that his proposed transfer to Abra constitutes a demotion in rank and diminution in pay and would cause personal inconvenience and hardship. For their part, respondents claimed that they were merely exercising their management prerogative to transfer employees for the purpose of advancing the schools interests. They argued that petitioners refusal to be transferred to Abra constitutes insubordination. Ruling: Petitioners transfer is not tantamount to constructive dismissal. Petitioner was originally appointed as instructor in 1991 and was given additional administrative functions as Head for Education during his stint in Laoag branch. He did not deny having been designated as Head for Education in a temporary capacity for which he cannot invoke any tenurial security. Hence, being temporary in character, such designation is terminable at the pleasure of respondents who made such appointment. Moreover, respondents right to transfer petitioner rests not only on contractual stipulation but also on jurisprudential authorities. The Labor Arbiter and the NLRC both relied on the condition laid down in petitioners employment contract that respondents have the prerogative to assign

Page 182

LABOR RELATIONS
Atty. Jefferson M. Marquez
petitioner in any of its branches or tie-up schools as the necessity demands. In any event, it is management prerogative for employers to transfer employees on just and valid grounds such as genuine business necessity. It is also important to stress at this point that respondents have shown that it was experiencing some financial constraints. Because of this, respondents opted to temporarily suspend the post-graduate studies of petitioner and some other employees who were given scholarship grants in order to prioritize more important expenditures.

76. Lopez vs. Keppel Bank Phils., G.R. No. 176800, September 5, 2011 Facts: Petitioner Elmer Lopez was the Branch Manager of the respondent Keppel Bank Philippines, Inc. (bank) in Iloilo City. Allegedly, through his efforts, Hertz Exclusive Cars, Inc. (Hertz) became a client of the bank. By a notice, the bank asked Lopez to explain in writing why he should not be disciplined for issuing, without authority, two purchase orders (POs) for the Hertz account amounting to a total of P6,493,000.00, representing the purchase price of 13 Suzuki Bravo and two Nissan Exalta vehicles. Lopez submitted his written explanation on the same day, but the bank refused to give it credit. Through respondents Manuel Bosano III (Vice-President and Head of Retail Banking Division/Consumer Banking Division) and Stefan Tong Wai Mun (VicePresident/Comptroller), the bank terminated Lopezs employment effective immediately. Lopez filed a complaint for illegal dismissal and money claims against the bank, Bosano and Tong. Lopez alleged before the labor arbiter that he issued the POs as part of his strategy to enhance the bank's business, in line with his duty as branch manager to promote the growth of the bank. For its part, the bank denied approving the first PO, arguing that Lopez did not have the authority to issue the POs for the Hertz account as there was a standing advice that no Hertz loan application was to be approved. It stressed that Lopez committed a serious violation of company rules when he issued the POs. Issue: Whether or not petitioner was illegally dismissed Held: The right of an employer to freely select or discharge his employee is a recognized prerogative of management; an employer cannot be compelled to continue employing one who has been guilty of acts inimical to its interests. When this happens, the employer can dismiss the employee for loss of confidence. At the same time, loss of confidence as a just cause of dismissal was never intended to provide employers with a blank check for terminating employment. Loss of confidence should ideally apply only (1) to cases involving employees occupying positions of trust and confidence, or (2) to situations where the employee is routinely charged with the care and custody of the employer's money or property. To the first class belong managerial employees, i.e., those vested with the powers and prerogatives to lay down management polices and/or to hire, transfer, suspend, lay-off, recall, discharge, assign or discipline employees, or effectively recommend such managerial actions. To the second class belong cashiers, auditors, property custodians, or those who, in the normal and routine exercise of their functions, regularly handle significant amounts of money or property. As branch manager, Lopez clearly occupies a "position of trust." His hold on his position and his stay in the service depend on the employer's trust and confidence in him and on his managerial services. According to the bank, Lopez betrayed this trust and confidence when he issued the subject POs without authority and despite the express directive to put the client's application on hold. In response, Lopez insists that he had sufficient authority to act as he did, as this authority is inherent in his position as bank manager. He points to his record in the past when he issued POs which were honored and paid by the bank and which constituted the arbiter's "overwhelming evidence" in support of the finding that "complainant's dismissal from work was without just cause, hence, illegal." As a bank official, the petitioner must have been aware that it is basic in every sound management that people under one's supervision and direction are bound to follow instructions or to inform their superior of what is going on in their respective areas of concern, especially regarding matters of vital interest to the enterprise. Under these facts, we find it undisputed that Lopez disobeyed the bank's directive to put the Hertz loan application on hold, and did not wait until its negative credit rating was cleared before proceeding to act. That he might have been proven right is immaterial. Neither does the submission that the bank honored and paid the first PO and even realized a profit from the transaction, mitigate the gravity of Lopez's defiance of the directive of higher authority on a business judgment. What appears clear is that the bank cannot in the future trust the petitioner as a manager who would follow directives from higher authorities on business policy and directions. The bank can be placed at risk if this kind of managerial attitude will be repeated, especially if it becomes an accepted rule among lower managers.

Page 183

LABOR RELATIONS
Atty. Jefferson M. Marquez
Under the circumstances of this case, we are convinced that the bank was justified in terminating Lopez's employment by reason of loss of trust and confidence. He admitted issuing the two POs, claiming merely that he had the requisite authority. He could not present any proof in this regard, however, except to say that it was part of his inherent duty as bank manager. He also claimed that the bank acquiesced to the issuance of the POs as it paid the first PO and the POs he issued in the past. This submission flies in the face of the bank's directive for him not to proceed unless matters are cleared with the bank's credit committee. The bank had a genuine concern over the issue as it found through its credit committee that Hertz was a credit risk. Whether the credit committee was correct or not is immaterial as the bank's direct order left Lopez without any authority to clear the loan application on his own. After this defiance, we cannot blame the bank for losing its confidence in Lopez and in separating him from the service.

77. St. Paul College Quezon City vs. Ancheta II, G.R. No. 169905, September 7, 2011 Facts: Respondent Remigio Michael Ancheta II and his wife, respondent Cynthia was hired by the same school as a part time teacher of the Mass Communication Department. A letter was sent to petitioner Sr. Bernadette and signed by some of the teachers of SPCQC, including the respondent spouses. The said letter contained the teachers' sentiments regarding two school policies, namely: first, the policy of penalizing the delay in encoding final grades and, second, the policy of withholding salaries of the teachers. The letter enumerated the departmental and instructional policies that respondent Remigio Michael failed to comply with, such as the late submission of final grades, failure to submit final test questions to the Program Coordinator, the giving of tests in the essay form instead of the multiple choice format as mandated by the school and the high number of students with failing grades in the classes that he handled. Mr. Ancheta failed 27 in a class of 44 students, and had a total number of 56 failures in his sections of Philippine History. Mrs. Ancheta failed 11 students in a class of 37, and had a total number of 16 failures in her 2 classes of Communication Theories. Subsequently, the respondent spouses received their respective letters of termination. Respondent spouses sent a letter for reconsideration to petitioner Sr. Lilia, but was eventually denied. Thus, respondent spouses filed a complaint for illegal dismissal with the NLRC but the Labor Arbiter dismissed the complaint. Issue: Whether or not the respondent spouses were validly dismissed Held: It is not disputed that respondent Remigio Michael was a full-time probationary employee and his wife, a part-time teacher of the petitioner school. A reality we have to face in the consideration of employment on probationary status of teaching personnel is that they are not governed purely by the Labor Code. The Labor Code is supplemented with respect to the period of probation by special rules found in the Manual of Regulations for Private Schools. On the matter of probationary period, Section 92 of these regulations provides: Section 92.Probationary Period. Subject in all instances to compliance with the Department and school requirements, the probationary period for academic personnel shall not be more than three (3) consecutive years of satisfactory service for those in the elementary and secondary levels, six (6) consecutive regular semesters of satisfactory service for those in the tertiary level, and nine (9) consecutive trimesters of satisfactory service for those in the tertiary level where collegiate courses are offered on a trimester basis. A probationary employee or probationer is one who is on trial for an employer, during which the latter determines whether or not he is qualified for permanent employment. The probationary employment is intended to afford the employer an opportunity to observe the fitness of a probationary employee while at work, and to ascertain whether he will become an efficient and productive employee. While the employer observes the fitness, propriety and efficiency of a probationer to ascertain whether he is qualified for permanent employment, the probationer, on the other hand, seeks to prove to the employer that he has the qualifications to meet the reasonable standards for permanent employment. Thus, the word probationary, as used to describe the period of employment, implies the purpose of the term or period, not its length. The common practice is for the employer and the teacher to enter into a contract, effective for one school year. At the end of the school year, the employer has the option not to renew the contract, particularly considering the teacher's performance. If the contract is not renewed, the employment relationship terminates. If the contract is renewed, usually for another school year, the probationary employment continues. Again, at the end of that period, the parties may opt to renew or not to renew the contract. If renewed, this second renewal of the contract for another school year would then be the last year since it would be the third school year of probationary employment. At the end of this third year,

Page 184

LABOR RELATIONS
Atty. Jefferson M. Marquez
the employer may now decide whether to extend a permanent appointment to the employee, primarily on the basis of the employee having met the reasonable standards of competence and efficiency set by the employer. For the entire duration of this three-year period, the teacher remains under probation. Upon the expiration of his contract of employment, being simply on probation, he cannot automatically claim security of tenure and compel the employer to renew his employment contract. Petitioner school contends that it did not extend the contracts of respondent spouses. It claims that, although, it has sent letters to the spouses informing them that the school is extending to them new contracts for the coming school year, the letters do not constitute as actual employment contracts but merely offers to teach on the said school year. Section 91 of the Manual of Regulations for Private Schools, states that: Section 91.Employment Contract. Every contract of employment shall specify the designation, qualification, salary rate, the period and nature of service and its date of effectivity, and such other terms and condition of employment as may be consistent with laws and rules, regulations and standards of the school. A copy of the contract shall be furnished the personnel concerned. It is important that the contract of probationary employment specify the period or term of its effectivity. The failure to stipulate its precise duration could lead to the inference that the contract is binding for the full three-year probationary period. Therefore, the letters sent by petitioner Sr. Racadio, which were void of any specifics cannot be considered as contracts. The closest they can resemble to are that of informal correspondence among the said individuals. As such, petitioner school has the right not to renew the contracts of the respondents, the old ones having been expired at the end of their terms. Assuming, arguendo, that the employment contracts between the petitioner school and the respondent spouses were renewed, this Court finds that there was a valid and just cause for their dismissal. The Labor Code commands that before an employer may legally dismiss an employee from the service, the requirement of substantial and procedural due process must be complied with. Under the requirement of substantial due process, the grounds for termination of employment must be based on just or authorized causes. The plain admissions of the charges against them were the considerations taken into account by the petitioner school in their decision not to renew the respondent spouses' employment contracts. This is a right of the school that is mandated by law and jurisprudence. It is the prerogative of the school to set high standards of efficiency for its teachers since quality education is a mandate of the Constitution. As long as the standards fixed are reasonable and not arbitrary, courts are not at liberty to set them aside. Schools cannot be required to adopt standards which barely satisfy criteria set for government recognition. The same academic freedom grants the school the autonomy to decide for itself the terms and conditions for hiring its teacher, subject of course to the overarching limitations under the Labor Code. The authority to hire is likewise covered and protected by its management prerogative the right of an employer to regulate all aspects of employment, such as hiring, the freedom to prescribe work assignments, working methods, process to be followed, regulation regarding transfer of employees, supervision of their work, lay-off and discipline, and dismissal and recall of workers.

78. Jumuad vs. Hi-Flyer Food, G.R. No. 187887, September 7, 2011 Facts: Petitioner Pamela Florentina P. Jumuad began her employment with respondent Hi-Flyer Food, Inc. as management trainee. Based on her performance through the years, Jumuad received several promotions until she became the area manager for the entire Visayas-Mindanao 1 region. Sometime on October 2004, Hi-Flyer conducted a food safety, service and sanitation audit and revealed several sanitation violations, such as the presence of rodents and the use of a defective chiller for the storage of food. When asked to explain, Jumuad first pointed out that she had already taken steps to prevent the further infestation of the branch. As to why the branch became infested with rodents, Jumuad faulted management's decision to terminate the services of the branch's pest control program and to rely solely on the pest control program of the mall. Hi-Flyer audited the account of one of its branches and found out irregularities of cash shortages. Another sanitation audit was made and signs of rodent gnawing/infestation were found. This time, Jumuad explained to management that she had been busy conducting management team meetings and that, at the date the audit was conducted, she had no scheduled visit. Hi-Flyer sent Jumuad an Irregularities Report and Notice of Charges. Jumuad submitted her written explanation. Hi-Flyer held an administrative hearing where Jumuad appeared with counsel. Apparently not satisfied with her explanations, Hi-Flyer served her a Notice of Dismissal. This prompted Jumuad to file a complaint against Hi-Flyer for illegal dismissal. Issue::

Page 185

LABOR RELATIONS
Atty. Jefferson M. Marquez
Whether Jumuad was illegally dismissed HELD: The Court is convinced that Jumuad cannot be dismissed on the ground of gross and habitual neglect of duty. The Court notes the apparent neglect of Jumuad of her duty in ensuring that her subordinates were properly monitored and that she had dutifully done all that was expected of her to ensure the safety of the consuming public. The nature of the anomalies uncovered were each of a different nature, the Court finds that her acts or lack of action in the performance of her duties is not born of habit. Despite saying this, it cannot be denied that Jumuad willfully breached her duties as to be unworthy of the trust and confidence of Hi-Flyer. Based on established facts, the mere existence of the grounds for the loss of trust and confidence justifies petitioner's dismissal. In the present case, the reports of Hi-Flyer show that there were anomalies committed in the branches managed by Jumuad. On the principle of respondeat superior or command responsibility alone, Jumuad may be held liable for negligence in the performance of her managerial duties. She may not have been directly involved in causing the cash shortages but her involvement in not performing her duty monitoring and supporting the day to day operations of the branches and ensure that all the facilities and equipment at the restaurant were properly maintained and serviced, could have truly prevented the whole debacle from ever occurring. Moreover, it is observed that rather than taking proactive steps to prevent the anomalies at her branches, Jumuad merely effected remedial measures. In the restaurant business where the health and well-being of the consuming public is at stake, this does not suffice. Thus, there is reasonable basis for Hi-Flyer to withdraw its trust in her and dismissing her from its service.

79. Nissan Motor Phils. Angelo, G.R. No. 164181, September 14, 2011 Facts: Respondent Victorino Angelo was employed by Nissan as one of its payroll staff. Respondent was not able to prepare the payroll for the said period as he was on sick leave. On his approved vacation leave, he was not again able to prepare the payroll for that particular period. Respondent received a Memorandum from the petitioner stating that the company is considering his dismissal from employment on the grounds of serious misconduct, willful disobedience and gross neglect of duties. It was stated in the memorandum that the supposed cut-off date for payroll purposes, the respondent went home early without finishing his work. The following day, he did not report for work, without any notice to the company or to any of his immediate superior section head. As a result, the company was doing the payroll thru IT and that the amount released to the employees were not accurate. Consequently, many employees got angry the deductions from salaries was not finished, the salaries of contractuals, apprentices were also not finished. Since the bank only reads account numbers of employees, the company experienced delay in the payroll processing. As a consequence of all these, the manufacturing employees, numbering about 350 people or about 65% of [Nissan's total population], have started to decline rendering overtime work, saying after their 15 days of work they received only less than P200 while some even received only P80. In sum, the company has suffered massive loss of opportunity to sell because of failure to produce in the production area due to non-availability of workers rendering overtime, high absenteeism rate among plant direct workers primarily due to the payroll problem. Finding that respondent's explanation was untrue and insufficient, petitioner issued a Notice of Termination. Respondent filed a complaint for illegal dismissal. Issue: Whether respondent was illegally dismissed Ruling: Going through the records, this Court found evidence to support the allegation of serious misconduct or insubordination. The language used by respondent in his Letter-Explanation is akin to a manifest refusal to cooperate with company officers, and resorted to conduct which smacks of outright disrespect and willful defiance of authority or insubordination. Accusatory and inflammatory language used by an employee to the employer or superior can be a ground for dismissal or termination. Another just cause cited by the petitioner is willful disobedience. Disobedience, to be a just cause for termination, must be willful or intentional, willfulness being characterized by a wrongful and perverse mental attitude rendering the employee's act inconsistent with proper subordination. This allegation of willful disobedience can still be adduced and proven from the same Letter-Explanation. Petitioner also dismissed respondent because of gross or habitual negligence. Neglect of duty, to be a ground for dismissal, must be both gross and habitual. In finding that petitioner was able to adduce evidence that would justify its dismissal of respondent, the NLRC correctly ruled that the latter's failure to turn over his functions to someone capable of performing the vital tasks which he could not effectively perform or undertake because of his heart ailment or condition constitutes gross neglect.

Page 186

LABOR RELATIONS
Atty. Jefferson M. Marquez
However, although the dismissal was legal, respondent is still entitled to a separation pay as a measure of financial assistance, considering his length of service and his poor physical condition which was one of the reasons he filed a leave of absence. As a general rule, an employee who has been dismissed for any of the just causes enumerated under Article 282 29 of the Labor Code is not entitled to separation pay. Although by way of exception, the grant of separation pay or some other financial assistance may be allowed to an employee dismissed for just causes on the basis of equity.

80. Phil. National Bank vs. Padao, G.R. No. 180849, November 16, 2011 Facts: On August 21, 1981, Padao was hired by PNB as a clerk at its Dipolog City Branch. He was later designated as a credit investigator in an acting capacity on November 9, 1993. On March 23, 1995, he was appointed regular Credit Investigator III, and was ultimately promoted to the position of Loan and Credit Officer IV. Sometime in 1994, PNB became embroiled in a scandal involving behest loans. A certain Sih Wat Kai complained to the Provincial Office of the Commission on Audit (COA) of Zamboanga del Norte that anomalous loans were being granted by its officers: Assistant Vice President (AVP) and Branch Manager Aurelio De Guzman (AVP de Guzman), Assistant Department Manager and Cashier Olson Sala (Sala), and Loans and Senior Credit Investigator Primitivo Virtudazo (Virtudazo). The questionable loans were reportedly being extended to select bank clients, among them Joseph Liong, Danilo Dangcalan, Jacinto Salac, Catherine Opulentisima, and Virgie Pango. The expos triggered the conduct of separate investigations by the COA and PNBs Internal Audit Department (IAD) from January to August 1995. Both investigations confirmed that the collateral provided in numerous loan accommodations were grossly over-appraised. The credit standing of the loan applicants was also fabricated, allowing them to obtain larger loan portfolios from PNB. These borrowers eventually defaulted on the payment of their loans, causing PNB to suffer millions in losses. On June 14, 1996, Padao was administratively charged with Dishonesty, Grave Misconduct, Gross Neglect of Duty, Conduct Prejudicial to the Best Interest of the Service, and violation of R.A. No. 3019. The case against Padao was grounded on his having allegedly presented a deceptively positive status of the business, credit standing/rating and financial capability of loan applicants Reynaldo and Luzvilla Baluma and eleven (11) others. It was later found that either said borrowers businesses were inadequate to meet their loan obligations, or that the projects they sought to be financed did not exist. Padao was also accused of having over-appraised the collateral of the spouses Gardito and Alma Ajero, the spouses Ibaba, and Rolly Pango. On January 10, 1997, after due investigation, PNB found Padao guilty of gross and habitual neglect of duty and ordered him dismissed from the bank. Issue: Won the court of appeals erred in treating the act of falsifying the credit and appraisal reports and that of merely affixing ones signature in a false report prepared by another as one and the same degree of misconduct which warrants legal dismissal. Held: In this case, Padao was dismissed by PNB for gross and habitual neglect of duties under Article 282 (b) of the Labor Code. Thus, in cases of regular employment, the employer is prohibited from terminating the services of an employee except for a just or authorized cause. Such just causes for which an employer may terminate an employee are enumerated in Article 282 of the Labor Code: (a) Serious misconduct or willful disobedience by the employee of the lawful orders of his employer or representative in connection with his work; (b) Gross and habitual neglect by the employee of his duties; (c) Fraud or willful breach by the employee of the trust reposed in him by his employer or duly authorized representative; (d) Commission of a crime or offense by the employee against the person of his employer or any immediate family member of his family or his

Page 187

LABOR RELATIONS
Atty. Jefferson M. Marquez
duly authorized representative; and (e) Other causes analogous to the foregoing. In this case, Padao was dismissed by PNB for gross and habitual neglect of duties under Article 282 (b) of the Labor Code. Gross negligence connotes want of care in the performance of ones duties, while habitual neglect implies repeated failure to perform ones duties for a period of time, depending on the circumstances. Gross negligence has been defined as the want or absence of or failure to exercise slight care or diligence, or the entire absence of care. It evinces a thoughtless disregard of consequences without exerting any effort to avoid them. Padaos repeated failure to discharge his duties as a credit investigator of the bank amounted to gross and habitual neglect of duties under Article 282 (b) of the Labor Code. He not only failed to perform what he was employed to do, but also did so repetitively and habitually, causing millions of pesos in damage to PNB. Thus, PNB acted within the bounds of the law by meting out the penalty of dismissal, which it deemed appropriate given the circumstances. However, Padao is not entitled to financial assistance. In Toyota Motor Phils. Corp. Workers Association v. NLRC, the Court reaffirmed the general rule that separation pay shall be allowed as a measure of social justice only in those instances where the employee is validly dismissed for causes other than serious misconduct, willful disobedience, gross and habitual neglect of duty, fraud or willful breach of trust, commission of a crime against the employer or his family, or those reflecting on his moral character. These five grounds are just causes for dismissal as provided in Article 282 of the Labor Code.

81. Tamsons Enterprises Inc. vs. CA, G.R. No. 192881, November 16, 2011 Facts: This case stemmed from a complaint for illegal dismissal with money claims filed by respondent Rosemarie L. Sy (Sy) before the Arbitration Branch, National Capital Region, NLRC, against petitioners Tamsons Enterprises, Inc. (Tamsons), Nelson Lee (Lee), the company President; and Lilibeth Ong (Ong) and Johnson Ng (Ng), her co-employees. On September 1, 2006, Sy was hired by Tamsons as Assistant to the President. Despite the title, she did not act as such because, per instruction of Lee, she was directed to act as payroll officer, though she actually worked as a payroll clerk. On February 24, 2007, four days before she completed her sixth month of working in Tamsons, Ng, the Sales Project Manager, called her to a meeting with him and Lee. During the meeting, they informed Sy that her services would be terminated due to inefficiency. She was asked to sign a letter of resignation and quitclaim. She was told not to report for work anymore because her services were no longer needed. On her last day of work, Ong humiliated her in front of her officemates by shouting at her and preventing her from getting her personal things or any other document from the office. During her pre-employment interview, Lee had nice comments about her good work experience and educational background. She was assured of a long-term employment with benefits. Throughout her employment, she earnestly performed her duties, had a perfect attendance record, worked even during brownouts and typhoons, and would often work overtime just to finish her work. Sy claimed that the remarks of her superiors about her alleged inefficiency were ill-motivated and made without any basis. She had been rendering services for almost six (6) months before she was arbitrarily and summarily dismissed. Her dismissal was highly suspicious as it took place barely four (4) days prior to the completion of her six-month probationary period. The petitioners did not show her any evaluation or appraisal report regarding her alleged inefficient performance. As she was terminated without an evaluation on her performance, she was deprived of the opportunity to be regularly part of the company and to be entitled to the benefits and privileges of a regular employee. Worse, she was deprived of her only means of livelihood. She further claimed was illegally terminated from service and insists that the petitioners cannot invoke her failure to qualify as she was not informed of the standards or criteria which she should have met for regular employment. Moreover, no proof was shown as to her alleged poor work performance. She was unceremoniously terminated to prevent her from becoming a regular employee and be entitled to the benefits as such. ISSUE:

Page 188

LABOR RELATIONS
Atty. Jefferson M. Marquez
WON the termination of Sy, a probationary employee, was valid or not. HELD: The Court finds the petition devoid of merit. The pertinent law governing the present case is Article 281 of the Labor Code which provides as follows: Art. 281. Probationary employment. Probationary employment shall not exceed six months from the date the employee started working, unless it is covered by an apprenticeship agreement stipulating a longer period. The services of an employee who has been engaged in a probationary basis may be terminated for a just cause or when he fails to qualify as a regular employee in accordance with reasonable standards made known by the employer to the employee at the time of his engagement. An employee who is allowed to work after a probationary period shall be considered a regular employee. (Underscoring supplied) It is settled that even if probationary employees do not enjoy permanent status, they are accorded the constitutional protection of security of tenure. This means they may only be terminated for a just cause or when they otherwise fail to qualify as regular employees in accordance with reasonable standards made known to them by the employer at the time of their engagement. Under the terms of the Labor Code, these standards should be made known to the [employees] on probationary status at the start of their probationary period, or xxx during which the probationary standards are to be applied. Of critical importance in invoking a failure to meet the probationary standards, is that the [employer] should show as a matter of due process how these standards have been applied. For failure of the petitioners to support their claim of unsatisfactory performance by Sy, this Court shares the view of the CA that Sys employment was unjustly terminated to prevent her from acquiring a regular status in circumvention of the law on security of tenure. As the Court previously stated, this is a common and convenient practice of unscrupulous employers to circumvent the law on security of tenure. Security of tenure, which is a right of paramount value guaranteed by the Constitution, should not be denied to the workers by such a stratagem. The Court can not permit such a subterfuge, if it is to be true to the law and social justice. The law is clear that in all cases of probationary employment, the employer shall make known to the employee the standards under which he will qualify as a regular employee at the time of his engagement. Where no standards are made known to the employee at that time, he shall be deemed a regular employee. The standards under which she would qualify as a regular employee not having been communicated to her at the start of her probationary period, Sy qualified as a regular employee. As held by this Court in the very recent case of Hacienda Primera Development Corporation v. Villegas. Being a regular employee whose termination was illegal, Sy is entitled to the twin relief of reinstatement and backwages granted by the Labor Code. Article 279 provides that an employee who is unjustly dismissed from work shall be entitled to reinstatement without loss of seniority rights and other privileges, to her full backwages, inclusive of allowances, and to her other benefits or their monetary equivalent computed from the time her compensation was withheld from her up to the time of actual reinstatement. Likewise, having been compelled to come to court and to incur expenses to protect her rights and interests, the award of attorneys fees is in order.

82. Concepcion vs. Minex Import Corp., G.R. No. 153569, January 24, 2012 Facts: Respondent is engaged in the retail of semi-precious stones, selling them in kiosks or stalls installed in various shopping centers. It employed the petitioner initially as a salesgirl then later on as supervisor. Working under her supervision were salesgirls Cristina Calung and Lida Baquilar. One day the petitioner and her salesgirls had sales of crystal items totaling P39,194.50. At the close of business that day, they conducted a cashcount of their sales proceeds, including those from the previous two days and determined their total for the three days to be P50,912.00. The petitioner wrapped the amount in a plastic bag and deposited it in the drawer of the locked wooden cabinet of the kiosk. The following day petitioner phoned respondent Vina Mariano to report that the P50,912.00 was missing, explaining how she and her salesgirls had placed the wrapped amount at the bottom of the cabinet the night before, and how she had found upon reporting to work that morning that the contents of the cabinet were in disarray and the money. Later, while the petitioner was giving a detailed statement on the theft to the security investigator of Harrison Plaza, Vina and Sylvia Mariano, her

Page 189

LABOR RELATIONS
Atty. Jefferson M. Marquez
superiors, arrived with a policeman who immediately placed the petitioner under arrest and brought her to a police station where she was investigated her and detained for a day. Subsequently petitioner filed a case for illegal dismissal against respondent and two days later respondent filed a criminal case for qualified theft against petitioner. The petitioner insisted on her innocence, reiterating that on the time the alleged crime took place she, together with her two salesgirls, had first counted the cash before placing it in a plastic bag that she deposited inside the drawer of the cabinet with the knowledge of the other salesgirls. One of the salesgirls however averred that she had left the petitioner alone because the latter had still to change her clothes; and that that was the first time that the petitioner had ever asked to be left behind, for they had previously left the kiosk together. Respondent Vina declared that the petitioner did not call the office of Minex for the pick-up of the P39,194.50 cash sales on that faithful day in violation of the standard operating procedure (SOP) requiring cash proceeds exceeding P10,000.00 to be reported for pick-up if the amount could not be deposited in the bank. After the preliminary investigation, the fiscal rendered a resolution finding probable cause for qualified theft and recommending the filing of an information against the petitioner. Thus, she was charged with qualified theft before the Regional Trial Court. The petitioner argued that there was no evidence at all upon which Minex could validly dismiss her considering that she had not yet been found guilty beyond reasonable doubt of the crime of qualified theft. Issues: Whether or not there was valid ground to terminate the petitioner. Ruling: The petitioners argument is not novel. It has been raised and rejected many times before on the basis that neither conviction beyond reasonable doubt for a crime against the employer nor acquittal after criminal prosecution was indispensable. Nor was a formal charge in court for the acts prejudicial to the interest of the employer a pre-requisite for a valid dismissal. The criminal charges initiated by the company against private respondents and the finding after preliminary investigation of their prima facie guilt of the offense charged constitute substantial evidence sufficient to warrant a finding by the Labor Tribunal of the existence of a just cause for their termination based on loss of trust and confidence. The Labor Tribunal need not have gone further as to require private respondents conviction of the crime charged, or inferred innocence on their part from their release from detention, which was mainly due to their posting of bail. While there is a valid ground to terminate petitioner, respondent however failed to comply with the requirements of due process prior to the termination under the implementing rules and regulations of the Labor Code. In all cases of termination of employment, the following standards of due process shall be substantially observed. For termination of employment based on just causes as defined in Article 282 of the Labor Code: (i) A written notice served on the employee specifying the ground or grounds for termination, and giving said employee reasonable opportunity within which to explain his side. (ii) A hearing or conference during which the employee concerned, with the assistance of counsel if he so desires is given opportunity to respond to the charge, present his evidence, or rebut the evidence presented against him. (iii) A written notice of termination served on the employee, indicating that upon due consideration of all the circumstances, grounds have been established to justify his termination In this case the respondents immediately had her arrested and investigated by the police authorities for qualified theft which constitutes a denial of her right to due process of law, consisting in the opportunity to be heard and to defend herself. In fact, their decision to dismiss her was already final even before the police authority commenced an investigation of the theft, the finality being confirmed by no less than Sylvia Mariano herself telling the petitioner during their phone conversation following the latters release from police custody that she (Sylvia) no longer wanted to see her. The fact that the petitioner was the only person suspected of being responsible for the theft aggravated the denial of due process.

83. Morales vs. Harbour Centre Port Terminal Inc., G.R. No. 174208, January 25, 2012 Facts:

Page 190

LABOR RELATIONS
Atty. Jefferson M. Marquez
Petitioner was hired by respondent Harbour Centre Port Terminal, Inc. (HCPTI) as an Accountant and Acting Finance Officer, with a monthly salary of P18,000. Morales was later on promoted to Division Manager of the Accounting Department, for which he was compensated a monthly salary of P33,700.00, plus allowances . Subsequent to HCPTIs transfer to its new offices at Vitas, Tondo, Manila Morales received an interoffice memorandum reassigning him to Operations Cost Accounting, tasked with the duty of monitoring and evaluating all consumables requests, gears and equipment related to the corporations operations and of interacting with its sub-contractor, Bulk Fleet Marine Corporation. Morales wrote Singson, protesting that his reassignment was a clear demotion since the position to which he was transferred was not even included in HCPTIs plantilla. In response to Morales grievance Singson issued an inter-office memorandum to the effect that transfer of employees is a management prerogative and that HCPTI had the right and responsibility to find the perfect balance between the skills and abilities of employees to the needs of the business. For the whole of the ensuing month Morales was absent from work and/or tardy. Singson issued to Morales a memorandum denominated as a First Warning reminding Morales that, as an employee of HCPTI, he was subject to its rules and regulations and could be disciplinarily dealt with pursuant to its Code of Conduct. In view of the absences Morales continued to incur, HCPTI issued a Second Warning and a Notice to Report for Work and Final Warning. In the meantime, Morales filed a complaint against HCPTI, Filart and Singson, for constructive dismissal. Respondent filed their position paper, arguing that Morales abandoned his employment and was not constructively dismissed. Issue: WON petitioner was constructively dismissed. Held: Constructive dismissal exists where there is cessation of work because "continued employment is rendered impossible, unreasonable or unlikely, as an offer involving a demotion in rank or a diminution in pay and other benefits. In cases of a transfer of an employee, the rule is settled that the employer is charged with the burden of proving that its conduct and action are for valid and legitimate grounds such as genuine business necessity and that the transfer is not unreasonable, inconvenient or prejudicial to the employee. If the employer cannot overcome this burden of proof, the employees transfer shall be tantamount to unlawful constructive dismissal. Petitioner was constructively dismissed. He was already occupying the position of Division Manager at HCPTIs Accounting Department and as a consequence of his promotion to said position. That the reassignment was a demotion is evident from Morales new duties which, far from being managerial in nature, were very simply and vaguely described as inclusive of monitoring and evaluating all consumables requests, gears and equipments related to [HCPTIs] operations as well as close interaction with its sub-contractor Bulk Fleet Marine Corporation. Morales demotion is evident from the fact that his reassignment entailed a transfer from a managerial position to one which was not even included in the corporations plantilla. While ordinarily management prerogative is not interfered with, it is also not absolute and is subject to limitations imposed by law, collective bargaining agreement, and general principles of fair play and justice. Thus, an employer may transfer or assign employees from one office or area of operation to another, provided there is no demotion in rank or diminution of salary, benefits, and other privileges, and the action is not motivated by discrimination, made in bad faith, or effected as a form of punishment or demotion without sufficient cause. Respondent however failed to justify the demotion of petitioner on the ground that it was reorganizing its business structure, this was evidenced by the fact that Morales was able to prove that HCPTIs existing plantilla did not include an Operations Cost Accounting Department and/or an Operations Cost Accountant. As the party belatedly seeking to justify the reassignment due to the supposed reorganization of its corporate structure, HCPTI, in contrast, did not even bother to show that it had implemented a corporate reorganization and/or approved a new plantilla of positions which included the one to which Morales was being transferred. On the allegation of abandonment on the part of petitioner As a just and valid ground for dismissal, at any rate, abandonment requires the deliberate, unjustified refusal of the employee to resume his employment, without any intention of returning. Since an employee like Morales who takes steps to protest his dismissal cannot logically be said to have abandoned his work, it is a settled doctrine that the filing of a complaint for illegal dismissal is inconsistent with abandonment of employment.

84. Mansion Printing Center vs. Bitara, G.R. No. 168120, January 25, 2012 Facts: Petitioner Mansion Printing Center is a single proprietorship registered under the name of its president and co-petitioner Clement Cheng. It is engaged in the printing of quality self-adhesive labels, brochures, posters, stickers, packaging and the like. Petitioners engaged the services of respondent as a helper (kargador), who was later as the company's sole driver tasked to pick-up raw materials for the printing business, collect account receivables and deliver the products to the clients within the delivery schedules.

Page 191

LABOR RELATIONS
Atty. Jefferson M. Marquez
Petitioners aver that the timely delivery of the products to the clients is one of the foremost considerations material to the operation of the business. It being so, they closely monitored the attendance of respondent. They noted his habitual tardiness and absenteeism. Thus, as early as 23 June 1999, petitioners issued a Memorandum requiring respondent to submit a written explanation why no administrative sanction should be imposed on him for his habitual tardiness. Despite having sent petitioners a letter stating his apologies and resolution to correct his tardiness and constant unauthorized absences, respondent still did not do what he had committed to do. Because of this, Davis Cheng, General Manager of the company and son of petitioner Cheng, issued another Memorandum (Notice to Explain) requiring respondent to explain why his services should not be terminated. He personally handed the Notice to Explain to respondent but the latter, after reading the directive, refused to acknowledge receipt thereof. He did not submit any explanation and, thereafter, never reported for work. Due to the actions of respondent, petitioner was urged to serve upon him another Memorandum, this time a Notice of Termination upon informing him that he was found grossly negligent of his duties. Respondent met with management, requesting that his termination from service would be reconsidered. After hearing the respondent, management still decided to implement the Memorandum, but out of the generosity of the management, respondent was offered financial assistance equivalent to P6,110.00 equivalent to his one month salary. Respondent demanded that he be given the amount equivalent to two (2) months' salary but the management declined as it believed it would, in effect, reward respondent for being negligent of his duties. Respondent filed a complaint illegal dismissal against petitioners before the LA, praying for reinstatement and payment of full backwages, legal holiday pay, SIL pay, damages and attorneys fees. LA dismissed the complaint for lack of merit. NLRC affirmed such decision and denied the motion for reconsideration. After raising the issue that NLRC rendered its decision with grave abuse of discretion and/or without or in excess of jurisdiction, the CA reversed the decision and found for the respondent. Hence, this petition. Issue: Whether or not respondents dismissal was illegal. Ruling: The Supreme Court rendered judgment for petitioners. In order to validly dismiss an employee, the employer is required to observe both substantive and procedural aspects the termination of employment must be based on a just or authorized cause of dismissal and the dismissal must be effected after due notice and hearing. Petitioners complied with substantive due process considering that his termination was not only due to his recent absences but this was because of his previous infractions capped by his recent unauthorized absences. Petitioners were even able to satisfactorily establish that respondents absences were indeed unauthorized. And they were able to establish that respondent was gross negligent of his duties as he was habitually tardy evidenced by his admission in his apology, and yet even after such apology, he continued to be tardy. Clearly, petitioners also complied with procedural due process since they served him with notice, before having terminated him. We said that procedural due process called for two requisites: (1) the employer must inform the employee of the specific acts or omissions for which his dismissal is sought; and (2) after the employee has been given the opportunity to be heard, the employer must inform him of the decision to terminate his employment Petitioners have repeatedly called the attention of respondent concerning his habitual tardiness, and the first Memorandum given to him required him to explain his tardiness. Having received this Memorandum, he submitted a letter offering an apology and undertook to henceforth report for duty on time, but afterward proved unavailing. The Supreme Court cited a previous jurisprudence defining gross negligence as "want of care in the performance of one's duties" 50 and habitual neglect as "repeated failure to perform one's duties for a period of time, depending upon the circumstances." 51 These are not overly technical terms, which, in the first place, are expressly sanctioned by the Labor Code of the Philippines. Bearing in mind that the tardiness and absences of respondent were not isolated incidents, but manifested a pattern of habituality, then petitioners were correct with their remedy of terminating him after having properly served him with a Notice of Termination that he refused to accept.

85. Manila Electric Co. vs. Beltran, G.R. No. 173774, January 30, 2012

Page 192

LABOR RELATIONS
Atty. Jefferson M. Marquez
Facts: Beltran was employed by MERALCO and at the time material to this case, she was holding the position of Senior Branch Clerk at MERALCOs Pasig branch. While rendering overtime work on September 28, 1996, a Saturday, Beltran accepted P15,164.48 from Collection Route Supervisor Berlin Marcos (Marcos), which the latter received from customer Andy Chang (Chang). The cash payment was being made in lieu of a returned check earlier issued as payment for Changs electric bill.Beltran received the payment and issued Auxiliary Receipt No. 87964 which she dated September 30, 1996, a Monday, instead of September 28, 1996. This was done to show that it was an accommodation, an accepted practice in the office. She thereafter placed the money and the original auxiliary receipt and other documents pertinent to the returned check underneath her other files inside the drawer of her table. Beltran, however, was only able to remit Changs payment on January 13, 1997. Thus, in a Memorandum dated January 16, 1997, she was placed under preventive suspension effective January 20, 1997 pending completion of an investigation. MERALCO considered as misappropriation or withholding of company funds her failure to immediately remit said payment in violation of its Code on Employee Discipline. Garcia, the Administrative Supervisor of MERALCOs Pasig branch, on the other hand, testified that while doing an accounting of all outstanding returned checks sometime in December 1996, she noticed that Changs returned check was missing. Upon further inquiry, she discovered that Chang had already redeemed the returned check after paying P15,164.48 to Beltran, who in turn issued an Auxiliary Receipt dated September 30, 1996. It was also discovered that the payment has not yet been remitted. This prompted her to inquire from Beltran on January 7, 1997 about the supposed payment and immediately ordered the remittance of the same. Beltran, however, failed to do so on that day and even on the next day when she reported for work. Beltran subsequently went on leave of absence on January 9 and 10, 1997. It was only on January 13, 1997 that the money with the pertinent documents was handed over. In a memorandum dated February 25, 1997, the investigator found Beltran guilty of misappropriating and withholding Changs payment of P15,164.48 and recommended her dismissal from service. Beltran filed a complaint for illegal dismissal against MERALCO. She argued that she had no intention to withhold company funds. Besides, it was not her customary duty to collect and remit payments from customers. She claimed good faith, believing that her acceptance of Changs payment is considered goodwill in favor of both MERALCO and its customer. If at all, her only violation was a simple delay in remitting the payment, which caused no considerable harm to the company. In a Decision of the Labor Arbiter regarded the penalty of dismissal as not commensurate to the degree of infraction committed as there was no adequate proof of misappropriation on the part of Beltran. If there was delay in Beltrans remittance of Changs payment, it was unintentional and same cannot serve as sufficient basis to conclude that there was misappropriation of company funds. In fact, Beltran did not even attempt to deny possession of, or refuse to hand in, the money. The Labor Arbiter thus gave compassionate consideration for the neglect to remit the money promptly, stating that it is excusable for Beltran to commit lapses in her work due to serious family difficulties. Upon appeal, the NLRC reversed the Labor Arbiters Decision and dismissed Beltrans complaint against MERALCO in its Decision. It found that Beltran withheld company funds by failing to remit it for almost four months. The NLRC thus ruled that MERALCO validly dismissed Beltran from the service in the exercise of its inherent right to discipline its employees. When Beltran brought the case to the CA the NLRCs ruling was reversed. The CA instead agreed with the findings of the Labor Arbiter that there were no serious grounds to warrant Beltrans dismissal. The CA held that the penalty of dismissal is harsh considering the infraction committed and Beltrans nine years of unblemished service with MERALCO. Issue: Whether or not Beltran dismissal is valid finding that she is guilty of withholding company funds. Ruling: Supreme Court support the CAs finding that there are no sufficient grounds to warrant Beltrans dismissal. For loss of trust and confidence to be a valid ground for dismissal, it must be based on a willful breach of trust and founded on clearly established facts. A breach is willful if it is done intentionally, knowingly and purposely, without justifiable excuse, as distinguished from an act done carelessly, thoughtlessly, heedlessly or inadvertently. In addition, loss of trust and confidence must rest on substantial grounds and not on the employers arbitrariness, whims, caprices or suspicion. In the case at bench, Beltran attributed her delay in turning over Changs payment to her difficult family situation as she and her husband were having marital problems and her child was suffering from an illness. Admittedly, she was reminded of Changs payment by her supervisor on January 7, 1997 but denied having been ordered to remit the money on that day. She then reasoned that her continued delay was caused by an inevitable need to take a leave of absence for her to attend to the needs of her child who was suffering from asthma. MERALCO cannot claim or conclude that Beltran misappropriated the money based on mere suspicion. And even if Beltran delayed handing over the funds to the company, MERALCO still has the burden of proof to show clearly that such act of negligence is sufficient to justify termination from employment. Beltran was

Page 193

LABOR RELATIONS
Atty. Jefferson M. Marquez
remiss in her duties for her failure to immediately turn over Changs payment to the company. Such negligence, however, is not sufficient to warrant separation from employment. To justify removal from service, the negligence should be gross and habitual. Gross negligence x x x is the want of even slight care, acting or omitting to act in a situation where there is duty to act, not inadvertently but willfully and intentionally, with a conscious indifference to consequences insofar as other persons may be affected. Habitual neglect, on the other hand, connotes repeated failure to perform ones duties for a period of time, depending upon the circumstances. No concrete evidence was presented by MERALCO to show that Beltrans delay in remitting the funds was done intentionally. Neither was it shown that same is willful, unlawful and felonious contrary to MERALCOs finging as stated in the letter of termination it sent to Beltran. Surely, Beltrans single and isolated act of negligence cannot justify her dismissal from service.

86. Bank of Lubao vs. Manabat, G.R. No. 188722, February 1, 2012 Facts: Sometime in 2001, Rommel J. Manabat (respondent) was hired by petitioner Bank of Lubao, a rural bank, as a Market Collector. Subsequently, the respondent was assigned as an encoder at the Bank of Lubaos Sta. Cruz Extension Office, which he manned together with two other employees, teller Susan P. Lingad (Lingad) and May O. Manasan. As an encoder, the respondents primary duty is to encode the clients deposits on the banks computer after the same are received by Lingad. In November 2004, an initial audit on the Bank of Lubaos Sta. Cruz Extension Office conducted by the petitioner revealed that there was a misappropriation of funds in the amount of P3,000,000.00, more or less. Apparently, there were transactions entered and posted in the passbooks of the clients but were not entered in the banks book of accounts. Further audit showed that there were various deposits which were entered in the banks computer but were subsequently reversed and marked as error in posting. The respondent, through a memorandum sent by the petitioner, was asked to explain in writing the discrepancies that were discovered during the audit. Respondent submitted to the petitioner his letter-explanation which, in essence, asserted that there were times when Lingad used the banks computer while he was out on errands. Administrative hearing was conducted by the banks investigating committee where the respondent was further made to explain his side. Subsequently, the investigating committee concluded that the respondent conspired with Lingad in making fraudulent entries disguised as error corrections in the banks computer.Thats why the petitioner filed several criminal complaints for qualified theft against Lingad and the respondent with the Municipal Trial Court (MTC) of Lubao, Pampanga. Thereafter, citing serious misconduct tantamount to willful breach of trust as ground, it terminated the respondents employment. But respondent filed a Complaint for illegal dismissal with the Regional Arbitration Branch of the National Labor Relations Commission (NLRC) in San Fernando City, Pampanga. In the said complaint, the respondent, to bolster his claim that there was no valid ground for his dismissal, averred that the charge against him for qualified theft was dismissed for lack of sufficient basis to conclude that he conspired with Lingad. The respondent sought an award for separation pay, full backwages, 13thmonth pay for 2004 and moral and exemplary damages. The Labor Arbiter (LA) rendered a decision sustaining the respondents claim of illegal dismissal thus ordering the petitioner to reinstate the respondent to his former position and awarding the latter backwages. The LA opined that the petitioner failed to adduce substantial evidence that there was a valid ground for the respondents dismissal. The NLRC rendered a Decision affirming the Decision of the LA. The NLRC held that it was sufficiently established that only Lingad was the one responsible for the said misappropriations However, the CA held that the respondent is entitled to separation pay equivalent to one-month salary for every year of service in lieu of reinstatement and backwages to be computed from the time of his illegal dismissal until the finality of the said decision. The CA agreed with the LA and the NLRC that the petitioner failed to establish by substantial evidence that there was indeed a valid ground for the respondents dismissal. Nevertheless, the CA held that the petitioner should pay the respondent separation pay since the latter did not pray for reinstatement before the LA and that the same would be in the best interest of the parties considering the animosity and antagonism that exist between them. Issues: Whether or not Rommel J. Manabat was illegally dismissed and will be entitled to separation pay in lieu of reinstatement and payment of backwages. Ruling: This Court notes that the LA, the NLRC and the CA unanimously ruled that the respondent was illegally dismissed. Factual findings of quasi-

Page 194

LABOR RELATIONS
Atty. Jefferson M. Marquez
judicial bodies like the NLRC, if supported by substantial evidence, are accorded respect and even finality by this Court, more so when they coincide with those of the LA. Such factual findings are given more weight when the same are affirmed by the CA. We find no reason to depart from the foregoing rule. Here, Court agree with the CA that the relations between the parties had been already strained thereby justifying the grant of separation pay in lieu of reinstatement in favor of the respondent. Undoubtedly, the petitioners filing of various criminal complaints against the respondent for qualified theft and the subsequent filing by the latter of the complaint for illegal dismissal against the latter, taken together with the pendency of the instant case for more than six years, had caused strained relations between the parties. considering that the respondents former position as bank encoder involves the handling of accounts of the depositors of the Bank of Lubao, it would not be equitable on the part of the petitioner to be ordered to maintain the former in its employ since it may only inspire vindictiveness on the part of the respondent. Then the refusal of the respondent to be re-admitted to work is in itself indicative of the existence of strained relations between him and the petitioner. The backwages that should be awarded to the respondent should be modified. Employees who are illegally dismissed are entitled to full backwages, inclusive of allowances and other benefits or their monetary equivalent, computed from the time their actual compensation was withheld from them up to the time of their actual reinstatement. But if reinstatement is no longer possible, the backwages shall be computed from the time of their illegal termination up to the finality of the decision. Thus, it is but fair that the backwages that should be awarded to the respondent be computed from the time that the respondent was illegally dismissed until the time when he was required to report for work, i.e. from September 1, 2005 until May 4, 2007. It is only during the said period that the respondent is deemed to be entitled to the payment of backwages.

87. Canadian Opportunities Unlimited vs. Dalangin, G.R. No. 172223, February 6, 2012 Facts: Dalangin was hired by Canadian Opportunities Unlimited Inc. on October 2001 as Immigration and Legal Manager. He was placed on probation for six months. His tasks involved principally the review of the clients' applications for immigration to Canada to ensure that they are in accordance with Canadian and Philippine laws. Barely a month later, on October 27, 2001, the company terminated Dalangin's employment, declaring him "unfit" and "unqualified" to continue his work. They alleged that during his brief employment in the company, Dalangin showed lack of enthusiasm towards his work and was indifferent towards his co-employees and the company clients. Dalangin refused to comply with the company's policies and procedures, routinely taking long lunch breaks, exceeding the one hour allotted to employees, and leaving the company premises without informing his immediate superior, only to call the office later and say that he would be unable to return because he had some personal matters to attend to. He also showed lack of interpersonal skills and initiative. Dalangin's lack of interest in the company was further manifested when he refused to attend company-sponsored seminars designed to acquaint or update the employees with the company's policies and objectives. Dalangin alleged that the company required its employees to attend a "Values Formation Seminar" scheduled for October 27, 2001 (a Saturday) at 2:00 p.m. onwards. He inquired from Abad, the Chief Operating Officer, about the subject and purpose of the seminar and when he learned that it bore no relation to his duties, he told Abad that he would not attend the seminar as he would have to leave at 2:00 p.m. in order to be with his family in the province. Dalangin claimed that Abad insisted that he attend the seminar. On October 26, 2001, Abad required him to explain why he could not attend the seminar scheduled for October 27, 2001 and the other forthcoming seminars. The following day, October 27, 2001, Abad informed him that Mr. Yadi N. Sichani, the company's Managing Director, wanted to meet with him regarding the matter. At that meeting, Sichani told him that his services were being terminated because Sichani could not keep in his company "people who are hard-headed and who refuse to follow orders from management." The Labor Arbiter declared Dalangin's dismissal illegal. He found that the charges against Dalangin were not established by clear and substantial proof. The NLRC reversed the labor arbiter's ruling. It found Dalangin's dismissal to be a valid exercise of the company's management prerogative because Dalangin failed to meet the standards for regular employment. The CA held that the NLRC erred when it ruled that Dalangin was not illegally dismissed. As the labor arbiter did, the CA found that the company failed to support, with substantial evidence, its claim that Dalangin failed to meet the standards to qualify as a regular employee. Issues: Whether Dalangin was illegally dismissed. Whether the requirements of notice and hearing in employee dismissals are applicable to Dalangin's case, being a probationary employee.

Page 195

LABOR RELATIONS
Atty. Jefferson M. Marquez
Held: The SC disagreed with the CA. The company is not liable for illegal dismissal. A probationary employee, as understood under Article 281 of the Labor Code, is one who is on trial by an employer, during which, the latter determines whether or not he is qualified for permanent employment. A probationary appointment gives the employer an opportunity to observe the fitness of a probationer while at work, and to ascertain whether he would be a proper and efficient employee. The essence of a probationary period of employment fundamentally lies in the purpose or objective of both the employer and the employee during the period. While the employer observes the fitness, propriety and efficiency of a probationer to ascertain whether he is qualified for permanent employment, the latter seeks to prove to the former that he has the qualifications to meet the reasonable standards for permanent employment. The word 'probationary,' as used to describe the period of employment, implies the purpose of the term or period, but not its length." Thus, the fact that Dalangin was separated from the service after only about four weeks does not necessarily mean that his separation from the service is without basis. Dalangins refusal to attend the seminar brings into focus and validates what was wrong with him, as Abad narrated in her affidavit and as reflected in the termination of employment memorandum. It highlights his lack of interest in familiarizing himself with the company's objectives and policies. In the face of Abad's direct statements, as well as those of his co-employees, it is puzzling that Dalangin chose to be silent about the charges, other than saying that the company could not cite any policy he violated. All along, he had been complaining that he was not able to explain his side, yet from the labor arbiter's level, all the way to this Court, he offered no satisfactory explanation of the charges. In this light, coupled with Dalangin's adamant refusal to attend the company's "Values Formation Seminar" and a similar program scheduled earlier, we find credence in the company's submission that Dalangin was unfit to continue as its Immigration and Legal Manager. As we stressed earlier, we are convinced that the company had seen enough from Dalangin's actuations, behavior and deportment during a four-week period to realize that Dalangin would be a liability rather than an asset to its operations. To our mind, four weeks was enough for the company to assess Dalangin's fitness for the job and he was found wanting. In separating Dalangin from the service before the situation got worse, we find the company not liable for illegal dismissal. As for the second issue, there is no need for a hearing. Section 2, Rule I, Book VI of the Labor Code's Implementing Rules and Regulations provides: If the termination is brought about by the completion of a contract or phase thereof, or by failure of an employee to meet the standards of the employer in the case of probationary employment, it shall be sufficient that a written notice is served the employee within a reasonable time from the effective date of termination. However, the notice served on Dalangin did not give him a reasonable time, from the effective date of his separation, as required by the rules. He was dismissed on the very day the notice was given to him, or on October 27, 2001. Although we cannot invalidate his dismissal in light of the valid cause for his separation, the company's non-compliance with the notice requirement entitles Dalangin to indemnity, in the form of nominal damages of P10,000.00.

88. Manila Electric Co. vs. Gala, G.R. No. 191288 & 191304, March 7, 2012 Facts: Respondent Jan Carlo Gala was hired by petitioner Meralco as a probationary lineman on March 2, 2006. On July 27, 2006, barely four months on the job, Gala was dismissed for alleged complicity in pilferages of Meralcos electrical supplies, particularly, for the incident which took place on May 25, 2006. On that day, Gala and other Meralco workers were instructed to replace a worn-out electrical pole at the Pacheco Subdivision in Valenzuela City. While the Meralco crew was at work, one Noberto Llanes, a non-Meralco employee, arrived. He appeared to be known to the Meralco foremen as they were seen conversing with him. Llanes boarded the trucks, without being stopped, and took out what were later found as electrical supplies. Aside from Gala, the foremen and the other linemen who were at the worksite when the pilferage happened were later charged with misconduct and dishonesty for their involvement in the incident. Unknown to them, a Meralco surveillance task force was monitoring their activities and recording everything with a video camera. Gala denied involvement in the pilferage, contending that even if his superiors might have committed a wrongdoing, he had no participation in what they did. He claimed that: (1) he was at some distance away from the trucks when the pilferage happened; (2) he did not have an inkling that an illegal activity was taking place since his supervisors were conversing with Llanes, giving him the impression that they knew him; (3) he did not call the attention of his superiors because he was not in a position to do so as he was a mere lineman; and (4) he was just following instructions in connection with his work and had no control in the disposition of company supplies and materials. He maintained that his mere

Page 196

LABOR RELATIONS
Atty. Jefferson M. Marquez
presence at the scene of the incident was not sufficient to hold him liable as a conspirator. Despite Galas explanation, Meralco terminated his employment. Gala responded by filing an illegal dismissal complaint against Meralco. The Labor Arbiter dismissed the complaint for lack of merit. She held that Galas participation in the pilferage of Meralcos property rendered him unqualified to become a regular employee. On appeal, the NLRC reversed the labor arbiters ruling. It found that Gala had been illegally dismissed, since there was no concrete showing of complicity with the alleged misconduct/dishonesty. The CA denied Meralcos petition for lack of merit and partially granted Galas petition. It concurred with the NLRC that Gala had been illegally dismissed. It opined that nothing in the records show Galas knowledge of or complicity in the pilferage. Issue: Whether or not Gala was illegally dismissed. Held: We find merit in the petition. Contrary to the conclusions of the CA and the NLRC, there is substantial evidence supporting Meralcos position that Gala had become unfit to continue his employment with the company. Gala was found, after an administrative investigation, to have failed to meet the standards expected of him to become a regular employee and this failure was mainly due to his undeniable knowledge, if not participation, in the pilferage activities done by their group, all to the prejudice of the Companys interests. As probationary employee, his overall job performance and his behavior were being monitored and measured in accordance with the standards (i.e., the terms and conditions) laid down in his probationary employment agreement. Under paragraph 8 of the agreement, he was subject to strict compliance with, and non-violation of the Company Code on Employee Discipline, Safety Code, rules and regulations and existing policies. Paragraph 10 required him to observe at all times the highest degree of transparency, selflessness and integrity in the performance of his duties and responsibilities, free from any form of conflict or contradicting with his own personal interest. On the whole, the totality of the circumstances obtaining in the case convinces us that Gala could not but have knowledge of the pilferage of company electrical supplies on May 25, 2006; he was complicit in its commission, if not by direct participation, certainly, by his inaction while it was being perpetrated and by not reporting the incident to company authorities. Thus, we find substantial evidence to support the conclusion that Gala does not deserve to remain in Meralcos employ as a regular employee. He violated his probationary employment agreement, especially the requirement for him to observe at all times the highest degree of transparency, selflessness and integrity in the performance of their duties and responsibilities. He failed to qualify as a regular employee.

89. Aro vs. NLRC, G.R. No. 174792, March 7, 2012 Facts: Several employees of private respondent Benthel Development Corporation, including the petitioners, filed a Complaint for illegal dismissal with various money claims and prayer for damages against the latter, in the NLRC Arbitration Branch No. VII in Cebu City and docketed as RAB Case No. 07-09-1222-97/12-1609-97. Thereafter, Labor Arbiter Ernesto F. Carreon rendered a decision finding private respondent guilty of illegal dismissal and ordering it to pay its thirty-six (36) employees P446,940.00 as separation pay. The employees, including the petitioners herein, appealed from the said decision. The NLRC, in NLRC Case No. V-000399-98, affirmed the decision of Labor Arbiter Carreon in its Decision dated January 12, 1999, with the modification that private respondent pay backwages computed from the respective dates of dismissal until finality of the decision. Private respondent, unsatisfied with the modification made by the NLRC, filed a motion for reconsideration with the contention that, since it has been found by the Labor Arbiter and affirmed in the assailed decision that the employees were project employees, the computation of backwages should be limited to the date of the completion of the project and not to the finality of the decision. The NLRC, however, denied the motion ruling that private respondent failed to establish the date of the completion of the project. As a recourse, private respondent filed a petition for certiorari with the CA, alleging that public respondent committed grave abuse of discretion in promulgating its assailed decision and denying its motion for reconsideration. The CA granted the petition, therefore, annulling and setting aside the decision and resolution of the NLRC as to the award for backwages and remanded the case to the same public respondent for the proper computation of the backwages due to each of the petitioners herein. Issue: Whether or not the CA gravely abused its discretion in declaring petitioners as project employees, such that the petitioners-employees are entitled to payment of backwages until the date of the completion of the project.

Page 197

LABOR RELATIONS
Atty. Jefferson M. Marquez
Rulings: It is not disputed that petitioners were hired for the construction of the Cordova Reef Village Resort in Cordova, Cebu. By the nature of the contract alone, it is clear that petitioners' employment was to carry out a specific project. Hence, the CA did not commit grave abuse of discretion when it affirmed the findings of the Labor Arbiter It is settled that, without a valid cause, the employment of project employees cannot be terminated prior to expiration. Otherwise, they shall be entitled to reinstatement with full backwages. However, if the project or work is completed during the pendency of the ensuing suit for illegal dismissal, the employees shall be entitled only to full backwages from the date of the termination of their employment until the actual completion of the work. While it may be true that in the proceedings below the date of completion of the project for which the private respondents were hired had not been clearly established, it constitutes grave abuse of discretion on the part of the public respondent for not determining for itself the date of said completion instead of merely ordering payment of backwages until finality of its decision. Therefore, being project employees, petitioners are only entitled to full backwages, computed from the date of the termination of their employment until the actual completion of the work. Illegally dismissed workers are entitled to the payment of their salaries corresponding to the unexpired portion of their employment where the employment is for a definite period. In this case, as found by the CA, the Cordova Reef Village Resort project had been completed in October 1996 and private respondent herein had signified its willingness, by way of concession to petitioners, to set the date of completion of the project as March 18, 1997; hence, the latter date should be considered as the date of completion of the project for purposes of computing the full backwages of petitioners.

90. Ymbong vs. ABS-CBN Broadcasting Corp., G.R. No. 184885, March 7, 2012 Facts: Petitioner Ernesto G. Ymbong started working for ABS-CBN Broadcasting Corporation (ABS-CBN) in 1993 at its regional station in Cebu as a television talent, co-anchoring Hoy Gising and TV Patrol Cebu. His stint in ABS-CBN later extended to radio when ABS-CBN Cebu launched its AM station DYAB in 1995 where he worked as drama and voice talent, spinner, scriptwriter and public affairs program anchor. Like Ymbong, Leandro Patalinghug also worked for ABS-CBN Cebu. Starting 1995, he worked as talent, director and scriptwriter for various radio programs aired over DYAB. On January 1, 1996, the ABS-CBN Head Office in Manila issued Policy No. HR-ER-016 or the "Policy on Employees Seeking Public Office." The pertinent portions read: 1.Any employee who intends to run for any public office position, must file his/her letter of resignation, at least thirty (30) days prior to the official filing of the certificate of candidacy either for national or local election. xxx xxx xxx 3.Further, any employee who intends to join a political group/party or even with no political affiliation but who intends to openly and aggressively campaign for a candidate or group of candidates (e.g., publicly speaking/endorsing candidate, recruiting campaign workers, etc.) must file a request for leave of absence subject to management's approval. For this particular reason, the employee should file the leave request at least thirty (30) days prior to the start of the planned leave period. Because of the impending May 1998 elections and based on his immediate recollection of the policy at that time, Dante Luzon, Assistant Station Manager of DYAB issued the following memorandum: TO:ALL CONCERNED FROM:DANTE LUZON DATE:MARCH 25, 1998 SUBJECT:AS STATED Please be informed that per company policy, any employee/talent who wants to run for any position in the coming election will have to file a leave

Page 198

LABOR RELATIONS
Atty. Jefferson M. Marquez
of absence the moment he/she files his/her certificate of candidacy. The services rendered by the concerned employee/talent to this company will then be temporarily suspended for the entire campaign/election period. After the issuance of the March 25, 1998 Memorandum, Ymbong got in touch with Luzon. Luzon claims that Ymbong approached him and told him that he would leave radio for a couple of months because he will campaign for the administration ticket. It was only after the elections that they found out that Ymbong actually ran for public office himself at the eleventh hour. Ymbong, on the other hand, claims that in accordance with the March 25, 1998 Memorandum, he informed Luzon through a letter that he would take a few months leave of absence from March 8, 1998 to May 18, 1998 since he was running for councilor of Lapu-Lapu City. As regards Patalinghug, Patalinghug approached Luzon and advised him that he will run as councilor for Naga, Cebu. According to Luzon, he clarified to Patalinghug that he will be considered resigned and not just on leave once he files a certificate of candidacy Later, Ymbong and Patalinghug both tried to come back to ABS-CBN Cebu. According to Luzon, he informed them that they cannot work there anymore because of company policy. As a result, they filed as illegal dismissal suit against ABS-CBN. Issues: (1) Whether Policy No. HR-ER-016 is valid; (2) Whether the March 25, 1998 Memorandum issued by Luzon superseded Policy No. HR-ER-016; and (3) Whether Ymbong, by seeking an elective post, is deemed to have resigned and not dismissed by ABS-CBN. Rulings: Policy No. HR-ER-016 is valid. We have consistently held that so long as a company's management prerogatives are exercised in good faith for the advancement of the employer's interest and not for the purpose of defeating or circumventing the rights of the employees under special laws or under valid agreements, this Court will uphold them. In the instant case, ABS-CBN validly justified the implementation of Policy No. HR-ER-016. It is well within its rights to ensure that it maintains its objectivity and credibility and freeing itself from any appearance of impartiality so that the confidence of the viewing and listening public in it will not be in any way eroded. Even as the law is solicitous of the welfare of the employees, it must also protect the right of an employer to exercise what are clearly management prerogatives. The free will of management to conduct its own business affairs to achieve its purpose cannot be denied. Policy No. HR-ER-016 was not superseded by the March 25, 1998 Memorandum The CA correctly ruled that though Luzon, as Assistant Station Manager for Radio of ABS-CBN, has policy-making powers in relation to his principal task of administering the network's radio station in the Cebu region, the exercise of such power should be in accord with the general rules and regulations imposed by the ABS-CBN Head Office to its employees. Clearly, the March 25, 1998 Memorandum issued by Luzon which only requires employees to go on leave if they intend to run for any elective position is in absolute contradiction with Policy No. HR-ER-016 issued by the ABS-CBN Head Office in Manila which requires the resignation, not only the filing of a leave of absence, of any employee who intends to run for public office. Having been issued beyond the scope of his authority, the March 25, 1998 Memorandum is therefore void and did not supersede Policy No. HR-ER-016. Ymbong is deemed resigned when he ran for councilor. As Policy No. HR-ER-016 is the subsisting company policy and not Luzon's March 25, 1998 Memorandum, Ymbong is deemed resigned when he ran for councilor. Ymbong's overt act of running for councilor of Lapu-Lapu City is tantamount to resignation on his part. He was separated from ABS-CBN not because he was dismissed but because he resigned. Since there was no termination to speak of, the requirement of due process in dismissal cases cannot be applied to Ymbong. Thus, ABS-CBN is not duty-bound to ask him to explain why he did not tender his resignation before he ran for public office as mandated by the subject company policy. In addition, we do not subscribe to Ymbong's claim that he was not in a position to know which of the two issuances was correct. Ymbong most likely than not, is fully aware that the subsisting policy is Policy No. HR-ER-016 and not the March 25, 1998 Memorandum and it was for this reason that, as stated by Luzon in his Sworn Statement, he only told the latter that he will only campaign for the administration ticket and not actually run for an elective post.

Page 199

LABOR RELATIONS
Atty. Jefferson M. Marquez
91. Blue Sky Trading Co. vs. Blas, G.R. No. 190559, March 7, 2012 Facts: Petitioner Blue Sky Trading Company, Inc. (Blue Sky) is a duly registered domestic corporation engaged in the importation and sale of medical supplies and equipment. The respondents Arlene P. Blas (Arlene) and Joseph D. Silvano (Joseph) were regular employees of Blue Sky and they respectively held the positions of stock clerk and warehouse helper before they were dismissed from service on February 5, 2005. An incident occurred where six pairs of intensifying screens were missing. On February 3, 2005, Jean B. De La Paz (Jean), Human Resource Department Head issued notices to explain/preventive suspension to Arlene, Joseph, delivery personnel Jayde Tano-an (Jayde) and maintenance personnel/driver Wilfredo Fasonilao (Wilfredo). The notices informed them that they were being accused of gross dishonesty in connection with their alleged participation in and conspiracy with other employees in committing theft against company property, specifically relative to the loss of the six intensifying screens. On February 5, 2005, Jean issued to Arlene, Joseph, Jayde and Wilfredo notices of dismissal for cause stating therein that evidence that they had conspired with each other to commit theft against company property was too glaring to ignore. Blue Sky had lost its trust and confidence on them and as an act of self-preservation, their termination from service was in order. On February 8, 2005, Arlene, Joseph, Helario, Jayde and Wilfredo filed with the National Labor Relations Commission (NLRC) a complaint for illegal dismissal and suspension, underpayment of overtime pay, and non-payment of emergency cost of living allowance (ECOLA), with prayers for reinstatement and payment of full backwages. Meanwhile, an entrapment operation was conducted by the police during which Jayde and Helario were caught allegedly attempting to sell to an operative an ultrasound probe worth around P400,000.00 belonging to Blue Sky. Though eventually, Jayde and Helario executed affidavits of desistance stating that their dismissal was for cause. The Labor Arbiter denied the claims of the respondents of illegal suspension and dismissal since they failed in their duties to exercise utmost protection, care, or custody of respondent's property. Hence, their dismissal from the service is warranted. The first decision of the NLRC ruled that respondents were not holding positions of trust and must therefore be reinstated and be paid their backwages. Their second decision on the other hand reversed the previous one which in turn reinstated the Labor Arbiters dismissal of the complaint saying that respondents were holding positions of trust and that the loss of the companys property are substantially proven. The CA on the other hand found merit on their claims, though found respondents to have positions of trust and confidence, petitioner in this case failed to sufficiently establish the charge against respondents which was the basis for its loss of trust and confidence that warranted their dismissal. Issue: Whether or not respondents Blas and Silvano committed a breach of trust Ruling: The rule is long and well settled that, in illegal dismissal cases like the one at bench, the burden of proof is upon the employer to show that the employee's termination from service is for a just and valid cause. The employer's case succeeds or fails on the strength of its evidence and not on the weakness of that adduced by the employee, in keeping with the principle that the scales of justice should be tilted in favor of the latter in case of doubt in the evidence presented by them. Often described as more than a mere scintilla, the quantum of proof is substantial evidence which is understood as such relevant evidence as a reasonable mind might accept as adequate to support a conclusion, even if other equally reasonable minds might conceivably opine otherwise. Failure of the employer to discharge the foregoing onus would mean that the dismissal is not justified and therefore illegal. We find no error in the CA's findings that the petitioners had not adequately proven by substantial evidence that Arlene and Joseph indeed participated or cooperated in the commission of theft relative to the six missing intensifying screens so as to justify the latter's termination from employment on the ground of loss of trust and confidence. We note that the parties disagree as to what tasks were actually and regularly performed by Arlene and Joseph. They are at odds as to the issue of whether or not Arlene and Joseph had custody of the missing screens. We observe though that neither of the parties presented any documentary evidence, such as employment contracts, to establish their claims relative to the actual nature of Arlene and Joseph's daily tasks. The petitioners also argue that if Arlene and Joseph had not been grossly negligent in the performance of their duties, Blue Sky would not have incurred the loss. We observe though that in the notices sent to Arlene and Joseph, first charging them with theft, and later, informing them of their dismissal from service, gross negligence was not stated therein as a ground. Hence, Arlene and Joseph could not have defended themselves against the charge of gross negligence. They cannot be dismissed on that ground lest due process be violated.

Page 200

LABOR RELATIONS
Atty. Jefferson M. Marquez
Other Matters: (For Discussion Purposes) Impropriety of the Preventive Suspension The purpose of the suspension is to prevent an employee from causing harm or injury to his colleagues and to the employer. The maximum period of suspension is 30 days, beyond which the employee should either be reinstated or be paid wages and benefits due to him. While we do not agree with Blue Sky's subsequent decision to terminate them from service, we find no impropriety in its act of imposing preventive suspension upon the respondents since the period did not exceed the maximum imposed by law and there was a valid purpose for the same. In lieu of reinstatement, separation pay If reinstatement proves impracticable, and hardly in the best interest of the parties, perhaps due to the lapse of time since the employee's dismissal, or if the employee decides not to be reinstated, the latter should be awarded separation pay in lieu of reinstatement. In the case at bar, Arlene and Joseph were dismissed from service on February 5, 2005. We find that the lapse of more than seven years already renders their reinstatement impracticable. Further, from the stubborn stances of the parties, to wit, the petitioners' insistence that dismissal was valid on one hand, and the respondents' express prayer for the payment of separation pay on the other, we find that reinstatement would no longer be in the best interest of the contending parties. Liability of Corporate Officers As a general rule, a corporate officer cannot be held liable for acts done in his official capacity because a corporation, by legal fiction, has a personality separate and distinct from its officers, stockholders, and members. In illegal dismissal cases, corporate officers may only be held solidarily liable with the corporation if the termination was done with malice or bad faith. We find that the aforementioned circumstance did not obtain in the case of Jose (vice-president) and Linda (secretary) relative to Arlene and Joseph's dismissal from service.

92. Internation management Services vs. Logarta, G.R. No. 163657, April 18, 2012 Facts: Recruitment agency, International Management Services (IMS), owned and operated by Marilyn C. Pascual, deployed respondent Roel P. Logarta to work for Petrocon Arabia Limited (Petrocon) in Alkhobar, Kingdom of Saudi Arabia, in connection with general engineering services of Petrocon for the Saudi Arabian Oil Company (Saudi Aramco). Respondent was employed for a period of two (2) years, commencing on October 2, 1997, with a monthly salary of eight hundred US Dollars (US$800.00). On April 29, 1998, Saudi Aramco notified Petrocon that due to changes in the general engineering services work forecast for 1998, the manhours that were formerly allotted to Petrocon is going to be reduced by 40% which constrained Petrocon to reduce its personnel. Thus, on June 1, 1998, Petrocon gave respondent a written notice informing the latter that due to the lack of project works related to his expertise, he is given a 30-day notice of termination, and that his last day of work with Petrocon will be on July 1, 1998. Petrocon also informed respondent that all due benefits in accordance with the terms and conditions of his employment contract will be paid to respondent, including his ticket back to the Philippines. Before his departure from Saudi Arabia, respondent received his final paycheck from Petrocon amounting SR7,488.57. Upon his return, respondent filed a complaint with the Regional Arbitration Branch VII, National Labor Relations Commission (NLRC), Cebu City, against petitioner as the recruitment agency which employed him for employment abroad. In filing the complaint, respondent sought to recover his unearned salaries covering the unexpired portion of his employment contract with Petrocon on the ground that he was illegally dismissed. The Labor Arbiter rendered judgment in favor of the respondent and ordered petitioner to pay the peso equivalent of US$5,600.00 based on the rate at the time of actual payment, as payment of his wages for the unexpired portion of his contract of employment. The NLRC on appeal affirmed the Labor Arbiters decision but reduced the award to only US$4,800.00 or its peso equivalent at the time of payment. The CA likewise dismissed the petition and affirmed the NLRC decision. Issue: Whether or not respondents dismissal through retrenchment illegal. Ruling: No. Retrenchment is the reduction of work personnel usually due to poor financial returns, aimed to cut down costs for operation particularly on

Page 201

LABOR RELATIONS
Atty. Jefferson M. Marquez
salaries and wages. It is one of the economic grounds to dismiss employees and is resorted by an employer primarily to avoid or minimize business losses. Retrenchment programs are purely business decisions within the purview of a valid and reasonable exercise of management prerogative. It is one way of downsizing an employer's workforce and is often resorted to by the employer during periods of business recession, industrial depression, or seasonal fluctuations, and during lulls in production occasioned by lack of orders, shortage of materials, conversion of the plant for a new production program, or introduction of new methods or more efficient machinery or automation. It is a valid management prerogative, provided it is done in good faith and the employer faithfully complies with the substantive and procedural requirements laid down by law and jurisprudence. Philippine Law recognizes retrenchment as a valid cause for the dismissal of a migrant or overseas Filipino worker under Article 283 of the Labor Code. Thus, retrenchment is a valid exercise of management prerogative subject to the strict requirements set by jurisprudence, to wit: (1)That the retrenchment is reasonably necessary and likely to prevent business losses which, if already incurred, are not merely de minimis, but substantial, serious, actual and real, or if only expected, are reasonably imminent as perceived objectively and in good faith by the employer; (2)That the employer served written notice both to the employees and to the Department of Labor and Employment at least one month prior to the intended date of retrenchment; (3)That the employer pays the retrenched employees separation pay equivalent to one month pay or at least 1/2 month pay for every year of service, whichever is higher; (4)That the employer exercises its prerogative to retrench employees in good faith for the advancement of its interest and not to defeat or circumvent the employees' right to security of tenure; and (5)That the employer used fair and reasonable criteria in ascertaining who would be dismissed and who would be retained among the employees, such as status,efficiency, seniority, physical fitness, age, and financial hardship for certain workers. 28 Applying the above-stated requisites for a valid retrenchment in the case at bar, it is apparent that the first, fourth and fifth requirements were complied with by respondent's employer. However, the second and third requisites were absent when Petrocon terminated the services of respondent. As aptly found by the NLRC and justly sustained by the CA, Petrocon exercised its prerogative to retrench its employees in good faith and the considerable reduction of work allotments of Petrocon by Saudi Aramco was sufficient basis for Petrocon to reduce the number of its personnel. As for the notice requirement, however, contrary to petitioner's contention, proper notice to the DOLE within 30 days prior to the intended date of retrenchment is necessary and must be complied with despite the fact that respondent is an overseas Filipino worker. In the present case, although respondent was duly notified of his termination by Petrocon 30 days before its effectivity, no allegation or proof was advanced by petitioner to establish that Petrocon ever sent a notice to the DOLE 30 days before the respondent was terminated. Thus, this requirement of the law was not complied with. In the case at bar, despite the fact that respondent was employed by Petrocon as an OFW in Saudi Arabia, still both he and his employer are subject to the provisions of the Labor Code when applicable. The basic policy in this jurisdiction is that all Filipino workers, whether employed locally or overseas, enjoy the protective mantle of Philippine labor and social legislations. Also, respondent is entitled to the payment of his separation pay. However, this Court disagrees with the conclusion of the Labor Arbiter, the NLRC and the CA, that respondent should be paid his separation pay in accordance with the provision of Section 10 of R.A. No. 8042. A plain reading of the said provision clearly reveals that it applies only to an illegally dismissed overseas contract worker or a worker dismissed from overseas employment without just, valid or authorized cause. In the case at bar, notwithstanding the fact that respondent's termination from his employment was procedurally infirm, having not complied with the notice requirement, nevertheless the same remains to be for a just, valid and authorized cause, i.e., retrenchment as a valid exercise of management prerogative. To stress, despite the employer's failure to comply with the one-month notice to the DOLE prior to respondent's termination, it is only a procedural infirmity which does not render the retrenchment illegal. In Agabon v. NLRC, this Court ruled that when the dismissal is for a just cause, the absence of proper notice should not nullify the dismissal or render it illegal or ineffectual. Instead, the employer should indemnify the employee for violation of his statutory rights. Consequently, it is Article 283 of the Labor Code and not Section 10 of R.A. No. 8042 that is controlling. Thus, respondent is entitled to payment of separation pay equivalent to one (1) month pay, or at least one-half (1/2) month pay for every year of service, whichever is higher. Considering that respondent was employed by Petrocon for a period of eight (8) months, he is entitled to receive one (1) month pay as separation pay. In addition, pursuant to current jurisprudence, for failure to fully comply with the statutory due process of sufficient notice, respondent is entitled to nominal damages in the amount P50,000.00.

Page 202

LABOR RELATIONS
Atty. Jefferson M. Marquez

93. Jiao vs. NLRC, G.R. No. 182331, April 18, 2012 Facts: The petitioners were regular employees of the Philippine Banking Corporation (Philbank), each with at least ten years of service in the company. Pursuant to its Memorandum dated August 28, 1970, Philbank established a Gratuity Pay Plan (Old Plan) for its employees. The Old Plan provided: 1. Any employee who has reached the compulsory retirement age of 60 years, or who wishes to retire or resign prior to the attainment of such age or who is separated from service by reason of death, sickness or other causes beyond his/her control shall for himself or thru his/her heirs file with the personnel office an application for the payment of benefits under the plan On March 8, 1991, Philbank implemented a new Gratuity Pay Plan (New Gratuity Plan). In particular, the New Gratuity Plan stated thus: x x x An Employee who is involuntarily separated from the service by reason of death, sickness or physical disability, or for any authorized cause under the law such as redundancy, or other causes not due to his own fault, misconduct or voluntary resignation, shall be entitled to either one hundred percent (100%) of his accrued gratuity benefit or the actual benefit due him under the Plan, whichever is greater.3[7] In February 2000, Philbank merged with Global Business Bank, Inc. (Globalbank), with the former as the surviving corporation and the latter as the absorbed corporation, but the bank operated under the name Global Business Bank, Inc. As a result of the merger, complainants respective positions became redundant. A Special Separation Program (SSP) was implemented and the petitioners were granted a separation package equivalent to one and a half months pay (or 150% of one months salary) for every year of service based on their current salary. Before the petitioners could avail of this program, they were required to sign two documents, namely, an Acceptance Letter and a Release, Waiver, Quitclaim (quitclaim). As their positions were included in the redundancy declaration, the petitioners availed of the SSP, signed acceptance letters and executed quitclaims in Globalbanks favor in consideration of their receipt of separation pay equivalent to 150% of their monthly salaries for every year of service. Subsequently, the petitioners filed separate complaints for non-payment of separation pay with prayer for damages and attorneys fees before the National Labor Relations Commission (NLRC) The petitioners asserted that, under the Old Plan, they were entitled to an additional 50% of their gratuity pay on top of 150% of one months salary for every year of service they had already received. They insisted that 100% of the 150% rightfully belongs to them as their separation pay. Thus, the remaining 50% was only half of the gratuity pay that they are entitled to under the Old Plan. The petitioners further argued that the quitclaims they signed should not bar them from claiming their full entitlement under the law. They also claimed that they were defrauded into signing the same without full knowledge of its legal implications. Issues: The petitioners are now before this Court raising the following errors supposedly committed by the CA: In holding that the bank had abandoned the old plan (referring to the old Gratuity Pay Plan) and replaced it with a Special Separation Program under which [the] petitioners would be receiving more than the rate in the old plan and higher than the legal rate for redundant employees. In holding that the benefits under the Special Separation Program legally replaced not only the gratuity pay plan to which [the] petitioners were entitled under the old and new Gratuity Pay Plans but also all other benefits including separation pay under the law. In not holding that when [the] petitioners were separated due to redundancy they were entitled per provision of Article 283 of the Labor Code to separation pay equivalent to one month pay for every year of service.

Page 203

LABOR RELATIONS
Atty. Jefferson M. Marquez
In holding that [the] petitioners are bound under the Acceptance and Release, Waiver and Quitclaim that they had executed and [cannot] question the same, hence they [cannot] claim benefits in addition to those they had received from the bank. Ruling: The petitioners receipt of separation pay equivalent to their one and a half months salary for every year of service as provided in the SSP and the New Gratuity Plan more than sufficiently complies with the Labor Code, which only requires the payment of separation pay at the rate of one month salary for every year of service. Globalbanks right to replace the Old Plan and the New Gratuity Plan is within legal bounds as the terms thereof are in accordance with the provisions of the Labor Code and complies with the minimum requirements thereof. Contrary to the petitioners claim, they had no vested right over the benefits under the Old Plan considering that none of the events contemplated thereunder occurred prior to the repeal thereof by the adoption of the New Gratuity Plan. Such right accrues only upon their separation from service for causes contemplated under the Old Plan and the petitioners can only avail the benefits under the plan that is effective at the time of their dismissal. In this case, when the merger and the redundancy program were implemented, what was in effect were the New Gratuity Plan and the SSP; the petitioners cannot, thus, insist on the provisions of the Old Plan which is no longer existent. The SSP did not revoke or supersede the New Gratuity Plan. The SSP was not intended to supersede the New Gratuity Plan. On the contrary, the SSP was issued to make the benefits under the New Gratuity Plan available to employees whose positions had become redundant because of the merger between Philbank and Globalbank, subject to compliance with certain requirements such as age and length of service, and to improve such benefits by increasing or rounding it up to an amount equivalent to the affected employees one and a half monthly salary for every year of service. In other words, the benefits to which the redundated employees are entitled to, including the petitioners, are the benefits under the New Gratuity Plan, albeit increased by the SSP. Considering that the New Gratuity Plan still stands and has not been revoked by the SSP, does this mean that the petitioners can claim the benefits thereunder in addition to or on top of what is required under the Article 283 of the Labor Code? For as long as the minimum requirements of the Labor Code are met, it is within the management prerogatives of employers to come up with separation packages that will be given in lieu of what is provided under the Labor Code. Article 283 of the Labor Code provides only the required minimum amount of separation pay, which employees dismissed for any of the authorized causes are entitled to receive. Employers, therefore, have the right to create plans, providing for separation pay in an amount over and above what is imposed by Article 283. There is nothing therein that prohibits employers and employees from contracting on the terms of employment, or from entering into agreements on employee benefits, so long as they do not violate the Labor Code or any other law, and are not contrary to morals, good customs, public order, or public policy. In the absence of proof that any of the vices of consent are present, the petitioners acceptance letters and quitclaims are valid; thus, barring them from claiming additional separation pay. The Court, in other cases, has upheld quitclaims if found to comply with the following requisites: (1) the employee executes a deed of quitclaim voluntarily; (2) there is no fraud or deceit on the part of any of the parties; (3) the consideration of the quitclaim is credible and reasonable; and (4) the contract is not contrary to law, public order, public policy, morals or good customs or prejudicial to a third person with a right recognized by law. We hold that Metrobank cannot be held liable for the petitioners claims. 94. Realda vs. New Age Graphics Inc., G.R. No. 192190, April 25, 2012 Facts: Petitioner Realda was dismissed by Respondent New Age Graphics Inc. for unjustified refusal to render overtime work, unexplained failure to observe prescribed work standards, habitual tardiness and chronic absenteeism despite warning and non-compliance with the directive for him to explain his numerous unauthorized absences. The Court of Appeals recognized the existence of just causes for petitioners dismissal, however, the appellate court found that the respondent failed to observe the procedural requirements of due process and, as a consequence, awarded the petitioner P5,000.00 as Nominal Damages. Issues: WoN the dismissal based on the grounds cited constituted just causes; and WoN the amount awarded as Nominal Damages of P5,000.00 was valid

Page 204

LABOR RELATIONS
Atty. Jefferson M. Marquez
Ruling: First, the petitioners arbitrary defiance to Graphics, Inc.s order for him to render overtime work constitutes willful disobedience. Taking this in conjunction with his inclination to absent himself and to report late for work despite being previously penalized, the CA correctly ruled that the petitioner is indeed utterly defiant of the lawful orders and the reasonable work standards prescribed by his employer. Second, the petitioners failure to observe Graphics, Inc.s work standards constitutes inefficiency that is a valid cause for dismissal. Failure to observe prescribed standards of work, or to fulfill reasonable work assignments due to inefficiency may constitute just cause for dismissal. Such inefficiency is understood to mean failure to attain work goals or work quotas, either by failing to complete the same within the alloted reasonable period, or by producing unsatisfactory results. As the operator of Graphics, Inc.s printer, he is mandated to check whether the colors that would be printed are in accordance with the clients specifications and for him to do so, he must consult the General Manager and the color guide used by Graphics, Inc. before making a full run. Unfortunately, he failed to observe this simple procedure and proceeded to print without making sure that the colors were at par with the clients demands. This resulted to delays in the delivery of output, client dissatisfaction, and additional costs on Graphics, Inc.s part. While a penalty in the form of suspension had already been imposed on the petitioner for his habitual tardiness and repeated absenteeism, the principle of totality of infractions sanctions the act of Graphics, Inc. of considering such previous infractions in decreeing dismissal as the proper penalty for his tardiness and unauthorized absences incurred afterwards, in addition to his refusal to render overtime work and conform to the prescribed work standards. This Court cannot likewise agree to the petitioners attempt to brush aside his refusal to render overtime work as inconsequential when Graphics, Inc.s order for him to do so is justified by Graphics, Inc.s contractual commitments to its clients. Such an order is legal under Article 89 of the Labor Code and the petitioners unexplained refusal to obey is insubordination that merits dismissal from service. Nonetheless, while the CA finding that the petitioner is entitled to nominal damages as his right to procedural due process was not respected despite the presence of just causes for his dismissal is affirmed, this Court finds the CA to have erred in fixing the amount that the Company is liable to pay. The CA should have taken cognizance of the numerous cases decided by this Court where the amount of nominal damages was fixed at P30,000.00 if the dismissal was for a just cause.

95. Kakampi and Its Members Panuelos vs. Kingspoint Express & Logistics, G.R. No. 194813, April 25, 2012 Facts: Petitioners were former drivers of the respondent Kingspoint Express, a sole proprietorship under the name of Co which is engaged in the business of transporting goods. They were dismissed from service on January 20, 2006 on the grounds of serious misconduct, dishonesty, loss of trust and confidence and commission of acts inimical to the interest of Kingspoint Express. Kingspoint Express issued separate notices to explain to the individual petitioners on January 16, 2006 the charges of dishonesty, serious misconduct and loss of confidence by filing with the NLRC false, malicious and fabricated cases against the company, and their allegedly unwarranted refusal to undergo drug testing. They were required to submit their answer to the charges within forty-eight (48) hours from receipt of the notices with a warning that failure to do so would mean waiver of their answer. They were also placed under preventive suspension in the meantime. Petitioners failed to submit their written explanation within the stated period. Subsequently, Kingspoint Express issued to them separate yet uniformly worded notices on January 20, 2006, informing them of their dismissal for the abovementioned charges based on the following acts: fabrication of baseless money claims against the company, misleading fellow co-workers to sign the malicious complaint for money claims against the company, refusal to undergo the company's general drug test, and extorting money from co-workers to fund activities that they were never fully informed of. Also, petitioner Dacara was dismissed for consummating his sexual relations with Cos helper inside her residence and thus impregnating the help. A complaint for illegal dismissal was subsequently filed, alleging that the charges against them were fabricated and that their dismissal was prompted by Kingspoint Express' aversion to their union activities. The Labor Arbiter ruled in favor of the petitioners as the charges are purportedly mere unsubstantiated allegations. This was affirmed by the NLRC on appeal but the latter reversed itself on a subsequent MR filed by Kingspoint. The CA initially reversed the NLRCs ruling but on an MR, they too reversed their earlier ruling and favored Kingspoint. Thus, this petition for certiorari before the SC.

Page 205

LABOR RELATIONS
Atty. Jefferson M. Marquez
Issue: WON the dismissal was valid. Ruling: Yes, the dismissal was valid. It is fundamental that in order to validly dismiss an employee, the employer is required to observe both substantive and procedural due process the termination of employment must be based on a just or authorized cause and the dismissal must be effected after due notice and hearing. As to the substantive requirements of due process, the employees' refusal to submit themselves to drug test is a just cause for their dismissal. An employer may terminate an employment on the ground of serious misconduct or willful disobedience by the employee of the lawful orders of his employer or representative in connection with his work. Willful disobedience requires the concurrence of two elements: (1) the employee's assailed conduct must have been willful, that is, characterized by a wrongful and perverse attitude; and, (2) the order violated must have been reasonable, lawful, made known to the employee, and must pertain to the duties which he had been engaged to discharge. Both elements are present in this case. As to the first element, the dismissed employees did not deny their refusal to undergo drug testing nor did they explain their refusal. The utter lack of reason or justification for their insubordination indicates that it was prompted by mere obstinacy, hence, willful and warranting of dismissal. As to the second element, the subject order is relevant in the performance of their functions as drivers of Kingspoint Express. As the NLRC correctly pointed out, drivers are indispensable to Kingspoint Express' primary business of rendering door-to-door delivery services. It is common knowledge that the use of dangerous drugs has adverse effects on driving abilities that may render the dismissed employees incapable of performing their duties to Kingspoint Express and acting against its interests, in addition to the threat they pose to the public. The existence of a single just cause is enough to order their dismissal and it is now inconsequential if the other charges against them do not merit their dismissal from service. Nonetheless, while Kingspoint Express had reason to sever their employment relations, this Court finds its supposed observance of the requirements of procedural due process pretentious. While Kingspoint Express required the dismissed employees to explain their refusal to submit to a drug test, the two (2) days afforded to them to do so cannot qualify as "reasonable opportunity", which the Court construed in King of Kings Transport, Inc. v. Mamac as a period of at least five (5) calendar days from receipt of the notice. Thus, even if Kingspoint Express' defective attempt to comply with procedural due process does not negate the existence of a just cause for their dismissal, Kingspoint Express is still liable to indemnify the dismissed employees, with the exception of Panuelos, Dizon and Dimabayao, who did not appeal the dismissal of their complaints, with nominal damages in the amount of P30,000.00.

Page 206

LABOR RELATIONS
Atty. Jefferson M. Marquez
SUSPENSION OF BUSINESS OPERATIONS

1. JPL Marketing Promotion vs. Court of Appeals, G.R. No. 151966, July 8, 2005 Facts: Petitioner is the employer of private respondents Gonzales, Abesa and Aninipot. The three are assigned as attendants in various firms where the products of California Marketing Corp., one of petitioners clients, are being displayed. On 13 August 1996, petitioner issued a memorandum to the three employees informing them that CMC would stop its direct merchandising activity after two days. Petitioner then advised them to wait for further notice as they would be transferred to other clients. Without waiting for six months, the three got employed with some other employer. However, on 17 October 1996, Abesa and Gonzales filed before the National Labor Relations Commission Regional Arbitration Branch (NLRC) complaints for illegal dismissal, praying for separation pay, 13th month pay, service incentive leave pay and payment for moral damages. Aninipot filed a similar case thereafter. The Labor Arbiter dismissed the complaint. Private respondents appealed to the NLRC which agreed with the Labor Arbiter's finding that when private respondents filed their complaints, the six-month period had not yet expired, and that CMC's decision to stop its operations in the areas was beyond the control of petitioner, thus, there was no illegal dismissal committed by petitioner. Aggrieved, petitioner filed a petition for certiorari under Rule 65 of the Rules of Court with the Court of Appeals, imputing grave abuse of discretion on the part of the NLRC. It claimed that private respondents are not by law entitled to separation pay, service incentive leave pay and 13th month pay. The Court of Appeals dismissed the petition and affirmed in toto the NLRC resolution. Issue: Were the private respondents illegally dismissed which would entitle them to claim separation pay? Ruling: The common denominator of the instances where payment of separation pay is warranted is that the employee was dismissed by the employer. In the instant case, there was no dismissal to speak of. Private respondents were simply not dismissed at all, whether legally or illegally. What they received from petitioner was not a notice of termination of employment, but a memo informing them of the termination of CMC's contract with petitioner. More importantly, they were advised that they were to be reassigned. At that time, there was no severance of employment to speak of. In addition, the doctrine enunciated in the case of Serrano 37 cited by private respondents has already been abandoned by our ruling in Agabon v. National Labor Relations Commission. There we ruled that an employer is liable to pay indemnity in the form of nominal damages to a dismissed employee if, in effecting such dismissal, the employer failed to comply with the requirements of due process. However, private respondents are not entitled to the payment of damages considering that there was no violation of due process in this case.

2. Pido vs NLRC, G.R. No. 169812, February 23, 2007 Facts: Federito B. Pido was employed by Cherubim Security and General Services, Inc. as a security guard. He was assigned at the Ayala Museum, but was later transferred to the Tower and Exchange Plaza of Ayala Center where he worked as a computer operator at the Console Room. Like the other guards deployed by respondent at the Ayala Center, petitioner was under the operational control and supervision of the Ayala Security Force (ASF) of the Ayala Group of Companies. On January 21, 2000, petitioner had an altercation with Richard Alcantara of the ASF, arising from a statement of Alcantara that petitioners security license for his .38 caliber revolver service firearm and duty detail order had already expired. Alcantara filed a complaint for Gross Misconduct, recommended that petitioner be relieved from his post, and that immediate disciplinary action against him be taken. Respondent thus conducted an investigation on January 25, 2000 during which petitioner echoed his tale in his January 21, 2000 information report.

Page 207

LABOR RELATIONS
Atty. Jefferson M. Marquez
Petitioner was later to claim that he was suspended by respondent following his argument with Alcantara. As more than nine months had elapsed since the investigation was conducted by respondent with no categorical findings thereon made, petitioner filed on October 23, 2000 a complaint for illegal constructive dismissal, illegal suspension, and non-payment and underpayment of salaries, holiday pay, rest day, service incentive leave, 13th month pay, meal and travel allowance and night shift differential against respondent, along with its employee Rosario K. Balais who was allegedly responsible for running the day to day affairs of respondents business. Petitioner likewise prayed for reinstatement and payment of full backwages, attorneys fees and other money claims. In its position paper, respondent denied that it dismissed petitioner from the service, it claiming that while it was still in the process of investigating the January 21, 2000 incident, it offered petitioner another assignment which he declined, saying "pahinga muna ako [I will in the meantime take a rest]." The Labor Arbiter ruled for separation pay. The NLRC, on appeal, ruled reinstatement without granting the other monetary claims. The ruling of the NLRC was affirmed by the Court of Appeals, hence the petition with the Supreme Court. Issue: 1. Whether the petitioners nine-month suspension is tantamount to constructive dismissal. 2. Whether the petitioner should be paid his backwages aside from his separation pay. 3. Whether the payment of separation pay is more viable than the order of reinstatement. Ruling: it is gathered that respondent intended to put petitioner under preventive suspension for an indefinite period of time pending the investigation of the complaint against him. The allowable period of suspension in such a case is not six months but only 30 days, following Sections 8 and 9 of Rule XXIII, Book V of the Omnibus Rules Implementing the Labor Code (Implementing Rules), SEC. 8. Preventive suspension. - The employer may place the worker concerned under preventive suspension if his continued employment poses a serious and imminent threat to the life or property of the employer or of his co-workers. SEC. 9. Period of suspension. - No preventive suspension shall last longer than thirty (30) days. The employer shall thereafter reinstate the worker in his former or in a substantially equivalent position or the employer may extend the period of suspension provided that during the period of extension, he pays the wages and other benefits due to the worker. In such case, the worker shall not be bound to reimburse the amount paid to him during the extension if the employer decides, after completion of the hearing, to dismiss the worker. Respondent did not inform petitioner that it was extending its investigation, nor did it pay him his wages and other benefits after the lapse of the 30-day period of suspension. Neither did respondent issue an order lifting petitioners suspension, or any official assignment, memorandum or detail order for him to assume his post or another post. Respondent merely chose to dawdle with the investigation, in absolute disregard of petitioners welfare. At the time petitioner filed the complaint for illegal suspension and/or constructive dismissal on October 23, 2000, petitioner had already been placed under preventive suspension for nine months. The Supreme Court ruled that the preventive suspension which lasted for nine months amounted to constructive dismissal. Petitioner, who is a regular employee of respondent, is entitled to reinstatement without loss of seniority and payment of backwages from the time his compensation was withheld up to the time of his actual reinstatement by virtue of Art. 279. The Court also ruled that there exists no exception to the general rule that award of separation pay would be proper in lieu of reinstatement. Respondent is ordered to reinstate petitioner together with the payment of the corresponding backwages.

3. Megaforce Security & Allied Services vs. Lactao, G.R. No. 160940, July 21, 2008 Facts:

Page 208

LABOR RELATIONS
Atty. Jefferson M. Marquez
On April 28, 1998, Megaforce hired Lactao as a security guard. He was detailed at Merville Park Subdivision in Paraaque City. On April 4, 2000, he filed with the Arbitration Branch of the NLRC a complaint against Megaforce for underpayment of wages, non-payment of overtime pay, service incentive leave pay and 13th month pay. On May 3, 2000, Lactao was reassigned to ABB Industry, Inc. in Sucat, Paraaque City but wasa recalled by Megaforce directing him to report to the Headquarters for proper disposition and new assignment. When Lactao reported to the Headquarters but he was not given a new assignment. Believing he was terminated, Lactao amended his complaint on June 7, 2000 to one for illegal dismissal with prayer for reinstatement with the same prayer for underpayment of wages, non-payment of overtime pay, service incentive leave pay and 13th month pay, plus moral and exemplary damages and attorney's fees. Lactao claims that in retaliation to his filing of the complaint Megaforce constructively dismissed him by relieving him from his post and not giving him a new assignment. In its Position Paper , Megaforce denied the illegal dismissal charge. The Labor Arbiter dismissed the complaint for lack of merit. On appeal, the NLRC set aside the LAs decision, ruling that Lactao was constructively dismissed. Hence, the present petition. Issue: WHETHER OR NOT LACTAO WAS CONSTRUCTIVELY DISMISSED MAKING THE DISMISSAL ILLEGAL. Ruling: Megaforce contends that it is not guilty of illegal dismissal because Lactao was merely recalled from his post and the failure to give him a new assignment within seven days from his recall is not constructive dismissal because a security guard may be placed on "floating status" for a period not exceeding six months under prevailing jurisprudence; Lactao never reported back for reassignment and his refusal to report back to work should not be taken against it. Lactao insists that he was constructively dismissed when he was recalled from his post at ABB Industry, Inc. without being informed that he was being placed on "floating status" or given a new assignment. YES. In cases involving security guards, a relief and transfer order in itself does not sever employment relationship between a security guard and his agency. 16 An employee has the right to security of tenure, but this does not give him such a vested right in his position as would deprive the company of its prerogative to change his assignment or transfer him where his service, as security guard, will be most beneficial to the client. Temporary "off-detail" or the period of time security guards are made to wait until they are transferred or assigned to a new post or client does not constitute constructive dismissal as their assignments primarily depend on the contracts entered into by the security agencies with third parties. 18 Indeed, the Court has repeatedly recognized that "off-detailing" is not equivalent to dismissal, so long as such status does not continue beyond a reasonable time; when such a "floating status" lasts for more than six months, the employee may be considered to have been constructively dismissed. However, in the present case, while the charge of illegal dismissal may have been premature because Lactao has not been given a new assignment or temporary "off-detail" for a period of seven days only when he amended his complaint, the continued failure of Megaforce to offer him a new assignment during the proceedings of the case before the LA and beyond the reasonable six-month period makes it liable for constructive dismissal. There is constructive dismissal if an act of clear discrimination, insensibility, or disdain by an employer becomes so unbearable on the part of the employee that it would foreclose any choice by him except to forego his continued employment. It exists where there is cessation of work because continued employment is rendered impossible, unreasonable or unlikely, as an offer involving a demotion in rank and a diminution in pay. The Court cannot accept the contention of Megaforce that Lactao did not report to work after his recall and had abandoned his job since it failed to present credible proof of any act on the part of Lactao to abandon his employment. Moreover, it is a settled doctrine that the filing of a complaint for illegal dismissal is inconsistent with abandonment of employment. An employee who takes steps to protest his dismissal cannot logically be said to have abandoned his work. The filing of such complaint is proof enough of his desire to return to work, thus negating any suggestion of abandonment. Under Article 279 of the Labor Code, as amended, an employee who is unjustly dismissed from work shall be entitled to reinstatement without loss of seniority rights and other privileges; to his full backwages, inclusive of allowances; and to other benefits or their monetary equivalent computed from the time his compensation was withheld from him up to the time of his actual reinstatement. Thus, Lactao is entitled to reinstatement and backwages as a necessary consequence.

Page 209

LABOR RELATIONS
Atty. Jefferson M. Marquez
4. National Mines and Allied Workers Union vs. Marcopper Mining Corp., G.R. No. 174641, Nov. 11, 2008 Facts: Department of Environment and Natural Resources (DENR) ordered the indefinite suspension of MARCOPPER's operations for causing damage to the environment of the Province of Marinduque by spilling the company's mine waste or tailings from an old underground impounding area into the Boac River, in violation of its Environmental Compliance Certificate. NAMAWU was the exclusive bargaining representative of the rank-and-file workers of MARCOPPER. filed a complaint with the Regional Arbitration Branch against MARCOPPER for nonpayment of wages, separation pay, damages, and attorney's fees; the case is hereinafter referred to as the "environmental incident case."NAMAWU claimed that due to the indefinite suspension of MARCOPPER's operations, its members were not paid the wages due them for six months (from April 12, 1996 to October 12, 1996). It further claimed that its members are also entitled to be paid their separation pay pursuant to their collective bargaining agreement with MARCOPPER. MARCOPPER denied liability, contending that NAMAWU had not been authorized by the individual employees - the real parties-in-interest - to file the complaint; and that the complaint should be dismissed for lack of certification of non-forum shopping, for the pendency of another action between the same parties, and for lack of factual and legal basis. Labor Arbiter Pedro C. Ramos ruled in NAMAWU's favor. MARCOPPER appealed the decision to the NLRC. In this appeal, it also moved that it be allowed not to post an appeal bond for 615 NAMAWU members - former MARCOPPER employees who had been dismissed effective March 7, 1995 due to an earlier illegal strike. MARCOPPER, however, posted the required bond for three non-striking employees. The NLRC dismissed MARCOPPER's appeal. The NLRC subsequently denied MARCOPPER's motion for reconsideration. MARCOPPER thus sought relief from the CA through a petition for certiorari under Rule 65 of the Rules of Court. The CA granted MARCOPPER's petition in the currently assailed decision promulgated on October 14, 2004. Accordingly, it nullified the NLRC resolution of February 28, 2002 and the order dated April 16, 2002, and ordered the NLRC to give due course to MARCOPPER's appeal. The CA found the non-filing of the appeal bond for the 615 NAMAWU members covered by the Labor Arbiter's award to be justified since their employment had been terminated as early as March 7, 1995, i.e., prior to the suspension of operations for which wages and separation pay were being claimed. The CA's denial of NAMAWU's motion for the reconsideration of the CA's October 14, 2004 decision. Issue:S: 1. WON there is necessity for the filing of an appeal bond considering that the employment of petitioner NAMAWU's members was terminated even before the issuance by the DENR of its order on April 1, 1996. 2. WON the employees are entitled to their wages due them for six months? WON they are entitled to separation pay? Ruling: On the First Issue The employment of the NAMAWU officers and members had been declared terminated on March 7, 1995 as a result of their failure to return to work after their strike of February 27, 1995. Thereafter, the illegal strike litigation commenced, resulting in a decision by the NLRC on November 11, 1996 declaring the strike illegal. MARCOPPER had every reason to claim in its April 10, 2000 appeal to the NLRC that it should be excused from filing an appeal bond with respect to the NAMAWU members who were no longer company employees. The CA decision decreeing the termination of employment of those involved in the illegal strike case had already been issued at that time. We subsequently ruled on the same issue during the time the environmental incident case was pending before the NLRC. Thus, when the NLRC dismissed MARCOPPER's appeal for failure to file the requisite appeal bond corresponding to the 615 NAMAWU members, the termination of employment of these NAMAWU members was already a settled matter that the NLRC was in no position to disregard. In this light, the CA was correct in reversing the dismissal of MARCOPPER's appeal for failure to file an appeal bond. Pursued to its logical end, the CA conclusions should lead to the dismissal of NAMAWU's complaint with respect to its 615 previously dismissed members. On the Second Issue The suspension of MARCOPPER's operations was decreed in an Ex-Parte Order dated April 1, 1996 issued by the Pollution Adjudication Board of the DENR. Separately from this Order, the DENR Secretary ordered on June 21, 1996 the cancellation of MARCOPPER's ECC without which MARCOPPER could not continue to undertake its mining operations. Thus, as of that date (June 21, 1996), the temporary suspension of operations that started on April 12, 1996 became permanent so that MARCOPPER did not have to wait for the end of the six-month suspension

Page 210

LABOR RELATIONS
Atty. Jefferson M. Marquez
of operations before the services of the three employees were deemed terminated. In Labor Code terms, the cancellation of the ECC on June 21, 1996 amounted to a company closure governed by Article 283 of the Labor Code - the provision that governs the relationship of employers and employees in closure situations. While the mine tailing leakage and pollution of the Boac River cannot but affect the health and safety of those in the MARCOPPER vicinity, particularly its employees, we find that the Department of Labor and Employment (DOLE) Regional Director - at whose initiative a suspension of operation must originate for the above-quoted provision to apply - did not act as envisioned by the above rule. Specifically, there was no ruling or directive from the DOLE that the environmental incident was a workplace health and safety concern that required a suspension of operation. There is likewise nothing in the laws applicable to pollution, specifically, P.D. No. 984 and P.D. No. 1586 and their implementing rules, that speak of the consequences of a DENR-ordered suspension of operations on employment relationships. Neither does the CBA between MARCOPPER and NAMAWU provide for the consequences of a suspension of operation due to environmental causes. Under the circumstances, we can only conclude that the general "no work, no pay" rule should prevail with respect to employees' wages during the suspension period, subject to existing CBA terms on leave credits and similar benefits of employees. Because the initial suspension of operations that the DENR imposed eventually turned into an involuntary closure as discussed above, Article 283 of the Labor Code comes into play entitling the three remaining employees the payment of separation pay computed under the terms of that Article. The termination of employment date, for separation pay purposes, should be computed from June 21, 1996 and not from October 12, 1996 (or six months from the April 12, 1996 suspension of operation date); June 21, 1996 must be the closure date as it is from this date that MARCOPPER, by law, ceased to have any authority to conduct its mining operations. In the absence of any showing that NAMAWU could represent "other similarly situated employees" who are not its members, we cannot consider these other employees (whose circumstances have never been discussed and who all remain unnamed) to be covered by the terms of this Decision.

5. Eagle Star Security Services Inc. vs. Mirando et al., G.R. No. 179512, July 30, 2009 Facts: Mirando, who was hired by Eagle Star Security Services, Inc. as a security guard on July 29, 1997, was posted at the Heroes Hill Branch of Equitable-PCI Bank (now Banco de Oro-EPCI Bank) with a 9:00 a.m.-to-5:00 p.m. shift and a daily wage of P250.00. On December 14, 2001, respondent was made to sign a duty schedule for December 15 (a Saturday). When he reported for work on December 15, 2001, he was told by the detachment commander, (Endencio), not to report for duty per instruction of the head office. Respondent thus called up the head office and was that he was removed from duty by (Agodilla), petitioners operations manager.As respondent was thereafter no longer asked to report for duty, he filed on December 18, 2001 a complaint for illegal dismissal. He later amended his complaint on February 1, 2002 to include a prayer for reinstatement and payment of full backwages, damages and attorneys fees. Responding to the complaint, petitioner alleged that respondent went on absence without official leave (AWOL) on December 16, 2001 and had not since reported for work, drawing it to send him a notice on December 26, 2001 to explain his absence, but he failed to respond thereto. Respondent argues that the present petition must be treated as a "mere scrap of paper" since the one who signed it was "not properly authorized by the [p]etitioner to file [it] before this [Court]." Issues: Whether or not the person who signed the present petition was duly authorized. Whether or not he was illegally dismissed. Ruling: The petition must be denied. There is no proof that petitioners representative Reynaldo G. Tauro was authorized to file the petition on its behalf.The Board Resolution which was adopted during petitioners Special Board Meeting of May 20, 2006, states: RESOLVED as it is hereby resolved that the corporation shall elevate on Certiorari before the Court of Appeals NLRC NCR Case No. 039872-04 entitled "Bonifacio L. Mirando, complainant, versus Eagle Star Security Services, Inc., respondent."

Page 211

LABOR RELATIONS
Atty. Jefferson M. Marquez
RESOLVED further as it is hereby resolved that Mr. REYNALDO G. TAURO, shall be appointed as authorized representative of the Corporation, to represent and sign in behalf of the corporation the Verification and Certification of the petition for afore-mentioned case. Clearly, it was adopted for the purpose of authorizing Tauro to file petitioners petition for "Certiorari before the Court of Appeals." Despite petitioners awareness in its Reply to respondents Comment filed before this Court of the defect in Tauro authority to sign for and in its behalf the Verification and Certification against Non-Forum Shopping, it failed even to belatedly file the requisite authority. Fuentebella and Rolling Hills Memorial Park v. Castro,on the requirement of a certification against forum shopping, explains: The reason for this is that the principal party has actual knowledge whether a petition has previously been filed involving the same case or substantially the same issues. If, for any reason, the principal party cannot sign the petition, the one signing on his behalf must have been duly authorized. . . . Where the petitioner is a corporation, the certification against forum shopping should be signed by its duly authorized director or representative [I]f the real party-in-interest is a corporate body, an officer of the corporation can sign the certification against forum shopping as long as he is authorized by a resolution of its board of directors. A certification without the proper authorization is defective and constitutes a valid cause for the dismissal of the petition. (Petitioners discourse on relaxation of technical rules of procedure in the interest of substantial justice does not impress. While there have been instances when the Court dispensed with technicalities on the basis of special circumstances or compelling reasons, there is no such circumstance or reason in the present case which warrants the liberal application of technical rules. AT ALL EVENTS, on the merits, the appellate court did not commit any reversible error in affirming the congruent findings of the Labor Arbiter and the NLRC that respondent was illegally dismissed. The persistence of respondent to resume his duties, not to mention his immediate filing of the illegal dismissal complaint, should dissipate any doubt that he did not abandon his job. Clutching at straws, petitioner argues that respondent was on temporary "off-detail," the period of time a security guard is made to wait until he is transferred or assigned to a new post or client; and since petitioners business is primarily dependent on contracts entered into with third parties, the temporary "off-detail" of respondent does not amount to dismissal as long as the period does not exceed 6 months, following Art. 286 of the Labor Code. Petitioners citation of Article 286 of the Labor Code reading: ART. 286. When employment not deemed terminated. The bona fide suspension of the operation of a business or undertaking for a period not exceeding six (6) months, or the fulfillment by the employee of a military or civic duty shall not terminate employment. In all such cases, the employer shall reinstate the employee to his former position without loss of seniority rights if he indicates his desire to resume his work not later than one (1) month from the resumption of operations of his employer or from his relief from the military or civic duty. (Emphasis in the original; underscoring supplied) is misplaced. Philippine Industrial Security Agency v. Dapiton teaches: We stress that Article 286 applies only when there is a bonafide suspension of the employers operation of a business or undertaking for a period not exceeding six (6) months. In such a case, there is no termination of employment but only a temporary displacement of employees, albeit the displacement should not exceed six (6) months. The paramount consideration should be the dire exigency of the business of the employer that compels it to put some of its employees temporarily out of work. In security services, the temporary "off-detail" of guards takes place when the security agencys clients decide not to renew their contracts with the security agency, resulting in a situation where the available posts under its existing contracts are less than the number of guards in its roster. In the present case, there is no showing that there was lack of available posts at petitioners clients or that there was a request from the clientbank, where respondent was last posted and which continued to hire petitioners services, to replace respondent with another. Petitioner suddenly prevented him from reporting on his tour of duty at the bank on December 15, 2001 and had not thereafter asked him to report for duty. WHEREFORE, the petition is DENIED.

6. Nationwide Security & Allied Services v. Valderama, G.R. No. 186614, February 23, 2011 Facts: In this case, petitioner and respondent presented two different sides of the story. RESPONDENT VALDERAMAS SIDE OF THE STORY PETITIONERS SIDE OF THE STORY

Page 212

LABOR RELATIONS
Atty. Jefferson M. Marquez
Valderama was hired by petitioner as security guard at the Philippine Heart Center. After he was relived from his job, no further assignment was given to him. Thus, he filed a complaint for constructive dismissal and nonpayment of 13th month pay, with prayer for damages against petitioner. Valderama was not constructively or illegally dismissed, but had voluntarily resigned. Respondent violated security rules of the work place. Furthermore, he was charged with conduct unbecoming which he was required to explain. For this, he was suspended for 7 days. Respondent displayed his discourteous and rude attitude upon his superior. He said to him in a high pitch "ano ba sir, personalan ba ito, sabihin mo lang kung ano gusto mo." Petitioner required him to explain why no disciplinary action should be meted against him. Seven security guards, including Valderama were made to explain their failure to report for duty without informing the office despite the instruction during their formation day which was held a day before. Detachment Commander, Roy Datiles, reported that Valderama confronted and challenged him in a high pitch and on top of his voice rudely showing discourtesy and rudeness. Being his superior, Datiles recommended the relief of Valderama. By order of the Operations Manager, he was relieved from his post at the Philippine Heart Center. He was then directed to report to the office where he got his cash bond and firearm deposit. Despite his voluntary resignation, petitioner sent him a letter through registered mail to report for the office and give information on whether or not he was still interested for report for duty or not. Valderama did not bother to reply. Neither did he report to the office.

The Labor Arbiter rendered a decision saying that Valderama was constructively dismissed. It said that the petitioners defense is unsubstantiated. On appeal, the NLRC modified the LA decision. It declared that respondent was neither constructively terminated nor did he voluntarily resign. As such, respondent remained an employee of petitioner. The NLRC thus ordered respondent to immediately report to petitioner and assume his duty. Respondent went to the CA via certiorari. CA rendered a Decision setting aside the resolutions of the NLRC and reinstating that of the LA. In gist, the CA sustained respondent's claim of constructive dismissal. Issue: Whether or not the respondent was constructively dismissed? Ruling: In cases involving security guards, a relief and transfer order in itself does not sever employment relationship between a security guard and his agency. An employee has the right to security of tenure, but this does not give him a vested right to his position as would deprive the company of its prerogative to change his assignment or transfer him where his service, as security guard, will be most beneficial to the client. Temporary "offdetail" or the period of time security guards are made to wait until they are transferred or assigned to a new post or client does not constitute constructive dismissal, so long as such status does not continue beyond six months. The onus of proving that there is no post available to which the security guard can be assigned rests on the employer. When a security guard is placed on a "floating status," he does not receive any salary or financial benefit provided by law. Due to the grim economic consequences to the employee, the employer should bear the burden of proving that there are no posts available to which the employee temporarily out of work can be assigned. Petitioner claims that respondent abandoned his job. The jurisprudential rule on abandonment is constant. It is a matter of intention and cannot lightly be presumed from certain equivocal acts. To constitute abandonment, two elements must concur: (1) The failure to report for work or absence without valid or justifiable reason (2) A clear intent, manifested through overt acts, to sever the employer-employee relationship. In this case, petitioner failed to establish clear evidence of respondent's intention to abandon his employment. Except for petitioner's bare assertion that respondent did not report to the office for reassignment, no proof was offered to prove that respondent intended to sever the employer-employee relationship. Besides, the fact that respondent filed the instant complaint negates any intention on his part to forsake his work. It is a settled doctrine that the filing of a complaint for illegal dismissal is inconsistent with the charge of abandonment, for an employee who takes steps to protest his dismissal cannot by logic be said to have abandoned his work.

Page 213

LABOR RELATIONS
Atty. Jefferson M. Marquez
Similarly, we cannot accept petitioner's argument that respondent voluntarily resigned. Resignation is the voluntary act of an employee who is in a situation where one believes that personal reasons cannot be sacrificed in favor of the exigency of the service, and one has no other choice but to dissociate oneself from employment. It is a formal pronouncement or relinquishment of an office, with the intention of relinquishing the office accompanied by the act of relinquishment. As the intent to relinquish must concur with the overt act of relinquishment, the acts of the employee before and after the alleged resignation must be considered in determining whether, he or she, in fact, intended to sever his or her employment. In this case, petitioner was firm in asserting that respondent voluntarily resigned. Oddly, it failed to present the alleged resignation letter of respondent. Jurisprudence is trite with pronouncements that the temporary inactivity or "floating status" of security guards should continue only for six months. Otherwise, the security agency concerned could be liable for constructive dismissal.[24] The failure of petitioner to give respondent a work assignment beyond the reasonable six-month period makes it liable for constructive dismissal. If there is a surplus of security guards caused by lack of clients or projects, the security agency may resort to retrenchment upon compliance with the requirements set forth in the Labor Code. In this way, the security agency will not to be held liable for constructive dismissal and be burdened with the payment of backwages. Under Article 279 of the Labor Code, an employee who is unjustly dismissed from work shall be entitled to reinstatement without loss of seniority rights and other privileges; to his full backwages, inclusive of allowances; and to other benefits or their monetary equivalent computed from the time his compensation was withheld from him up to the time of his actual reinstatement. PETITION DENIED. RESPONDENT WAS ILLEGALLY DISMISSED BY PETITIONER.

7. Nippon Housing Phils. vs. Leynes, G.R. No. 177816, August 3, 2011 Facts: Petitioner, originally engaged in the business of providing building maintenance From its original ventured into building management and gained Bay Gardens Condominium Project (the Project) of the Bay Gardens Condominium Corporation (BGCC) as its first and only building maintenance client. In this regard, petitioner hired respondent Maiah Angela Leynes on 26 March 2001 for the position of Property Manager, with a salary of P40,000.00 per month. Her responsibilities include surveying the requirements of the government and the client for said project, the formulation of house rules and regulations, the preparation of the annual operating and capital expenditure budget, hiring and deployment of manpower, salary and position determination as well as the assignment of the schedules and responsibilities of employees. Leynes had a misunderstanding with the building engineer of the project (Cantuba) and barred the latters entry to the site. The Engr. also accused the former of conceit, pride and poor managerial skills. Takada, the NHPI's Vice President issued a memorandum attributing the incident to "simple personal differences" and directing Leynes to allow Engr. Cantuba to report back for work. Disappointed with this management decision, she submitted a letter to NHPIs President (Ota) asking for an emergency leave of absence for the supposed purpose of coordinating with her lawyer regarding her resignation letter. NHPI offered the Property Manager position to Engr. Carlos Jose as a consequence Leynes' signification of her intention to resign. However, she sent another letter expressing her intention to return to work and to call off her planned resignation. However, she received a letter from the management to report instead to the main office as one in a floating status because someone already occupies her post. Aggrieved, Leynes filed a complaint against petitioner for illegal dismissal, unpaid salaries, benefits, damages and attorney's fees. The Labor arbiter found that the petitioners act of putting Leynes on a floating status was equivalent to termination without just cause. The NLRC ruled that NHPI's placement of Leynes on floating status was necessitated by the client's contractually guaranteed right to request for her relief. However, this was later on reversed by the CA, hence, this present petition before the SC. Issue: WON petitioners' decision to place respondent on floating status is tantamount to constructive dismissal. (Alternative: what is the effect of withdrawn resignation?) Ruling: No, the placement of Leynes on a floating status due to redundancy is valid. There is no constructive dismissal. The factual antecedents suggest that NHPI's immediate hiring of Engr. Jose as the new Property Manager for the Project was brought about by Leynes' own rash announcement of her intention to resign from her position. Although she subsequently changed her mind and sent Reyes a letter by telefax announcing the reconsideration of her planned resignation and her intention to return to work, Leynes evidently had only herself to blame for precipitately setting in motion the events which led to NHPI's hiring of her own replacement.

Page 214

LABOR RELATIONS
Atty. Jefferson M. Marquez
The record, moreover, shows that NHPI simply placed her on floating status "until such time that another project could be secured" for her. Traditionally invoked by security agencies when guards are temporarily sidelined from duty while waiting to be transferred or assigned to a new post or client, Article 286 of the Labor Code has been applied to other industries when, as a consequence of the bona fide suspension of the operation of a business or undertaking, an employer is constrained to put employees on floating status for a period not exceeding six months. In brushing aside respondents' reliance on said provision to justify the act of putting Leynes on floating status, the CA ruled that no evidence was adduced to show that there was a bona fide suspension of NHPI's business. What said court clearly overlooked, however, is the fact that NHPI had belatedly ventured into building management and, with BGCC as its only client in said undertaking, had no other Property Manager position available to Leynes. The rule is settled, however, that "off-detailing" is not equivalent to dismissal, so long as such status does not continue beyond a reasonable time and that it is only when such a "floating status" lasts for more than six months that the employee may be considered to have been constructively dismissed. A complaint for illegal dismissal filed prior to the lapse of said six-month and/or the actual dismissal of the employee is generally considered as prematurely filed. Since the petitioner has no other client for the building management side of its business, it acted within its prerogatives when it eventually terminated Leynes' services on the ground of redundancy. One of the recognized authorized causes for the termination of employment, redundancy exists when the service capability of the workforce is in excess of what is reasonably needed to meet the demands of the business enterprise. A redundant position is one rendered superfluous by any number of factors, such as overhiring of workers, decreased volume of business, dropping of a particular product line previously manufactured by the company or phasing out of service activity priorly undertaken by the business An employer has no legal obligation to keep more employees than are necessary for the operation of its business. Considering that Leynes was terminated from service upon an authorized cause, we find that the CA likewise erred in faulting NHPI for supposedly failing to notify said employee of the particular act or omission leveled against her and the ground/s for which she was dismissed from employment. Where dismissal, however, is for an authorized cause like redundancy, the employer is, instead, required to serve a written notice of termination on the worker concerned and the DOLE, at least one month from the intended date thereof. For its failure to comply strictly with the 30-day minimum requirement for said notice and effectively violating Leynes' right to due process, NHPI should be held liable to pay nominal damages in the sum of P50,000.00.

Page 215

LABOR RELATIONS
Atty. Jefferson M. Marquez
DISEASE AS A GROUND FOR TERMINATION

1. Sy vs. Court of Appeals, G.R. No. 142293, February 27, 2003

Facts: Private respondent Jaime Sahot has been working for petitioners family-owned trucking business named Vicente Sy Trucking starting in 1958. Since that time, the family business has changed names, first from T. Paulino Trucking Service, then to 6Bs Trucking Corporation, and finally to SBT Trucking Corporation. Throughout all these changes and for 36 years, Sahot remained with the business. When Sahot was already 59 years old, he had recurring absences due to his suffering various ailments. His left thigh, in particular, has been causing him pain, which greatly affected his performance as a driver. After inquiring with the SSS regarding his medical and retirement benefits, he found that his premium payments had not been remitted by his employer. Later, he filed a week-long leave during which time he was medically examined and treated for several illnesses. Upon the advice of SBT Trucking Service management, he filed a formal request for extension of his leave. It was at this time that Sahot was first threatened of termination from work, with his employers later carrying out this threat by dismissing him. Issue: Whether or not there was valid dismissal Ruling: Article 277(b) of the Labor Code puts the burden of proving that the dismissal of an employee was for a valid or authorized cause on the employer, without distinction whether the employer admits or does not admit the dismissal. For an employees dismissal to be valid, (a) the dismissal must be for a valid cause and (b) the employee must be afforded due process. Article 284 of the Labor Code authorizes an employer to terminate an employee on the ground of disease. However, in order to validly terminate employment on this ground, Book VI, Rule I, Section 8 of the Omnibus Implementing Rules of the Labor Code requires a medical certificate. This requirement cannot be dispensed with; otherwise, it would sanction the unilateral and arbitrary determination by the employer of the gravity or extent of the employees illness and thus defeat the public policy in the protection of labor. In the case at bar, the employer clearly did not comply with the medical certificate requirement before Sahots dismissal was effected. In addition, there is likewise the determination if the procedural aspect of due process had been complied with by the employer. From the records, it clearly appears that procedural due process was not observed in the separation of private respondent by the management of the trucking company. The employer is required to furnish an employee with two written notices before the latter is dismissed: (1) the notice to apprise the employee of the particular acts or omissions for which his dismissal is sought, which is the equivalent of a charge; and (2) the notice informing the employee of his dismissal, to be issued after the employee has been given reasonable opportunity to answer and to be heard on his defense. These, the petitioners failed to do, even only for record purposes. What management did was to threaten the employee with dismissal, then actually implement the threat when the occasion presented itself because of private respondents painful left thigh. All told, both the substantive and procedural aspects of due process were violated. Clearly, therefore, Sahots dismissal is tainted with invalidity.

2. Manly Express vs. Payong, G.R. No. 167462, October 25, 2005 Facts: Sometime in December 1999, Romualdo Payong, Jr., was complaining of eyesight problems. He was brought to an eye specialist by private respondent Manly Express, Inc. and/or Siy Eng T. Ching, he was diagnosed to be suffering from eye cataract. Despite having the cataract removed in January of 2000, he was disallowed to return to his work by Ching. Much later, on August 1, 2000, he was given a letter of

Page 216

LABOR RELATIONS
Atty. Jefferson M. Marquez
termination of employment. Thus, a complaint for illegal dismissal with money claims was filed against Manly. Issue: Whether or not the dismissal on the ground of disease was valid. Ruling: In order to validly terminate employment on this ground, Section 8, Rule I, Book VI of the Omnibus Rules Implementing the Labor Code requires: Sec. 8. Disease as a ground for dismissal. Where the employee suffers from a disease and his continued employment is prohibited by law or prejudicial to his health or to the health of his co-employees, the employer shall not terminate his employment unless there is a certification by a competent public health authority that the disease is of such nature or at such a stage that it cannot be cured within a period of six (6) months even with proper medical treatment. If the disease or ailment can be cured within the period, the employer shall not terminate the employee but shall ask the employee to take a leave. The employer shall reinstate such employee to his former position immediately upon the restoration of his normal health. The rule is explicit. For a dismissal on the ground of disease to be considered valid, two requisites must concur: (a) the employee suffers from a disease which cannot be cured within six months and his continued employment is prohibited by law or prejudicial to his health or to the health of his co-employees, and (b) a certification to that effect must be issued by a competent public health authority. In the present case, there was no proof that Payongs continued employment was prohibited by law or prejudicial to his health and that of his coemployees. No medical certificate by a competent public health authority was submitted that Payong was suffering from a disease that cannot be cured within a period of six months. In the absence of such certification, Payongs dismissal must necessarily be declared illegal. The burden of proving the validity of the dismissal rests on the employer. As such, the employer must prove that the requisites for a valid dismissal due to a disease have been complied with. In the absence of the required certification by a competent public health authority, this Court has ruled against the validity of the employees dismissal. The Supreme Court also note that Manly failed to comply with the procedure for terminating an employee. In dismissing an employee, the employer has the burden of proving that the employee has been served two notices: (1) one to apprise him of the particular acts or omissions for which his dismissal is sought, and (2) the other to inform him of his employers decision to dismiss him. The first notice must state that dismissal is sought for the act or omission charged against the employee, otherwise, the notice cannot be considered sufficient compliance with the rules. Payongs dismissal did not comply with both the substantive and procedural aspects of due process. Clearly, his dismissal is tainted with invalidity.

3. Duterte vs. Kingswood Trading Co., G.R. No. 160325, October 4, 2007 Facts: Petitioner was hired as truck/trailer driver by respondent Kingswood Trading Company, Inc. (KTC) of which co-respondent Filemon Lim is the President. Petitioner was on the 6:00 a.m. 6:00 p.m. shift. He averaged 21 trips per month, getting P700 per trip. When not driving, petitioner was assigned to clean and maintain respondent KTCs equipment and vehicles for which he was paid P125 per day. Regularly, petitioner would be seconded by respondent Filemon Lim to drive for one of KTCs clients, the Philippine National Oil Corporation, but always subject to respondents convenience. On November 8, 1998, petitioner had his first heart attack and was confined for two weeks at the Philippine Heart Center (PHC). This was confirmed by respondent KTC which admitted that petitioner was declared on sick leave with corresponding notification. A month later, petitioner returned to work armed with a medical certificate signed by his attending physician at the PHC, attesting to petitioners fitness to work. However, said certificate was not honored by the respondents who refused to allow petitioner to work. In February 1999, petitioner suffered a second heart attack and was again confined at the PHC. Upon release, he stayed home and spent time to recuperate. Petitioner attempted to report back to work but was told to look for another job because he was unfit. Respondents refused to declare petitioner fit to work unless physically examined by the company physician. Respondents promise to pay petitioner his separation pay turned out to be an empty one. Instead, petitioner was presented, for his signature, a document as proof of his receipt of the amount of P14,375.00 as first installment of his Social Security System (SSS) benefits. Having received no such amount, petitioner refused to affix his signature thereon and instead requested for the necessary documents from respondents to enable him to claim his SSS benefits, but the latter did not heed his request.

Page 217

LABOR RELATIONS
Atty. Jefferson M. Marquez
Ruling: The law is unequivocal: the employer, before it can legally dismiss its employee on the ground of disease, must adduce a certification from a competent public authority that the disease of which its employee is suffering is of such nature or at such a stage that it cannot be cured within a period of six months even with proper treatment. Here, the record does not contain the required certification. And when the respondents asked the petitioner to look for another job because he was unfit to work, such unilateral declaration, even if backed up by the findings of its company doctors, did not meet the quantum requirement mandated by the law, i.e., there must be a certification by a competent public authority. The requirement for a medical certificate under Article 284 of the Labor Code cannot be dispensed with; otherwise, it would sanction the unilateral and arbitrary determination by the employer of the gravity or extent of the employees illness and thus defeat the public policy on the protection of labor. All told, we rule and so hold that petitioners dismissal did not comply with both the substantive and procedural aspects of due process. Clearly, his dismissal is tainted with invalidity. WHEREFORE, the assailed decision of the CA is REVERSED and SET ASIDE. SO ORDERED.

4. Villaruel vs. Yeo Han Guan, G.R. No. 169191, June 1, 2011 Facts: Petitioner alleged that in June 1963, he was employed as a machine operator by Ribonette Manufacturing Company, an enterprise engaged in the business of manufacturing and selling PVC pipes and is owned and managed by herein respondent Yeo Han Guan. Over a period of almost twenty (20) years, the company changed its name four times. Starting in 1993 up to the time of the filing of petitioner's complaint in 1999, the company was operating under the name of Yuhans Enterprises. Despite the changes in the company's name, petitioner remained in the employ of respondent. Petitioner further alleged that on October 5, 1998, he got sick and was confined in a hospital; on December 12, 1998, he reported for work but was no longer permitted to go back because of his illness; he asked that respondent allow him to continue working but be assigned a lighter kind of work but his request was denied; instead, he was offered a sum of P15,000.00 as his separation pay; however, the said amount corresponds only to the period between 1993 and 1999; petitioner prayed that he be granted separation pay computed from his first day of employment in June 1963, but respondent refused. Aside from separation pay, petitioner prayed for the payment of service incentive leave for three years as well as attorney's fees. The Labor Arbiter found for the respondent, granting him separation pay from the June 1963 up to the time of separation, and service incentive leave equivalent to 15 days. The NLRC affirmed. On appeal, the CA reversed the NLRC on the issue of separation pay. Issue: The assigned errors in the instant petition essentially boil down to the question of whether petitioner is entitled to separation pay under the provisions of the Labor Code, particularly Article 284 thereof, which reads as follows: An employer may terminate the services of an employee who has been found to be suffering from any disease and whose continued employment is prohibited by law or is prejudicial to his health as well as to the health of his co-employees: Provided, That he is paid separation pay equivalent to at least one (1) month salary or to one-half () month salary for every year of service whichever is greater, a fraction of at least six months being considered as one (1) whole year. Held: A plain reading of the abovequoted provision clearly presupposes that it is the employer who terminates the services of the employee found to be suffering from any disease and whose continued employment is prohibited by law or is prejudicial to his health as well as to the health of his coemployees. It does not contemplate a situation where it is the employee who severs his or her employment ties. This is precisely the reason why Section 8, Rule 1, Book VI of the Omnibus Rules Implementing the Labor Code, directs that an employer shall not terminate the services of the employee unless there is a certification by a competent public health authority that the disease is of such nature or at such a stage that it cannot be cured within a period of six (6) months even with proper medical treatment. On the other hand, the Court agrees with the CA in its observation of the following circumstances as proof that respondent did not terminate petitioner's employment: first, the only cause of action in petitioner's original complaint is that he was offered a very low separation pay; second, there was no allegation of illegal dismissal, both in petitioner's original and amended complaints and position paper; and, third, there was no prayer for reinstatement. In consonance with the above findings, the Court finds that petitioner was the one who initiated the severance of his employment relations with

Page 218

LABOR RELATIONS
Atty. Jefferson M. Marquez
respondent. It is evident from the various pleadings filed by petitioner that he never intended to return to his employment with respondent on the ground that his health is failing. Indeed, petitioner did not ask for reinstatement. In fact, he rejected respondent's offer for him to return to work. This is tantamount to resignation. Resignation is defined as the voluntary act of an employee who finds himself in a situation where he believes that personal reasons cannot be sacrificed in favor of the exigency of the service and he has no other choice but to disassociate himself from his employment. It may not be amiss to point out at this juncture that aside from Article 284 of the Labor Code, the award of separation pay is also authorized in the situations dealt with in Article 283[16] of the same Code and under Section 4 (b), Rule I, Book VI of the Implementing Rules and Regulations of the said Code[17] where there is illegal dismissal and reinstatement is no longer feasible. By way of exception, this Court has allowed grants of separation pay to stand as a measure of social justice where the employee is validly dismissed for causes other than serious misconduct or those reflecting on his moral character.[18] However, there is no provision in the Labor Code which grants separation pay to voluntarily resigning employees. In fact, the rule is that an employee who voluntarily resigns from employment is not entitled to separation pay, except when it is stipulated in the employment contract or CBA, or it is sanctioned by established employer practice or policy. [19] In the present case, neither the abovementioned provisions of the Labor Code and its implementing rules and regulations nor the exceptions apply because petitioner was not dismissed from his employment and there is no evidence to show that payment of separation pay is stipulated in his employment contract or sanctioned by established practice or policy of herein respondent, his employer. Since petitioner was not terminated from his employment and, instead, is deemed to have resigned therefrom, he is not entitled to separation pay under the provisions of the Labor Code. The foregoing notwithstanding, this Court, in a number of cases, has granted financial assistance to separated employees as a measure of social and compassionate justice and as an equitable concession. Taking into consideration the factual circumstances obtaining in the present case, the Court finds that petitioner is entitled to this kind of assistance. In this regard, the Court finds credence in petitioner's contention that he is in the employ of respondent for more than 35 years. In the absence of a substantial refutation on the part of respondent, the Court agrees with the findings of the Labor Arbiter and the NLRC that respondent company is not distinct from its predecessors but, in fact, merely continued the operation of the latter under the same owners and the same business venture. The Court further notes that there is no evidence on record to show that petitioner has any derogatory record during his long years of service with respondent and that his employment was severed not by reason of any infraction on his part but because of his failing physical condition. Add to this the willingness of respondent to give him financial assistance. Hence, based on the foregoing, the Court finds that the award of P50,000.00 to petitioner as financial assistance is deemed equitable under the circumstances.

Page 219

LABOR RELATIONS
Atty. Jefferson M. Marquez
OTHER CAUSES OF SEVERANCE OF EMPLOYMENT RELATION

1. Pantranco North Express vs. NLRC, 259 SCRA 161 [1996] Facts: Private respondent was hired by petitioner in 1964 as a bus conductor. He eventually joined the Pantranco Employees Association, PTGWO. He continued in petitioners employ until August 12, 1989, when he was retired at the age of 52 after having rendered twenty five years service. The basis of his retirement was the compulsory retirement provision of the collective bargaining agreement between the petitioner and the aforenamed union. Private respondent received P49, 300.00 as retirement pay. Private respondent filed a complaint for illegal dismissal against petitioner with the Sub-Regional Arbitration Branch of the respondent Commission in Dagupan City. The complaint was consolidated with two other cases of illegal dismissal having similar facts and issues, filed by other employees, non union members. Issue: Is a Collective Bargaining Agreement provision allowing compulsory retirement before age 60 but after twenty five years of service legal and enforceable? Who has jurisdiction over a case involving such a question -- the labor arbiter or arbitrators authorized by such CBA? Ruling: On the first issue: The bone of contention in this case is the provision on compulsory retirement after 25 years of service. Article XI, Section 1 (e) (5) of the May 2, 1989 Collective Bargaining Agreement between petitioner company and the union states: "Section 1. The COMPANY shall formulate a retirement plan with the following main features: xxx xxx xxx (e) The COMPANY agrees to grant the retirement benefits herein provided to regular employees who may be separated from the COMPANY for any of the following reasons: xxx xxx xxx Upon reaching the age of sixty (60) years or upon completing twenty-five (25) years of service to the COMPANY, whichever comes first, and the employee shall be compulsory retired and paid the retirement benefits herein provided." Art. 287 of the Labor Code as worded permits employers and employees to fix the applicable retirement age at below 60 years. Moreover, providing for early retirement does not constitute diminution of benefits. In almost all countries today, early retirement, i.e., before age 60, is considered a reward for services rendered since it enables an employee to reap the fruits of his labor particularly retirement benefits, whether lump-sum or otherwise at an earlier age, when said employee, in presumably better physical and mental condition, can enjoy them better and longer. As a matter of fact, one of the advantages of early retirement is that the corresponding retirement benefits, usually consisting of a substantial cash windfall, can early on be put to productive and profitable uses by way of income-generating investments, thereby affording a more significant measure of financial security and independence for the retiree who, up till then, had to contend with life's vicissitudes within the parameters of his fortnightly or weekly wages. Thus we are now seeing many CBAs with such early retirement provisions. And the same cannot be considered a diminution of employment benefits. It is also further argued that, being a union member, private respondent is bound by the CBA because its terms and conditions constitute the law between the parties. The parties are bound not only to the fulfillment of what has been expressly stipulated but also to all the consequences which according to their nature, may be in keeping with good faith, usage and law. It binds not only the union but also its members. Thus, the Solicitor General said: "Private respondent cannot therefore claim illegal dismissal when he was compulsory retired after rendering twenty-five (25) years of service since his retirement is in accordance with the CBA." A CBA incorporates the agreement reached after negotiations between employer and bargaining agent with respect to terms and conditions of employment. A CBA is not an ordinary contract. "(A)s a labor contract within the contemplation of Article 1700 of the Civil Code of the

Page 220

LABOR RELATIONS
Atty. Jefferson M. Marquez
Philippines which governs the relations between labor and capital, (it) is not merely contractual in nature but impressed with public interest, thus it must yield to the common good. As such, it must be construed liberally rather than narrowly and technically, and the courts must place a practical and realistic construction upon it, giving due consideration to the context in which it is negotiated and purpose which it is intended to serve." Being a product of negotiation, the CBA between the petitioner and the union intended the provision on compulsory retirement to be beneficial to the employees-union members, including herein private respondent. When private respondent ratified the CBA with the union, he not only agreed to the CBA but also agreed to conform to and abide by its provisions. Thus, it cannot be said that he was illegally dismissed when the CBA provision on compulsory retirement was applied to his case. On the second issue: In Sanyo Philippines Workers Union PSSLU vs. Caizares, a case cited by the petitioner, this Court ruled: x x x Hence, only disputes involving the union and the company shall be referred to the grievance machinery or voluntary arbitrators. In the instant case, both the union and the company are united or have come to an agreement regarding the dismissal of private respondents. No grievance between them exists which could be brought to a grievance machinery. The problem or dispute in the present case is between the union and the company on the one hand and some union and non-union members who were dismissed, on the other hand. The dispute has to be settled before an impartial body. The grievance machinery with members designated by the union and the company cannot be expected to be impartial against the dismissed employees. Due process demands that the dismissed workers grievances be ventilated before an impartial body. Since there has already been an actual termination, the matter falls within the jurisdiction of the Labor Arbiter." Applying the same rationale to the case at bar, it cannot be said that the "dispute" is between the union and petitioner company because both have previously agreed upon the provision on "compulsory retirement" as embodied in the CBA. Also, it was only private respondent on his own who questioned the compulsory retirement. Thus, the case is properly denominated as a "termination dispute" which comes under the jurisdiction of labor arbiters.

2. Phil. Airlines vs. Airline Pilots Asso. Of Phils., G.R. No. 143686, January 15, 2002 Facts: This case was stemmed from petitioner's act of unilaterally retiring airline pilot Captain Albino Collantes own Retirement Plan. Contending, inter alia, that the retirement of Captain Collantes constituted illegal dismissal and union busting. The Secretary of the DOLE assumed jurisdiction over the labor dispute. Base on his decision, PAL should first consult the pilot concerned before implementing his retirement. Issue: Is the decision of the Secretary of DOLE correct in saying that in the exercise of retiring their employees, the employer should first consult their employee concerned before implementing the retirement? Ruling: NO. The retirement of an employee may be done upon initiative and option of the management. Surely, the requirement to consult the pilots prior to their retirement defeats the exercise by management of its option to retire the said employees. It gives the pilot concerned an undue prerogative to assail the decision of management. Due process only requires that notice be given to the pilot of petitioner's decision to retire him. Hence, the Secretary of Labor overstepped the boundaries of reason and fairness when he imposed on petitioner the additional requirement of consulting each pilot prior to retiring him. Furthermore, when the Secretary of Labor and Employment imposed the added requirement that petitioner should consult its pilots prior to retirement, he resolved a question which was outside of the issues raised, thereby depriving petitioner an opportunity to be heard on this point. 3. Cainta Catholic School vs. Cainta Catholic School Employees Union, G.R. No. 151021, May 4, 2006 citing 1996 Pantranco North Express Facts:

Page 221

LABOR RELATIONS
Atty. Jefferson M. Marquez
On 6 March 1986, a Collective Bargaining Agreement (CBA) was entered into between Cainta Catholic School (School) and the Cainta Catholic School Employees Union (Union) effective 1 January 1986 to 31 May 1989. On 15 October 1993, the School retired Llagas and Javier, who had rendered more than twenty (20) years of continuous service, pursuant to Section 2, Article X of the CBA, to wit: An employee may be retired, either upon application by the employee himself or by the decision of the Director of the School, upon reaching the age of sixty (60) or after having rendered at least twenty (20) years of service to the School the last three (3) years of which must be continuous. Three (3) days later, the Union filed a notice of strike with the National Conciliation and Mediation Board (NCMB). On 8 November 1993, the Union struck and picketed the Schools entrances. On 27 July 1994, the Union filed a complaint9 for unfair labor practice before the NLRC. Three (3) issues were passed upon by the NLRC, namely: (1) whether the retirement of Llagas and Javier is legal; (2) whether the School is guilty of unfair labor practice; and (3) whether the strike is legal. The NLRC ruled that the retirement of Llagas and Javier is legal as the School was merely exercising an option given to it under the CBA. The NLRC dismissed the unfair labor practice charge against the School for insufficiency of evidence. Furthermore, it was found that the strike declared by the Union from 8 to 12 November 1993 is illegal, thereby declaring all union officers to have lost their employment status. In reversing the decision of the NLRC, the Court of Appeals construed the retirement of Llagas and Javier as an act amounting to unfair labor practice when viewed against the backdrop of the relevant circumstances obtaining in the case. Issue: The key issue remains whether the forced retirement of Llagas and Javier was a valid exercise of management prerogative. Undoubtedly, the retirement of the two (2) union officers triggered the declaration of strike by the Union, and the ruling on whether the strike was legal is highly dependent on whether the retirement was valid. Ruling: We are impelled to reverse the Court of Appeals and affirm the validity of the termination of employment of Llagas and Javier, arising as it did from a management prerogative granted by the mutually-negotiated CBA between the School and the Union. Pursuant to the existing CBA, the School has the option to retire an employee upon reaching the age limit of sixty (60) or after having rendered at least twenty (20) years of service to the School, the last three (3) years of which must be continuous. Retirement is a different specie of termination of employment from dismissal for just or authorized causes under Articles 282 and 283 of the Labor Code. While in all three cases, the employee to be terminated may be unwilling to part from service, there are eminently higher standards to be met by the employer validly exercising the prerogative to dismiss for just or authorized causes. In those two instances, it is indispensable that the employer establish the existence of just or authorized causes for dismissal as spelled out in the Labor Code. Retirement, on the other hand, is the result of a bilateral act of the parties, a voluntary agreement between the employer and the employee whereby the latter after reaching a certain age agrees and/or consents to sever his employment with the former. Article 287 of the Labor Code, as amended, governs retirement of employees, stating: ART. 287. Retirement. Any employee may be retired upon reaching the retirement age established in the collective bargaining agreement or other applicable employment contract. In case of retirement, the employee shall be entitled to receive such retirement benefits as he may have earned under existing laws and any collective bargaining agreement and other agreements: Provided, however, That an employees retirement benefits under any collective bargaining agreement and other agreements shall not be less than those provided herein. In the absence of a retirement plan or agreement providing for retirement benefits of employees in the establishment, an employee upon reaching the age of sixty (60) years or more, but not beyond sixty-five (65) years which is hereby declared the compulsory retirement age, who has served at least five (5) years in the said establishment, may retire and shall be entitled to retirement pay equivalent to at least one-half (1/2) month salary for every year of service, a fraction of at least six (6) months being considered as one whole year. The CBA in the case at bar established 60 as the compulsory retirement age. However, it is not alleged that either Javier or Llagas had reached the compulsory retirement age of 60 years, but instead that they had rendered at least 20 years of service in the School, the last three (3) years continuous. Clearly, the CBA provision allows the employee to be retired by the School even before reaching the age of 60, provided that he/she had rendered 20 years of service. Would such a stipulation be valid? Jurisprudence affirms the position of the School.

Page 222

LABOR RELATIONS
Atty. Jefferson M. Marquez
By their acceptance of the CBA, the Union and its members are obliged to abide by the commitments and limitations they had agreed to cede to management. The questioned retirement provisions cannot be deemed as an imposition foisted on the Union, which very well had the right to have refused to agree to allowing management to retire retire employees with at least 20 years of service. It should not be taken to mean that retirement provisions agreed upon in the CBA are absolutely beyond the ambit of judicial review and nullification. A CBA, as a labor contract, is not merely contractual in nature but impressed with public interest. If the retirement provisions in the CBA run contrary to law, public morals, or public policy, such provisions may very well be voided. Certainly, a CBA provision or employment contract that would allow management to subvert security of tenure and allow it to unilaterally retire employees after one month of service cannot be upheld. Neither will the Court sustain a retirement clause that entitles the retiring employee to benefits less than what is guaranteed under Article 287 of the Labor Code, pursuant to the provisions express proviso thereto in the provision. Yet the CBA in the case at bar contains no such infirmities which must be stricken down. There is no essential difference between the CBA provision in this case and those we affirmed in Pantranco and Progressive. Twenty years is a more than ideal length of service an employee can render to one employer. Under ordinary contemplation, a CBA provision entitling an employee to retire after 20 years of service and accordingly collect retirement benefits is reward for services rendered since it enables an employee to reap the fruits of his labor particularly retirement benefits, whether lump-sum or otherwise at an earlier age, when said employee, in presumably better physical and mental condition, can enjoy them better and longer. Nonetheless, the premise warrants considering whether management may be precluded from retiring an employee whom it is entitled to retire upon a determination that the true cause for compulsory retirement is the employees union activities. The law and this Court frowns upon unfair labor practices by management, including so-called union-busting. Such illegal practices will not be sustained by the Court, even if guised under ostensibly legal premises. But with respect to an active unionized employee who claims having lost his/her job for union activities, there are different considerations presented if the termination is justified under just or authorized cause under the Labor Code; and if separation from service is effected through the exercise of a duly accorded management prerogative to retire an employee. There is perhaps a greater imperative to recognize the management prerogative on retirement than the prerogative to dismiss employees for just or authorized causes. For one, there is a greater subjectivity, not to mention factual dispute, attached to the concepts of just or authorized cause than retirement which normally contemplates merely the attainment of a certain age or a certain number of years in the service. It would be easier for management desirous to eliminate pesky union members to abuse the prerogative of termination for such purpose since the determination of just or authorized cause is rarely a simplistic question, but involves facts highly prone to dispute and subjective interpretation. On the other hand, the exercise by management of its retirement prerogative is less susceptible to dubitability as to the question whether an employee could be validly retired. The only factual matter to consider then is whether the employee concerned had attained the requisite age or number of years in service pursuant to the CBA or employment agreement, or if none, pursuant to Article 287 of the Labor Code. In fact, the question of the amount of retirement benefits is more likely to be questioned than the retirement itself. Evidently, it more clearly emerges in the case of retirement that management would anyway have the right to retire an employee, no matter the degree of involvement of said employee in union activities. There is another point that militates against the Union. A ruling in its favor is tantamount to a concession that a validly drawn management prerogative to retire its employees can be judicially interfered on a showing that the employee in question is highly valuable to the union. Such a rule would be a source of mischief, even if narrowly carved out by the Court, for it would imply that an active union member or officer may be, by reason of his/her importance to the union, somehow exempted from the normal standards of retirement applicable to the other, perhaps less vital members of the union. Indeed, our laws protection of the right to organize labor does not translate into perpetual job security for union leaders by reason of their leadership role alone. Should we entertain such a notion, the detriment is ultimately to the union itself, promoting as it would a stagnating entrenched leadership. We can thus can comfortably uphold the principle, as reiterated in Philippine Airlines, 34 that the exercise by the employer of a valid and duly established prerogative to retire an employee does not constitute unfair labor practice.

4. Jaculbe vs. Silliman University, G.R. No. 156934, March 16, 2007 Facts: Respondent, through its Human Resources Development Office, informed petitioner that she was approaching her 35th year of service with the university and was due for automatic retirement on November 18, 1993, at which time she would be 57 years old. This was pursuant to respondents retirement plan for its employees which provided that its members could be automatically retired "upon reaching the age of 65 or

Page 223

LABOR RELATIONS
Atty. Jefferson M. Marquez
after 35 years of uninterrupted service to the university." Respondent required certain documents in connection with petitioners impending retirement. Petitioner emphatically insisted that the compulsory retirement under the plan was tantamount to a dismissal and pleaded with respondent to be allowed to work until the age of 60 because this was the minimum age at which she could qualify for SSS pension. But respondent stood pat on its decision to retire her, citing "company policy." On November 15, 1993, petitioner filed a complaint in the National Labor Relations Commission (NLRC) for "termination of service with preliminary injunction and/or restraining order." On November 18, 1993, respondent compulsorily retired petitioner. Issue:S 1) did respondents retirement plan imposing automatic retirement after 35 years of service contravene the security of tenure clause in the 1987 Constitution and the Labor Code? 2) did respondent commit illegal dismissal by retiring petitioner solely by reason of such provision in its retirement plan? Labor Arbiter: found respondent guilty of illegal dismissal and ordered that petitioner be reinstated and paid full backwages. NLRC: reversed the labor arbiters decision and dismissed the complaint for lack of merit and likewise denied petitioners motion for reconsideration. CA: affirmed the NLRC. Hence this petition. Ruling: AFFIRMATIVE. Retirement plans allowing employers to retire employees who are less than the compulsory retirement age of 65 are not per se repugnant to the constitutional guaranty of security of tenure. Article 287 of the Labor Code provides: ART. 287. Retirement - Any employee may be retired upon reaching the retirement age established in the collective bargaining agreement or other applicable employment contract. Xxx By its express language, the Labor Code permits employers and employees to fix the applicable retirement age at below 60 years. However, after reviewing the assailed decision together with the rules and regulations of respondents retirement plan, we find that the plan runs afoul of the constitutional guaranty of security of tenure contained in Article XIII, also known as the provision on Social Justice and Human Rights. The CA, in ruling against petitioner, premised its decision to uphold the retirement plan on her voluntary participation therein: The petitioner in this case may, however, argue that the Pantranco case is not applicable in the case at bar as the controversy in the said case involves a compulsory retirement on the basis of the length of service rendered by the employee as agreed in an existing CBA, whereas in the present case, the private respondent compulsorily retired the petitioner not based on a CBA but on the retirement scheme provided for in the private respondents retirement plan. Nonetheless, this argument must fail. The contract fixing for retirement age as allowed under Article 287 of the Labor Code does not exclusively refer to CBA which provides for an agreed retirement age. The said provision explicitly allows, as well, other applicable employment contract to fix retirement age. The records disclose that the private respondents Retirement Plan has been in effect for more than 30 years. The said plan is deemed integrated into the employment contract between private respondent and its employees as evidenced by the latters voluntary contribution through monthly salary deductions. Previous retirees have already enjoyed the benefits of the retirement plan, and ever since the said plan was effected, no questions or disagreement have been raised, until the same was made to apply to the petitioner. The problem with this line of reasoning is that a perusal of the rules and regulations of the plan shows that participation therein was not voluntary at all. Rule III of the plan, on membership, stated: SECTION 1 MEMBERSHIP All full-time Filipino employees of the University will automatically become members of the Plan, provided, however, that those who have retired from the University, even if rehired, are no longer eligible for membership in the Plan. A member who continues to serve the University cannot withdraw from the Plan.

Page 224

LABOR RELATIONS
Atty. Jefferson M. Marquez
xxx xxx xxx SECTION 2 EFFECTIVITY OF MEMBERSHIP Membership in the Plan starts on the day a person is hired on a full-time basis by the University. SECTION 3 TERMINATION OF MEMBERSHIP Termination of membership in the Plan shall be upon the death of the member, resignation or termination of employees contract by the University, or retirement from the University. Rule IV, on contributions, stated: The Plan is contributory. The University shall set aside an amount equivalent to 3% of the basic salaries of the faculty and staff. To this shall be added a 5% deduction from the basic salaries of the faculty and staff. A member on leave with the University approval shall continue paying, based on his pay while on leave, his leave without pay should pay his contributions to the Plan. However, a member, who has been on leave without pay should pay his contributions based on his salary plus the Universitys contributions while on leave or the full amount within one month immediately after the date of his reinstatement. Provided[,] further that if a member has no sufficient source of income while on leave may pay within six months after his reinstatement. From the language of the foregoing retirement plan rules, the compulsory nature of both membership in and contribution to the plan debunked the CAs theory that petitioners "voluntary contributions" were evidence of her willing participation therein. It was through no voluntary act of her own that petitioner became a member of the plan. In fact, the only way she could have ceased to be a member thereof was if she stopped working for respondent altogether. Furthermore, in the rule on contributions, the repeated use of the word "shall" ineluctably pointed to the conclusion that employees had no choice but to contribute to the plan (even when they were on leave). According to the assailed decision, respondents retirement plan "ha(d) been in effect for more than 30 years." What was not pointed out, however, was that the retirement plan came into being in 1970 or 12 years after petitioner started working for respondent. In short, it was not part of the terms of employment to which petitioner agreed when she started working for respondent. Neither did it become part of those terms shortly thereafter, as the CA would have us believe. Retirement is the result of a bilateral act of the parties, a voluntary agreement between the employer and the employee whereby the latter, after reaching a certain age agrees to sever his or her employment with the former. In Pantranco North Express, Inc. v. NLRC, to which both the CA and respondent refer, the imposition of a retirement age below the compulsory age of 65 was deemed acceptable because this was part of the CBA between the employer and the employees. The consent of the employees, as represented by their bargaining unit, to be retired even before the statutory retirement age of 65 was laid out clearly in black and white and was therefore in accord with Article 287. In this case, neither the CA nor the respondent cited any agreement, collective or otherwise, to justify the latters imposition of the early retirement age in its retirement plan, opting instead to harp on petitioners alleged "voluntary" contributions to the plan, which was simply untrue. The truth was that petitioner had no choice but to participate in the plan, given that the only way she could refrain from doing so was to resign or lose her job. It is axiomatic that employer and employee do not stand on equal footing, a situation which often causes an employee to act out of need instead of any genuine acquiescence to the employer. This was clearly just such an instance. Not only was petitioner still a good eight years away from the compulsory retirement age but she was also still fully capable of discharging her duties as shown by the fact that respondents board of trustees seriously considered rehiring her after the effectivity of her "compulsory retirement." As already stated, an employer is free to impose a retirement age less than 65 for as long as it has the employees consent. Stated conversely, employees are free to accept the employers offer to lower the retirement age if they feel they can get a better deal with the retirement plan presented by the employer. Thus, having terminated petitioner solely on the basis of a provision of a retirement plan which was not freely assented to by her, respondent was guilty of illegal dismissal. At this point, reinstatement is out of the question. Petitioner is now 71 years old and therefore well over the statutory compulsory retirement age. For this reason, we grant her separation pay in lieu of reinstatement. It is also for this reason that we modify the award of backwages in her favor, to be computed from the time of her illegal dismissal on November 18, 1993 up to her compulsory retirement age. WHEREFORE, the petition is hereby GRANTED.

Page 225

LABOR RELATIONS
Atty. Jefferson M. Marquez
5. Globe Telecom vs. Crisologo, G.R. No. 17644, August 10, 2007 Facts: Respondent Jenette Marie B. Crisologo, a lawyer, joined Globe Telecom (Globe) on November 3, 1998 as a manager in its corporate legal services department.[6] Her tasks included negotiating, drafting and reviewing the companys supply contracts.[7] On April 5, 2002, respondent (who was then pregnant) was rushed to the Makati Medical Center due to profuse bleeding. It was later diagnosed as a possible miscarriage.[8] After a week-long absence, respondent reported back to work on April 12, 2002.[9] On the same day, she tendered her resignation letter explaining that she was advised by her doctor to rest for the duration of her pregnancy.[10] She also requested permission to exhaust her unused leaves until the effective date of her resignation on May 30, 2002.[11] Globe accepted her resignation. On April 30, 2002, respondent called on her immediate supervisor, petitioner Ma. Caridad Gonzales.[12] In the course of their conversation, petitioner Gonzales casually informed respondent of an e-mail circulating within the company[13] to the effect that she (respondent) allegedly solicited money from one of the companys suppliers.[14] Because the e-mail was not forwarded to her (being its subject), respondent requested a copy and an opportunity to confront the person(s) responsible. Petitioner Gonzales declined as there was no longer any reason to pursue the matter.[15] On May 2, 2002, respondent sent petitioner Gonzales a letter complaining of her ill-treatment by the company after she submitted her resignation letter.[16] She also confided that she resigned only because the e-mail damaged her name and reputation.[17] For that reason, she requested petitioner Gonzales to issue a certification clearing her of any wrongdoing, misconduct or transgression.[18] Petitioner Gonzales reminded respondent that, as a former executive, she should have been familiar with the company's standard operating procedure with regard to former employees. All employees basically undergo the same procedure upon separation from the company.[19] Gonzales also requested respondent to settle her debts and accountabilities to the company.[20] Meanwhile, Globe issued a certification attesting to respondents employment in the company from November 3, 1998 to May 30, 2002.[21] On May 2, 2002, respondent sent petitioners another letter. She insinuated that petitioners forced her to resign and reiterated her demand that Globe clear her name.[22] Petitioner Gonzales informed respondent that she had to settle her obligations to Globe first before it could issue the requested clearance.[23] Believing that Globe would not comply with her demands, respondent filed a complaint for illegal dismissal against petitioners on July 3, 2002.[24] According to respondent, petitioners fired her on the basis of a rumor whose veracity was never proven.[25] She was neither furnished a copy of the e-mail nor allowed to confront the person(s) who circulated it. Petitioner Gonzales immediately closed the matter with finality without conducting any inquiry.[26] Furthermore, petitioners failed not only to adduce clear and substantial proof of loss of confidence but also to observe due process[27] as petitioner Gonzales summarily forced her to resign.[28] Petitioners, on the other hand, contended that respondents clear and unequivocal resignation letter showed her unconditional desire to resign.[29] Ruling: To support their contention that respondent voluntarily resigned, petitioners presented her resignation letter dated April 12, 2002[46]: This is to inform you that as per my doctors advice, I have to take a long rest due to a very difficult pregnancy and other health reasons. I am therefore tendering my resignation effective 30 May 2002 and would like to request that I be allowed to exhaust all leaves due to me until such date. Furthermore, I hereby undertake to turn over all my pending work to other lawyers until said effective date of my termination. Thank you very much.[47] (emphasis supplied) Respondent personally drafted her resignation letter in a clear, concise and categorical language. Its content, as quoted above, confirmed her unequivocal intent to resign. An employee of respondents accomplished educational background and professional standing will not easily relinquish her legal rights unless she intends to.[48] Respondents resignation letter without doubt proved petitioners assertion that she voluntarily resigned from her job.

Page 226

LABOR RELATIONS
Atty. Jefferson M. Marquez
Moreover, the resignation letter was submitted by respondent and was accepted by Globe on April 12, 2002. This fact alone completely negated her claim that petitioners coerced her to resign on April 30, 2002. Indeed, how could she have been forced to resign on that date when she had already tendered her resignation more than two weeks earlier? Respondent Could Not Have Been Coerced or Intimidated. Coercion exists when there is a reasonable or well-grounded fear of an imminent evil upon a person or his property or upon the person or property of his spouse, descendants or ascendants.[51] No such situation existed in this case. As a matter of fact, respondents resignation letter[52] and May 2, 2002 letter[53] both contained expressions of gratitude. In her May 2, 2002 letter, she told petitioner Gonzales: I wish to express my appreciation for the training you readily gave me while I was under your supervision.[54] In St. Michael Academy v. NLRC,[55] we held that expressions of gratitude cannot possibly come from an employee who is just forced to resign as they belie allegations of coercion.[56] Moreover, the May 2, 2002 letter was sent after respondents April 30, 2002 conversation with petitioner Gonzales. Indeed, if something untoward really took place in the course of that conversation, experience dictates that respondent would not have bothered to thank petitioner Gonzales. Therefore, respondents assertion that she was forced to resign was simply not true.

6. BMG Records Phils et al., vs. Aparecio, et al., G.R. No. 153290, September 5, 2007, citing Phil Today vs. NLRC, 267 SCRA 202 [1996] Facts: Petitioner BMG Records (Phils.), Inc. (BMG) is engaged in the business of selling various audio records nationwide. On September 2, 1990, it hired private respondent Aida C. Aparecio (Aparecio) as one of the promo girls in its Cebu branch. For working from Monday to Sunday, she received a salary of P181.00 per day. On May 25, 1998, Aparecio filed a complaint against BMG and its Branch Manager, Jose Yap, Jr., co-petitioner herein, for illegal dismissal and non-payment of overtime pay, holiday pay, premium pay for rest day, 13th month pay, service incentive leave, and separation pay.[5] In her Position Paper, she alleged: That she was illegally dismissed or terminated [from] employment on April 30, 1998; that before said date[,] however, she was asked by respondent to resign and will be paid (sic) all her benefits due like a one-month pay for every year of service, payment of services rendered, overtime and holiday pay, rest day, 13th month, service incentive leave and separation pay and to [execute] a letter of resignation; That in view of respondents insistence to prepare and [execute] a letter-resignation[,] even without proper accounting of any accountability, the complainant was lured, induced and compelled to submit a letter of resignation believing on respondents promise and assurance to pay all the benefits due her as aforesaid; That after executing said resignation letter, the respondent did not make good its promise and [instead] did an accounting by themselves in the absence of herein complainant and arrived on a computation that complainants liability per their accounting reached to the staggering amount of P8,000.00; that since they offered to pay a separation pay of only P12,000.00, minus complainants alleged accountability of P8,000.00, they are ready to pay the balance thereof any time; That herein complainant was under respondents employ for seven (7) years, seven (7) months and twenty-eight (28) days when illegally terminated [from] her employment xxx. Petitioners, however, proffer a different version of the facts. They narrate that Aparecio was initially performing well as an employee but as years passed by she seemed to be complacent in the performance of her job and had been comparing the salaries of promo girls in other companies. It appeared that she was no longer interested in her job. In April 1998, Aparecio and two other promo girls, Jovelina V. Soco and Veronica P. Mutya, intimated to their supervisor that they were intending to resign and were requesting for some financial assistance. BMG made it clear that, as a company policy, an employee who resigns from service is not entitled to financial assistance, but considering the length of their service and due to humanitarian consideration it would accede to the request after they secure their respective clearances. Forthwith, the three employees tendered their resignations, which were accepted. When they processed the required individual clearance, it was found out that they had incurred some shortages after inventory. Per agreement, said shortages were deducted from the amounts due them. Thus, Soco and Mutya received their last salary, a proportion of the 13th month pay, tax refund and financial assistance less the deductions, and they executed their releases and quitclaims. Except for the financial assistance, Aparecio also obtained the same yet refused to sign the release and quitclaim, protesting the amount of P9,170.12 deducted from the financial assistance. She was adamant but BMG stood by the previous agreement. Ruling: After careful analysis, this Court finds and so holds that the submissions of Aparecio in all her pleadings failed to substantiate the allegation that her consent was vitiated at the time she tendered her resignation and that petitioners are guilty of illegal dismissal. In a nutshell, Aparecio submits that fraud, undue influence, intimidation, and/or mistake were attendant upon her resignation from BMG. As her consent was allegedly vitiated, the act of resigning became involuntary; hence, petitioners are guilty of illegal dismissal.

Page 227

LABOR RELATIONS
Atty. Jefferson M. Marquez
The argument is not tenable. Resignation is the voluntary act of an employee who is in a situation where one believes that personal reasons cannot be sacrificed in favor of the exigency of the service, and one has no other choice but to dissociate oneself from employment. It is a formal pronouncement or relinquishment of an office, with the intention of relinquishing the office accompanied by the act of relinquishment. As the intent to relinquish must concur with the overt act of relinquishment, the acts of the employee before and after the alleged resignation must be considered in determining whether in fact, he or she intended to sever from his or her employment. Thus, this Court agrees with petitioners contention that the circumstances surrounding Aparecios resignation should be given due weight in determining whether she had intended to resign. In this case, such intent is very evident: First, Aparecio already communicated to other people that she was about to resign to look for a better paying job since she had been complaining that employees like her in other companies were earning much more; Second, prior to the submission of her resignation letter, Aparecio and two other promo girls, Soco and Mutya, approached their supervisor, intimated their desire to resign, and requested that they be given financial assistance, which petitioners granted on the condition that deductions would be made in case of shortage after inventory; Third, Aparecio, Soco, and Mutya submitted their duly signed resignation letters, which were accepted by petitioners; and Fourth, Aparecio already initiated the processing of her clearance; thus, she was able to receive her last salary, 13th month pay, and tax refund but refused to receive the financial assistance less the deductions made. The foregoing facts were affirmatively narrated and attested to in the notarized affidavit of Soco and Cinco and have remained incontrovertible as they were never denied by Aparecio. The NLRC, thus, erred when it did not give probative weight to their testimonies even if belatedly presented in petitioners motion for reconsideration. Now, the acceptance by petitioners of Aparecios resignation rendered the same effective. Upon such acceptance, it may not be unilaterally withdrawn without the consent of petitioners. When the employee later signified the intention of continuing his or her work, it was already up to the employer to accept the withdrawal of his or her resignation. The mere fact that the withdrawal was not accepted does not constitute illegal dismissal, the acceptance of the withdrawal of the resignation being the employers sole prerogative. As held in Intertrod Maritime, Inc. v. NLRC: Once an employee resigns and his resignation is accepted, he no longer has any right to the job. If the employee later changes his mind, he must ask for approval of the withdrawal of his resignation from his employer, as if he were re-applying for the job. It will then be up to the employer to determine whether or not his service would be continued. If the employer accepts said withdrawal, the employee retains his job. If the employer does not x x x the employee cannot claim illegal dismissal for the employer has the right to determine who his employees will be. To say that an employee who has resigned is illegally dismissed, is to encroach upon the right of employers to hire persons who will be of service to them. A resigned employee who desires to take his job back has to re-apply therefor, and he shall have the status of a stranger who cannot unilaterally demand an appointment. He cannot arrogate unto himself the same position which he earlier decided to leave. To allow him to do so would be to deprive the employer of his basic right to choose whom to employ. Such is tantamount to undue oppression of the employer. It has been held that an employer is free to regulate, according to his own discretion and judgment, all aspects of employment including hiring. The law, in protecting the rights of the laborer, impels neither the oppression nor self-destruction of the employer.

7. Blue Angel Manpower and Security Services vs. CA, G.R. No. 161196, July 28, 2008 Facts: Blue Angel, a messengerial and security agency, hired private respondents Romel Castillo, Wilson Ciriaco, Gary Garces, and Chesterfield Mercader as security guards and detailed them at the National College of Business and Arts (NCBA) in Cubao, Quezon City. Castillo and Mercader, later joined by Ciriaco and Garces, filed a complaint for illegal deductions and other money claims against Blue Angel. Eventually, they amended their complaint to include illegal dismissal. According to the four guards, they were required, while still with Blue Angel, to work from 7:00 a.m. to 7:00 p.m. without overtime and premium holiday pay, among other benefits. They also alleged receiving only PhP 5,000 a month or PhP 166 per day and, from this amount, Blue Angel deducted PhP 100 as cash bond. They further averred that Blue Angel,

Page 228

LABOR RELATIONS
Atty. Jefferson M. Marquez
when apprised of their original complaint, illegally terminated Garces and Ciriaco, respectively, and Castillo and Mercader. The four guards prayed for (1) payment of backwages, wage differentials, premium and overtime pay for holidays, and 13th month pay; (2) reimbursement of their cash bond; (3) reinstatement or separation pay; and (4) damages. Issue: Whether or not private respondents were illegally dismissed Ruling: We rule that the resignations were involuntary and the termination of private respondents was illegal. Blue Angel insists that the guards had pleaded to be allowed to resign when they were told of the pending investigation, and that they eventually tendered their pro-forma resignation letters followed by their own handwritten resignation letters. Our review of the circumstances surrounding these resignation letters does not support Blue Angel's contentions that these letters are indications that private respondents had voluntarily resigned. We agree with the labor arbiter when he pointed out that the undated, similarly worded resignation letters tended to show that the guards were made to copy the pro-forma letters, in their own hand, to make them appear more convincing that the guards had voluntarily resigned. As the labor arbiter noted, the element of voluntariness of the resignations is even more suspect considering that the second set of resignation letters were pre-drafted, similarly worded, and with blank spaces filled in with the effectivity dates of the resignations.[5] In their Comment, private respondents claimed being forced to sign and copy the pro-forma resignation letters and quitclaims on pain that they would not get their remaining compensations. With the finding that private respondents were illegally dismissed, they are entitled to reinstatement to their positions without loss of their seniority rights and with full backwages, inclusive of allowances, and to other benefits or their monetary equivalent computed from the time private respondents' compensation was withheld from them up to the time of their actual reinstatement as provided for in Article 279 of the Labor Code.

8. Guerzon Jr et al vs. Pasig Industries Inc., et al., G.R. No. 170266, Sept. 12, 2008 Facts: Petitioners were employees of respondent Pasig Industries, Inc. (PII) stationed in its Makati office. Guerzon was PII's export/import manager for 21 years; Cruz was the company's chief accountant for 20 years and Bauyon was a member of PII's accounting staff since 1989. In 1995, respondent Yoshikitsu Fujita informed petitioners that PII's parent company had decided to close the Makati office. To streamline operations, functions performed by the Makati office would be transferred to its facilities in the Bataan Export Processing Zone. For this reason, petitioners were given the option to resign, in which case they would be entitled to a special separation package (SSP) equivalent to one-month basic salary for each year of service. Petitioners decided to resign but requested a recomputation of their respective separation pay based on the monthly gross pay (i.e., basic pay plus all allowances). Despite voluntarily availing of the SSP, petitioners filed a complaint for illegal dismissal and payment of separation pay, retirement benefits, leave pay and 13th month pay against PII, its president Masahiro Fukada and Fujita in the National Labor Relations Commission (NLRC). Because petitioners filed the complaint two days after they were "terminated," the labor arbiter found respondents guilty of illegal dismissal. Accordingly, he awarded backwages, separation pay and attorneys' fees to petitioners. Respondents appealed. The NLRC found that petitioners voluntarily accepted the terms of the SSP offered by PII. It noted that they negotiated to improve PII's offered SSP. Thus, the NLRC reversed the decision of the labor arbiter. Aggrieved, petitioners filed a petition for certiorari in the Court of Appeals (CA) asserting that the NLRC committed grave abuse of discretion in reversing the decision of the labor arbiter.

Page 229

LABOR RELATIONS
Atty. Jefferson M. Marquez
Hence, petitioners availed of this recourse contending that the CA erred in affirming the decision of the NLRC. Respondents allegedly failed to prove that PII had been incurring losses to justify its reorganization. They claimed they were dismissed without just or authorized cause. Issues: Whether or not the petitioners were illegally dismissed. Is the streamlining an authorized cause for the termination of the petitoners? Ruling: The petitioners were NOT illegally dismissed. As they voluntarily resigned from PII. Petitioners held responsible positions in PII. Employees of their educational backgrounds and professional standing do not easily relinquish their legal rights unless they intend to. In fact, petitioners even bargained to improve the terms of the SSP and, after successfully doing so, voluntarily resigned from PII. Consequently, whether the streamlining of PII's operations constituted an authorized cause for petitioners' termination became immaterial in view of their voluntary resignation.

9. Suarez Jr. et al., vs. National Steel Corp., G.R. No. 150180, Oct. 17, 2008 Facts: National Steel Corporation was engaged in the business of manufacturing steel products needed for pipe making, ship building, can-making and production of appliances. Sometime in 1994, respondent suffered substantial financial losses due to an increase in the volume of steel products manufactured by foreign countries. With this development, respondent adopted an organizational streamlining program that resulted in the retrenchment of seven hundred (700) employees in its main plant in Iligan City, among whom were herein petitioners. One month prior to its effectivity, respondent sent out individual notices to the seven hundred (700) employees affected by the retrenchment, including petitioners. The notices specifically stated that their services were terminated effective on said date and they will each receive a separation package in accordance with the retrenchment program. The separation package consisted of the following: (1) separation pay equivalent to two (2) months salary for every year of service; (2) leave balance credits; (3) 13th month pay; and (4) uniform plus rice subsidy differential. After having been paid their separation benefits, the employees, including herein petitioners, each executed and signed a release and quitclaim, written in English and containing a translation in the Visayan dialect in the same document. Nothing was heard from the retrenched employees, until about two and half years after their separation from the company, when herein petitioners wrote respondent demanding payment of retirement benefits under the CBA. They claimed that they were qualified for optional retirement after having rendered services for at least ten (10) years when they were retrenched. Respondent rejected petitioners' claim, forcing petitioners to file a complaint for payment of retirement benefits against respondent. The Labor Arbiter dismissed the complaint for lack of merit. Upon appeal, NLRC granted the appeal and reversed the ruling of the Labor Arbiter. By way of Petition for Certiorari, CA declared that petitioners were no longer entitled to retirement benefits after having received the separation pay, and were precluded from claiming such benefits because of their quitclaims. Issue: Whether or not petitioners (retrenched employees) can still recover retirement benefits in addition to their separation pay. Ruling: No, petitioners are no longer entitled to recover retirement benefits. Having been separated from employment due to an authorized cause, petitioners are barred from receiving retirement benefits pursuant to Article X(E) of respondent's retirement plan. With the inclusion of such provision in the retirement plan, respondent categorically disallows payment of retirement benefits to retrenched employees. They are only entitled to payment of separation pay in accordance with Article 283 of the Labor Code. The CA committed no error in considering the affidavits as contemporaneous and subsequent acts from which the intention of the parties to the CBA can be inferred. While the CBA, on its face, does not contain an express prohibition of payment of retirement benefits to retrenched employees, the parties may still prove it by means of contemporaneous and subsequent acts of the parties to the agreement, such as the execution of the affidavits. In their affidavits, they attested that under the CBA, an employee who is separated pursuant to a retrenchment

Page 230

LABOR RELATIONS
Atty. Jefferson M. Marquez
program and who received the corresponding separation package is completely proscribed from demanding and claiming payment of retirement benefits. Further, petitioners voluntarily executed and signed a release and quitclaim after receiving their separation package, acknowledging full and final payment of all benefits that they may be entitled to in relation to their employment. Thus, Petition for review is denied.

10. Goodrich Mfg Corp vs. Ativo et al., G.R. No. 188002, Feb. 1, 2010 Facts: Emerlina Ativo et al., are former employees of petitioner Goodrich Manufacturing Corporation (Goodrich) assigned as machine or maintenance operators. In the last quarter of 2004, Goodrich suffered financial constraints and gave all its employees the option to voluntarily resign from the company. Respondents were among those who availed of that option and were paid their separation pay. Ativo et al., executed their waivers and quitclaims. However, they changed their minds and filed for illegal dismissal against Goorichwith prayer for payment of their full monetary benefits before the NLRC. The Labor Arbiter held that there was no illegal dismissal but ruled that Goodrich was still liable for the employees SIL, ECOLA, and 13th month pay, and that the separation pay was insufficient. Mutually unhappy, both parties appealed to the NLRC which reversed the LAs decision. The NLRC said that the considerations they received are not unreasonable, vis--vis the awards granted [to] them in the assailed Decision. Notably, the awards even include the 13th month pays for 2002 and 2003 which, by respondents proof appear already paid. We also noted that complainants are not shown to have signed the deeds of waiver and quitclaim involuntarily, without understanding the implication and consequences thereof. The case was brought before the CA which renderred a decision in favor of Ativo et. al., holding that they are entitled to receive their unpaid 13th month pay, SIL, and ECOLA. And so the issue is now before the Supreme Court. Issue: W/N the release, waiver and quitclaim signed by respondents are valid and binding; and whether respondents may still receive the deficiency amounts due them. Ruling: The release, waiver and quitclaim are valid and binding. Capital wins this time. Requisites of a valid quitclaim It is true that the law looks with disfavor on quitclaims and releases by employees who have been inveigled or pressured into signing them by unscrupulous employers seeking to evade their legal responsibilities and frustrate just claims of employees. In certain cases, however, the Court has given effect to quitclaims executed by employees if the employer is able to prove the following requisites, to wit: (1) the employee executes a deed of quitclaim voluntarily; (2) there is no fraud or deceit on the part of any of the parties; (3) the consideration of the quitclaim is credible and reasonable; and (4) the contract is not contrary to law, public order, public policy, morals or good customs, or prejudicial to a third person with a right recognized by law. Not all waivers and quitclaims are invalid as against public policy. If the agreement was voluntarily entered into and represents a reasonable settlement, it is binding on the parties and may not later be disowned simply because of a change of mind. In their Comment19 dated October 1, 2009, respondents themselves admitted that they were not coerced to sign the quitclaims. They, however, maintain that two (2) reasons moved them to sign the said documents: first, they believed Goodrich was terminating its business on account of financial hardship; and second, they thought petitioners will pay them the full amount of their compensation. 21 Respondents insist that they were deceived into signing the quitclaims when they learned that they were not paid their full monetary benefits and after discovering that the company did not really close shop, but instead only assumed a different company name. Quitclaims were simple, clear and unequivocal The records of the case are bereft of any substantial evidence to show that respondents did not know that they were relinquishing their right short of what they had expected to receive and contrary to what they have so declared. Put differently, at the time they were signing their

Page 231

LABOR RELATIONS
Atty. Jefferson M. Marquez
quitclaims, respondents honestly believed that the amounts received by them were fair and reasonable settlements of the amounts which they would have received had they refused to voluntarily resign from the said company. Respondents were not deceived Ativo and company claim that they were deceived because petitioners did not really terminate their business since Mr. Chua Goy had set up another company with the same line of business as Goodrich. Such contention, however, was not proven during the hearing before the Labor Arbiter and the NLRC. Hence, such claim is based only on respondents surmises and speculations which, unfortunately, can never be used as a valid and legal ground to repudiate respondents quitclaims. Considerations received were not grossly inadequate As correctly pointed out by the NLRC, the total awards computed by the Labor Arbiter will definitely even be lesser after deducting the 13th month pay for the years 2002 and 2003, which have already been received by the respondents prior to the filing of their complaints, but which the Labor Arbiter still included in his computation. The difference between the amounts expected from those that were received may, therefore, be considered as a fair and reasonable bargain on the part of both parties. Petition is granted. CA decision is reversed and set aside. The NLRCs decision is reinstated. 11. Korean Air Co. Ltd. v. Yuson, G.R. No. 170369, June 16, 2010 Facts: Korean Air hired Yuson as reservations agent. Korean Air promoted Yuson to assistant manager in 1993, and to passenger sales manager in 1999. In April 2001, Yuson requested Korean Air that she be transferred from the passenger sales department to the cargo department, and so Korean Air temporarily transferred Yuson to the cargo department as "cargo dispatch." Yuson continued to receive the same compensation and exercise the same authority as passenger sales manager. In order to cut costs, Korean Air offered its employees an early retirement program (ERP). Yuson accepted the offer for early retirement. Suk (manager) informed Yuson that she was excluded from the ERP because she was retiring on 8 January 2002. Yuson claimed that Korean Air was bound by the perfected contract and accused the company of harassment and discrimination. On 28 November 2001, Yuson filed with the arbitration branch of the NLRC a complaint against Korean Air and Suk for payment of benefit under the ERP, moral damages, exemplary damages, and attorney's fees. LA denied for lack of merit Yuson's claims and held that the ERP memorandum included only rank-and-file, and excluded managerial, employees and such memo was reserved to Korean Air discretion in approving applications for the ERP and that approval of applications for the ERP was a valid exercise of Korean Air's management prerogative. A compromise agreement was entered into between Korean Air and Yuson, on 14 February 2003, whereby the agreement included among others the payment of P1,671,546.92, representing her retirement benefit pursuant to Article 287 of the Labor Code, as amended. NLRC affirmed LA decision, however CA set aside the decision. Issue: WON Yuson may still claim benefit under the ERP. Ruling: No, Yuson may no longer claim the benefit under the ERP. Third paragraph of Article 287 states that: In the absence of a retirement plan or agreement providing for retirement benefits of employees in the establishment, an employee upon reaching the age of sixty (60) years or more, but not beyond sixty-five (65) years which is hereby declared the compulsory retirement age, who has served at least five (5) years in the said establishment, may retire and shall be entitled to retirement pay equivalent to at least one-half (1/2) month salary for every year of service, a fraction of at least six (6) months being considered as one whole year.

Page 232

LABOR RELATIONS
Atty. Jefferson M. Marquez
On 14 February 2003, Yuson accepted P1,671,546.92 as retirement benefit under Article 287. Her claim for benefit under the ERP became moot when she availed of the optional retirement under Article 287 and accepted the benefit. By her acceptance of the benefit, Yuson is deemed to have opted to retire under Article 287. The Court of Appeals held that Yuson may claim benefit under the ERP because "the offer was certain and the acceptance is absolute; hence, there is a valid contract pursuant to the last paragraph of Article 1315 of the New Civil Code." The Court disagrees. In the present case, the offer is not certain: (1) the 21 August 2001 memorandum clearly states that, "MNLSM Management, on its discretion, is hereby offering the said early retirement program to its staff"; (2) applications for the ERP were forwarded to the head office for approval, and further acts on the offeror's part were necessary before the contract could come into existence; and (3) the 21 August 2001 memorandum clearly states Korean Air's intention, which was, "to prevent further losses." Korean Air could not have intended to ministerially approve all applications for the ERP.

12. Cercado v. Uniprom Inc., G.R. No. 188154, October 13, 2010 Facts: Petitioner Lourdes A. Cercado (Cercado) started working for respondent UNIPROM on December 15, 1978 as a ticket seller assigned at Fiesta Carnival, Araneta Center, Quezon City. Later on, she was promoted as cashier and then as clerk typist. UNIPROM instituted an Employees NonContributory Retirement Plan which provides that any participant with twenty (20) years of service, regardless of age, may be retired at his option or at the option of the company. It amended the retirement plan in compliance with Republic Act (R.A.) No. 7641. Under the revised retirement plan, UNIPROM reserved the option to retire employees who were qualified to retire under the program. UNIPROM implemented a company-wide early retirement program for its 41 employees, including herein petitioner, who, at that time, was 47 years old, with 22 years of continuous service to the company. She was offered an early retirement package amounting to P171,982.90, but she rejected the same. UNIPROM exercised its option under the retirement plan, and decided to retire Cercado effective at the end of business hours on February 15, 2001. A check of even date in the amount of P100,811.70, representing her retirement benefits under the regular retirement package, was issued to her. Cercado refused to accept the check. UNIPROM nonetheless pursued its decision and Cercado was no longer given any work assignment after February 15, 2001. This prompted Cercado to file a complaint for illegal dismissal before the Labor Arbiter (LA), alleging, among others, that UNIPROM did not have a bona fide retirement plan, and that even if there was, she did not consent thereto. The National Labor Relations Commission (NLRC) affirmed the LAs decision that Cercado was illegally dismissed. However, the CA set aside the decisions of the LA and the NLRC. Hence, this petition. Issue:S: Whether UNIPROM has a bona fide retirement plan Whether petitioner was validly retired pursuant thereto Ruling: The assailed retirement plan of UNIPROM is not embodied in a CBA or in any employment contract or agreement assented to by petitioner and her co-employees. On the contrary, UNIPROMs Employees Non-Contributory Retirement Plan was unilaterally and compulsorily imposed on them. This is evident in the following provisions of the 1980 retirement plan and its amended version in 2000: ARTICLE III ELIGIBILITY FOR PARTICIPATION Section 1. Any regular employee, as of the Effective Date, shall automatically become a Participant in the Plan, provided the Employee was hired below age 60. Verily, petitioner was forced to participate in the plan, and the only way she could have rejected the same was to resign or lose her job. The assailed CA Decision did not really make a finding that petitioner actually accepted and consented to the plan. The law demands more than a passive acquiescence on the part of employees, considering that an employers early retirement age option involves a concession of the formers

Page 233

LABOR RELATIONS
Atty. Jefferson M. Marquez
constitutional right to security of tenure. Retirement is the result of a bilateral act of the parties, a voluntary agreement between the employer and the employee whereby the latter, after reaching a certain age, agrees to sever his or her employment with the former. Acceptance by the employees of an early retirement age option must be explicit, voluntary, free, and uncompelled. While an employer may unilaterally retire an employee earlier than the legally permissible ages under the Labor Code, this prerogative must be exercised pursuant to a mutually instituted early retirement plan. In other words, only the implementation and execution of the option may be unilateral, but not the adoption and institution of the retirement plan containing such option. For the option to be valid, the retirement plan containing it must be voluntarily assented to by the employees or at least by a majority of them through a bargaining representative. Hence, consistent with the Courts ruling in Jaculbe case, having terminated petitioner merely on the basis of a provision in the retirement plan which was not freely assented to by her, UNIPROM is guilty of illegal dismissal.

13. Bilbao vs. Saudi Arabian Airlines, G.R. No. 183915, December 14, 2011 Facts Bilbao was a former employee of respondent Saudia, having been hired as a Flight Attendant on May 13, 1986 until her separation from Saudia in September 2004. During the course of her employment, Bilbao was assigned to work at the Manila Office, although the nature of her work as a flight attendant entailed regular flights from Manila to Jeddah, Saudi Arabia, and back. On August 25, 2004, the In-Flight Service Senior Manager of Saudia assigned in Manila received an inter-office Memorandum dated August 17, 2004 from its Jeddah Office regarding the transfer of 10 flight attendants from Manila to Jeddah effective September 1, 2004. The said memorandum explained that such transfer was made due to operational requirements. [3] Bilbao was among the 10 flight attendants to be transferred.Bilbao initially complied with the transfer order and proceeded to Jeddah for her new assignment. However, on September 7, 2004, she opted to resign and relinquish her post by tendering a resignation letter. On October 28, 2004, Bilbao executed and signed an Undertaking [5] similar to that of a Receipt, Release and Quitclaim wherein she acknowledged receipt of a sum of money as full and complete end-of-service award with final settlement and have no further claims whatsoever against Saudi Arabian Airlines. [6] In spite of this signed Undertaking, however, on July 20, 2005, Bilbao filed with the NLRC a complaint for reinstatement and payment of full backwages; moral, exemplary and actual damages; and attorneys fees. For her part, Bilbao maintained that her resignation from Saudia was not voluntary. Upon the other hand, Saudia averred that the resignation letters from Bilbao and her co-complainants were voluntarily made since they were actually hand-written and duly signed. Saudia asserted that Bilbao and her co-complainants were not subjected to any force, intimidation, or coercion when they wrote said resignation letters and even their undertakings, after receiving without protest a generous separation package despite the fact that employees who voluntarily resign are not entitled to any separation pay. Saudia also added that the transfer of flight attendants from their Manila Office to the Jeddah Office was a valid exercise of its management prerogative. On August 31, 2006, Labor Arbiter Reyes rendered a Decision [7] declaring that Bilbao, together with co-complainants Centi-Mandanas and Castells, was illegally dismissed . Saudia filed an appeal before the NLRC , Bilbao followed suit and also appealed before the NLRC . On June 25, 2007, the NLRC granted Saudias appeal, and reversed and set aside the decision of the Labor Arbiter Likewise, the Motion for Reconsideration of Maria Joy Teresa Bilbao is DENIED. Bilbao went to the Court of Appeals via a petition for certiorari . the Court of Appeals affirmed the Resolutions of the NLRC dated June 25, 2007 and October 26, 2007, and held that the resignation of Bilbao was of her own free will and intelligent act. Bilbao filed a motion for reconsideration which was denied by the Court of Appeals. Issue: Did Ms. Bilbao vountarilty resign from Saudi Arabian Airlines Ruling: After a review of the case, we uphold the findings of the Court of Appeals that Bilbao voluntarily resigned from her employment with Saudia. Her resignation letter and undertaking that evidenced her receipt of separation pay, when taken together with her educational attainment and the circumstances surrounding the filing of the complaint for illegal dismissal, comprise substantial proof of Bilbaos voluntary resignation.

Page 234

LABOR RELATIONS
Atty. Jefferson M. Marquez
Resignation is the voluntary act of an employee who is in a situation where one believes that personal reasons cannot be sacrificed in favor of the exigency of the service, and one has no other choice but to dissociate oneself from employment. It is a formal pronouncement or relinquishment of an office, with the intention of relinquishing the office accompanied by the act of relinquishment. As the intent to relinquish must concur with the overt act of relinquishment, the acts of the employee before and after the alleged resignation must be considered in determining whether he or she, in fact, intended to sever his or her employment. In the instant case, Bilbao tendered her resignation letter a week after her transfer to the Jeddah office. In the said letter, Bilbao expressed her gratitude for the support which Saudia had given her for her eighteen years of service. Clearly, her use of words of appreciation and gratitude negates the notion that she was forced and coerced to resign. Besides, the resignation letter was hand-written by Bilbao on a Saudia form and was in English, a language she is conversant in. Additionally, instead of immediately filing a complaint for illegal dismissal after she was allegedly forced to resign, Bilbao executed an Undertaking in favor of Saudia, wherein she declared that she received her full and complete endof-service award with final settlement. What is more, Bilbao waited for more than 10 months after her separation from Saudia to file a complaint for illegal dismissal. Despite the foregoing circumstances, Bilbao maintains that she was forced and coerced into writing the said resignation letter in the form prepared by Saudia, and that she was left with no other option but to resign. Saudia, on the other hand, claims that Bilbaos resignation was voluntary, thus, there could be no illegal dismissal. Even assuming that Saudia prepared the form in which Bilbao wrote her resignation letter as claimed, this Court is not convinced that she was coerced and intimidated into signing it. Bilbao is no ordinary employee who may not be able to completely comprehend and realize the consequences of her acts. She is an educated individual. It is highly improbable that with her long years in the profession and her educational attainment, she could be tricked and forced into doing something she does not intend to do. Under these circumstances, it can hardly be said that Bilbao was coerced into resigning from Saudia. Besides, Bilbao did not adduce any competent evidence to prove that she was forced or threatened by Saudia. It must be remembered that for intimidation to vitiate consent, the following requisites must be present: (1) that the intimidation caused the consent to be given; (2) that the threatened act be unjust or unlawful; (3) that the threat be real or serious, there being evident disproportion between the evil and the resistance which all men can offer, leading to the choice of doing the act which is forced on the person to do as the lesser evil; and (4) that it produces a well-grounded fear from the fact that the person from whom it comes has the necessary means or ability to inflict the threatened injury to his person or property. [19] In the instant case, Bilbao did not prove the existence of any one of these essential elements. Anent the Undertaking signed by Bilbao, this Court is of the opinion that the same was validly and voluntarily executed. Indeed, not all waivers and quitclaims are invalid as against public policy. There are legitimate waivers and quitclaims that represent a voluntary and reasonable settlement of workers claims which should be respected by the courts as the law between the parties. [20] And if such agreement was voluntarily entered into and represented a reasonable settlement, it is binding on the parties and should not later be disowned. Clearly then, Bilbaos claim that she was illegally dismissed cannot be sustained. There is no showing that the Undertaking and resignation letter were executed by Bilbao under force or intimidation. Bilbaos claims for reinstatement, payment of backwages without loss of seniority rights and with interest, moral and exemplary damages, and attorneys fees must inevitably fail.

14. San Miguel Properties vs. Gucaban, G.R. No. 153982, July 18, 2011 Facts: Respondent Gucaban, civil engineer, joined the workforce of petitioner San Miguel Properties Philippines, Inc. (SMPI) in 1991. Initially engaged as a construction management specialist, she, by her satisfactory performance on the job, was promoted in 1994 and 1995, respectively, to the position of technical services manager, and then of project development manager. As project development manager, she also sat as a member of the company's management committee. She had been in continuous service in the latter capacity until her severance from the company in February 1998. In her complaint for illegal dismissal, Gucaban alleged that her separation from service was practically forced upon her by management. She claimed that on January 27, 1998, she was informed by SMPI's President and Chief Executive Officer that the company was planning to reorganize its manpower in order to cut on costs, and that she must file for resignation or otherwise face termination. Gucaban complained of the ugly treatment which she had since received from Gonzalez and the management supposedly on account of her refusal to sign the resignation letter. She claimed she had been kept off from all the meetings of the management committee. Her performance of duties had been reported to be negligent and unsatisfactory. She found said report to be unfounded and unfair, because no less than the company's Vice-President for Property Management in a subsequent memorandum, had actually vouched for her competence and efficiency on the job. It was supposedly the extreme humiliation and alienation that impelled her to submit a signed resignation letter on February 18, 1998.

Page 235

LABOR RELATIONS
Atty. Jefferson M. Marquez
Gucaban surmised that she had merely been tricked by SMPI into filing her resignation letter because it never actualized its reorganization and streamlining plan; on the contrary, SMPI allegedly expanded its employee population and also made new appointments and promotions to various other positions. She felt that she had been dismissed without cause and, hence, prayed for reinstatement and payment of backwages and damages. The Labor Arbiter dismissed the complaint for lack of merit, finding no proven force, coercion, intimidation or any other circumstance which could otherwise invalidate Gucaban's resignation. He likewise dismissed her claim that SMPI merely feigned the necessity of reorganization in that while the company indeed made new other appointments following Gucaban's resignation, still, this measure was an implementation of its reorganization plan. Gucaban appealed to the NLRC which reversed the ruling of the Labor Arbiter. Finding that Gucaban has been illegally dismissed, it ordered her reinstatement without loss of seniority rights and with full backwages, as well as ordered the award of damages and attorney's fees. It elevated the matter to the Court of Appeals via a petition for certiorari. The Court of Appeals issued the assailed Decision finding partial merit in the petition. It affirmed the NLRC's finding of illegal/constructive dismissal, but modified the monetary award. Issue: Whether or not the resignation of Gucaban was tendered voluntarily. Ruling: Resignation the formal pronouncement or relinquishment of a position or office is the voluntary act of an employee who is in a situation where he believes that personal reasons cannot be sacrificed in favor of the exigency of the service, and he has then no other choice but to disassociate himself from employment. The intent to relinquish must concur with the overt act of relinquishment; hence, the acts of the employee before and after the alleged resignation must be considered in determining whether he in fact intended to terminate his employment. In illegal dismissal cases, fundamental is the rule that when an employer interposes the defense of resignation, on him necessarily rests the burden to prove that the employee indeed voluntarily resigned. Guided by these principles, we agree with the Court of Appeals that with the availing evidence, SMPI was unable to discharge this burden. The question of whether or not there was such reorganization plan in place at the time of Gucaban's separation from the company, is material to the determination of whether her resignation was of her own volition as claimed by SMPI, inasmuch as the facts of this case tell that Gucaban could not have filed for resignation had Gonzalez not communicated to her the alleged reorganization plan for the company. True, while a reorganization of SMPI's corporate structure might have indeed taken place as shown by the notices, nevertheless, it happened only in the latter part of 1999 or more than a year after Gucaban's separation from the company and incidentally, after she filed the instant complaint. It is not difficult to see that, shortly prior to and at the time of Gucaban's alleged resignation, there was actually no genuine corporate restructuring plan in place as yet. In other words, although the company might have been suffering from losses due to market decline as alleged, there was still no concrete plan for a corporate reorganization at the time Gonzalez presented to Gucaban the seemingly last available alternative options of voluntary resignation and termination by abolition of her office. It is then understandable for Gucaban, considering the attractive financial package which SMPI admittedly offered to her, to opt for resignation instead of suffer termination a consequence the certainty of which she was made to believe. As respondent was dismissed without cause, the NLRC ruling is correct that she is entitled to reinstatement and backwages, the latter to be computed from her dismissal up to the time of her actual reinstatement pursuant to Art. 279 of the Labor Code. However, there is the possibility that Gucaban's rejoining SMPI's workforce would only exacerbate the tension and strained relations which in the first place had given rise to this incident. Thus, the ruling of the Court of Appeals is modified in this respect. In lieu of reinstatement, an award of separation pay is in order, equivalent to one (1) month salary for every year of service.

15. Skippers United Pacific vs. Doza, G.R. No. 175558, February 8, 2012

Page 236

LABOR RELATIONS
Atty. Jefferson M. Marquez
Facts: Petitioner deployed De Gracia, Lata and Aprosta to work on board the vessel MV Wisdom Star. On December 3 1998, Skippers alleges that De Garcia smelling strongly of alcohol, went to the cabin of Gabriel Oleszek, MV Wisdom Stars Master. Skippers claims that he was rude and shouted noisily to the master. De Gracia left the masters cabin after a few minutes and was heard shouting very loudly somewhere down the corridors. The incident was evidenced by the Captains Report sent on said date. Furthermore, Skippers also claim that on January 22, 1999, Aprosta, De Gracia, Lata and Daza arrived in the masters cabin and demanded immediate repatriation because they were not satisfied with the ship. De Gracia, et al. threatened that they may become crazy any moment and demanded for all outstanding payments due to them. The incident is evidenced by a telex of Cosmoship MV Wisdom to skippers but had conflicting dates. De Gracia claims that Skippers failed to remit their respective allotments, compelling them to vent their grievances with the Romanian Seafarers Union. On January 28, 1999, the Filipino seafarers were unceremoniously discharged and immediately repatriated. Upon arrival in the Philippines, they filed a complaint for illegal dismissal with the LA. The LA dismissed the seafarers complaint as the seafarers demand for immediate repatriation due to the dissatisfaction with the ship is considered a voluntary pre-termination of employment. Such act was deemed akin to resignation recognized under Article 285 of the LC. The LA gave credence to the telex of the masters report that the seafarers indeed demanded immediate repatriation. The NLRC agreed with the LAs decision. The CA however reversed the LAs and the NLRCs decision. The Court deemed the telex message as a self-serving document that does not satisfy the requirement of substantial evidence, or that amount of relevant evidence which a reasonable mind might accept as adequate to justify the conclusion that petitioners indeed voluntarily demanded their immediate repatriation. Aggrieved, Skippers appeals the case with the Supreme Court. Issue: Whether or not the seafarers demand for immediate repatriation can be considered an act of voluntary resignation. Ruling: For a worker's dismissal to be considered valid, it must comply with both procedural and substantive due process. The legality of the manner of dismissal constitutes procedural due process, while the legality of the act of dismissal constitutes substantive due process. Procedural due process in dismissal cases consists of the twin requirements of notice and hearing. The employer must furnish the employee with two written notices before the termination of employment can be effected: (1) the first notice apprises the employee of the particular acts or omissions for which his dismissal is sought; and (2) the second notice informs the employee of the employer's decision to dismiss him. Before the issuance of the second notice, the requirement of a hearing must be complied with by giving the worker an opportunity to be heard. It is not necessary that an actual hearing be conducted. Substantive due process, on the other hand, requires that dismissal by the employer be made under a just or authorized cause under Articles 282 to 284 of the Labor Code. In this case, there was no written notice furnished to De Gracia, et al., regarding the cause of their dismissal. Cosmoship furnished a written notice (telex) to Skippers, the local manning agency, claiming that De Gracia, et al., were repatriated because the latter voluntarily pre-terminated their contracts. This telex was given credibility and weight by the Labor Arbiter and NLRC in deciding that there was pre-termination of the employment contract "akin to resignation" and no illegal dismissal. However, as correctly ruled by the CA, the telex message is "a biased and self-serving document that does not satisfy the requirement of substantial evidence." If, indeed, De Gracia, et al., voluntarily pre-terminated their contracts, then De Gracia, et al., should have submitted their written resignations. Article 285 of the Labor Code recognizes termination by the employee of the employment contract by "serving written notice on the employer at least one (1) month in advance." Given that provision, the law contemplates the requirement of a written notice of resignation. In the absence of a written resignation, it is safe to presume that the employer terminated the seafarers. In addition, the telex message relied upon by the Labor Arbiter and NLRC bore conflicting dates of 22 January 1998 and 22 January 1999, giving doubt to the veracity and authenticity of the document. In 22 January 1998, De Gracia, et al., were not even employed yet by the foreign principal.

Page 237

LABOR RELATIONS
Atty. Jefferson M. Marquez
PRESCRIPTION OF CLAIMS CASES: 1. Ludo & Luym Corp. vs Saornido, G.R. No. 140960, January 20, 2003 Facts: Petitioner LUDO & LUYM CORPORATION is engaged in the manufacture of coconut oil, corn starch, glucose and related products. It operates a manufacturing plant and a wharf where raw materials and finished products are shipped out. LUDO engaged the arrastre services of Cresencio Lu Arrastre Services (CLAS) for the loading and unloading of its finished products at the wharf. Accordingly, several arrastre workers were deployed by CLAS to perform the services needed by LUDO. These arrastre workers were subsequently hired, on different dates, as regular rank-and-file employees of LUDO every time the latter needed additional manpower services. Said employees thereafter joined respondent union, the LUDO Employees Union (LEU), which acted as the exclusive bargaining agent of the rank-and-file employees. Respondent union entered into a collective bargaining agreement with LUDO which provides certain benefits to the employees, the amount of which vary according to the length of service rendered by the availing employee. Thereafter, the union requested LUDO to include in its members period of service the time during which they rendered arrastre services to LUDO through the CLAS so that they could get higher benefits. LUDO failed to act on the request. Thus, the matter was submitted for voluntary arbitration. Issue: Whether or not benefits consisting of salary increases, vacation leave and sick leave benefits for the years 1977 to 1987 are already barred by prescription when private respondents filed their case in January 1995; Ruling: No. As regards petitioners contention that the money claim in this case is barred by prescription, we hold that this contention is without merit. So is petitioners stance that the benefits claimed by the respondents, i.e., sick leave, vacation leave and 13th-month pay, had already prescribed, considering the three-year period for the institution of monetary claims. Such determination is a question of fact which must be ascertained based on the evidence, both oral and documentary, presented by the parties before the Voluntary Arbitrator. In this case, the Voluntary Arbitrator found that prescription has not as yet set in to bar the respondents claims for the monetary benefits awarded to them. Basic is the rule that findings of fact of administrative and quasi-judicial bodies, which have acquired expertise because their jurisdiction is confined to specific matters, are generally accorded not only great respect but even finality. Here, the Voluntary Arbitrator received the evidence of the parties firsthand. No compelling reason has been shown for us to diverge from the findings of the Voluntary Arbitrator, especially since the appellate court affirmed his findings, that it took some time for respondent employees to ventilate their claims because of the repeated assurances made by the petitioner that it would review the company records and determine therefrom the validity of the claims, without expressing a categorical denial of their claims. As elucidated by the Voluntary Arbitrator: The respondents had raised prescription as defense. The controlling law, as ruled by the High Court, is: "The cause of action accrues until the party obligated refuses xxx to comply with his duty. Being warded off by promises, the workers not having decided to assert [their] right[s], [their] causes of action had not accrued" (Citation omitted.) Since the parties had continued their negotiations even after the matter was raised before the Grievance Procedure and the voluntary arbitration, the respondents had not refused to comply with their duty. They just wanted the complainants to present some proofs. The complainants cause of action had not therefore accrued yet. Besides, in the earlier voluntary arbitration case aforementioned involving exactly the same issue and employees similarly situated as the complainants, the same defense was raised and dismissed by Honorable Thelma Jordan, Voluntary Arbitrator.

Page 238

LABOR RELATIONS
Atty. Jefferson M. Marquez
2. Degamo vs. Avantgarde Shipping corp., G.R. no. 154460, November 22, 2005 Facts: On November 8, 1994, respondent Avantgarde Shipping Corporation, acting in behalf of its foreign principal, respondent Sembawang Johnson Management, Pte., Ltd., hired petitioner Lauro Degamo as Oiler of the vessel Nippon Reefer. While working in the vessels engine room, a spanner dropped and hit petitioner on his right thigh. He required surgery and hospitalization. He was repatriated to the Philippines on March 4, 1995. Petitioner was again operated and Avantgarde paid all his hospital bills and promised to work out his sickness benefit with Sembawang as soon as he was declared fit to work. On September 11, 1997, petitioner was declared fit to work. On December 24, 1997, petitioner asked Avantgarde to pay his sickness benefits. On January 6, 1998, Avantgarde replied that it could no longer act on petitioners claim as he had deviated from the legal procedure and, should he wish, he could personally follow-up with Sembawang. On March 4, 1998 and May 5, 1998, petitioner wrote a letter to Sembawang regarding his claim. Sembawang did not reply. On March 2, 2001, petitioner lodged a complaint for payment of disability benefits and other money claims against the respondents. The labor arbiter dismissed the case without prejudice, stating that the action had already prescribed. On appeal, the National Labor Relations Commission (NLRC) likewise ruled that petitioners cause of action had prescribed. Issue: Whether petitioners cause of action had already prescribed. Arguments: Petitioner, citing Article 1155 of the New Civil Code, contends that his cause of action had not prescribed as the running of the prescriptive period was tolled by his extrajudicial demand for unpaid sickness benefits on December 24, 1997. Respondents counter that the Civil Code provision on extinctive prescription applies only to obligations that are intrinsically civil in nature and is inapplicable to labor cases. Respondents assert that petitioners demand was made more than one year from his date of arrival in the Philippines, contrary to what is prescribed in Section 28 of the Philippine Overseas Employment Administration (POEA) Memorandum Circular No. 55, Series of 1996. They add that the institution of the action was beyond the three-year period prescribed in Article 291 of the Labor Code as his employment with the respondents ended on March 4, 1995 but the complaint was filed only on March 2, 2001. Ruling: Yes. We note that POEA Circular No. 55, Series of 1996 became effective only on January 1, 1997 while the employment contract between the parties was entered earlier on November 8, 1994. The earlier standard employment contract issued by the POEA did not have a provision on prescription of claims. Hence, the applicable provision in this case is Article 291 of the Labor Code which we shall now discuss. In Cadalin v. POEAs Administrator, we held that Article 291 covers all money claims from employer-employee relationship and is broader in scope than claims arising from a specific law. It is not limited to money claims recoverable under the Labor Code, but applies also to claims of overseas contract workers. Article 291 provides that all money claims arising from employer-employee relations shall be filed within three years from the time the cause of action accrued, otherwise, these shall be forever barred. A cause of action accrues upon the categorical denial of claim. Petitioners cause of action accrued only on January 6, 1998, when Avantgarde denied his claim and so breached its obligation to petitioner. Petitioner could not have a cause of action prior to this because his earlier requests were warded off by indefinite promises. The complaint filed on March 2, 2001 is beyond the three-year period mandated by the Labor Code.

3. Intercontinental Broadcasting Corp. vs. Panganiban, G.R. No. 151407, February 6, 2007 Facts: Respondent Ireneo Panganiban was employed as Assistant General Manager of the petitioner Intercontinental Broadcasting Corporation from

Page 239

LABOR RELATIONS
Atty. Jefferson M. Marquez
May 1986 until his preventive suspension on August 26, 1988. Respondent resigned from his employment on September 2, 1988. On April 12, 1989, respondent filed with the trial court a case against the members of the Board of Administrators (BOA) of petitioner alleging, among others, non-payment of his unpaid commissions. A motion to dismiss was filed by Joselito Santiago, one of the defendants, on the ground of lack of jurisdiction, as respondent's claim was a labor money claim, but this was denied by the RTC. Thus, Santiago filed a petition for certiorari with the CA, which granted Santiago's petition for lack of jurisdiction and set aside the RTC's Orders. Thereafter, respondent was elected by the BOA as Vice-President for Marketing in July 1992. He resigned in April 1993. On July 24, 1996, respondent filed against petitioner a complaint for illegal dismissal, separation pay, retirement benefits, unpaid commissions, and damages. Issue: Whether or not respondent's claim for unpaid commissions has already prescribed. Ruling: Yes. The applicable law in this case is Article 291 of the Labor Code which provides that "all money claims arising from employer-employee relations accruing during the effectivity of this Code shall be filed within three (3) years from the time the cause of action accrued; otherwise they shall be forever barred." The term "money claims" covers all money claims arising from an employer-employee relation. Like other causes of action, the prescriptive period for money claims is subject to interruption, and in the absence of an equivalent Labor Code provision for determining whether the said period may be interrupted, Article 1155 of the Civil Code may be applied. Thus, the prescription of an action is interrupted by (a) the filing of an action, (b) a written extrajudicial demand by the creditor, and (c) a written acknowledgment of the debt by the debtor. On this point, the Court ruled that although the commencement of a civil action stops the running of the statute of prescription or limitations, its dismissal or voluntary abandonment by plaintiff leaves the parties in exactly the same position as though no action had been commenced at all. Hence, while the filing of the civil could have interrupted the running of the three-year prescriptive period, its consequent dismissal by the CA due to lack of jurisdiction effectively canceled the tolling of the prescriptive period within which to file his money claim, leaving respondent in exactly the same position as though no civil case had been filed at all. The running of the three-year prescriptive period not having been interrupted by the filing of the civil case, respondent's cause of action had already prescribed on September 2, 1991, three years after his cessation of employment on September 2, 1988. Consequently, when respondent filed his complaint for illegal dismissal, separation pay, retirement benefits, and damages in July 24, 1996, his claim, clearly, had already been barred by prescription.

4. Far East Agricultural Supply vs. Lebatique, G.R. No. 162813, February 12, 2007 Facts: Petitioner Far East hired on March 4, 1996 private respondent Jimmy Lebatique as truck driver with a daily wage of P223.50. He delivered animal feeds to the companys clients. On January 24, 2000, Lebatique complained of nonpayment of overtime work particularly on January 22, 2000, when he was required to make a second delivery in Novaliches, Quezon City. That same day, Manuel Uy, brother of Far Easts General Manager and petitioner Alexander Uy, suspended Lebatique apparently for illegal use of company vehicle. Even so, Lebatique reported for work the next day but he was prohibited from entering the company premises. On January 26, 2000, Lebatique sought the assistance of the Department of Labor and Employment (DOLE) Public Assistance and Complaints Unit concerning the nonpayment of his overtime pay. According to Lebatique, two days later, he received a telegram from petitioners requiring him to report for work. When he did the next day, January 29, 2000, Alexander asked him why he was claiming overtime pay. Lebatique explained that he had never been paid for overtime work since he started working for the company. He also told Alexander that Manuel had fired him. After talking to Manuel, Alexander terminated Lebatique and told him to look for another job. On March 20, 2000, Lebatique filed a complaint for illegal dismissal and nonpayment of overtime pay.

Page 240

LABOR RELATIONS
Atty. Jefferson M. Marquez
Petitioners maintain that Lebatique, as a driver, is not entitled to overtime pay since he is field personnel whose time outside the company premises cannot be determined with reasonable certainty. According to petitioners, the drivers do not observe regular working hours unlike the other office employees. The drivers may report early in the morning to make their deliveries or in the afternoon, depending on the production of animal feeds and the traffic conditions. Petitioners also aver that Lebatique worked for less than eight hours a day. Respondent, on his part claims that he is not a field personnel, thus, he is entitled to overtime pay and service incentive leave pay. Issue: Whether or not Lebatique is estopped from claiming that he was illegally dismissed since his complaint before the DOLE was only on the nonpayment of his overtime pay; Ruling: All money claims arising from an employer-employee relationship shall be filed within three years from the time the cause of action accrued; otherwise, they shall be forever barred. Further, if it is established that the benefits being claimed have been withheld from the employee for a period longer than three years, the amount pertaining to the period beyond the three-year prescriptive period is therefore barred by prescription. The amount that can only be demanded by the aggrieved employee shall be limited to the amount of the benefits withheld within three years before the filing of the complaint. Lebatique timely filed his claim for service incentive leave pay, considering that in this situation, the prescriptive period commences at the time he was terminated. On the other hand, his claim regarding nonpayment of overtime pay since he was hired in March 1996 is a different matter. In the case of overtime pay, he can only demand for the overtime pay withheld for the period within three years preceding the filing of the complaint on March 20, 2000.

5. Victory Liner, Inc. vs. Race, G.R. No. 164820, March 28, 2007 Facts: In June 1993, respondent was employed by the petitioner as a bus driver. On the night of 24 August 1994, the bus he was driving was bumped by a Dagupan-bound bus. As a consequence thereof, respondent suffered a fractured left leg and was rushed to the Country Medical and Trauma Center in Tarlac City where he was operated on and confined from 24 August 1994 up to 10 October 1994. One month after his release from the said hospital, the respondent was confined again for further treatment of his fractured left leg at the Specialist Group Hospital in Dagupan City. His confinement therein lasted a month. Petitioner shouldered the doctors professional fee and the operation, medication and hospital expenses of the respondent in the aforestated hospitals. In January 1998, the respondent, still limping heavily, went to the petitioners office to report for work. He was, however, informed by the petitioner that he was considered resigned from his job. Respondent refused to accede and insisted on having a dialogue with the petitioners officer named Yolanda Montes. During their meeting, Montes told him that he was deemed to have resigned from his work and to accept a consideration of P50,000.00. Respondent rejected the explanation and offer. Thereafter, before Christmas of 1998, he again conversed with Montes who reiterated to him that he was regarded as resigned but raised the consideration therein to P100,000.00. Respondent rebuffed the increased offer. On 30 June 1999, respondent, through his counsel, sent a letter to the petitioner demanding employment-related money claims. There being no response from the petitioner, the respondent filed before the Labor Arbiter on 1 September 1999 a complaint for (1) unfair labor practice; (2) illegal dismissal; (3) underpayment of wages; (4) nonpayment of overtime and holiday premium, service incentive leave pay, vacation and sick leave benefits, 13th month pay; (5) excessive deduction of withholding tax and SSS premium; and (6) moral and exemplary damages and attorneys fees. In its Position Paper dated 27 March 2000, petitioner claimed that the respondents cause of action against petitioner had already prescribed because when the former instituted the aforesaid complaint on 1 September 1999, more than five years had already lapsed from the accrual of his cause of action on 24 August 1994. Issue: Whether or not the cause of action of respondent has already prescribed;

Page 241

LABOR RELATIONS
Atty. Jefferson M. Marquez
Ruling: In illegal dismissal cases, the employee concerned is given a period of four years from the time of his dismissal within which to institute a complaint. This is based on Article 1146 of the New Civil Code which states that actions based upon an injury to the rights of the plaintiff must be brought within four years. The four-year prescriptive period shall commence to run only upon the accrual of a cause of action of the worker. It is settled that in illegal dismissal cases, the cause of action accrues from the time the employment of the worker was unjustly terminated. Thus, the four-year prescriptive period shall be counted and computed from the date of the employees dismissal up to the date of the filing of complaint for unlawful termination of employment. Proceeding therefrom, we shall now discuss and determine when the respondents cause of action accrued in order to ascertain whether the same had already prescribed. It is error to conclude that the employment of the respondent was unjustly terminated on 10 November 1994 because he was, at that time, still confined at the Specialist Group Hospital, Dagupan City, for further treatment of his fractured left leg. He must be considered as merely on sick leave at such time. Likewise, the respondent cannot also be deemed as illegally dismissed from work upon his release from the said hospital in December 1994 up to December 1997 since the records show that the respondent still reported for work to the petitioner and was granted sick and disability leave by the petitioner during the same period. The respondent must be considered as unjustly terminated from work in January 1998 since this was the first time he was informed by the petitioner that he was deemed resigned from his work. During that same occasion, the petitioner, in fact, tried to convince the respondent to accept an amount of P50,000.00 as a consolation for his dismissal but the latter rejected it. Thus, it was only at this time that the respondents cause of action accrued. Consequently, the respondents filing of complaint for illegal dismissal on 1 September 1999 was well within the fouryear prescriptive period. It is also significant to note that from 10 November 1994 up to December 1997, the petitioner never formally informed the respondent of the fact of his dismissal either through a written notice or hearing. Indeed, it cannot be gainfully said that respondent was unlawfully dismissed on 10 November 1994 and that the cause of action accrued on that date.

6. J.K. Mercado & Sons Agricultural Enterprises vs. Sto. Tomas, G.R. No. 158084, August 29, 2008 Facts: On December 3, 1993, the Regional Tripartite Wages and Productivity Board, Region XI, issued Wage Order No. RTWPB-XI-03, granting a Cost of Living Allowance (COLA) to covered workers. On January 28, 1994, petitioner filed an application for exemption from the coverage of the aforesaid wage order. Thus, however, was denied by the regional wage board in an Order dated April 11, 1994, Notwithstanding the said order, private respondents were not given the benefits due them under Wage Order No. RTWPB-XI-03. On July 10, 1998, private respondents filed an Urgent Motion for Writ of Execution, and Writ of Garnishment in RTWPB-XI-03-CBBE-94 NWPBC Case No. E-95-087 Case No. R1100 seeking the enforcement of subject wage order against several entities including herein petitioner. On October 7, 1998, the OIC-Regional Director, Region XI, issued a Writ of Execution for the enforcement of the Order dated April 11, 1994 of the Regional Tripartite Wages and Productivity Board. On November 17, 1998 and November 23, 1998, respectively, petitioner filed a Motion to Quash the Writ of Execution and a Supplemental Motion to the Motion to Quash. Petitioner argued that herein private respondents right had already prescribed due to their failure to move for the execution of the April 11, 1994 Order within the period provided under Article 291 of the Labor Code, as amended, or within three (3) years from the finality of the said order. Issue: Whether or not the claim of respondents have already prescribed.

Page 242

LABOR RELATIONS
Atty. Jefferson M. Marquez
Ruling: Art. 291 of the Labor Code applies to money claims in general and provides for a 3-year prescriptive period to file them. On the other hand, respondent employees money claims in this case had been reduced to a judgment, in the form of a Wage Order, which has become final and executory. The prescription applicable, therefore, is not the general one that applies to money claims, but the specific one applying to judgments. Thus, the right to enforce the judgment, having been exercised within five years, has not yet prescribed. Stated otherwise, a claimant has three years to press a money claim. Once judgment is rendered in her favor, she has five years to ask for execution of the judgment, counted from its finality. This is consistent with the rule on statutory construction that a general provision should yield to a specific one and with the mandate of social justice that doubts should be resolved in favor of labor.

7. Reyes vs. Nlrc, G.R. No. 180551, February 10, 2009 Facts: The present Petition arose from a Complaint for illegal dismissal with claims for moral and exemplary damages and attorneys fees filed by petitioner against respondents Coca Cola Bottlers Philippines (CCBP) and Rotaida Taguibao (Taguibao) before the Labor Arbiter on 14 June 2004. Respondent CCBP is a corporation engaged in the business of production and distribution of carbonated drinks, and Taguibao is its Human Resource Manager. In his Complaint, petitioner alleged that he was first employed by respondent CCBP, through Interserve Manpower Agency (Interserve), as a Leadman in February 1988. Petitioner was initially assigned to the Mendiola Sales Office of respondent CCBP. Petitioners employment contract was renewed every five months and he was assigned a different task every time. Such an arrangement continued until petitioner was directly hired by respondent CCBP as a Route Salesman on 15 September 2000. Exactly one year from the time of petitioners employment as a Route Salesman, respondent CCBP, thru Taguibao, terminated his services on 15 September 2001. Since he already acquired the status of a regular employee, petitioner asserted that his dismissal from employment without the benefit of due process was unlawful. Issue: Whether or not respondents claim for backwages has already prescribed. Ruling: The Court was more emphatic in Philippine Industrial Security Agency Corporation v. Dapiton, when it ruled that backwages had to be paid by the employer as part of the price or penalty he had to pay for illegally dismissing his employee. It was to be computed from the time of the employees illegal dismissal (or from the time his compensation was withheld from him) up to the time of his reinstatement. One of the natural consequences of a finding that an employee has been illegally dismissed is the payment of backwages corresponding to the period from his dismissal up to actual reinstatement. The statutory intent of this matter is clearly discernible. The payment of backwages allows the employee to recover from the employer that which he has lost by way of wages as a result of his dismissal. Logically, it must be computed from the date of petitioners illegal dismissal up to the time of actual reinstatement. There can be no gap or interruption, lest we defeat the very reason of the law in granting the same. That petitioner did not immediately file his Complaint should not affect or diminish his right to backwages, for it is a right clearly granted to him by law -- should he be found to have been illegally dismissed -- and for as long as his cause of action has not been barred by prescription. The law fixes the period of time within which petitioner could seek remedy for his illegal dismissal and for as long as he filed his Complaint within the prescriptive period, he shall be entitled to the full protection of his right to backwages. In illegal dismissal cases, the employee concerned is given a period of four years from the time of his illegal dismissal within which to institute the complaint. This is based on Article 1146 of the New Civil Code which states that actions based upon an injury to the rights of the plaintiff must be brought within four years. The four-year prescriptive period shall commence to run only upon the accrual of a cause of action of the worker. Here, petitioner was dismissed from service on 15 September 2001. He filed his complaint for illegal dismissal on 14 June 2004. Clearly, then, the instant case was filed within the prescriptive period.

Page 243

LABOR RELATIONS
Atty. Jefferson M. Marquez
8. LWV Construction Corp. vs. Dupo, G.R. No. 172342, July 13, 2009 Facts: Petitioner, a domestic corporation which recruits Filipino workers, hired respondent Marcelo Dupo as Civil Structural Superintendent to work in Saudi Arabia for its principal, Mohammad Al-Mojil Group/Establishment (MMG). On February 26, 1992, respondent signed his first overseas employment contract, renewable after one year. It was renewed five times on the following dates: May 10, 1993, November 16, 1994, January 22, 1996, April 14, 1997, and March 26, 1998. All were fixed-period contracts for one year. The sixth and last contract stated that respondents employment starts upon reporting to work and ends when he leaves the work site. Respondent left Saudi Arabia on April 30, 1999 and arrived in the Philippines on May 1, 1999. Respondent then has signed 6 overseas contracts and worked for seven years in Saudi Arabia. On July 6, 1999, respondent through a letter resigned from his work and asked MMG to give him his long service award in accordance with the Article 87 of Saudi Law which states that; Article 87 Where the term of a labor contract concluded for a specified period comes to an end or where the employer cancels a contract of unspecified period, the employer shall pay to the workman an award for the period of his service to be computed on the basis of half a months pay for each of the first five years and one months pay for each of the subsequent years. The last rate of pay shall be taken as basis for the computation of the award. For fractions of a year, the workman shall be entitled to an award which is proportionate to his service period during that year. Furthermore, the workman shall be entitled to the service award provided for at the beginning of this article in the following cases: If he is called to military service. If a workman resigns because of marriage or childbirth. If the workman is leaving the work as a result of a force majeure beyond his control. (Emphasis supplied.) However, MMG did not reply to the letter of the respondent Dupo which led to the filing of the case before the labor arbiter for the payment of the long service award in the amount of US$12,640.33. On the other hand, petitioner presented two defenses namely payment and prescription. Firstly, petitioner said the long service award has already been paid every time each of the contracts of employment of the respondent comes to an end, since, the contract is for a fixed period of time. In effect, the severance pay received by the respondent every time each of the 6 contracts of employment comes to an end, is also the longevity service award. Petitioner claimed that long service award is the same with severance pay. Secondly, petitioner insists that prescription barred respondents claim for long service award because under Article 13 of the Saudi Labor Law it provides that no case or claim relating to any of the rights provided for under said law shall be heard after the lapse of 12 months from the date of the termination of the contract. Respondents sixth contract ended on April 30, 1999 the date he left his work which was also in effect the date of the termination of his contract, and he filed the case on December 11, 2000 which is 1 year and seven months from the date of the termination of his contract. The labor arbiter, NLRC, and CA decided all in favor of the respondent. Issues: Whether or not CA erred in ruling that respondent is entitled to long service pay which is different from severance pay. 2. Whether or not the cause of action has prescribed. Ruling: SC said that CA has committed an error in ruling that the long service pay is different from severance pay. According to SC the severance pay received by the respondent at the end of each of the six contracts of employment is equivalent to the long service pay. This is the reason why the formula in computing the severance pay is the same with the computation of the long service award. Moreover, SC said that respondents employment contracts expressly stated that his employment ended upon his departure from work. Each year he departed from work and successively new contracts were executed before he reported for work anew. His service was not cumulative. Pertinently, in Brent School, Inc. v. Zamora, we said that a fixed term is an essential and natural appurtenance of overseas employment contracts as in this case. We also said in that case that under American law, [w]here a contract specifies the period of its duration, it terminates on the expiration of such period. A contract of employment for a definite period terminates by its own terms at the end of such period. As it is, Article 72 of the Saudi Labor Law is also of similar import. It reads: A labor contract concluded for a specified period shall terminate upon the expiry of its term. If both parties continue to enforce the contract, thereafter, it shall be considered renewed for an unspecified period.

Page 244

LABOR RELATIONS
Atty. Jefferson M. Marquez
SC ruled that the claim has not yet prescribed because the law that should be applied on prescription is not the Saudi Law which grants 12 months period of time to file the claim from the time of the termination of contract but it should be the Labor Code particularly ART. 291. Money claims. All money claims arising from employer-employee relations accruing during the effectivity of this Code shall be filed within three (3) years from the time the cause of action accrued; otherwise they shall be forever barred. The reason is because prescription is a procedural law and under the conflict of laws rules of the Philippines the procedural law of the lex fori or law of the forum (law of the place where the case is filed) must be applied. However, an argument can be raised that even if the conflict of laws rule provides that the procedural law of the lex fori must be followed, Sec. 48 of our Code of Civil Procedure which is a borrowing statute provides that If by the laws of the state or country where the cause of action arose, the action is barred, it is also barred in the Philippine Islands. Section 48 has not been repealed or amended by the Civil Code of the Philippines. Article 2270 of said Code repealed only those provisions of the Code of Civil Procedure as to which were inconsistent with it. There is no provision in the Civil Code of the Philippines, which is inconsistent with or contradictory to Section 48 of the Code of Civil Procedure (Paras, Philippine Conflict of Laws, 104 [7th ed.]). In the light of the 1987 Constitution, however, Section 48 [of the Code of Civil Procedure] cannot be enforced ex proprio vigore insofar as it ordains the application in this jurisdiction of [Article] 156 of the Amiri Decree No. 23 of 1976. The courts of the forum will not enforce any foreign claim obnoxious to the forums public policy. To enforce the one-year prescriptive period of the Saudi Law as regards the claims in question would contravene the public policy on the protection to labor. Respondents complaint was filed well within the three-year prescriptive period under Article 291 of our Labor Code. This point, however, has already been mooted by SCs finding that respondents service award had been paid, albeit the payroll termed such payment as severance pay. Petition is Granted.

9. PLDT v. Pingol, G.R. No. 182622, September 8, 2010 Facts: In 1979, respondent Roberto R. Pingol (Pingol) was hired by petitioner PLDT in 1979, as a maintenance technician. On April 13, 1999, while still under the employ of PLDT, Pingol was admitted at The Medical City, Mandaluyong City, for paranoid personality disorder due to financial and marital problems. He was discharged from the hospital. Thereafter, he reported for work but frequently absented himself due to his poor mental condition. Pingol was absent from work without official leave from September 16, 1999 to December 31, 1999. PLDT, sent him notices with a stern warning that he would be dismissed from employment if he continued to be absent without official leave pursuant to PLDT Systems Practice A-007 which provides that Absence without authorized leaves for seven (7) consecutive days is subject to termination from the service. January 1, 2000, PLDT terminated his services on the grounds of unauthorized absences and abandonment of office. On March 29, 2004, four years later, Pingol filed a Complaint for Constructive Dismissal and Monetary Claims[6] against PLDT. In his complaint, he alleged that he was hastily dismissed from his employment on January 1, 2000. In response, PLDT filed a motion to dismiss claiming, among others, that respondents cause of action had already prescribed as the complaint was filed four (4) years and three (3) months after his dismissal. Labor Arbiter (LA) issued an order granting petitioners Motion to Dismiss on the ground of prescription. As correctly cited by (PLDT), as ruled by the Supreme Court in the case of Callanta vs. Carnation Phils., 145 SCRA 268, the complaint for illegal dismissal must be filed within four (4) years from and after the date of dismissal. The NLRC in its November 15, 2006 Resolution reversed the LAs resolution and favored Pingol. Let the entire records of the case be REMANDED to the Labor Arbiter a quo for further proceedings. Issues: Whether or not respondent Pingol filed his complaint for constructive dismissal and money claims within the prescriptive period of four (4) years as provided in Article 1146 of the Civil Code[11][12] respectively and three (3) years as provided in Article 291 of the Labor Code, When is the pivotal date when the cause of action of respondent Pingol accrued?

Page 245

LABOR RELATIONS
Atty. Jefferson M. Marquez
Ruling: Parties apparently do not dispute the applicable prescriptive period. Article 1146 of the New Civil Code provides: Art. 1146. The following actions must be instituted within four years: (1) Upon an injury to the rights of the plaintiff; In Callanta v. Carnation,[16] when one is arbitrarily and unjustly deprived of his job or means of livelihood, the action instituted to contest the legality of one's dismissal from employment constitutes, in essence, an action predicated "upon an injury to the rights of the plaintiff," as contemplated under Art. 1146 of the New Civil Code, which must be brought within four (4) years. With regard to the prescriptive period for money claims, Article 291 of the Labor Code states: Article 291. Money Claims. All money claims arising from employer-employee relations accruing during the effectivity of this Code shall be filed within three (3) years from the time the cause of action accrued; otherwise they shall be barred forever. It is a settled jurisprudence that a cause of action has three (3) elements, to wit: (1) a right in favor of the plaintiff by whatever means and under whatever law it arises or is created; (2) an obligation on the part of the named defendant to respect or not to violate such right; and (3) an act or omission on the part of such defendant violative of the right of the plaintiff or constituting a breach of the obligation of the defendant to the plaintiff. Pingol asserts that his complaint was filed within the prescriptive period of four (4) years. He claims that his cause of action did not accrue on January 1, 2000 because he was not categorically and formally dismissed or his monetary claims categorically denied by petitioner PLDT on said date. Further, respondent Pingol posits that the continuous follow-up of his claim with petitioner PLDT from 2001 to 2003 should be considered in the reckoning of the prescriptive period. Petitioner PLDT, on the other hand, contends that respondent Pingol was dismissed from the service on January 1, 2000 and such fact was even alleged in the complaint he filed before the LA. He never contradicted his previous admission that he was dismissed on January 1, 2000. Such admitted fact does not require proof. The Court agrees with petitioner PLDT. Judicial admissions made by parties in the pleadings, or in the course of the trial or other proceedings in the same case are conclusive and so does not require further evidence to prove them. These admissions cannot be contradicted unless previously shown to have been made through palpable mistake or that no such admission was made.[18] In Pepsi Cola Bottling Company v. Guanzon,] it was written: that the dismissal of the private respondent's complaint was still proper since it is apparent from its face that the action has prescribed. Private respondent himself alleged in the complaint that he was unlawfully dismissed in 1979 while the complaint was filed only on November 14, 1984. xxx (Emphasis supplied. Citations omitted.) Pingol himself alleged the date January 1, 2000 as the date of his dismissal in his complaint[20] filed on March 29, 2004, exactly four (4) years and three (3) months later. Respondent never denied making such admission or raised palpable mistake as the reason therefor. Thus, the petitioner correctly relied on such allegation in the complaint to move for the dismissal of the case on the ground of prescription. The Labor Code has no specific provision on when a claim for illegal dismissal or a monetary claim accrues. Thus, the general law on prescription applies. Article 1150 of the Civil Code states: Article 1150. The time for prescription for all kinds of actions, when there is no special provision which ordains otherwise, shall be counted from the day they may be brought. (Emphasis supplied) The day the action may be brought is the day a claim starts as a legal possibility. In the present case, January 1, 2000 was the date that respondent Pingol was not allowed to perform his usual and regular job as a maintenance technician. Respondent Pingol cited the same date of dismissal in his complaint before the LA. As, thus, correctly ruled by the LA, the complaint filed had already prescribed. Respondent claims that between 2001 and 2003, he made follow-ups with PLDT management regarding his benefits. This, to his mind, tolled the running of the prescriptive period. The rule in this regard is covered by Article 1155 of the Civil Code. Its applicability in labor cases was upheld in the case of International Broadcasting Corporation v. Panganiban where it was written: Like other causes of action, the prescriptive period for money claims is subject to interruption, and in the absence of an equivalent Labor Code provision for determining whether the said period may be interrupted, Article 1155 of the Civil Code may be applied, to wit: ART. 1155. The prescription of actions is interrupted when they are filed before the Court, when there is a written extrajudicial demand by the creditors, and when there is any written acknowledgment of the debt by the debtor. Thus, the prescription of an action is interrupted by (a) the filing of an action, (b) a written extrajudicial demand by the creditor, and (c) a written acknowledgment of the debt by the debtor.

Page 246

LABOR RELATIONS
Atty. Jefferson M. Marquez
Pingol never made any written extrajudicial demand. Neither did petitioner make any written acknowledgment of its alleged obligation. Thus, the claimed follow-ups could not have validly tolled the running of the prescriptive period. It is worthy to note that respondent never presented any proof to substantiate his allegation of follow-ups. Unfortunately, respondent Pingol has no one but himself to blame for his own predicament. By his own allegations in his complaint, he has barred his remedy and extinguished his right of action. Although the Constitution is committed to the policy of social justice and the protection of the working class, it does not necessary follow that every labor dispute will be automatically decided in favor of labor. The management also has its own rights. Out of Its concern for the less privileged in life, this Court, has more often than not inclined, to uphold the cause of the worker in his conflict with the employer. Such leaning, however, does not blind the Court to the rule that justice is in every case for the deserving, to be dispensed in the light of the established facts and applicable law and doctrine. 10. Medline Management Inc. vs. Roslinda, G.R. No. 168715, September 15, 2010 Facts: Petitioner Medline Management, Inc. (MMI), on behalf of its foreign principal, petitioner Grecomar Shipping Agency (GSA), hired Juliano Roslinda (Juliano) to work on board the vessel MV "Victory." Juliano was previously employed by the petitioners under two successive separate employment contracts of varying durations. His latest contract was approved by the POEA on September 9, 1998 for a duration of nine months. In accordance with which, he boarded the vessel MV "Victory" on October 25, 1998 as an oiler and, after several months of extension, was discharged on January 20, 2000. Months after his repatriation, or on March 6, 2000, Juliano consulted Dr. Pamela R. Lloren (Dr. Lloren) of Metropolitan Hospital. He complained about abdominal distention which is the medical term for a patient who vomits previously ingested foods. From March 8 to August 24, 2000, Juliano has undergone Hemodialysis, a method of removing waste products such as creatinine and urea, as well as freeing water from the blood, when the kidneys are in renal failure. On August 27, 2001, Juliano died. On September 4, 2003, his wife Gliceria Roslinda and son Ariel Roslinda, respondents herein, filed a complaint against MMI and GSA for payment of death compensation, reimbursement of medical expenses, damages, and attorney's fees before the Labor Arbitration Branch of the NLRC. Instead of filing an answer, they filed a Motion to Dismiss on the grounds of prescription, lack of jurisdiction and prematurity. Petitioners contended that the action has already prescribed because it was filed three years, seven months and 22 days from the time the deceased seafarer reached the point of hire. Issue: Whether or not the claim is not yet barred by prescription despite the fact that it was filed beyond the one-year prescriptive period provided by the POEA Standard Employment Contract. Ruling: The employment contract signed by Juliano stated that "Upon approval, the same shall be deemed an integral part of the Standard Employment Contract (SEC) for seafarers." Section 28 of the POEA SEC states: SECTION 28.JURISDICTION XXX Recognizing the peculiar nature of overseas shipboard employment, the employer and the seafarer agree that all claims arising from this contract shall be made within one (1) year from the date of the seafarer's return to the point of hire. (Emphasis supplied) On the other hand, the Labor Code states: ART. 291.Money claims. All money claims arising from employer-employee relations accruing during the effectivity of this Code shall be filed within three (3) years from the time the cause of action accrued; otherwise they shall forever be barred. In Southeastern Shipping v. Navarra, Jr., we ruled that "Article 291 is the law governing the prescription of money claims of seafarers, a class of overseas contract workers. This law prevails over Section 28 of the Standard Employment Contract for Seafarers which provides for claims to be brought only within one year from the date of the seafarer's return to the point of hire." We further declared that "for the guidance of all, Section 28 of the Standard Employment Contract for Seafarers, insofar as it limits the prescriptive period within which the seafarers may file their money claims, is hereby declared null and void. The applicable provision is Article 291 of the Labor Code, it being more favorable to the seafarers and more in accord with the State's declared policy to afford full protection to labor. The prescriptive period in the present case is thus three years from the time the cause of action accrues."

Page 247

LABOR RELATIONS
Atty. Jefferson M. Marquez
In the present case, the cause of action accrued on August 27, 2001 when Juliano died. Hence, the claim has not yet prescribed, since the complaint was filed with the arbitration branch of the NLRC on September 4, 2003.

11. University of East vs. University of East Employees Assoc., G.R. No. 179593, September 14, 2011 Facts: Petitioner University of the East (UE) is an educational institution duly organized and existing under Philippine laws. On the other hand, respondent University of the East Employees' Association (UEEA) is a duly registered labor union of the rank-and-file employees of UE. It appears from the records that prior to school year (SY) 1983-1984, the 70% incremental proceeds from tuition fee increases as mandated by Presidential Decree No. 451 (P.D. No. 451), as amended, was distributed by UE in proportion to the average number of academic and nonacademic personnel. The distribution scheme became the subject of an Agreement dated October 18, 1983 signed by the management, faculty association and respondent. Starting SY 1994-1995, however, the 70% incremental proceeds from the tuition fee increase was distributed by UE to its covered employees based on a new formula of percentage of salary. On June 19, 1995, a tripartite meeting was held among the representatives of management, faculty union and UEEA. In the said meeting, it was agreed that the distribution of the incremental proceeds would now be based on percentage of salary, and not anymore on the average number of personnel. The Minutes of the June 19, 1995 meeting was signed and attested to by UEEA officers who attended. On April 27, 1999, UEEA filed a complaint before the NLRC for non-payment/underpayment of the rank-and-file employees' share of the tuition fee increases against UE pursuant to P.D. No. 451, as amended, and Republic Act (R.A.) No. 6728 otherwise known as Government Assistance to Students and Teachers in Private Education Act. UE asserted that the claim of the UEEA was already barred since it was filed three (3) years from the time its supposed cause of action accrued. ISSUE: Whether or not prescription has already set in. RULING: The Court agrees with UE and holds that UEEA's right to question the distribution of the incremental proceeds for SY 1994-1995 has already prescribed. Article 291 of the Labor Code provides that money claims arising from an employer-employee relationship must be filed within three (3) years from the time the cause of action accrued. In the present case, the cause of action accrued when the distribution of the incremental proceeds based on percentage of salary of the covered employees was discussed in the tripartite meeting held on June 19, 1995. UEEA did not question the manner of its distribution and only on April 27, 1999 did it file an action based therein. Hence, prescription had set in.

Page 248

You might also like